compendium ome - toomateseotvos de hungría, que se iniciaron en 1894, precisamente durante la...

903
COMPENDIUM OME Olimpiada Matemática Española 1964 2021 Con todas las soluciones oficiales Gerard Romo Garrido

Upload: others

Post on 09-Mar-2021

6 views

Category:

Documents


0 download

TRANSCRIPT

Page 1: COMPENDIUM OME - ToomatesEotvos de Hungría, que se iniciaron en 1894, precisamente durante la efervescencia de fin de siglo, consecuencia de la cual fue también el proceso iniciado

COMPENDIUM OME

Olimpiada Matemática Española

1964 – 2021

Con todas las soluciones oficiales

Gerard Romo Garrido

Page 2: COMPENDIUM OME - ToomatesEotvos de Hungría, que se iniciaron en 1894, precisamente durante la efervescencia de fin de siglo, consecuencia de la cual fue también el proceso iniciado

Toomates Coolección

Los documentos de Toomates son materiales digitales y gratuitos. Son digitales porque están pensados para ser consultados

mediante un ordenador, tablet o móvil. Son gratuitos porque se ofrecen a la comunidad educativa sin coste alguno. Los libros de

texto pueden ser digitales o en papel, gratuitos o en venta, y ninguna de estas opciones es necesariamente mejor o peor que las otras. Es más: Suele suceder que los mejores docentes son los que piden a sus alumnos la compra de un libro de texto en papel, esto es un

hecho. Lo que no es aceptable, por inmoral y mezquino, es el modelo de las llamadas "licencias digitales" con las que las editoriales

pretenden cobrar a los estudiantes, una y otra vez, por acceder a los mismos contenidos (unos contenidos que, además, son de una bajísima calidad). Este modelo de negocio es miserable, pues impide el compartir un mismo libro, incluso entre dos hermanos,

pretende convertir a los estudiantes en un mercado cautivo, exige a los estudiantes y a las escuelas costosísimas líneas de Internet,

pretende pervertir el conocimiento, que es algo social, público, convirtiéndolo en un producto de propiedad privada, accesible solo a aquellos que se lo puedan permitir, y solo de una manera encapsulada, fragmentada, impidiendo el derecho del alumno de poseer

todo el libro, de acceder a todo el libro, de moverse libremente por todo el libro.

Nadie puede pretender ser neutral ante esto: Mirar para otro lado y aceptar el modelo de licencias digitales es admitir un mundo más injusto, es participar en la denegación del acceso al conocimiento a aquellos que no disponen de medios económicos, en un mundo

en el que las modernas tecnologías actuales permiten, por primera vez en la historia de la Humanidad, poder compartir el

conocimiento sin coste alguno, con algo tan simple como es un archivo "pdf". El conocimiento no es una mercancía. El proyecto Toomates tiene como objetivo la promoción y difusión entre el profesorado y el colectivo de estudiantes de unos

materiales didácticos libres, gratuitos y de calidad, que fuerce a las editoriales a competir ofreciendo alternativas de pago atractivas

aumentando la calidad de unos libros de texto que actualmente son muy mediocres, y no mediante retorcidas técnicas comerciales. Este documento se comparte bajo una licencia “Creative Commons”: Se permite, se promueve y se fomenta cualquier uso,

reproducción y edición de todos estos materiales siempre que sea sin ánimo de lucro y se cite su procedencia. Todos los documentos

se ofrecen en dos versiones: En formato “pdf” para una cómoda lectura y en el formato “doc” de MSWord para permitir y facilitar su edición y generar versiones parcial o totalmente modificadas. Se agradecerá cualquier observación, comentario o colaboración a

[email protected]

La biblioteca Toomates Coolección consta de los siguientes libros:

Bloques temáticos: Problem-solving Libros de texto (en catalán)

Geometría Axiomática pdf 1 2 ... 23

Problemas de Geometría pdf 1 2 3 4 5 6 7 8 9

Introducción a la Geometría pdf doc

Teoría de números

pdf 1 2 3

Trigonometría pdf doc pdf doc

Desigualdades pdf doc

Números complejos pdf doc pdf doc

Álgebra pdf doc pdf 1 2 3 4

Combinatoria

pdf doc

Probabilidad

pdf doc

Guía del estudiante de Olimpiadas Matemáticas

pdf

Combinatòria i Probabilitat pdf doc

Estadística pdf doc

Funcions pdf doc

Geometria analítica pdf 1 2

Àlgebra Lineal 2n batxillerat pdf doc

Geometria Lineal 2n batxillerat pdf doc

Càlcul Infinitesimal 2n batxillerat pdf 1 2

Programació Lineal 2n batxillerat pdf doc

Recopilaciones de pruebas PAU:

Catalunya TEC , Catalunya CCSS , Galicia , Portugal A , Portugal B

Recopilaciones de problemas olímpicos y preolímpicos (España):

OME , OMEFL , OMEC , OMEM , Canguro , Cangur

Recopilaciones de problemas olímpicos y preolímpicos (Internacional):

IMO , OMI , USAMO , AIME , AMC 8 , AMC 12 , SMT , Kangourou, Kangaroo , Mathcounts

Versión de este documento: 09/05/2021

Todos estos documentos se actualizan constantemente. ¡No utilices una versión anticuada! Descarga totalmente gratis la última

versión de los documentos en los enlaces superiores.

www.toomates.net

Page 3: COMPENDIUM OME - ToomatesEotvos de Hungría, que se iniciaron en 1894, precisamente durante la efervescencia de fin de siglo, consecuencia de la cual fue también el proceso iniciado

Presentación.

La Olimpiada Matemática Española (OME).

Es un concurso matemático para alumnos de enseñanza secundaria del sistema

educativo español organizado por la Real Sociedad Matemática Española. Se

celebra desde el año 1964. Esta competición selecciona los estudiantes que

forman el equipo español que participa en la Olimpiada Matemática

Internacional y en la Iberoamericana.

La competición se desarrolla en dos fases. La fase local, que se hace en cada

distrito universitario, y la fase nacional, que se celebra en una sede itinerante.

Los problemas de todas las fases no requieren conocimientos especiales de

Matemáticas, por el contrario se intenta que para resolverlos el alumno deba

utilizar capacidad de raciocinio y habilidad para enfrentarse a situaciones

nuevas.

La fase Local de la OME.

Podrán participar en la Fase Local de la OME los alumnos del sistema educativo

español o equivalente, matriculados en centros españoles o en centros

extranjeros homologados establecidos en España, que cursen Bachillerato en el

presente curso académico. Con carácter excepcional, y si son avalados por

escrito por su profesor, también podrán tomar parte alumnos ESO, de excelentes

capacidades, matriculados en alguno de los centros descritos en el párrafo

anterior. Suele celebrarse en el mes de enero y la participación es individual.

Consta de dos pruebas escritas en las que han de resolverse un total de seis

problemas. Solamente se permitirá la utilización de útiles de dibujo y escritura.

En particular, no está permitido usar calculadoras, aparatos electrónicos,

teléfonos móviles, libros, tablas u otros documentos distintos de los que

proporcione el Tribunal.

Tribunales de la Fase Local.

En cada Comunidad Autónoma, un tribunal designado por la Real Sociedad

Matemática Española calificará los ejercicios y propondrá a los ganadores de la

primera fase, en número de tres como máximo por cada Universidad pública

existente en la Comunidad Autónoma o por Distrito Universitario. El fallo del

Tribunal es inapelable.

Fase Nacional de la OME.

En cada Comunidad o Ciudad Autónoma tendrá lugar a continuación una

selección de concursantes, entre los ganadores de la primera fase en los Distritos

que corresponden a dicha Comunidad. Estos seleccionados serán los

representantes de la correspondiente Comunidad o Ciudad Autónoma en la Fase

Nacional de la OME, que suele celebrarse en el mes de marzo.

Consta de dos pruebas escritas de cuatro horas y media de duración cada una, en

el transcurso de las cuales los participantes deben enfrentarse a un total de seis

problemas propuestos por un tribunal.

En la Fase Nacional se premiará a los 36 primeros clasificados con medallas de

oro, plata y bronce, en la proporción 1:2:3, respectivamente.

Page 4: COMPENDIUM OME - ToomatesEotvos de Hungría, que se iniciaron en 1894, precisamente durante la efervescencia de fin de siglo, consecuencia de la cual fue también el proceso iniciado

Olimpiada Internacional de Matemáticas (IMO).

Los estudiantes españoles que hayan obtenido Medalla de Oro en la fase

nacional formarán parte del Equipo Olímpico de España que ostentará su

representación en la IMO.

Suele celebrase a mediados de Julio; consta de dos pruebas escritas de cuatro

horas y media de duración cada una, en el transcurso de las cuales, los

participantes deben enfrentarse a un total de seis problemas propuestos por un

tribunal.

Las primeras competiciones matemáticas nacionales fueron los concursos

Eotvos de Hungría, que se iniciaron en 1894, precisamente durante la

efervescencia de fin de siglo, consecuencia de la cual fue también el proceso

iniciado por el Barón de Coubertin que desembocó en las Olimpiadas de la

época moderna (Atenas 1896). A principios de nuestro siglo este tipo de

competiciones se extendió por todo el centro y el este de Europa. La forma

actual del concurso data de 1938 y fue establecida en las competiciones W. L.

Putnam, organizadas en Estados Unidos y Canadá. El nombre de Olimpiadas

data de 1958, año de celebración de las primeras Olimpiadas Matemáticas

Internacionales por iniciativa de Rumania.

Olimpiada Iberoamericana de Matemáticas.

La Comisión de Olimpiadas de la RSME decidirá la composición del equipo que

representará a España en la Olimpiada Iberoamericana de Matemáticas. Para ello

se atenderá a los siguientes criterios:

- Haber obtenido medalla de oro en la fase nacional de la OME.

- Resultados en la Olimpiada Internacional de Matemáticas.

- Aprovechamiento de las correspondientes sesiones de preparación.

Podrán resultar elegidos para formar parte del equipo español en la Olimpiada

Iberoamericana de Matemáticas, si se diera el caso, los estudiantes que,

habiendo obtenido Medalla de Oro en la edición anterior de la Olimpiada, y que

por haber completado sus estudios de Bachillerato no puedan participar en la

edición actual de la OME, cumplan sin embargo los requisitos de participación

en la Olimpiada Iberoamericana de Matemáticas. La Olimpiada Iberoamericana

se celebra en el mes de septiembre.

Olimpiada Femenina Europea (EGMO).

Las alumnas mejor clasificadas en las fases locales, hasta un máximo de 15,

podrán participar en la prueba de selección del equipo español que representará a

España en la Olimpiada Femenina Europea (EGMO)

Page 5: COMPENDIUM OME - ToomatesEotvos de Hungría, que se iniciaron en 1894, precisamente durante la efervescencia de fin de siglo, consecuencia de la cual fue también el proceso iniciado

Índice.

Enunciados Sugerencias Soluciones

1 - I - (1963-64) 8 90 372

2 - II - (1964-65) 10 98 382

3 - III - (1965-66) 12 106 391

4 - IV - (1966-67) 14 114 399

5 - V - (1967-68) 18 122 407

6 - VI - (1968-69) 20 130 416

7 - VII - (1969-70) 22 138 425

8 - VIII - (1970-71) 24 146 433

9 - IX - (1971-72) 26 154 442

10 - X - (1972-73) 28 162 450

11 - XI - (1973-74) 30 170 458

12 - XII - (1974-75) 32 178 466

13 - XIII - (1975-76) 34 186 474

14 - XIV - (1976-77) 36 194 482

15 - XV - (1978-79) 38 202 491

16 - XVI - (1979-80) 40 210 499

17 - XVII - (1980-81) 42 218 507

18 - XVIII - (1981-82) 44 226 515

19 - XIX - (1982-83) 46 234 524

20 - XX - (1983-84) 48 242 534

21 - XXI - (1984-85) 50 250 543

22 - XXII - (1985-86) 52 258 558

23 - XXIII - (1986-87) 54 264 564

24 - XXIV - (1987-88) 56 270 571

25 - XXV - (1988-89) 58 276 577

26 - XXVI - (1989-90) 60 282 583

27 - XXVII - (1990-91) 62 288 589

28 - XXVIII - (1991-92) 64 294 595

29 - XXIX - (1992-93) 66 300 603

30 - XXX - (1993-94) 68 306 610

31 - XXXI - (1994-95) 70 312 616

32 - XXXII - (1995-96) 72 318 623

33 - XXXIII - (1996-97) 74 324 630

34 - XXXIV - (1997-98) 76 330 636

35 - XXXV - (1998-99) 78 336 643

36 - XXXVI - (1999-00) 80 342 650

37 - XXXVII - (2000-01) 82 348 657

38 - XXXVIII - (2001-02) 84 354 664

39 - XXXIX - (2002-03) 86 360 670

40 - XL - (2003-04) 88 366 676

41 - XLI - (2004-05) 684 687

42 - XLII - (2005-06) 693 697

43 - XLIII - (2006-07) 701 705

44 - XLIV - (2007-08) 710 712

45 - XLV - (2008-09) 718 720

46 - XLVI - (2009-10) 733 735

47 - XLVII - (2010-11) 740 742

Page 6: COMPENDIUM OME - ToomatesEotvos de Hungría, que se iniciaron en 1894, precisamente durante la efervescencia de fin de siglo, consecuencia de la cual fue también el proceso iniciado

48 - XLVIII - (2011-12) 749 751

49 - XLIX - (2012-13) 756 758

50 - L - (2013-14) 765 767

51 - LI - (2014-15) 772 774

52 - LII - (2015-16) 780 782

53 - LIII - (2016-17) 787 789

54 - LIV - (2017-18) 797 799

55 - LV - (2018-19) 806 808

56 - LVI - (2019-20) 816 820

Dossier Actividades 2019 830 Presentación 832

Fase Local Enunciados 838

Fase Local Soluciones 840 Fase Nacional Enunciados 846

Fase Nacional Soluciones 847

Mathcontest 853 Mediterranean Mathematical Olympiad 857

Barcelona Contest 861

LX Olimpiada Internacional de Matemáticas 866 XXXIV Olimpiada Iberoamericana 875

57 - LVII - (2020-21) 882 883

Documentos de las primeras Olimpiadas 899

Page 7: COMPENDIUM OME - ToomatesEotvos de Hungría, que se iniciaron en 1894, precisamente durante la efervescencia de fin de siglo, consecuencia de la cual fue también el proceso iniciado

Fuentes.

De 1964 al 2004: El documento de la Real Sociedad Matemática Española http://www.olimpiadamatematica.es/platea.pntic.mec.es/_csanchez/olimp_1963-2004/OME2004.pdf

Del 2005 al 2019: Los 99 archivos que se encuentran en la página web http://www.olimpiadamatematica.es/platea.pntic.mec.es/_csanchez/olimprab.htm

También te puede interesar...

El Compendium OMEFL con todas las fases Locales de las olimpiadas

www.toomates.net/biblioteca/CompendiumOMEFL.pdf

El Compendium OMC con la fase local de Cataluña:

www.toomates.net/biblioteca/CompendiumOMC.pdf

Todo este material ha sido agrupado en un único archivo "pdf" mediante la aplicación

online

https://www.ilovepdf.com/

Page 8: COMPENDIUM OME - ToomatesEotvos de Hungría, que se iniciaron en 1894, precisamente durante la efervescencia de fin de siglo, consecuencia de la cual fue también el proceso iniciado

1963-64 I Olimpiada Matematica

Espanola1

Primera sesion

A1/1. Dada la ecuacion x2 + ax + 1 = 0, determinar

a) El intervalo en que debe mantenerse el numero real a para que las raıces de esa

ecuacion sean imaginarias.

b) El lugar geometrico de los puntos representativos de esas raıces en la representacion

grafica habitual de los numeros complejos, cuando a recorre el intervalo anterior.

A1/2. El impuesto sobre el Rendimiento del Trabajo Personal es una funcion f(x)

del total x de las retribuciones anuales (en pesetas). Sabiendo que

a) f(x) es una funcion continua.

b) La derivada df(x)/dx en el intervalo 0 ≤ x < 60000 es constante e igual a cero;

en el intervalo 60000 < x < P es constante e igual a 1; y para x > P es constante e

igual a 0.14.

c) f(0) = 0 y f(140000) = 14000.

Determinar el valor de la cantidad P de pesetas y representar graficamente la funcion

y = f(x) .

A1/3. Se considera un polıgono convexo de n lados. Se trazan todas sus rectas

diagonales y se supone que en ningun caso concurren tres de ellas en un punto que no

sea un vertice, y que tampoco hay diagonales que sean paralelas. En estas condiciones

se desea calcular:

a) El numero total de puntos de interseccion de estas diagonales, excluidos los vertices.

b) Cuantos de estos puntos son interiores al polıgono, y cuantos exteriores.

A1/4. Dados el triangulo equilatero ABC , de lado a , y su circunferencia circun-

scrita, se considera el segmento de cırculo limitado por la cuerda AB y el arco (de

120 ) con los mismos extremos. Al cortar este segmento circular con rectas paralelas al

lado BC , queda determinado sobre cada una de ellas un segmento de puntos interiores

al segmento circular mencionado. Determinar la longitud maxima de esos segmentos

rectilıneos.

Page 9: COMPENDIUM OME - ToomatesEotvos de Hungría, que se iniciaron en 1894, precisamente durante la efervescencia de fin de siglo, consecuencia de la cual fue también el proceso iniciado

Segunda sesion

A1/5. Dado un pentagono regular, se dibujan sus cinco segmentos diagonales. Se

pide determinar el numero total de triangulos que aparecen construidos en la figura y

clasificar este conjunto de triangulos en clases de triangulos iguales (directa o inversa-

mente) entre sı.

A1/6. Representar graficamente la funcion

y =∣∣∣∣∣|x − 1| − 2

∣∣ − 3∣∣∣

en el intervalo −8 ≤ x ≤ 8.

A1/7. Se considera un fichero con 1000 fichas numeradas, ordenadas en su orden

natural. A ese fichero se le aplica la siguiente operacion:

La primera ficha del fichero se coloca intercalada entre la penultima y la ultima del

mismo, y la segunda, al final de todas, quedando, por tanto, en primer lugar la que

antes ocupaba el tercero.

Observando la sucesion de posiciones ocupadas por cada una de las fichas, demostrar

que al cabo de 1000 operaciones analogas, aplicadas sucesivamente (cada una a la

ordenacion resultante de la operacion anterior), el fichero vuelve a estar en su orden

natural.

Comprobar que no podrıa obtenerse un resultado analogo (n operaciones para un

fichero de n fichas) si se tratase de un fichero con un numero impar n de fichas.

A1/8. En un plano vertical se consideran los puntos A y B situados sobre una

recta horizontal, y la semicircunferencia de extremos A, B situada en el semiplano

inferior. Un segmento de longitud a , igual al diametro de la semicircunferencia, se

mueve de manera que contiene siempre el punto A , y que uno de sus extremos recorre

la semicircunferencia dada. Determinar el valor del coseno del angulo que debe formar

ese segmento con la recta horizontal, para que su punto medio este lo mas bajo posible.

Premiados: Eugenio J. Miranda Palacios, Alberto de la Torre, Antoni Oliva Cuyas.

Page 10: COMPENDIUM OME - ToomatesEotvos de Hungría, que se iniciaron en 1894, precisamente durante la efervescencia de fin de siglo, consecuencia de la cual fue también el proceso iniciado

1964-65 II Olimpiada Matematica

Espanola2

Primera sesion

A2/1. Un triangulo equilatero inscrito en una circunferencia de centro O y radio

igual a 4 cm, se gira un angulo recto en torno a O . Hallar el area de la parte comun al

triangulo dado y al obtenido en ese giro.

A2/2. ¿Cuantos numeros de tres cifras (es decir, mayores que 99 y menores que

1000) hay que tengan su cifra central mayor que las otras dos? ¿Cuantos de ellos tienen

ademas las tres cifras distintas?

A2/3. Un disco microsurco gira a velocidad de 33 1

3 revoluciones por minuto y su

audicion dura 24 min 30 s. La parte grabada tiene 29 cm de diametro exterior y 11.5

cm de diametro interior. Con estos datos, calcular la longitud del surco grabado.

A2/4. Hallar todos los intervalos de valores de x para los cuales

cos x + sen x > 1;

el mismo problema para

cos x + |sen x| > 1.

Page 11: COMPENDIUM OME - ToomatesEotvos de Hungría, que se iniciaron en 1894, precisamente durante la efervescencia de fin de siglo, consecuencia de la cual fue también el proceso iniciado

Segunda sesion

A2/5. Es bien sabido que si p/q = r/s , ambas razones son iguales a (p−r)/(q−s) .

Escribimos ahora la igualdad

3x − b

3x − 5b=

3a − 4b

3a − 8b.

Por la propiedad anterior, ambas fracciones deben ser iguales a

3x − 5b − 3a + 8b

3x − b − 3a + 4b=

3x − 3a + 3b

3x − 3a + 3b= 1

mientras que las propuestas son de ordinario distintas de la unidad. Explicar con

claridad a que se debe este resultado.

A2/6. Se construye con alambre un triangulo equilatero de lado y se deposita

sobre una esfera maciza de radio r (que no pasa a traves del triangulo anterior). ¿A

que distancia del centro de la esfera quedan los vertices del triangulo?

A2/7. Un tronco de cono de revolucion tiene su base mayor de radio r y sus gen-

eratrices forman con el plano de la base un angulo cuya tangente vale m . Este tronco

de cono esta formado por un material de densidad d y su base menor esta recubierta

por una lamina cuya masa es de p g/cm2 . ¿Cual es la altura del tronco para la cual la

masa total es maxima? Discusion completa del problema.

A2/8. Sea γ1 una circunferencia de radio r y P un punto exterior que dista a de

su centro. Se suponen construidas las dos rectas tangentes a γ1 desde P , y sea γ2

una circunferencia de radio menor que el de γ1 , tangente a esas dos rectas y a γ1 ; en

general, una vez construida la circunferencia γn se construye otra γn+1 de radio menor

que el de γn , tangente a las dos rectas citadas y a γn . Determinar

a) El radio de γ2 .

b) La expresion general del radio de γn .

c) El lımite de la suma de las longitudes de las circunferencias γ1 , γ2 , . . . , γn , . . .

Premiados: Luis Puig Espinosa, Jaime Vinuesa Tejedor, Andres Mendez Rutllan.

Page 12: COMPENDIUM OME - ToomatesEotvos de Hungría, que se iniciaron en 1894, precisamente durante la efervescencia de fin de siglo, consecuencia de la cual fue también el proceso iniciado

1965-66 III Olimpiada Matematica

Espanola3

Primera sesion

A3/1. A un fabricante de tres productos cuyos precios por unidad son de 50, 70 y

65 pta, le pide un detallista 100 unidades, remitiendole en pago de las mismas 6850 pta,

con la condicion de que mande el mayor numero posible del producto de precio superior

y las restantes de los otros dos. ¿Cuantas debera enviar de cada producto para servir

el pedido?

A3/2. Un numero de tres cifras se escribe xyz en el sistema de base 7 y zyx en el

sistema de base 9. ¿Cual es el numero?

A3/3. Dado un pentagono regular se considera el pentagono convexo limitado por

sus diagonales. Se pide calcular:

a) La relacion de semejanza entre los dos pentagonos convexos.

b) La relacion de sus areas.

c) La razon de la homotecia que transforma el primero en el segundo.

A3/4. Se quiere colgar un peso P de modo que quede 7 m por debajo de un techo.

Para ello se suspende mediante un cable vertical sujeto al punto medio M de una

cadena colgada por sus extremos de dos puntos del techo A y B distantes entre sı 4

m. El precio del cable PM es p pta/m y el de la cadena AMB es q pta/m. Se pide:

a) Determinar las longitudes del cable y de la cadena para obtener el precio mas

economico de la instalacion.

b) Discutir la solucion para los distintos valores de la relacion p/q de ambos precios.

(Se supone que el peso es lo suficientemente grande para poder considerar como rec-

tilıneos los segmentos de cadena AM y MB ).

Page 13: COMPENDIUM OME - ToomatesEotvos de Hungría, que se iniciaron en 1894, precisamente durante la efervescencia de fin de siglo, consecuencia de la cual fue también el proceso iniciado

Segunda sesion

A3/5. La longitud de la hipotenusa BC de un triangulo rectangulo ABC es a ,

y sobre ella se toman los puntos M y N tales que BM = NC = k , con k < a/2.

Supuesto que se conocen (tan solo) los datos a y k , calcular:

a) El valor de la suma de los cuadrados de las longitudes AM y AN .

b) La razon de las areas de los triangulos ABC y AMN .

c) El area encerrada por la circunferencia que pasa por los puntos A , M ′ , N ′ , siendo

M ′ la proyeccion ortogonal de M sobre AC y N ′ la de N sobre AB .

A3/6. Nos indican que un matrimonio tiene 5 hijos. Calcular la probabilidad de que

entre ellos haya por lo menos dos varones y por lo menos una mujer. La probabilidad

de nacer varon se considera 1/2.

A3/7. Determinar una progresion geometrica de siete terminos, conociendo la suma,

7, de los tres primeros, y la suma, 112, de los tres ultimos.

A3/8. Determinar los valores de a , b , c , para que la representacion grafica de la

funcion

y = ax3 + bx2 + cx

tenga una inflexion en el punto de abscisa x = 3, con tangente en el de ecuacion

x − 4y + 1 = 0.

Dibujese despues la grafica correspondiente.

Premiados: Jose L. Rubio de Francia, Manuel Gamella Bacete, Antonio Vazquez

Rodrıguez.

Page 14: COMPENDIUM OME - ToomatesEotvos de Hungría, que se iniciaron en 1894, precisamente durante la efervescencia de fin de siglo, consecuencia de la cual fue también el proceso iniciado

1966-67 IV Olimpiada Matematica

Espanola4

Primera sesion

A4/1. Se sabe que la funcion real f(t) es monotona creciente en el intervalo −8 ≤

t ≤ 8, pero no se sabe nada de lo que ocurre fuera de este. ¿En que intervalo de valores

de x se puede asegurar que sea monotona creciente la funcion y = f(2x − x2)?

A4/2. Determinar los polos de las inversiones que transforman cuatro puntos A ,

B , C , D , alineados en este orden, en cuatro puntos A′ , B′ , C′ , D′ que sean vertices

de un paralelogramo rectangulo, y tales que A′ y C′ sean vertices opuestos.

A4/3. Un semaforo instalado en un cruce principal de una vıa, en la que se circula

en ambos sentidos, permanece en rojo 30 s y en verde otros 30 s, alternativamente. Se

desea instalar otro semaforo en la misma vıa, para un cruce secundario, situado a 400

m de distancia del primero, que funcione con el mismo perıodo de 1 min de duracion.

Se quiere que los coches que circulan a 60 Km/h por la vıa en cualquiera de los dos

sentidos y que no se tienen que parar si solo hubiese el semaforo del cruce principal,

tampoco se tengan que parar despues de instalar el del cruce secundario. ¿Cuantos

segundos puede estar encendido el rojo en el semaforo secundario?

Nota: Se sugiere razonar sobre una representacion cartesiana de la marcha de los coches,

tomando un eje de distancias y otro de tiempos.

A4/4. Se tiene un botella de fondo plano y circular, cerrada y llena parcialmente

de vino, de modo que su nivel no supere la parte cilındrica. Discutir en que casos

se puede calcular la capacidad de la botella sin abrirla, disponiendo solamente de un

doble decımetro graduado; y en caso de que sea posible, describir como se calcularıa.

(Problema de la Gara Matematica italiana).

Page 15: COMPENDIUM OME - ToomatesEotvos de Hungría, que se iniciaron en 1894, precisamente durante la efervescencia de fin de siglo, consecuencia de la cual fue también el proceso iniciado

Segunda sesion

A4/5. Sea γ una semicircunferencia de diametro AB . Se construye una quebrada

con origen en A , que tiene sus vertices alternativamente en el diametro AB y en la

semicircunferencia γ , de modo que sus lados forman angulos iguales α con el diametro

(pero alternativamente de uno y otro sentido). Se pide:

a) Valores del angulo α para que la quebrada pase por el otro extremo B del diametro.

b) La longitud total de la quebrada, en el caso que termine en B , en funcion de la

longitud d del diametro y del angulo α .

A Bα

Problema 4/5

OA

B

C

Problema 4/6

A4/6. Se da un triangulo equilatero ABC de centro O y radio OA = R , y se

consideran las siete regiones que las rectas de los lados determinan sobre el plano. Se

pide dibujar y describir la region del plano tansformada de las dos regiones sombreadas

en la figura adjunta, por la inversion de centro O y potencia R2 .

A4/7. Por una carretera circula una caravana de coches, todos a la misma veloci-

dad, manteniendo la separacion mınima entre uno y otro senalada por el Codigo de

Circulacion. Esta separacion es, en metros,

v2

100,

donde v es la velocidad expresada en Km/h. Suponiendo que la longitud de cada coche

es de 2.89 m, calcular la velocidad a la que deben circular para que la capacidad de

trafico resulte maxima, es decir, para que en un tiempo fijado pasen el maximo numero

de vehıculos por un punto de la carretera.

Page 16: COMPENDIUM OME - ToomatesEotvos de Hungría, que se iniciaron en 1894, precisamente durante la efervescencia de fin de siglo, consecuencia de la cual fue también el proceso iniciado

A4/8. Para obtener el valor de un polinomio de grado n , cuyos coeficientes son

a0, a1, . . . , an

(comenzando por el termino de grado mas alto), cuando a la variable x se le da el valor

b , se puede aplicar el proceso indicado en el organigrama adjunto, que desarrolla las

acciones requeridas para aplicar la regla de Ruffini. Se pide construir otro organigrama

analogo que permita expresar el calculo del valor de la derivada del polinomio dado,

tambien para x = b .

Hacer A igual a a0

Hacer i igual a 1

Hacer P igual a Ab

Hacer

A igual a P +ai

¿Es i = n?

si

noIncrementar i

en 1 unidad

El valor es A

Premiados: Bernardo Lopez Melero, Arturo Fraile Perez, Julio Falivene Raboso.

Page 17: COMPENDIUM OME - ToomatesEotvos de Hungría, que se iniciaron en 1894, precisamente durante la efervescencia de fin de siglo, consecuencia de la cual fue también el proceso iniciado
Page 18: COMPENDIUM OME - ToomatesEotvos de Hungría, que se iniciaron en 1894, precisamente durante la efervescencia de fin de siglo, consecuencia de la cual fue también el proceso iniciado

1967-68 V Olimpiada Matematica

Espanola5

Primera sesion

A5/1. En una noche la temperatura del aire se mantuvo constante, varios grados

bajo cero, y la del agua de un estanque cilındrico muy extenso, que formaba una capa

de 10 cm de profundidad, llego a ser de cero grados, comenzando entonces a formarse

una capa de hielo en la superficie. En estas condiciones puede admitirse que el espesor

de la capa de hielo formada es directamente proporcional a la raız cuadrada del tiempo

transcurrido. A las 0 h, el espesor del hielo era de 3 cm y a las 4 h justamente se acabo

de helar el agua del estanque. Calcular a que hora comenzo a formarse la capa de hielo,

sabiendo que la densidad del hielo formado era de 0.9.

A5/2. Razonar si puede afirmarse, negarse o no puede decidirse la continuidad en

el punto x = 0 de una funcion real f(x) de variable real, en cada uno de los tres casos

(independientes).

a) Se sabe unicamente que para todo n natural

f

(12n

)= 1 y f

(1

2n + 1

)= −1.

b) Se sabe que para todo x real no negativo es f(x) = x2 y para x real negativo es

f(x) = 0.

c) Se sabe unicamente que para todo n natural es

f

(1n

)= 1.

A5/3. Dado un cuadrado cuyo lado mide a , se considera el conjunto de todos los

puntos de su plano por los que pasa una circunferencia de radio a cuyo cırculo contenga

al cuadrado citado. Se pide probar que el contorno de la figura formada por los puntos

con esa propiedad esta formado por arcos de circunferencia, y determinar las posiciones

de sus centros, sus radios y sus longitudes.

A5/4. En los dos extremos A , B de un diametro (de longitud 2r ) de un pavimento

circular horizontal se levantan sendas columnas verticales, de igual altura h , cuyos

extremos soportan una viga A′B′ de longitud igual al diametro citado. Se forma una

cubierta colocando numerosos cables tensos (que se admite que quedan rectilıneos),

uniendo puntos de la viga A′B′ con puntos de la circunferencia borde del pavimento,

de manera que los cables queden perpendiculares a la viga A′B′ . Se desea averiguar el

volumen encerrado entre la cubierta y el pavimento.

Page 19: COMPENDIUM OME - ToomatesEotvos de Hungría, que se iniciaron en 1894, precisamente durante la efervescencia de fin de siglo, consecuencia de la cual fue también el proceso iniciado

Segunda sesion

A5/5. Hallar el lugar geometrico del centro de un rectangulo, cuyos cuatro vertices

describen el contorno de un triangulo dado.

A5/6. Razonar si en todo tetraedro son concurrentes:

a) Las perpendiculares a las caras en sus circuncentros.

b) Las perpendiculares a las caras en sus ortocentros.

c) Las perpendiculares a las caras en sus incentros.

En caso afirmativo, caracterizar con alguna propiedad geometrica sencilla el punto en

que concurren. En caso negativo mostrar un ejemplo en el que se aprecie claramente la

no concurrencia.

A5/7. En la succesion de potencias de 2 (escritas en el sistema decimal, comenzando

con 21 = 2) hay tres terminos de una cifra, otros tres de dos cifras, otros tres de 3,

cuatro de 4, tres de 5, etc. Razonar claramente las respuestas a las cuestiones siguientes:

a) ¿Puede haber solamente dos terminos con un cierto numero de cifras?

b) ¿Puede haber cinco terminos consecutivos con el mismo numero de cifras?

c) ¿Puede haber cuatro terminos de n cifras, seguidos de cuatro con n + 1 cifras?

d) ¿Cual es el numero maximo de potencias consecutivas de 2 que pueden encontrarse

sin que entre ellas haya cuatro con el mismo numero de cifras?

A5/8. Supondremos que los lados de un cuadrado son reflectantes y los designare-

mos con los nombres de los cuatro puntos cardinales. Senalando un punto en el lado N,

determinar en que direccion debe salir un rayo de luz (hacia el interior del cuadrado)

para que retorne a el despues de haber sufrido n reflexiones en el lado E, otras n en

el lado W, m en el S y m − 1 en el N, siendo n y m numeros naturales conocidos.

¿Que ocurre si m y n no son primos entre sı? Calcular la longitud del rayo luminoso

considerado en funcion de m y n , y de la longitud del lado del cuadrado.

Premiados: Francisco J. Vives Arumı, Roberto Moriyon Salomon, Carlos A. Lucio

Fernandez.

Page 20: COMPENDIUM OME - ToomatesEotvos de Hungría, que se iniciaron en 1894, precisamente durante la efervescencia de fin de siglo, consecuencia de la cual fue también el proceso iniciado

1968-69 VI Olimpiada Matematica

Espanola6

Primera sesion

A6/1. Hallar el lugar geometrico de los centros de las inversiones que transforman

dos punto A , B de una circunferencia dada γ , en puntos diametralmente opuestos de

las circunferencias inversas de γ .

A6/2. Hallar el lugar geometrico del afijo M , del numero complejo z , para que este

alineado con los afijos de i y de iz .

A6/3. Una bolsa contiene cubos de plastico del mismo tamano, cuyas caras han

sido pintadas de colores: blanco, rojo, amarillo, verde, azul y violeta (sin repetir un

color en dos caras del mismo cubo). ¿Cuantos de estos cubos puede haber distinguibles

entre sı?

A6/4. Se divide una circunferencia de radio R en 8 partes iguales. Los puntos de

division se designan sucesivamente por A , B , C , D , E , F , G y H . Hallar el area

del cuadrado formado al dibujar las cuerdas AF , BE , CH y DG .

Segunda sesion

A6/5. Demostrar que un polıgono convexo de mas de cuatro lados no puede ser

descompuesto en otros dos, ambos semejantes al primero (directa o inversamente), por

medio de un solo corte rectilıneo. Precisar razonadamente cuales son los cuadrilateros

y triangulos que admiten una descomposicion de este tipo.

Page 21: COMPENDIUM OME - ToomatesEotvos de Hungría, que se iniciaron en 1894, precisamente durante la efervescencia de fin de siglo, consecuencia de la cual fue también el proceso iniciado

A6/6. Dado un polinomio de coeficientes reales P (x) , ¿se puede afirmar que para

todo valor real de x es cierta alguna de las desigualdades siguientes:

P (x) ≤ P (x)2; P (x) < 1 + P (x)2; P (x) ≤ 12

+12P (x)2.

Encontrar un procedimiento general sencillo (entre los muchos existentes) que permita,

siempre que nos den dos polinomis P (x) y Q(x) , encontrar otro M(x) tal que para

todo valor de x , sea a la vez

−M(x) < P (x) < M(x) y − M(x) < Q(x) < M(x).

A6/7. Un polıgono convexo A1A2 . . .An de n lados e inscrito en una circunferencia,

tiene sus lados que satisfacen las desigualdades

AnA1 > A1A2 > A2A3 > · · · > An−1An.

Demostrar que sus angulos interiores satisfacen las desigualdades

A1 < A2 < A3 < · · · < An−1, An−1 > An > A1.

A6/8. La casa SEAT recomienda a los usuarios, para la correcta conservacion de

las ruedas, substituciones periodicas de las mismas en la forma R →3 →2 →1 →4 →R, segun la numeracion de la figura. Llamando G a este cambio de ruedas, G2 = G Ga la realizacion de este cambio dos veces, y ası sucesivamente para las demas potencias

de G ,

a) Demostrar que el conjunto de esas potencias forma un grupo, y estudiarlo.

b) Cada pinchazo de una de las ruedas equivale tambien a una sustitucion en la que

dicha rueda es reemplazada por la de repuesto (R) y, una vez reparada, pasa a ocupar

el lugar de esta. Obtener G como producto de transformaciones pinchazo. ¿Forman

estas un grupo?

1

2

3

4

R

Premiados: Jaume Lluıs Garcıa Roig, Dolores Carrillo Gallego, Jorge Bustos Puche.

Page 22: COMPENDIUM OME - ToomatesEotvos de Hungría, que se iniciaron en 1894, precisamente durante la efervescencia de fin de siglo, consecuencia de la cual fue también el proceso iniciado

1969-70 VII Olimpiada Matematica

Espanola7

Primera sesion

A7/1. Un recipiente cilındrico de revolucion esta parcialmente lleno de un lıquido

cuya densidad ignoramos. Situandolo con el eje inclinado 30 respecto de la vertical, se

observa que al sacar lıquido de modo que el nivel descienda 1 cm, el peso del contenido

disminuye 40 g. ¿Cuanto disminuira el peso de ese contenido por cada centımetro que

descienda el nivel si el eje forma un angulo de 45 con la vertical? Se supone que la

superficie horizontal del lıquido no llega a tocar ninguna de les bases del recipiente.

A7/2. Una planta crece del modo que describimos a continuacion. Tiene un tronco

que se bifurca en dos ramas; cada rama de la planta puede, a su vez, bifurcarse en otras

dos ramas, o bien acabar en una yema. Llamaremos carga de una rama al numero total

de yemas que soporta, es decir, el numero de yemas alimentadas por la savia que pasa

por esa rama;y llamaremos alejamiento de una yema al numero de bifurcaciones que la

savia tiene que atravesar para llegar desde el tronco a esa yema.

Si n es el numero de bifurcaciones que tiene una determinada planta de ese tipo, se

pide

a) el numero de ramas de la planta,

b) el numero de yemas,

c) demostrar que la suma de las cargas de todas las ramas es igual a la suma de los

alejamientos de todas las yemas.

Sugerencia: Puede procederse por induccion, demostrando que si unos resultados son

correctos para una determinada planta, siguen siendolo para la planta que se obtiene

sustituyendo en ella una yema por un par de ramas terminadas en sendas yemas.

A7/3. Se da un triangulo arbitrario ABC y un punto P situado en el lado AB .

Se pide trazar por P una recta que divida al triangulo en dos figuras de la misma area.

A7/4. Sabiendo que los polinomios

2x5 − 13x4 + 4x3 + 61x2 + 20x−25

x5 − 4x4 − 13x3 + 28x2 + 85x+50

tienen dos raıces dobles comunes, determinar todas sus raıces.

Page 23: COMPENDIUM OME - ToomatesEotvos de Hungría, que se iniciaron en 1894, precisamente durante la efervescencia de fin de siglo, consecuencia de la cual fue también el proceso iniciado

Segunda sesion

A7/5. En los examenes de sexto curso de un Centro, aprueban la Fısica, al menos,

el 70% de los alumnos; las Matematicas, al menos, el 75%; la Filosofıa, al menos, el

90%; y el Idioma, al menos, el 85%. ¿Cuantos alumnos, al menos, aprueban esas cuatro

asignaturas?

A7/6. Dada una circunferencia γ y dos punts A y B en su plano, se traza por B

una secante variable que corta γ en dos puntos M y N . Determinar el lugar geometrico

de los centros de les circunferencias circunscritas al triangulo AMN .

A7/7. Calcular los valores de los cosenos de los angulos x que satisfacen la ecuacion

siguiente:

sen2 x − 2 cos2 x +12sen 2x = 0.

A7/8. Se da un punto M en el interior de una circunferencia, a una distancia

OM = d del centro O . Por M se trazan dos cuerdas AB y CD que forman angulo

recto. Se une A con C y B con D . Determinar el coseno del angulo que ha de formar

la cuerda AB con OM para que la suma de les areas de los triangulos AMC y BMD

sea mınima.

Premiados: Enrique Rodrıguez Bono, Francisco J. Corella Monzon, Ignacio Alegre de

Miguel.

Page 24: COMPENDIUM OME - ToomatesEotvos de Hungría, que se iniciaron en 1894, precisamente durante la efervescencia de fin de siglo, consecuencia de la cual fue también el proceso iniciado

1970-71 VIII Olimpiada Matematica

Espanola8

Primera sesion

A8/1. Calcular

k=49∑k=5

11(k

23√

1331(k

sabiendo que los numeros 11 y 1331 estan escritos en base k ≥ 4.

A8/2. En una cierta geometrıa operamos con dos tipos de elementos, puntos y

rectas, relacionados entre sı por los axiomas siguientes:

I. Dados dos puntos A y B , existe una unica recta (AB) que pasa por ambos.

II. Sobre una recta existen al menos dos puntos. Existen tres puntos no situados sobre

una recta.

III. Cuando un punto B esta situado entre A y C , entonces B esta tambien entre

C y A . (A , B , C son tres puntos diferentes de una recta.)

IV. Dados dos puntos A y C existe al menos un punto B en la recta (AC) de forma

que C esta entre A y B .

V. De entre tres puntos situados sobre una misma recta, uno como maximo, esta entre

los otros dos.

VI. Si A , B , C son tres puntos no situados sobre la misma recta y a es una recta que

no contiene ninguno de los tres, cuando la recta pasa por un punto del segmento [AB] ,

entonces pasa por uno del [BC] , o pasa por uno del [AC] . (Designamos por [AB] al

conjunto de puntos que estan entre A y B .)

A partir de los axiomas anteriores, demostrar las proposiciones siguientes:

Teorema 1. Entre los puntos A y C existe al menos un punto B .

Teorema 2. De entre tres puntos situados sobre una recta, uno esta siempre entre los

otros dos.

A8/3. Si 0 < p , 0 < q y p + q < 1 demostrar

(px + qy)2 ≤ px2 + qy2.

A8/4. Demostrar que en todo triangulo de lados a , b , c y angulos opuestos A , B ,

C , se cumple (midiendo los angulos en radianes)

aA + bB + cC

a + b + c≥ π

3.

Indicacion: Utilizar a ≥ b ≥ c =⇒ A ≥ B ≥ C .

Page 25: COMPENDIUM OME - ToomatesEotvos de Hungría, que se iniciaron en 1894, precisamente durante la efervescencia de fin de siglo, consecuencia de la cual fue también el proceso iniciado

Segunda sesion

A8/5. Demostrar que cualquiera que sea el numero complejo z , se cumple

(1 + z2n)(

1 − z2n)= 1 − z2n+1

.

Escribiendo las igualdades que resultan al dar a n los valores 0, 1, 2, . . . y multi-

plicandolas, demostrar que para |z| < 1 se cumple

11 − z

= limk→∞

(1 + z

)(1 + z2

)(1 + z22) · · · (1 + z2k)

.

A8/6. Las velocidades de un submarino sumergido y en superficie son, respectiva-

mente, v y kv . Esta situado en un punto P a 30 millas del centro O de un cırculo de

radio 60 millas. La vigilancia de una escuadra enemiga le obliga a navegar sumergido

mientras esta dentro del cırculo. Discutir, segun los valores de k , el camino mas rapido

para trasladarse al extremo opuesto del diametro que pasa por P . (Considerar el caso

particular k =√

5.)

A8/7. Transformar por inversion dos circunferencies concentricas y coplanarias en

dos iguales.

A8/8. De entre los 2n numeros 1, 2, 3, . . . , 2n se eligen de cualquier forma n + 1

numeros distintos. Demostrar que entre los numeros elegidos hay por lo menos dos,

tales que uno divide al otro.

Premiados: M. Isabel Corella Monzon, Vicente Frances Tortosa, Jose M. Gil Martınez.

Page 26: COMPENDIUM OME - ToomatesEotvos de Hungría, que se iniciaron en 1894, precisamente durante la efervescencia de fin de siglo, consecuencia de la cual fue también el proceso iniciado

1971-72 IX Olimpiada Matematica

Espanola9

Primera sesion

A9/1. Sea K un anillo con unidad y M el conjunto de las matrices 2×2 constituidas

con elementos de K . Se define en M una adicion y una multiplicacion de la forma usual

entre matrices. Se pide:

a) Comprobar que M es un anillo con unidad y no conmutativo respecto de las leyes

de composicion definidas.

b) Comprobar que si K es un cuerpo conmutativo, los elements de M que tienen inverso

estan caracterizados por la condicion ad − bc = 0.

c) Demostrar que el subconjunto de M formado por los elementos que tienen inverso

es un grupo multiplicativo.

A9/2. Un punto se mueve sobre los lados del triangulo ABC , definido por los

vertices A(−1.8, 0), B(3.2, 0), C(0, 2.4). Determinar las posiciones de dicho punto, en

las que la suma de su distancia a los tres vertices es maxima o mınima absoluta.

A(−1.8 , 0) B(3.2 , 0)

C(0 , 2.4)

A9/3. Sea un prisma hexagonal regular. ¿Cual es la poligonal que, partiendo de

un vertice de la base, recorre todas las caras laterales y acaba en el vertice de la cara

superior, situado en la misma arista que el vertice de partida, y tiene longitud mınima.

A9/4. Se consideran en el plano los siguientes conjuntos de puntos:

A =afijos de los complejos z tales que arg

(z − (2 + 3i)

)= π/4

,

B =afijos de los complejos z tales que mod

(z − (2 + i)

)< 2

.

Determinar la proyeccion ortogonal sobre el eje X de A ∩ B .

Page 27: COMPENDIUM OME - ToomatesEotvos de Hungría, que se iniciaron en 1894, precisamente durante la efervescencia de fin de siglo, consecuencia de la cual fue también el proceso iniciado

Segunda sesion

A9/5. Dadas dos rectas paralelas r y r′ y un punto P sobre el plano que las contine

y que no esta sobre ellas, determinar un triangulo equilatero que tenga por vertice el

punto P , y los otros dos, uno sobre cada una de las dos rectas.

P

r

r′

A9/6. Dadas tres circunferencias de radios r , r′ y r′′ , cada una tangente exterior-

mente a las otras dos, calcular el radio del cırculo inscrito al triangulo cuyos vertices

son los tres centros de aquellas.

A9/7. Demostrar que para todo entero positivo n , el numero

An = 5n + 2 · 3n−1 + 1

es multiplo de 8.

A9/8. Sabemos que R3 = (x1, x2, x3) | xi ∈ R, i = 1, 2, 3 es un espacio vectorial

respecto de las leyes de composicion

(x1, x2, x3) + (y1, y2, y3) = (x1 + y1, x2 + y2, x3 + y3),

λ(x1, x2, x3) = (λx1, λx2, λx3), λ ∈ R.

Consideramos el siguiente subconjunto de R3 :

L =(x1, x2, x3) ∈ R3 | x1 + x2 + x3 = 0

.

a) Demostrar que L es un subespacio vectorial de R3 .

b) En R3 se define la relacion siguiente

xRy ⇐⇒ x − y ∈ L; x, y ∈ R3.

Demostrar que se trata de una relacion de equivalencia.

c) Hallar dos vectores de R3 que pertenezcan a la misma clase que el vector (−1, 3, 2).

Premiados: Josep Gelonch Anye, Jose I. Querol Bravo, Jose Bonet Solves.

Page 28: COMPENDIUM OME - ToomatesEotvos de Hungría, que se iniciaron en 1894, precisamente durante la efervescencia de fin de siglo, consecuencia de la cual fue también el proceso iniciado

1972-73 X Olimpiada Matematica

Espanola10

Primera sesion

A10/1. Dada la sucesion (an), en la que

an =14n4 − 10n2(n − 1), con n = 0, 1, 2 . . .

determinar el termino menor de la sucesion.

A10/2. Determinar todas las soluciones del sistema

2x − 5y + 11z − 6 = 0

−x + 3y − 16z + 8 = 0

4x − 5y − 83z + 38 = 0

3x + 11y − z + 9 > 0

en el que las tres primeras son ecuaciones y la ultima una inecuacion lineal.

A10/3. Se considera en el plano complejo la sucesion (an) de numeros complejos,

en la que es:

a0 = 1, y an = an−1 +1n

(cos 45 + i sen45

)n.

Probar que la sucesion de las partes reales de los terminos de (an) es convergente y su

lımite es un numero comprendido entre 0.85 y 1.15.

A10/4. Sean C y C ′ dos circunferencias concentricas de radios r y r′ respectiva-

mente. Determinar cuanto ha de valer el cociente r′/r para que en la corona limitada

por C y C ′ existan ocho circunferencias Ci , i = 1, . . . , 8, que sean tangentes a C y a

C ′ , y tambien que Ci sea tangente a Ci+1 para i = 1, . . . , 7 y C8 tangente a C1 .

Segunda sesion

A10/5. Se considera el conjunto de todos los polinomios de grado menor o igual

que 4 con coeficientes racionales.

a) Probar que tiene estructura de espacio vectorial sobre el cuerpo de los numeros

racionales.

Page 29: COMPENDIUM OME - ToomatesEotvos de Hungría, que se iniciaron en 1894, precisamente durante la efervescencia de fin de siglo, consecuencia de la cual fue también el proceso iniciado

b) Probar que los polinomios 1, x− 2, (x − 2)2 , (x − 2)3 y (x − 2)4 forman una base

de este espacio.

c) Expresar el polinomio 7 + 2x − 45x2 + 3x4 en la base anterior.

A10/6. Se considera un triangulo equilatero de altura 1. Para todo punto P del

interior del triangulo, se designan por x , y , z las distancias del punto P a los lados

del triangulo.

a) Probar que para todo punto P interior del triangulo se cumple que x + y + z = 1.

b) ¿Para que puntos del triangulo se cumple que la distancia a un lado es mayor que

la suma de las distancias a los otros dos?

c) Tenemos una barra de longitud 1 y la rompemos en tres trozos. Hallar la probabilidad

de que con estos trozos se pueda formar un triangulo.

A10/7. En el plano se consideran los dos puntos P (8, 2) y Q(5, 11). Un movil se

desplaza de P a Q segun un camino que ha de cumplir las condiciones seguientes: El

movil parte de P y llega a un punto del eje x , a lo largo del cual recorre un segmento

de longitud 1; despues se separa de este eje y se dirige hacia un punto del eje y , sobre

el cual recorre un segmento de longitud 2; se separa del eje y finalmente y va hacia

el punto Q . Entre todos los caminos posibles, determinar el de longitud mınima, ası

como esta misma longitud.

A10/8. En un espacio euclidiano de tres dimensiones se designan por u1 , u2 , u3

los tres vectores unitarios ortogonales sobre los ejes x, y, z , respectivamente.

a) Probar que el punto P (t) = (1− t)u1 +(2− 3t)u2 +(2t− 1)u3 , donde t toma todos

los valores reales, describe una recta (que designaremos por L).

b) ¿Que describe el punto Q(t) = (1− t2)u1 +(2−3t2)u2 +(2t2 −1)u3 si t toma todos

los valores reales?

c) Hallar un vector paralelo a L .

d) ¿Para que valores de t esta el punto P (t) sobre el plano 2x + 3y + 2z + 1 = 0?

e) Hallar la ecuacion cartesiana del plano paralelo al anterior y que contenga el punto

P (3).

f) Hallar la ecuacion cartesiana del plano perpendicular a L que contenga el punto

P (2).

Premiados: Antonio Garcıa Fernandez, Miguel Castano Gracia, Enrique Frau Pico.

Page 30: COMPENDIUM OME - ToomatesEotvos de Hungría, que se iniciaron en 1894, precisamente durante la efervescencia de fin de siglo, consecuencia de la cual fue también el proceso iniciado

1973-74 XI Olimpiada Matematica

Espanola11

Primera sesion

A11/1. Se sabe que un dodecaedro regular es un poliedro regular con 12 caras

pentagonales iguales y concurriendo 3 aristas en cada vertice. Se pide calcular, razon-

adamente,

a) el numero de vertices,

b) el numero de aristas,

c) el numero de diagonales de todas las caras,

d) el numero de segmentos rectilıneos determinados por cada dos vertices,

d) el numero de diagonales del dodecaedro.

A11/2. En un disco metalico se quita un sector circular, de modo que con la parte

restante se pueda formar un vaso conico de volumen maximo. Calcular, en radianes, el

angulo del sector que se quita.

A11/3. Designaremos por Z(5) un cierto subconjunto del conjunto Q de los numeross

racionales. Un racional pertenece a Z(5) si y solo si existen fracciones pertenecientes a

este racional tales que 5 no sea divisor de su denominador. (Por ejemplo, el numero

racional 13/10 no pertenece a Z(5) , ya que el denominador de todas las fracciones

iguales a 13/10 es un multiplo de 5. En cambio, el racional 75/10 pertenece a Z(5) ya

que 75/10 = 15/12).

Contestar razonadamente las siguientes cuestiones:

a) ¿Que estructura algebraica (semigrupo, grupo, etc.) tiene Z(5) respecto de la suma?

b) ¿Y respecto del producto?

c) ¿Es Z(5)un subanillo de Q?

d) ¿Es Z(5) un Z(5) -espacio vectorial?

A11/4. Los tres lados de un triangulo equilatero se suponen reflectantes (excepto en

los vertices), de forma que reflejen hacia dentro del triangulo los rayos de luz situados

en su plano, que incidan sobre ellos y que salgan de un punto interior del triangulo.

Determinar el recorrido de un rayo de luz que, partiendo de un vertice del triangulo

alcance a otro vertice del mismo despues de reflejarse sucesivamente en los tres lados.

Calcular la longitud del camino seguido por la luz suponiendo que el lado del triangulo

mide 1 m.

Page 31: COMPENDIUM OME - ToomatesEotvos de Hungría, que se iniciaron en 1894, precisamente durante la efervescencia de fin de siglo, consecuencia de la cual fue también el proceso iniciado

Segunda sesion

A11/5. Sea (G, ·) un grupo y e un elemento neutro. Probar que si todos los

elementos x de G cumplen

x · x = e

entonces (G, ·) es abeliano (o sea, conmutativo).

A11/6. En una circunferencia de radio igual a la unidad se trazan dos cuerdas, AB

y AC de igual longitud.

a) Averiguar como se puede construir una tercera cuerda DE que quede dividida en

tres partes iguales por las intersecciones con AB y AC .

b) Si AB = AC =√

2, ¿cuanto valen las longitudes de los dos segmentos que la cuerda

DE determina sobre AB ?

A11/7. Un deposito tiene forma de prisma exagonal regular, cuyas bases son de 1

m de lado y su altura es de 10 m. Se situan las aristas laterales en posicion oblicua y

se llena parcialmente con 9 m3 de agua. El plano de la superficie libre del agua corta

a todas las aristas laterales. Una de ellas queda con una parte de 2 m bajo el agua.

¿Que parte queda bajo el agua en la arista lateral opuesta del prisma?

A11/8. Los lados de un polıgono regular convexo de L + M + N lados se han de

dibujar en tres colores: L de ellos con trazo rojo, M con trazo amarillo, y N con trazo

azul. Expresar, por medio de desigualdades, las condiciones necesarias y suficientes

para que tenga solucion (varias, en general) el problema de hacerlo sin que queden dos

lados contiguos dibujados con el mismo color.

Premiados: Juan M. Sueiro Bal, Jesus Alcazar Moreno, Luis Narvaez Macarro.

Page 32: COMPENDIUM OME - ToomatesEotvos de Hungría, que se iniciaron en 1894, precisamente durante la efervescencia de fin de siglo, consecuencia de la cual fue también el proceso iniciado

1974-75 XII Olimpiada Matematica

Espanola12

Primera sesion

A12/1. Calcular el lımite

limn→∞

1n

(1nk

+2k

nk+ · · · + (n − 1)k

nk+

nk

nk

).

(Para el calculo del lımite se puede seguir el procedimento de construccion de la inte-

gral).

A12/2. Estudiar la funcion real

f(x) =(

1 +1x

)x

definida para x ∈ R − [−1, 0] . Representacion grafica.

A12/3. Designaremos por Z(5) un cierto subconjunto del conjunto Q de los numeross

racionales. Un racional pertenece a Z(5) si y solo si existen fracciones pertenecientes a

este racional tales que 5 no sea divisor de su denominador. (Por ejemplo, el numero

racional 13/10 no pertenece a Z(5) , ya que el denominador de todas las fracciones

iguales a 13/10 es un multiplo de 5. En cambio, el racional 75/10 pertenece a Z(5) ya

que 75/10 = 15/12).

Contestar razonadamente las siguientes cuestiones:

a) ¿Que estructura algebraica (semigrupo, grupo, etc.) tiene Z(5) respecto de la suma?

b) ¿Y respecto del producto?

c) ¿Es Z(5)un subanillo de Q?

d) ¿Es Z(5) un Z(5) -espacio vectorial?

A12/4. Probar que si el producto de n numeros reales y positivos es igual a 1, su

suma es mayor o igual que n .

Page 33: COMPENDIUM OME - ToomatesEotvos de Hungría, que se iniciaron en 1894, precisamente durante la efervescencia de fin de siglo, consecuencia de la cual fue también el proceso iniciado

Segunda sesion

A12/5. En el plano tenemos una recta r y dos puntos A y B exteriores a la recta

y en el mismo semiplano. Determinar un punto M de la recta tal que el angulo de r

con AM sea doble del de r con BM . (Considerese como angulo de dos rectas el menor

de los angulos que forman).

A12/6. Sean xn e yn dos sucesiones de numeros naturales definidas como

sigue:x1 = 1, x2 = 1, xn+2 = xn+1 + 2xn para n = 1, 2, 3, . . .y1 = 1, y2 = 7, yn+2 = 2yn+1 + 3yn para n = 1, 2, 3, . . .

Demostrar que, salvo el caso x1 = y1 = 1, no existe ningun valor natural que se presente

en las dos sucesiones.

A12/7. Se considera la funcion real definida por

f(x) =1

|x + 3| + |x + 1| + |x − 2| + |x − 5|

para todo x ∈ R .

a) Determinar su maximo.

b) Representacion grafica.

A12/8. Se eligen aleatoriamente dos numeros reales entre 0 y 1. Calcular la prob-

abilidad de que uno cualquiera de ellos sea menor que el cuadrado del otro.

Premiados: Agustın Llerena Achutegui, Federico Cuco Pardillos, Enrique Uzabal

Amores.

Page 34: COMPENDIUM OME - ToomatesEotvos de Hungría, que se iniciaron en 1894, precisamente durante la efervescencia de fin de siglo, consecuencia de la cual fue también el proceso iniciado

1975-76 XIII Olimpiada Matematica

Espanola13

Primera sesion

A13/1. En un plano se dan cuatro puntos fijos A , B , C , D no alineados tres a

tres. Construir un cuadrado de lados a , b , c , d de forma que A ∈ a , B ∈ b , C ∈ c ,

D ∈ d .

A13/2. Se considera el conjunto C de todas las r -plas cuyas componentes son

1 o −1. Calcular la suma de todas las componentes de todos los elementos de C

excluyendo la r -pla (1, 1, 1, . . . , 1).

A13/3. A traves de una lente que invierte la imagen miramos el espejo retrovisor

de nuestro coche. Si en el se refleja la matrıcula del coche que nos sigue, CS-3965-EN,

dibujar la imagen que nosotros recibimos. Dibujar tambien la obtenida permutando

las anteriores transformaciones, es decir, reflejando en el retrovisor la imagen que de

la matrıcula da la lente. ¿Es conmutativo el producto de ambas transformaciones, la

reflexion en el espejo y la refraccion a traves de la lente?

A13/4. Demostrar que la expresion

n5 − 5n3 + 4n

n + 2

donde n es un entero cualquiera, es siempre divisible por 24.

Page 35: COMPENDIUM OME - ToomatesEotvos de Hungría, que se iniciaron en 1894, precisamente durante la efervescencia de fin de siglo, consecuencia de la cual fue también el proceso iniciado

Segunda sesion

A13/5. Demostrar que la ecuacion

z4 + 4(i + 1)z + 1 = 0

tiene una raız en cada cuadrante del plano complejo.

A13/6. Dada una matriz cuadrada M de orden n sobre el cuerpo de los numeros

reales, encontrar, en funcion de M , dos matrices, una simetrica y una antisimetrica,

tales que su suma sea precisamente M .

A13/7. El precio de un diamante es proporcional al cuadrado de su peso. Demostrar

que, rompiendolo en dos partes, existe una depreciacion de su valor. ¿Cuando es maxima

la depreciacion?

A13/8. Se da la funcion

y =∣∣x2 − 4x + 3

∣∣ .Estudiar su continuidad y derivabilidad en el punto de abcisa 1. Su grafica determina

con el eje X una figura cerrada. Determinar el area de dicha figura.

Premiados: Serafın Moral Callejon, Antonio J. Rodrıguez de la Cruz, Antonio Barreiro

Blas.

Page 36: COMPENDIUM OME - ToomatesEotvos de Hungría, que se iniciaron en 1894, precisamente durante la efervescencia de fin de siglo, consecuencia de la cual fue también el proceso iniciado

1976-77 XIV Olimpiada Matematica

Espanola14

Primera sesion

A14/1. Dado el determinante de orden n∣∣∣∣∣∣∣∣∣∣

8 3 3 . . . 33 8 3 . . . 33 3 8 . . . 3...

......

. . ....

3 3 3 . . . 8

∣∣∣∣∣∣∣∣∣∣calcular su valor y determinar para que valores de n dicho valor es multiplo de 10.

A14/2. Demostrar que todas las matrices cuadradas de la forma (con a, b ∈ R),(

a b−b a

)forman un cuerpo conmutativo K cuando se consideran las operaciones de suma y

producto de matrices.

Probar tambien que si A ∈ K es un elemento de dicho cuerpo, existen dos matrices de

K tales que el cuadrado de cada una sea igual a A .

A14/3. Demostrar que en una reunion de 285 personas, una al menos de ellas ha

dado un numero par de apretones de mano (0 se considera un numero par y corresponde

a un asistente que no estrecha ninguna mano).

A14/4. Demostrar que la suma de los cuadrados de cinco enteros consecutivos no

puede ser un cuadrado perfecto.

Page 37: COMPENDIUM OME - ToomatesEotvos de Hungría, que se iniciaron en 1894, precisamente durante la efervescencia de fin de siglo, consecuencia de la cual fue también el proceso iniciado

Segunda sesion

A14/5. Utilizando una escalera mecanica para bajar a la estacion del Metro y an-

dando con paso regular, observo que necesito 50 escalones para bajar. Si luego vuelvo

a subirla corriendo, a una velocidad 5 veces mi paso normal anterior, compruebo que

necesito 125 escalones para llegar arriba. ¿Cuantos escalones visibles tiene la escalera

mecanica cuando se encuentra parada?

A14/6. Se considera un triangulo ABC , y sea D el punto de corte de la bisectriz

correspondiente al angulo A con el lado BC . Demostrar que la circunferencia que

pasa por A y es tangente a la recta BC en D , tambien es tangente a la circunferencia

circunscrita al triangulo ABC .

A14/7. Se dan los numeros A1 , A2 , . . . , An . Demostrar, sin necesidad de calcular

derivadas, que el valor de X que hace mınima la suma

(X − A1)2 + (X − A2)2 + · · · + (X − An)2

es precisamente la media aritmetica de los numeros dados.

A14/8. Determinar una condicion necesaria y suficiente para que los afijos de tres

numeros complejos z1 , z2 y z3 sean los vertices de un triangulo equilatero.

Premiados: Alberto Elduque Palomo, Francisco J. Palma Molina, Jose Pena Gamarra.

Page 38: COMPENDIUM OME - ToomatesEotvos de Hungría, que se iniciaron en 1894, precisamente durante la efervescencia de fin de siglo, consecuencia de la cual fue también el proceso iniciado

1978-79 XV Olimpiada Matematica

Espanola15

Primera sesion. Junio de 1979.

A15/1. Calcular el area de la interseccion del interior de la elipse

x2

16+

y2

4= 1

con el cırculo limitado por la circunferencia (x − 2)2 + (y − 1)2 = 5.

A15/2. Cierto profesor de Oxford, destinado a los servicios de criptografıa del es-

pionaje britanico, papel interpretado por Dirk Bogarde en una pelıcula, recluta su

personal proponiendo pequenos ejercicios de atencion, como leer mentalmente una pal-

abra al reves. Frecuentemente lo hace con su propio nombre: SEBASTIAN, que habra

que leer NAITSABES.

Se pregunta si hay algun movimiento del plano o del espacio que transforme una de

estas palabras en la otra, tal como aparecen escritas. ¿Y si se hubiera llamado AVITO,

como un cierto personaje de Unamuno? Explıquese razonadamente cada respuesta.

A15/3. Demostrar la igualdad(

n

0

)2

+(

n

1

)2

+(

n

2

)2

+ · · ·+(

n

n

)2

=(

2n

n

).

A15/4. Si z1 , z2 son las raıces de la ecuacion con coeficientes reales z2+az+b = 0,

probar que zn1 + zn

2 es un numero real para cualquier valor natural de n . En el caso

particular de la ecuacion z2 − 2z + 2 = 0, expresar, en funcion de n , dicha suma.

Nota: Durante el curso 1977-78 no se celebro Olimpiada Matematica.

Page 39: COMPENDIUM OME - ToomatesEotvos de Hungría, que se iniciaron en 1894, precisamente durante la efervescencia de fin de siglo, consecuencia de la cual fue también el proceso iniciado

Segunda sesion. Junio de 1979.

A15/5. Calcular la integral definida∫ 4

2

sen((x − 3)3

)dx.

A15/6. Una urna se lleno con tres bolas por el siguiente procedimento: se lanzo

una moneda tres veces, introduciendo, cada vez que salio cara una bola blanca, y

cada vez que salio cruz, una bola negra. Extraemos de esta urna, por cuatro veces

consecutivas, una bola; la devolvemos a la urna antes de la extraccion siguiente. ¿Cual

es la probabilidad de que en las cuatro extracciones se obtenga bola blanca?

A15/7. Probar que el volumen de un neumatico (toro) es igual al volumen de un

cilindro cuya la base es una seccion meridiana de aquel y que tiene por altura la longitud

de la circunferencia formada por los centros de las secciones meridianas.

A15/8. Dado el polinomio

P (x) = 1 + 3x + 5x2 + 7x3 + · · ·+ 1001x500,

expresar el valor numerico de su derivada de orden 325 para x = 0.

Premiados: Carles Casacuberta Verges, Jesus Nievas Espuelas, Jorge Mas Trullenque.

Page 40: COMPENDIUM OME - ToomatesEotvos de Hungría, que se iniciaron en 1894, precisamente durante la efervescencia de fin de siglo, consecuencia de la cual fue también el proceso iniciado

1979-80 XVI Olimpiada Matematica

Espanola16

Primera sesion.

A16/1. De entre los triangulos que tienen un lado de 5 m de longitud y el angulo

opuesto de 30 , determinar el de area maxima, calculando el valor de los otros dos

angulos y el area del triangulo.

A16/2. Una urna contiene los votos para la eleccion de dos candidatos A y B .

Se sabe que el candidato A cuenta con 6 votos y el candidato B con 9. Hallar la

probabilidad de que, al efectuar el escrutinio, siempre vaya por delante el candidato B .

A16/3. Demostrar que si a1 , a2 , . . . , an son numeros reales positivos, entonces

(a1 + a2 + · · · + an)(

1a1

+1a2

+ · · ·+ 1an

)≥ n2.

¿Cuando es valida la igualdad?

A16/4. Hallar la funcion f(x) que cumple la ecuacion

f ′(x) + x2f(x) = 0

sabiendo que f(1) = e . Representar graficamente esta funcion y calcular la tangente

en el punto de la curva de abscisa 1.

Page 41: COMPENDIUM OME - ToomatesEotvos de Hungría, que se iniciaron en 1894, precisamente durante la efervescencia de fin de siglo, consecuencia de la cual fue también el proceso iniciado

Segunda sesion.

A16/5. Demostrar que si x es tal que

x +1x

= 2 cos α

entonces, para todo n = 0, 1, 2, . . . ,

xn +1xn

= 2 cosnα.

A16/6. Demostrar que si al producto de cuatro numeros naturales consecutivos se

anade una unidad, el resultado es un cuadrado perfecto.

A16/7. El punto M varıa sobre el segmento AB que mide 2 m.

a) Hallar la ecuacion y la representacion grafica del lugar geometrico de los puntos del

plano cuyas coordenadas, x , y , son, respectivamente, las areas de los cuadrados de

lados AM y MB .

b) Averiguar que clase de curva es. (Sugerencia: hacer un giro de ejes de 45 ).

c) Hallar el area del recinto comprendido entre la curva obtenida y los ejes de coorde-

nadas.

A16/8. Determinar todos los triangulos tales que las longitudes de los tres lados y

su area esten dados por cuatro numeros naturales consecutivos.

Premiados: Guillermo Rozas Rodrıguez, Pedro Carrion Rodrıguez de Guzman, Jose

Fernando Lopez Blazquez.

Page 42: COMPENDIUM OME - ToomatesEotvos de Hungría, que se iniciaron en 1894, precisamente durante la efervescencia de fin de siglo, consecuencia de la cual fue también el proceso iniciado

1980-81 XVII Olimpiada Matematica

Espanola17

Primera sesion. Junio de 1981.

A17/1. Calcular la suma de n sumandos

7 + 77 + 777 + · · ·+ 7 . . .7.

A17/2. Un vaso de vidrio cilındrico tiene 8 cm de altura y su borde 12 cm de

circunferencia. En su interior, a 3 cm del borde, hay una diminuta gota de miel. En un

punto de su superficie exterior, perteneciente al plano que pasa por el eje del cilindro

y por la gota de miel, y situado a 1 cm de la base (o fondo) del vaso, hay una mosca.

¿Cual es el camino mas corto que la mosca debe recorrer, andando sobre la superficie

del vaso, hasta la gota de miel, y que longitud tiene dicho camino?

A17/3. Dadas las rectas que se cruzan r y s , se consideran las rectas u y v tales

que:

a) u es simetrica de r respecto de s ,

b) v es simetrica de s respecto de r .

Determinar el angulo que deben formar las rectas dadas para que u y v sean copla-

narias.

A17/4. Calcular la integral∫

dx

sen (x − 1) sen (x − 2).

Sugerencia: Cambio tanx = t .

Page 43: COMPENDIUM OME - ToomatesEotvos de Hungría, que se iniciaron en 1894, precisamente durante la efervescencia de fin de siglo, consecuencia de la cual fue también el proceso iniciado

Segunda sesion. Junio de 1981.

A17/5. Dado un numero natural no nulo n , sea fn la funcion del intervalo cerrado

[0, 1] en R definida ası:

fn(x) =

n2x, si 0 ≤ x < 1/n

3/n, si 1/n ≤ x ≤ 1.

a) Representar graficamente la funcion.

b) Calcular An =∫ 1

0fn(x) dx .

c) Hallar, si existe, limn→∞ An .

A17/6. Demostrar que la transformacion producto de la simetrıa de centro (0, 0)

por la simetrıa de eje la recta de ecuacion x = y + 1, puede expresarse como producto

de una simetrıa de eje la recta e por una traslacion de vector v , con e paralela a v .

Determinar una recta e y un vector v que cumplan las condiciones indicadas. ¿Han de

ser unicos e y v?

A17/7. En una fabrica de bolas de tenis hay 4 maquinas m1 , m2 , m3 , m4 , que

producen, respectivamente, el 10%, 20%, 30% y 40% de las bolas que salen de la

fabrica. La maquina m1 introduce defectos en un 1% de las bolas que fabrica, la

maquina m2 en el 2%, la m3 en el 4% y la m4 en el 15%. De las pelotas fabricadas

en un dia, se elige una al azar y resulta ser defectuosa. ¿Cual es la probabilidad de que

esta bola haya sido elaborada por la maquina m3 ?

A17/8. Si a es un numero impar, demostrar que

a4 + 4a3 + 11a2 + 6a + 2

es una suma de tres cuadrados y que es divisible por 4.

Premiados: Pablo Alvarez Royo-Villanova, Fernando Barbero Gonzalez, Fernando

Etayo Gordejuela.

Page 44: COMPENDIUM OME - ToomatesEotvos de Hungría, que se iniciaron en 1894, precisamente durante la efervescencia de fin de siglo, consecuencia de la cual fue también el proceso iniciado

1981-82 XVIII Olimpiada Matematica

Espanola18

Primera sesion. Junio de 1982.

A18/1. En la pagina de pasatiempos de un periodico se propone este problema:

“Dos ninos, Antonio y Jose, tienen 160 tebeos. Antonio cuenta los suyos de 7 en 7 y le

sobran 4. Jose cuenta los suyos de 8 en 8 y tambien le sobran 4. ¿Cuantos tebeos tiene

cada uno?” En el siguiente numero del periodico se da esta solucion: “Antonio tiene

60 tebeos y Jose tiene 100.” Analiza esta solucion e indica que harıa un matematico

con este problema.

A18/2. Al componer una simetrıa de eje r con un giro de angulo recto alrededor

de un punto P que no pertenece a la recta, resulta otro movimiento M .

¿Es M una simetrıa axial? ¿Hay alguna recta invariante por M ?

A18/3. Se lanza un cohete y alcanza los 120 m de altura; en la caıda pierde 60 m,

a continuacion recupera 40 m, vuelve a perder 30, gana 24, pierde 20, etc.

Si el proceso sigue indefinidamente, ¿a que altura tiende a estabilitzarse?

A18/4. Determinar un polinomio de coeficientes reales no negativos que cumpa las

dos condiciones siguientes:

p(0) = 0, p(|z|

)≤ x4 + y4,

siendo |z| el modulo del numero complejo z = x + iy .

Page 45: COMPENDIUM OME - ToomatesEotvos de Hungría, que se iniciaron en 1894, precisamente durante la efervescencia de fin de siglo, consecuencia de la cual fue también el proceso iniciado

Segunda sesion. Junio de 1982.

A18/5. Construir un cuadrado conociendo la suma de la diagonal y el lado.

A18/6. Demostrar que si u , v son numeros reales no negativos cualesquiera, y a ,

b numeros reales positivos tales que a + b = 1, entonces

ua vb ≤ au + bv.

A18/7. Sea S el subconjunto de numeros racionales que pueden escribirse en la

forma a/b , donde a es un entero cualquiera y b un entero impar. ¿Pertenece a S la

suma de dos de sus elementos? ¿Y el producto? ¿Hay en S elementos cuyo inverso

pertenezca a S ?

A18/8. Dado un conjunto C de puntos del plano, se llama distancia de un punto

P del plano al conjunto C a la menor de las distancias de P a cada uno de los puntos

de C . Sean los conjuntos C = A, B , con A = (1, 0) y B = (2, 0); y C′ = A′, B′con A′ = (0, 1) y B′ = (0, 7), en un sistema de referencia ortogonal.

Hallar y dibujar el conjunto M de puntos del plano que equidistan de C y C′ .

Estudiar si es derivable la funcion cuya grafica es el conjunto M antes obtenido.

Premiados: Javier Caballero Guerrero, Jose Sanchez Lacuesta, Patrik Simonetta.

Page 46: COMPENDIUM OME - ToomatesEotvos de Hungría, que se iniciaron en 1894, precisamente durante la efervescencia de fin de siglo, consecuencia de la cual fue también el proceso iniciado

1982-83 XIX Olimpiada Matematica

Espanola19

Primera sesion. Febrero de 1983.

A19/1. Mientras Teofrasto hablaba con Aristoteles sobre la clasificacion de las plan-

tas, tenıa un perro atado a una columna cilındrica perfectamente lisa de radio r , con

una cuerda muy fina que envolvıa la columna y con un lazo. El perro tenıa el extremo

libre de la cuerda cogido a su cuello. Al intentar alcanzar a Teofrasto, puso la cuerda

tirante y esta se rompio. Averiguar a que distancia de la columna estaba el nudo en el

momento de romperse la cuerda.

A19/2. Construir un triangulo conociendo un angulo, la razon de los lados que lo

forman y el radio del cırculo inscrito.

A19/3. Una semicircunferencia de radio r se divide en n + 1 partes iguales y se

une un punto cualquiera k de la division con los extremos de la semicircunferencia,

formandose ası un triangulo Ak . Calcular el lımite, cuando n tiende a infinito, de la

media aritmetica de las areas de los triangulos.

A19/4. Determinar el numero de raıces reales de la ecuacion

16x5 − 20x3 + 5x + m = 0.

Page 47: COMPENDIUM OME - ToomatesEotvos de Hungría, que se iniciaron en 1894, precisamente durante la efervescencia de fin de siglo, consecuencia de la cual fue también el proceso iniciado

Segunda sesion. Febrero de 1983.

A19/5. Hallar las coordenadas de los vertices de un cuadrado ABCD , sabiendo

que A esta sobre la recta y − 2x − 6 = 0, C en x = 0 y B es el punto (a, 0), siendo

a = log2/3(16/81).

A19/6. En una cafeterıa, un vaso de limonada, tres bocadillos y siete bizcochos han

costado 1 chelın y 2 peniques; y un vaso de limonada, cuatro bocadillos y 10 bizcochos

valen 1 chelın y 5 peniques. Hallar el precio de:

a) un vaso de limonada, un bocadillo y un bizcocho;

b) dos vasos de limonada, tres bocadillos y cinco bizcochos.

(1 chelın = 12 peniques).

A19/7. Un tetraedro regular de arista 30 cm descansa sobre una de sus caras.

Suponiendolo hueco, se le echan 2 litros de agua. Se pide la altura que alcanza el

lıquido y el area de la superficie libre del agua.

A19/8. En 1960, el mayor de tres hermanos tiene una edad que es la suma de las

de sus hermanos mas pequenos. Unos anos despues, la suma de las edades de dos de

los hermanos es doble de la del otro. Ha pasado ahora un numero de anos desde 1960,

que es igual a dos tercios de la suma de las edades que los tres hermanos tenıan en ese

ano, y uno de ellos ha alcanzado los 21 anos. ¿Cual es la edad de cada uno de los otros

dos?

Premiados: Josep Burillo Puig, Roberto Selva Gomis, Francisco J. Dıez Vegas, Jose

Maranon Mora.

Page 48: COMPENDIUM OME - ToomatesEotvos de Hungría, que se iniciaron en 1894, precisamente durante la efervescencia de fin de siglo, consecuencia de la cual fue también el proceso iniciado

1983-84 XX Olimpiada Matematica

Espanola20

Primera sesion. Febrero de 1984.

A20/1. En una posicion O de un aeropuerto de campana esta emplazado un canon

que puede girar 360 . Dos tanques atacan dicho lugar siguiendo trayectorias rectas

AB y CD dadas. Hallar graficamente el alcance del canon sabiendo que la suma de

los trozos de trayectorias de ambos tanques en los cuales estos estan bajo el fuego del

canon, es una longitud conocida .

A20/2. Determinar un numero de cinco cifras tal que su cuadrado termine en las

mismas cinco cifras colocadas en el mismo orden.

A20/3. Dados dos numeros reales positivos p , q tales que p + q = 1, y sabiendo

que todo par de numeros reales x, y cumple (x − y)2 ≥ 0, se pide demostrar

a)x + y

2≥ √

xy

b)x2 + y2

2≥

(x + y

2

)2

a)(

p +1p

)2

+(

q +1q

)2

≥ 252

A20/4. Calcular

limn→∞

cosx

2· cos

x

22· · · cos

x

2n.

Page 49: COMPENDIUM OME - ToomatesEotvos de Hungría, que se iniciaron en 1894, precisamente durante la efervescencia de fin de siglo, consecuencia de la cual fue también el proceso iniciado

Segunda sesion. Febrero de 1984.

A20/5. Llevense arcos iguales AB = A′B′ = x sobre dos circunferencias iguales a

partir de dos puntos fijos A , A′ sobre cada una de ellas. Hallar el lugar geometrico de

los puntos medios del segmento BB′ al variar x :

a) cuando los arcos se llevan en el mismo sentido,

b) cuando los arcos se llevan en sentidos opuestos.

A20/6. Se considera una circunferencia γ de centro (3, 0) y radio 3, y la recta

r paralela al eje Ox y que dista 3 del origen. Se traza una recta variable por el

origen que corta a γ en el punto M y corta a la recta r en P . Determinar el lugar

geometrico de los puntos de interseccion de las paralelas a Ox y Oy trazadas por M

y P respectivamente.

A20/7. Se consideran los numeros naturales escritos en el sistema de base 10.

a) Encontrar el menor numero que al suprimirle la primera cifra quede reducido a su

quinta parte. ¿De que forma son todos los numeros que tienen esta propiedad?

b) Demostrar que no existe ningun numero que al suprimirle la primera cifra quede

dividido por 12.

c) Formular un criterio general que nos permita afirmar cuando un numero queda

dividido por k al suprimir su primera cifra.

A20/8. Hallar el resto de la division por x2 − 1 del determinante∣∣∣∣∣∣∣

x3 + 3x 2 1 0x2 + 5x 3 0 2x4 + x2 + 1 2 1 3x5 + 1 1 2 3

∣∣∣∣∣∣∣ .

Premiados: Pablo Novaes Ledieu, Andres Garcıa Parrilla, Miguel Aparisi Botella,

Gonzalo Genova Fuster, Agustın Rafael Tejera Gomez, Miguel Brandt Sanz.

Page 50: COMPENDIUM OME - ToomatesEotvos de Hungría, que se iniciaron en 1894, precisamente durante la efervescencia de fin de siglo, consecuencia de la cual fue también el proceso iniciado

1984-85 XXI Olimpiada Matematica

Espanola21

Primera session. Febrero de 1985.

A21/1. Sea P el conjunto de los puntos del plano y f : P → P una aplicacion que

cumple las tres condiciones siguientes:

a) f es biyectiva.

b) Para cada recta r del plano, f(r) es una recta.

c) Para cada recta r , la recta f(r) es paralela o coincidente con r .

¿Que posibles transformaciones pueden ser f ?

A21/2. Sea Z el conjunto de los enteros y Z × Z el conjunto de pares ordenados

de enteros. La suma de estos pares se define por

(a, b) + (a′, b′) = (a + a′, b + b′),

siendo (−a,−b) el opuesto de (a, b) .

Estudiar si existe un subconjunto E de Z × Z que cumpla las condiciones siguientes:

a) La suma de dos pares de E tambien es de E .

b) El par (0, 0) pertenece a E .

c) Si (a, b) no es (0, 0), entonces o bien (a, b) pertenece a E , o bien (−a,−b) pertenece

a E , pero no ambos.

A21/3. Resolver la ecuacion

tan2 2x + 2 tan 2x tan 3x − 1 = 0.

A21/4. Consideremos tres numeros naturales a , b , c tales que la razon

a + b + c

abc

sea el inverso de un numero k entero y positivo. Se pide demostrar:

a) a3 + b3 + c3 no es primo.

b) Para cada k ∈ N existen ternas de naturales a , b , c que cumplen las condiciones.

Page 51: COMPENDIUM OME - ToomatesEotvos de Hungría, que se iniciaron en 1894, precisamente durante la efervescencia de fin de siglo, consecuencia de la cual fue también el proceso iniciado

Segunda sesion. Febrero de 1985.

A21/5. Hallar la ecuacion de la circunferencia que pasa por los afijos de las solu-

ciones de la ecuacion

z3 + (−1 + i)z2 + (1 − i)z + i = 0.

A21/6. Se consideran las semirrectas no alineadas Ox , Oy . Por el punto A ∈ Ox

se trazan pares de rectas r1 , r2 , antiparalelas respecto al angulo xOy ; sean M , N

las intersecciones de r1 con Oy y de r2 con Ox , respectivamente. Sea P el punto de

interseccion de las bisectrices de los angulos AMy , ANy . Hallar el lugar geometrico

de P al variar A .

A21/7. Dada la ecuacion x5−px−1 = 0, estudiar el valor de p de forma que existan

dos soluciones de la ecuacion, x1 , x2 , que a la vez sean soluciones de x2 − ax + b = 0,

con a , b enteros.

A21/8. Diremos que una matriz cuadrada es de suma constante si la suma de

los elementos de cada fila, de cada columna, y de cada diagonal, son valores iguales.

Analogamente, una matriz cuadrada es de producto constante si son iguales los pro-

ductos de los elementos de cada fila, de cada columna y de cada diagonal. Determinar

las matrices cuadradas de orden 3 sobre R que son, a la vez, de suma y de producto

constante.

Premiados: Ricardo Perez Marco, Ignacio Garijo Amilburo, Juan Acuaron Joven,

Ana Jose Reguera Lopez, Jose Luis Ansorena Barasoain, Antonio Gomez Amigo.

Page 52: COMPENDIUM OME - ToomatesEotvos de Hungría, que se iniciaron en 1894, precisamente durante la efervescencia de fin de siglo, consecuencia de la cual fue también el proceso iniciado

1985-86 XXII Olimpiada Matematica

Espanola22

Primera sesion. Febrero de 1986.

A22/1. Indicaremos por [x], x las partes entera y decimal del numero real x .

Definimos una distancia entre los numeros reales x e y

d(x, y) =√(

[x] − [y])2 +

(x − y

)2.

Determinar (como union de intervalos) el conjunto de los numeros reales que distan del

numero 3/2 menos de 202/100.

A22/2. Un segmento d divide al segmento s si existe un natural n tal que

nd = d + d+n· · · +d = s.

a) Demostrar que si el segmento d divide a los segmentos s y s′ con s < s′ , entonces

divide al segmento diferencia s′ − s .

b) Demostrar que ningun segmento divide al lado s y a la diagonal s′ de un pentagono

regular (razonar sobre el pentagono regular cuyos lados estan contenidos en las diago-

nales del pentagono dado, sin efectuar calculos numericos).

A22/3. Hallar los valores de n ∈ N tales que 5n + 3 es una potencia de 2 de

exponente natural.

Page 53: COMPENDIUM OME - ToomatesEotvos de Hungría, que se iniciaron en 1894, precisamente durante la efervescencia de fin de siglo, consecuencia de la cual fue también el proceso iniciado

Segunda sesion. Febrero de 1986.

A22/4. Indicamos por m(a, b) la media aritmetica de los numeros reales positivos

a y b . Dada la funcion real positiva g que tiene la primera y la segunda derivada

positivas, definimos la media µ(a, b) relativa a la funcion g mediante

2 g(µ(a, b)

)= g(a) + g(b).

Decir, razonadamente, cual de las dos medias m y µ es mayor.

A22/5. Consideramos la curva Γ definida por la ecuacion y2 = x3 + bx+ b2 , donde

la constante b es un numero racional no nulo. Inscribir en la curva Γ un triangulo

cuyos vertices tengan coordenadas racionales.

A22/6. Calcular

14∏k=1

cos(kπ

15

).

Premiados: Carlos Ueno Jacue, Alberto Garrido Arribas, Juan David Gonzalez Cobas,

Jaume Amoros Torrent, Joaquim Ortega Cerda, Juan Cuenca Gonzalez.

Page 54: COMPENDIUM OME - ToomatesEotvos de Hungría, que se iniciaron en 1894, precisamente durante la efervescencia de fin de siglo, consecuencia de la cual fue también el proceso iniciado

1986-87 XXIII Olimpiada Matematica

Espanola23

Primera sesion. Febrero de 1987.

A23/1. Sean a , b , c las longitudes de los lados de un triangulo no isosceles. Se

dan tres cırculos concentricos de radios a , b y c .

a) ¿Cuantos triangulos equilateros de areas distintas pueden construirse, de modo que

las rectas que contienen sus lados sean tangentes a cada cırculo?

b) Hallar las superficies de estos triangulos.

A23/2. Demostrar que para todo numero natural n > 1 se cumple

1 ·

√(n

1

)+ 2 ·

√(n

2

)+ · · ·+ n ·

√(n

n

)<

√2n−1n3.

A23/3. Un triangulo dado T se descompone en triangulos T1 , T2 , . . . , Tn de

manera que:

a) Ningun par de triangulos Ti tiene puntos interiores comunes.

b) La union de todos los triangulos Ti es T .

c) Cada segmento que es lado de algun triangulo Ti , o bien es lado de otro triangulo

Tj , o bien es lado del triangulo T .

Sean s el numero total de lados (cada uno contado una sola vez, aunque sea comun a

dos triangulos), y v el numero total de vertices (cada uno contado una sola vez, aunque

sea comun a varios triangulos).

Demostrar que si n es impar, existen varias descomposiciones de esta clase, y todas

tienen el mismo numero v de vertices y el mismo numero s de lados. Expresar v y s

en funcion de n . Demostrar tambien que si n es par no existe tal descomposicion.

Page 55: COMPENDIUM OME - ToomatesEotvos de Hungría, que se iniciaron en 1894, precisamente durante la efervescencia de fin de siglo, consecuencia de la cual fue también el proceso iniciado

Segunda sesion. Febrero de 1987.

A23/4. Si a y b son dos numeros reales diferentes, resolver el sistema

x + y = 1

(ax + by)2 ≤ a2x + b2y.

Resolver tambien el sistema

x + y = 1

(ax + by)4 ≤ a4x + b4y.

A23/5. En un triangulo ABC tenemos puntos D y E respectivamente sobre AB

y AC . Conocemos la medida de los angulos indicados a continuacion: ABE = 30 ,

EBC = 50 , ACD = 20 y DCB = 60 . Hallar el valor del angulo EDC .

A23/6. Para cada numero natural n se considera el polinomio

Pn(x) = xn+2 − 2x + 1.

a) Demostrar que la ecuacion Pn(x) = 0 tiene una raız cn y solo una en el intervalo

(0, 1).

b) Calcular

limn→∞

cn.

Premiados: Fernando Galve Mauricio, Salvador Villegas Barranco, Santiago Vila Don-

cel, Juan R. Valderrama Alcalde, Pablo Benıtez Gimenez, Carlos J. Perez Jimenez.

Page 56: COMPENDIUM OME - ToomatesEotvos de Hungría, que se iniciaron en 1894, precisamente durante la efervescencia de fin de siglo, consecuencia de la cual fue también el proceso iniciado

1987-88 XXIV Olimpiada Matematica

Espanola24

Primera sesion. Febrero de 1988.

A24/1. Sea (xn) , n ∈ N , una sucesion de numeros enteros tal que

x1 = 1,

xn+1 > xn, para n ≥ 1,

xn+1 ≤ 2n, para n ≥ 1.

Demostrar que para todo entero natural k existen dos terminos de la sucesion xr y xs

tales que xr − xs = k .

A24/2. Sobre una circunferencia se eligen n > 3 puntos y se numeran de 1 a n

en cualquier orden. Diremos que dos puntos no consecutivos a y b estan relacionados

si en uno de los dos arcos de extremos a y b , todos los puntos estan marcados con

numeros menores que las marcas de a y b .

Demostrar que el numero de pares de puntos relacionados es exactamente n − 3.

A24/3. Probar que los binomios 25x + 31y y 3x + 7y son multiplos de 41 para

los mismos valores de x e y .

Page 57: COMPENDIUM OME - ToomatesEotvos de Hungría, que se iniciaron en 1894, precisamente durante la efervescencia de fin de siglo, consecuencia de la cual fue también el proceso iniciado

Segunda sesion. Febrero de 1988.

A24/4. Se atribuye al matematico renacentista Leonardo da Pisa (mas conocido

como Fibonacci) la sucesion definida de la manera siguiente

a1 = 1,

a2 = 1,

ai = ai−1 + ai−2 para i > 2.

Expresar a2n en funcion solamente de los tres terminos an−1 , an , an+1 .

A24/5. Es muy conocido el puzzle consis-

tente en descomponer la cruz griega de la

izquierda de la figura en cuatro partes con las que

se pueda componer un cuadrado. Una solucion

habitual es la de la figura de la derecha. De-

mostrar que hay una infinidad de soluciones

diferentes.

4

3

12

3 24

1

¿Hay alguna solucion que de lugar a cuatro partes iguales?

A24/6. Calcular, para cualquier valor del parametro entero t , soluciones enteras x ,

y de la ecuacion

y2 = x4 − 22x3 + 43x2 + 858x + t2 + 10452(t + 39).

Premiados: Javier Campins Pascual, Ramon Esteban Romero, Santiago Perez-Cacho

Fernando-Arguelles, Jose Ignacio Nogueira Coriba, Boris Bartolome Mana, Fernando

Martınez Puente.

Page 58: COMPENDIUM OME - ToomatesEotvos de Hungría, que se iniciaron en 1894, precisamente durante la efervescencia de fin de siglo, consecuencia de la cual fue también el proceso iniciado

1988-89 XXV Olimpiada Matematica

Espanola25

Primera sesion. Febrero de 1989.

A25/1. El programa de una asignatura consta de n preguntas; el examen consiste en

desarrollar una de esas preguntas, elegida al azar. Un alumno solo se sabe una pregunta,

pero puede repetir el examen n veces. Expresar, en funcion de n , la probabilidad pn

de que el alumno apruebe el examen. ¿Crece o decrece pn al aumentar n? Calcular

limn→∞

pn.

¿Cual es la mayor de las cotas inferiores de las probabilidades pn ?

A25/2. Los puntos A′ , B′ , C′ de los lados BC , CA , AB del triangulo ABC

cumplenAC′

C′B=

BA′

A′C=

CB′

B′A= k.

Las rectas AA′ , BB′ , CC′ forman un triangulo A1B1C1 . Dados k y el area S del

triangulo ABC , calcular el area del triangulo A1B1C1 .

A25/3. Demostrar que

110

√2

<1 · 3 · 5 · · ·992 · 4 · 6 · · · 100

<110

.

Page 59: COMPENDIUM OME - ToomatesEotvos de Hungría, que se iniciaron en 1894, precisamente durante la efervescencia de fin de siglo, consecuencia de la cual fue también el proceso iniciado

Segunda sesion. Febrero de 1989.

A25/4. Demostrar que el numero 1989 y todas sus potencias enteras 1989n se

pueden escribir como suma de dos cuadrados de enteros positivos, y como mınimo, de

dos formas diferentes.

A25/5. Sea D el conjunto de los numeros complejos que se pueden escribir en la

forma a + b√−13, con a , b enteros. El numero 14 = 14 + 0

√−13 puede escribirse

como producto de dos elementos de D : 14 = 2 · 7. Expresar 14 como producto de dos

elementos de D de todas las formas posibles.

A25/6. Demostrar que dados siete numeros reales cualesquiera, se pueden elegir

dos, digamos a y b , de manera que

√3 |a − b| < |1 + ab|.

Dar un ejemplo de seis numeros reales que no cumplan esta propiedad.

Premiados: Vicente Munoz Velazquez, Enrique Garcıa Lopez, Alberto Garcıa Martınez,

Cristina Draper Fontanales, Leandro Marın Munoz, Javier Portela Lemos.

Page 60: COMPENDIUM OME - ToomatesEotvos de Hungría, que se iniciaron en 1894, precisamente durante la efervescencia de fin de siglo, consecuencia de la cual fue también el proceso iniciado

1989-90 XXVI Olimpiada Matematica

Espanola26

Primera sesion. 16 de Marzo de 1990.

A26/1. Sean x e y dos numeros reales positivos. Probar que la expresion

A =√

x +√

y +√

xy

se puede escribir en la forma

B =√

x +√

y + xy + 2y√

x

y comparar los numeros

L =√

3 +√

10 + 2√

3 y M =√

5 +√

22 +

√8 −

√22 + 2

√15 − 3

√22.

A26/2. Cada punto de un plano esta pintado de un color elegido entre tres distintos.

¿Existen necesariamente dos puntos de ese plano que disten 1 cm y que esten pintados

del mismo color?

A26/3. Se llama parte entera de un numero real a (y se escribe [a] ), al mayor

numero entero menor o igual que a . Si n es un numero natural, demostrar que la parte

entera de (4 +√

11)n es un numero impar.

Page 61: COMPENDIUM OME - ToomatesEotvos de Hungría, que se iniciaron en 1894, precisamente durante la efervescencia de fin de siglo, consecuencia de la cual fue también el proceso iniciado

Segunda sesion. 17 de Marzo de 1990.

A26/4. Demostrar que la suma

3

√a + 1

2+

a + 36

√4a + 3

3+

3

√a + 1

2− a + 3

6

√4a + 3

3

es independiente del valor de a , para todo valor real a ≥ −3/4, y hallar el valor de

dicha suma.

A26/5. Tres puntos A′ , B′ , C′ estan situados, respectivamente, sobre los lados

BC , CA y AB de un triangulo dado ABC de area S , de forma que

AC′

AB=

BA′

BC=

CB′

CA= p,

siendo p un parametro variable, 0 < p < 1. Determinar

1) El area del triangulo A′B′C′ en funcion de p .

2) El valor de p que minimiza el area anterior.

3) El lugar geometrico de los puntos P de interseccion de las paralelas trazadas por

A′ y C′ , repectivamente a los lados AB y AC , cuando p esta entre 0 y 1.

A26/6. Se consideran n puntos del plano de forma que no existan dos parejas

equidistantes. Por cada punto se traza el segmento que le une al mas proximo. De-

mostrar que ningun punto esta unido a mas de cinco puntos.

Premiados: Francisco Ogando Serrano, Daniel Lasaosa Medarde, Marco Castrillon

Lopez, Javier Arregui Garcıa, Jose F. Herrador Barrios, Jose M. Gordillo Arias de

Saavedra.

Page 62: COMPENDIUM OME - ToomatesEotvos de Hungría, que se iniciaron en 1894, precisamente durante la efervescencia de fin de siglo, consecuencia de la cual fue también el proceso iniciado

1990-91 XXVII Olimpiada Matematica

Espanola27

Primera sesion. 15 de Febrero de 1991.

A27/1. En el plano, donde se ha tomado un sistema de referencia ortonormal,

se consideran todos los puntos (m, n) cuyas coordenadas son numeros enteros. Se

suponen trazados todos los segmentos que unen pares cualesquiera de estos puntos y

cuya longitud es entera. Probar que no hay dos de esos segmentos que formen un angulo

de 45 .

Si se hace lo mismo con los puntos (m, n, k) del espacio. ¿Habra algun par de esos

segmentos que formen un angulo de 45 ?

A27/2. Sean a y b enteros diferentes de 0, 1 y −1, y consideremos la matriz

a + b a + b2 a + b3 · · · a + bm

a2 + b a2 + b2 a2 + b3 · · · a2 + bm

a3 + b a3 + b2 a3 + b3 · · · a3 + bm

......

......

an + b an + b2 an + b3 · · · an + bm

.

Determinar un subconjunto S de filas de esa matriz, lo menor posible, tal que cualquier

otra fila se pueda expresar como suma de las filas de S multiplicadas por numeros

enteros apropiados (es decir, como combinacion lineal con coeficientes enteros de las

filas de S ). Explicitar dichas combinaciones lineales.

A27/3. Supongamos que la ecuacion x3 + px2 + qx + r = 0 con r = 0, admite tres

raıces reales y positivas. Determinar la relacion que debe ligar los numeros reales p , q

y r a fin de que las tres raıces puedan ser las longitudes de los lados de un triangulo.

Page 63: COMPENDIUM OME - ToomatesEotvos de Hungría, que se iniciaron en 1894, precisamente durante la efervescencia de fin de siglo, consecuencia de la cual fue también el proceso iniciado

Segunda sesion. 16 de Febrero de 1991.

A27/4. Sean A′ , B′ y C′ los puntos de tangencia de los lados BC , CA y AB de

un triangulo con su circunferencia inscrita. Sea D el punto de interseccion de C′A′

con la bisectriz del angulo del vertice A . Calcular el valor del angulo ADC .

A27/5. Dado un numero natural n , se designa por s(n) la suma de las cifras del

numero n , expresado en el sistema de numeracion binario, es decir, el numero de cifras

1 que tiene. Determinar, para todo numero natural k

σ(k) = s(1) + s(2) + · · ·+ s(2k).

A27/6. Calcular la parte entera de

S =1√1

+1√2

+ · · ·+ 1√10000

.

Premiados: Ignasi Mundet Riera, Roger Espel Llima, Marcos Durantez Gamzukoff,

Ignacio Uriarte Tuero, Alberto Bravo de Mansilla Jimenez, Ignacio Marcos Primo.

Page 64: COMPENDIUM OME - ToomatesEotvos de Hungría, que se iniciaron en 1894, precisamente durante la efervescencia de fin de siglo, consecuencia de la cual fue también el proceso iniciado

1991-92 XXVIII Olimpiada Matematica

Espanola28

Primera sesion. 14 de Febrero de 1992.

A28/1. Un numero N , multiplo de 83, es tal que su cuadrado tiene 63 divisores.

Hallar N , sabiendo que es el menor numero que cumple las condiciones anteriores.

A28/2. Dadas dos circunferencias exteriores de radios r y r′ (r = r′) , se pide

dibujar, razonadamente, una recta paralela a una direccion dada, tal que determine

sobre las dos circunferencias dos cuerdas tales que la suma de sus longitudes sea igual

a una longitud dada .

A28/3. Probar que si a , b , c y d son numeros enteros no negativos, y es

(a + b)2 + 2a + b = (c + d)2 + 2c + d, (∗)

necesariamente debe ser a = c y b = d .

Probar la misma conclusion si, en lugar de (*) se cumple

(a + b)2 + 3a + b = (c + d)2 + 3c + d.

Ver que, en cambio, existen numeros enteros no negativos a = c , b = d , tales que

(a + b)2 + 4a + b = (c + d)2 + 4c + d.

Page 65: COMPENDIUM OME - ToomatesEotvos de Hungría, que se iniciaron en 1894, precisamente durante la efervescencia de fin de siglo, consecuencia de la cual fue también el proceso iniciado

Segunda sesion. 15 de Febrero de 1992.

A28/4. Sea la sucesion (progresion aritmetica)

3, 7, 11, 15, . . .

Demostrar que en dicha sucesion hay infinitos numeros primos.

A28/5. Dibujado el triangulo de vertices A , B , C , se pide determinar graficamente

el punto P tal que

PAB = PBC = PCA.

Expresar una funcion trigonometrica de este angulo PAB en funcion de las funciones

trigonometricas de los angulos A , B y C .

A28/6. Dados un numero natural n > 0 y un numero complejo z = x + iy de

modulo unidad, x2 + y2 = 1, se puede cumplir o no la igualdad(z +

1z

)n

= 2n−1(zn +

1zn

).

Fijado n , designaremos per S(n) al subconjunto de complejos de modulo unidad para

los que se cumple la igualdad dada. Se pide

a) Calcular razonadamente S(n) , para n = 2, 3, 4, 5.

b) Acotar superiormente el numero de elementos de S(n) en funcion de n , para n > 5.

Premiados: Alvaro Begue Aguado, Javier Ribon Herguedas, Jose Miguel Atienza

Riera, Raquel Barco Moreno, Vicente Giner Bosch, Manuel M. Aguado Martınez.

Page 66: COMPENDIUM OME - ToomatesEotvos de Hungría, que se iniciaron en 1894, precisamente durante la efervescencia de fin de siglo, consecuencia de la cual fue también el proceso iniciado

1992-93 XXIX Olimpiada Matematica

Espanola29

Primera sesion. 26 de Febrero de 1993.

A29/1. En una reunion hay 201 personas de 5 nacionalidades diferentes. Se sabe

que, en cada grupo de 6, al menos 2 tienen la misma edad. Demostrar que hay al menos

5 personas del mismo paıs, de la misma edad y del mismo sexo.

A29/2. Escrito el triangulo aritmetico

0 1 2 3 4 . . . 1991 1992 1993

1 3 5 7 . . . 3983 3985

4 8 12 . . . 7968

. . . . . . . . .

donde cada numero es igual a la suma de los dos que tiene encima (como es evidente,

cada fila consta de un numero menos que la anterior, y por lo tanto la ultima fila estara

formada por un unico numero), razonar que el ultimo numero es multiplo de 1993.

A29/3. Justificar razonadamente que en cualquier triangulo el diametro de la cir-

cunferencia inscrita no es mayor que el radio de la circunferencia circunscrita.

Page 67: COMPENDIUM OME - ToomatesEotvos de Hungría, que se iniciaron en 1894, precisamente durante la efervescencia de fin de siglo, consecuencia de la cual fue también el proceso iniciado

Segunda sesion. 27 de Febrero de 1993.

A29/4. Demostrar que todo numero primo p distinto de 2 y de 5 tiene infinitos

multiplos escritos solo con unos (es decir, de la forma 111 . . .1).

A29/5. Se dan 16 puntos que forman una cuadrıcula como en la figura:

• D

A • De ellos, se han destacado dos: A y D . Se pide fijar de todos los modos posibles otros

dos puntos B y C con la condicion de que las 6 distancias determinadas por los cuatro

puntos sean distintas. En este conjunto de cuaternas, estudiar:

1) Cuantas figuras de 4 puntos existen con las condiciones del enunciado.

2) Cuantas de ellas son geometricamente diferentes, es decir, no deducibles una de otra

por una transformacion de igualdad.

3) Si cada punto se designa por un par de enteros (Xi, Yi) , razonar que es constante la

suma |Xi −Xj |+ |Yi − Yj | , extendida a los seis pares AB , AC , AD , BC , BD , CD .

A29/6. Una maquina de juego de un casino tiene una pantalla en la que se ofrece

un esquema como el de la figura. Al comenzar el juego aparece una bola en el punto

S .

S

A

C

D

B

G

A cada impulso del jugador, la bola se mueve hasta

uno de los cırculos inmediatos, con la misma proba-

bilidad para cada uno de ellos. La partida acaba al

ocurrir el primero de los dos sucesos siguientes: (1)

La bola vuelve a S , y el jugador pierde. (2) La bola

llega a G , y entonces el jugador gana. Se pide la

probabilidad de que el jugador gane, y la duracion

media de las partidas.

Premiados: Alvaro Begue Aguado, Miguel Carrion Alvarez, Antonio Rojas Leon,

David Sevilla Gonzalez, Antonio Sanchez Esguevillas, David Castell Burgaleta.

Page 68: COMPENDIUM OME - ToomatesEotvos de Hungría, que se iniciaron en 1894, precisamente durante la efervescencia de fin de siglo, consecuencia de la cual fue también el proceso iniciado

1993-94 XXX Olimpiada Matematica

Espanola30

Primera sesion. 25 de Febrero de 1994.

A30/1. Demostrar que si entre los infinitos terminos de una progesion aritmetica de

numeros enteros hay un cuadrado perfecto, entonces infinitos terminos de la progresion

son cuadrados perfectos.

A30/2. Sea O.XY Z un triedro trirrectangulo de vertice O y aristas X , Y y Z .

Sobre la arista Z se fija un punto C tal que OC = c . Sobre X e Y ss consideran,

respectivamente, puntos variables P y Q de manera que OP + OQ sea una constante

dada k . Para cada par de puntos P y Q , los cuatro puntos O , C , P y Q determinan

una esfera, cuyo centro W se proyecta sobre el plano OXY . Razonar cual es el lugar

geometrico de esa proyeccion. Razonar tambien cual es el lugar geometrico de W .

A30/3. Una Oficina de Turismo va a realizar una encuesta sobre el numero de dias

soleados y de dias lluviosos al lo largo de un ano. Para ello recurre a seis regiones, que

le transmiten los datos de la tabla siguiente:

Region Sol o lluvia Inclasificable

A 336 29

B 321 44

C 335 30

D 343 22

E 329 36

F 330 35

La persona encargada de la encuesta, que tiene datos mas detallados, no es imparcial.

Se da cuenta de que, prescindiendo de una de las regiones, la observacion da un numero

de dias lluviosos que es la tercera parte del numero de dias de sol. Razonar cual es la

region de la que prescindira.

Page 69: COMPENDIUM OME - ToomatesEotvos de Hungría, que se iniciaron en 1894, precisamente durante la efervescencia de fin de siglo, consecuencia de la cual fue también el proceso iniciado

Segunda sesion. 26 de Febrero de 1994.

A30/4. El angulo A de un triangulo isosceles ABC mide 2/5 de recto, siendo los

angulos B y C iguales. La bisectriz del angulo C corta al lado opuesto en el punto

D . Calcular las medidas de los angulos del triangulo BCD . Expresar la medida a del

lado BC en funcion de la medida b del lado AC , sin que en la expresion aparezcan

razones trigonometricas.

A30/5. Con 21 fichas de damas, unas blancas y otras negras, se forma un rectangulo

3×7. Demostrar que siempre hay cuatro fichas del mismo color situadas en los vertices

de un rectangulo.

A30/6. Un polıgono convexo de n lados se descompone en m triangulos con inte-

riores disjuntos, de modo que cada lado de esos m triangulos, lo es tambien de otro

triangulo contiguo o del polıgono dado. Demostrar que m + n es par. Conocidos m

y n , hallar el numero de lados distintos que quedan en el interior del polıgono y el

numero de vertices distintos que quedan en ese interior.

Premiados: David Sevilla Gonzalez, Tomas Baeza Oliva, Miguel Catalina Gallego,

Alfonso Gracia Saz, Jeronimo Arenas Garcıa, Miguel A. Bermudez Carro.

Page 70: COMPENDIUM OME - ToomatesEotvos de Hungría, que se iniciaron en 1894, precisamente durante la efervescencia de fin de siglo, consecuencia de la cual fue también el proceso iniciado

1994-95 XXXI Olimpıada Matematica Espanola

Castellon31

Primera sesion. 24 de Febrero de 1995.

A31/1. Se consideran conjuntos A de cien numeros naturales distintos, que tengan

la propiedad de que si a , b , c son elementos cualesquiera (iguales o distintos) de A ,

existe un triangulo no obtusangulo cuyos lados miden a , b y c unidades.

Se denomina S(A) a la suma de los perımetros considerados en la definicion de A .

Calcular el valor mınimo de S(A) .

A31/2. Recortamos varios cırculos de papel (no necesariamente iguales) y los exten-

demos sobre una mesa de modo que haya algunos solapados (con parte interior comun),

pero de tal forma que no haya ningun cırculo dentro de otro.

Probar que es imposible ensamblar las piezas que resultan de recortar las partes no

solapadas y componer con ellas cırculos disjuntos.

A31/3. Por el baricentro G de un triangulo ABC se traza una recta que corta el

lado AB en P y el lado AC en Q . Demostrar que

PB

PA· QC

QA≤ 1

4.

Page 71: COMPENDIUM OME - ToomatesEotvos de Hungría, que se iniciaron en 1894, precisamente durante la efervescencia de fin de siglo, consecuencia de la cual fue también el proceso iniciado

Segunda sesion. 25 de Febrero de 1995.

A31/4. Hallar las soluciones enteras de la ecuacion p(x + y) = xy donde p es un

numero primo.

A31/5. Demostrar que en el caso de que las ecuaciones

x3 + mx − n = 0

nx3 − 2m2x2 − 5mnx − 2m3 − n2 = 0

(n = 0), tengan una raız comun, la primera tendra dos raıces iguales, y determinar

entonces las raıces de las dos ecuaciones en funcion de n .

A31/6. En la figura, AB es un segmento fijo y C un punto variable dentro de

el. Se construyen triangulos equilateros ACB′ y CBA′ de lados AC y CB en el

mismo semiplano definido por AB , y otro triangulo equilatero ABC′ de lado AB en

el semiplano opuesto. Demostrar:

a) Las rectas AA′ , BB′ y CC′ son concurrentes.

b) Si llamamos P al punto comun a las tres rectas del apartado a), hallar el lugar

geometrico de P cuando C varıa en el segmento AB .

c) Los centros A′′ , B′′ y C′′ de los tres triangulos

forman un triangulo equilatero.

d) Los punts A′′ , B′′ , C′′ y P son concıclicos.

A BC

A′

B′

C′

B′′

A′′

C′′

P

Premiados: Angel Paredes Galan, Jeronimo Arenas Garcıa, Luis Fabiani Bendicho,

Jaume Andreu Pascual, Alejandro Garcıa Gil, Ignacio Fernandez Galvan.

Page 72: COMPENDIUM OME - ToomatesEotvos de Hungría, que se iniciaron en 1894, precisamente durante la efervescencia de fin de siglo, consecuencia de la cual fue también el proceso iniciado

1995-96 XXXII Olimpıada Matematica Espanola

Tarragona32

Primera sesion. 22 de Febrero de 1996.

A32/1. Los numeros naturales a y b son tales que

a + 1b

+b + 1

a

es un entero. Demostrar que el maximo comun divisor de a y b no es mayor que√

a + b .

A32/2. Sea G el baricentro del triangulo ABC . Demostrar que si

AB + GC = AC + GB,

entonces el triangulo es isosceles.

A32/3. Sean a , b y c tres numeros reales. Se consideran las funciones

f(x) = ax2 + bx + c y g(x) = cx2 + bx + a.

Sabiendo que

|f(−1)| ≤ 1, |f(0)| ≤ 1, y |f(1)| ≤ 1,

probar que si −1 ≤ x ≤ 1, entonces |f(x)| ≤ 5/4 y |g(x)| ≤ 2.

Page 73: COMPENDIUM OME - ToomatesEotvos de Hungría, que se iniciaron en 1894, precisamente durante la efervescencia de fin de siglo, consecuencia de la cual fue también el proceso iniciado

Segunda sesion. 23 de Febrero de 1996.

A32/4. Discutir la existencia de soluciones reales x de la ecuacion

√x2 − p + 2

√x2 − 1 = x

segun los valores reales del parametro p , y resolverla en aquellos casos en que tenga

solucion.

A32/5. En Port Aventura hay 16 agentes secretos. Cada uno de ellos vigila a alguno

de sus colegas. Se sabe que si el agente A vigila al agente B , entonces B no vigila

a A . Ademas, 10 agentes cualesquiera pueden ser numerados de forma que el primero

vigila al segundo, este vigila al tercero, . . . , el decimo vigila al primero. Demostrar que

tambien se pueden numerar de esa manera 11 agentes cualesquiera.

A32/6. La figura adjunta se compone de seis pentagonos regulares de lado un metro.

Se dobla por las lıneas de puntos hasta que coincidan las aristas no punteadas que

confluyen en cada vertice. ¿Que volumen de agua cabe en el recipiente ası formado?

Premiados: Sergi Elizalde Torrent, Tomas Palacios Gutierrez, Fernando Rambla

Blanco, Antonio Jara de las Heras, Patricia Sebastian Celorrio, Vıctor Martınez de

Albeniz Margalef.

Page 74: COMPENDIUM OME - ToomatesEotvos de Hungría, que se iniciaron en 1894, precisamente durante la efervescencia de fin de siglo, consecuencia de la cual fue también el proceso iniciado

1996-97 XXXIII Olimpıada Matematica Espanola

Valencia33

Primera sesion. 7 de Marzo de 1997.

A33/1. Calcular la suma de los cuadrados de los cien primeros terminos de una

progresion aritmetica, sabiendo que la suma de ellos vale −1, y la suma de los terminos

de lugar par vale +1.

A33/2. Un cuadrado de lado 5 se divide en 25 cuadrados unidad por medio de rectas

paralelas a los lados. Sea A el conjunto de los 16 puntos interiores, que son vertices de

los cuadrados unidad, pero que no estan en los lados del cuadrado inicial.

¿Cual es el mayor numero de puntos de A que se pueden elegir de manera que tres

cualesquiera de ellos no sean vertices de un triangulo rectangulo isosceles?

A33/3. Se consideran las parabolas y = x2 + px + q que cortan a los ejes de coor-

denadas en tres puntos diferentes, por los que se traza una circunferencia. Demostrar

que todas las circunferencias trazadas al variar p y q en R pasan por un punto fijo,

que se determinara.

Page 75: COMPENDIUM OME - ToomatesEotvos de Hungría, que se iniciaron en 1894, precisamente durante la efervescencia de fin de siglo, consecuencia de la cual fue también el proceso iniciado

Segunda sesion. 8 de Marzo de 1997.

A33/4. Sea p un numero primo. Determinar todos los enteros k ∈ Z tales que√

k2 − pk es un entero positivo.

A33/5. Demostrar que en un cuadrilatero convexo de area unidad, la suma de las

longitudes de todos los lados y diagonales no es menor que 2(2 +√

2).

A33/6. Para dar una vuelta completa en un coche a un circuito circular, la cantidad

exacta de gasolina esta distribuida en depositos fijos situados en n puntos distintos

cualesquiera del circuito. Inicialmente el deposito del coche esta vacıo. Demostrar que

cualquiera que sea la distribucion del combustible en los depositos, siempre existe un

punto de partida de forma que se puede dar la vuelta completa.

Aclaraciones:

Se supone que el consumo es uniforme y proporcional a la distancia.

El deposito del coche tiene capacidad suficiente para contener toda la gasolina.

Premiados: Anatoli Segura Velez, Miguel Lobo Lopez, Mario Andres Montes Garcıa,

Max Bernstein Obiols, Joseba Villate Bejarano, Xavier Perez Gimenez.

Page 76: COMPENDIUM OME - ToomatesEotvos de Hungría, que se iniciaron en 1894, precisamente durante la efervescencia de fin de siglo, consecuencia de la cual fue también el proceso iniciado

1997-98 XXXIV Olimpıada Matematica Espanola

Tarazona34

Primera sesion. 13 de Marzo de 1998.

A34/1. Un cuadrado ABCD de centro O y lado gira un angulo α en torno a

O . Hallar el area comun a los dos cuadrados.

O

AB

C D

A′

B′

C′

D′

A34/2. Hallar todos los numeros naturales de cuatro cifras, escritos en base 10, que

sean iguales al cubo de la suma de sus cifras.

A34/3. Se considera un triangulo ABC y su circunferencia circunscrita. Si D y E

son puntos sobre el lado BC tales que AD y AE son, respectivamente, paralelas a las

tangentes en C y B a la circunferencia circunscrita, demostrar que

BE

CD=

AB2

AC2.

Page 77: COMPENDIUM OME - ToomatesEotvos de Hungría, que se iniciaron en 1894, precisamente durante la efervescencia de fin de siglo, consecuencia de la cual fue también el proceso iniciado

Segunda sesion. 14 de Marzo de 1998.

A34/4. Hallar las tangentes de los angulos de un triangulo sabiendo que son numeros

enteros positivos.

A34/5. Hallar todas las funciones f : N → N estrictamente crecientes y tales que

f(n + f(n)

)= 2f(n)

para n = 1, 2, 3, . . .

A34/6. Determinar los valores de n para los que es posible construir un cuadrado

n × n ensamblando piezas del tipo

Premiados: Mario Andres Montes Garcıa, Ramon Jose Aliaga Varea, David Martın

Clavo, Marıa Pe Pereira, Beatriz Sanz Merino, Jaime Vinuesa del Rio.

Page 78: COMPENDIUM OME - ToomatesEotvos de Hungría, que se iniciaron en 1894, precisamente durante la efervescencia de fin de siglo, consecuencia de la cual fue también el proceso iniciado

1998-99 XXXV Olimpıada Matematica Espanola

Granada35

Primera sesion. 12 de Marzo de 1999.

A35/1. Las recta t y t′ tangentes a la parabola de ecuacion y = x2 en los puntos

A y B se cortan en el punto C . La mediana del triangulo ABC correspondiente al

vertice C tiene longitud m . Determinar el area del triangulo ABC en funcion de m .

A35/2. Probar que existe una sucesion de enteros positivos a1, a2, . . . , an, . . . tal

que

a21 + a2

2 + · · ·+ a2n

es un cuadrado perfecto para todo entero positivo n .

A35/3. Sobre un tablero en forma de triangulo equilatero con un numero par de

filas n (tal como se indica en la figura), se juega un solitario.

Sobre cada casilla se coloca una ficha. Cada ficha es blanca por un lado y negra por el

otro. Inicialmente, solo una ficha, que esta situada en un vertice, tiene la cara negra

hacia arriba; las demas fichas tienen la cara blanca hacia arriba. En cada movimiento

del juego se retira solamente una ficha negra del tablero y se da la vuelta a cada una

de las fichas que ocupa una casilla vecina. (Casillas vecinas son las que estan unidas

por un segmento.)

Despues de varios movimientos ¿sera posible quitar todas las fichas del tablero?

Page 79: COMPENDIUM OME - ToomatesEotvos de Hungría, que se iniciaron en 1894, precisamente durante la efervescencia de fin de siglo, consecuencia de la cual fue también el proceso iniciado

Segunda sesion. 13 de Marzo de 1999.

A35/4. Una caja contiene 900 tarjetas numeradas del 100 al 999. Se sacan al azar

(sin reposicion) tarjetas de la caja y se anota la suma de los dıgitos de cada tarjeta

extraıda. ¿Cual es la menor cantidad de tarjetas que se deben sacar, para garantizar

que al menos tres de esas sumas sean iguales?

A35/5. El baricentro del triangulo ABC es G . Denotemos por ga , gb , gc las dis-

tancias desde G a los lados a , b , c , respectivamente. Sea r el radio de la circunferencia

inscrita.

a) Probar que

ga ≥ 2r

3, gb ≥

2r

3, gc ≥

2r

3.

b) Probar quega + gb + gc

r≥ 3.

A35/6. Se divide el plano en un numero finito de regiones n mediante tres familias

de rectas paralelas. No hay tres rectas que pasen por el mismo punto. ¿Cual es el

mınimo numero de rectas necesarias para que n > 1999?

Premiados: Ramon Aliaga Varea, Andres Tallos Tanarro, Enrique Vallejo Gutierrez,

Alvaro Navarro Tobar, Javier Mugica de Ribera, Nestor Sancho Bejarano.

Page 80: COMPENDIUM OME - ToomatesEotvos de Hungría, que se iniciaron en 1894, precisamente durante la efervescencia de fin de siglo, consecuencia de la cual fue también el proceso iniciado

1999-2000 XXXVI Olimpıada Matematica Espanola

Palma de Mallorca36

Primera sesion. 30 de Marzo de 2000.

A36/1. Sean los polinomios:

P (x) = x4 + ax3 + bx2 + cx + 1;

Q(x) = x4 + cx3 + bx2 + ax + 1.

Halla las condiciones que deben cumplir los parametros reales a , b y c , (a = c) , para

que P (x) y Q(x) tangan dos raıces comunes, y resuelve en ese caso las ecuaciones

P (x) = 0; Q(x) = 0.

A36/2. La figura muestra un plano con calles que delimitan 12 manzanas cuadradas.

Una persona P va desde A hasta B y otra Q desde B hasta A . Ambas parten a la

vez siguiendo caminos de longitud mınima con la misma velocidad constante. En cada

punto con dos posibles direcciones a tomar, ambas tienen la misma probabilidad.

Halla la probabilidad de que P y Q se crucen.

A

B

A36/3. Dos circunferencias C1 y C2 de radios r1 y r2 se cortan en los puntos A

y B . Por B se traza una recta variable que corta de nuevo a C1 y C2 en dos puntos

que llamaremos Pr y Qr , respectivamente.

Demuestra la siguiente propiedad: Existe un punto M , que depende solo de C1 y C2 ,

tal que la mediatriz del segmento PrQr pasa por M .

Page 81: COMPENDIUM OME - ToomatesEotvos de Hungría, que se iniciaron en 1894, precisamente durante la efervescencia de fin de siglo, consecuencia de la cual fue también el proceso iniciado

Segunda sesion. 31 de Marzo de 2000.

A36/4. Encuentra el mayor numero entero N que cumpla las siguientes condiciones:

a) E(N/3) tiene sus tres cifras iguales.

b) E(N/3) es suma de numeros naturales consecutivos comenzando en 1, es decir,

existe un natural n tal que

E(N/3) = 1 + 2 + 3 + · · ·+ n.

Nota: E(x) es la parte entera de x .

A36/5. Tomemos cuatro puntos situados en el interior o el borde de un cuadrado

de lado 1. Demuestra que al menos dos de ellos estan a una distancia menor o igual

que 1.

A36/6. Demuestra que no existe ninguna funcion f : N → N que cumpla

f(f(n)

)= n + 1.

Premiados: Carlos Gomez Rodrıguez, Luis Emilio Garcıa Martınez, Alberto Suarez

Real, Jose Marıa Cantarero Lopez, Manuel Perez Molina, Roberto Rubio Nunez.

Page 82: COMPENDIUM OME - ToomatesEotvos de Hungría, que se iniciaron en 1894, precisamente durante la efervescencia de fin de siglo, consecuencia de la cual fue también el proceso iniciado

2000-01 XXXVII Olimpıada Matematica Espanola

Murcia37

Primera sesion. 23 de Marzo de 2001, de 16.00 a 19.30.

A37/1. Probar que la grafica del polinomio P (x) es simetrica respecto del punto

A(a, b) si y solo si existe un polinomio Q(x) tal que

P (x) = b + (x − a)Q((x − a)2)

).

A37/2. Sea P un punto en el interior del triangulo ABC , de modo que el triangulo

ABP cumple

AP = BP.

Sobre cada uno de los otros lados de ABC se construyen exteriormente triangulos BQC

y CRA , ambos semejantes al triangulo ABP cumpliendo:

BQ = QC y CR = RA.

Probar que los puntos P , Q , C y R o estan alineados o son los vertices de un paralel-

ogramo.

A37/3. Se tienen cinco segmentos de longitudes a1 , a2 , a3 , a4 y a5 tales que con

tres cualesquiera de ellos es posible construir un triangulo.

Demostrar que al menos uno de esos triangulos tiene todos los angulos agudos.

Page 83: COMPENDIUM OME - ToomatesEotvos de Hungría, que se iniciaron en 1894, precisamente durante la efervescencia de fin de siglo, consecuencia de la cual fue también el proceso iniciado

Segunda sesion. 24 de Marzo de 2001, de 9.30 a 13.00

A37/4. Los numeros enteros desde 1 hasta 9 se distribuyen en las casillas de una

tabla 3 × 3. Despues se suman seis numeros de tres cifras: los tres que se leen en filas

de izquierda a derecha y los tres que se leen en columnas de arriba abajo.

¿Hay alguna disposicion para la cual el valor de esa suma sea 2001?

A37/5. Sea ABCD un cuadrilatero inscrito en una circunferencia de radio 1 de

modo que AB es un diametro y el cuadrilatero admite circunferencia inscrita.

Probar que CD ≤ 2√

5 − 4.

A37/6. Determinar la funcion f : N → N (siendo N = 1, 2, 3, . . . el conjunto de

los numeros naturales) que cumple, para cualesquiera s, n ∈ N , las siguientes condi-

ciones:

f(1) = f(2s) = 1 y si n < 2s , entonces f(2s + n) = f(n) + 1.

Calcular el valor maximo de de f(n) cuando n ≤ 2001.

Hallar el menor numero natural n tal que f(n) = 2001.

Premiados: Javier Coppola Rodrıguez, Martı Prats Soler, Luis Hernandez Corbato,

Sergio Millan Lopez, Ignacio Cascudo Pueyo, Miquel Oliu Barton.

Page 84: COMPENDIUM OME - ToomatesEotvos de Hungría, que se iniciaron en 1894, precisamente durante la efervescencia de fin de siglo, consecuencia de la cual fue también el proceso iniciado

2001-02 XXXVIII Olimpıada Matematica Espanola

Logrono38

Primera sesion. 5 de Abril de 2002, de 16.00 a 19.30.

A38/1. Hallar todos los polinomios P (t) de una variable, que cumplen

P(x2 − y2

)= P (x + y)P (x − y)

para todos los numeros reales x e y .

A38/2.

En un triangulo ABC , A′ es el pie de la altura relativa al vertice A , y H el ortocentro.

a) Dado un numero real positivo k =AA′

HA′ , hallar la relacion entre los angulos B y C

en funcion de k .

b) Si B y C son fijos, hallar el lugar geometrico del vertice A para cada valor de k .

B A′ C

A

H

A38/3. La funcion g se define sobre los numeros naturales y satisface las condiciones

g(2) = 1

g(2n) = g(n)

g(2n + 1) = g(2n) + 1.

Sea n un numero natural tal que 1 ≤ n ≤ 2002. Calcula el valor maximo M de g(n) .

Calcula tambien cuantos valores de n satisfacen la condicion g(n) = M .

Page 85: COMPENDIUM OME - ToomatesEotvos de Hungría, que se iniciaron en 1894, precisamente durante la efervescencia de fin de siglo, consecuencia de la cual fue también el proceso iniciado

Segunda sesion. 6 de Abril de 2002, de 9.30 a 13.00

A38/4. Sea n un numero natural, y m el que resulta al escribir en orden inverso las

cifras de n . Determinar, si existen, los numeros de tres cifras que cumplen 2m+S = n ,

siendo S la suma de las cifras de n .

A38/5. Se consideran 2002 segmentos en el plano, tales que la suma de sus longituds

es la unidad. Probar que existe una recta r tal que la suma de las longitudes de las

proyecciones de los 2002 segmentos dados sobre r es menor que 2/3.

A38/6. En un polıgono regular H de 6n + 1 lados (n es un entero positivo),

pintamos r vertices de color rojo, y el resto de azul. Demostrar que el numero de

triangulos isosceles que tienen sus tres vertices del mismo color no depende del modo

de distribuir los colores en los vertices de H .

Premiados: Daniel Rodrigo Lopez, Luis Hernandez Corbato, Sergio Millan Lopez,

David Garcıa Soriano, Susana Ladra Gonzalez, Jose Miguel Manzano Prego.

Page 86: COMPENDIUM OME - ToomatesEotvos de Hungría, que se iniciaron en 1894, precisamente durante la efervescencia de fin de siglo, consecuencia de la cual fue también el proceso iniciado

2002-03 XXXIX Olimpıada Matematica Espanola

La Laguna39

Primera sesion. 3 de Marzo de 2003, de 9.00 a 13.00.

A39/1. Probar que para cualquier primo p distinto de 2 y 5 existe un multiplo de

p cuyas cifras son todas nueves. Por ejemplo, si p = 13, es 999999 = 13 · 76923.

A39/2. ¿Existe algun conjunto finito de numeros reales M que contenga al menos

dos elementos distintos y que cumpla la propiedad de que para dos numeros a , b

cualesquiera de M , el numero 2a − b2 sea tambien un elemento de M ?

A39/3. Las alturas del triangulo ABC se cortan en el punto H . Se sabe que

AB = CH . Determinar el valor del angulo BCA .

Page 87: COMPENDIUM OME - ToomatesEotvos de Hungría, que se iniciaron en 1894, precisamente durante la efervescencia de fin de siglo, consecuencia de la cual fue también el proceso iniciado

Segunda sesion. 4 de Marzo de 2003, de 9.00 a 13.00

A39/4. Sea x un numero real tal que x3 + 2x2 + 10x = 20. Demostrar que tanto

x como x2 son irracionales.

A39/5. ¿Cuales son las posibles areas de un hexagono convexo con todos los angulos

iguales y cuyos lados miden 1, 2, 3, 4, 5 y 6, en algun orden?

A39/6. Ensartamos 2n bolas blancas y 2n bolas negras formando una cadena

abierta. Demuestra que, se haga en el orden en que se haga, siempre es posible cortar

un segmento de cadena que contenga exactamente n bolas blancas y n bolas negras.

Premiados: Daniel Rodrigo Lopez, Luis Hernandez Corbato, Mohamed Blanca Ruiz,

Vıctor Gonzalez Alonso, Javier Gomez Serrano, Maite Pena Alcaraz.

Page 88: COMPENDIUM OME - ToomatesEotvos de Hungría, que se iniciaron en 1894, precisamente durante la efervescencia de fin de siglo, consecuencia de la cual fue también el proceso iniciado

2003-04 XL Olimpıada Matematica Espanola

Ciudad Real40

Primera sesion. 26 de Marzo de 2004, de 9.00 a 13.00.

A40/1. Tenemos un conjunto de 221 numeros reales cuya suma es 110721. Los

disponemos formando una tabla rectangular de modo que todas las filas y la primera

y ultima columnas son progresiones aritmeticas de mas de un elemento. Probar que la

suma de los elementos de las cuatro esquinas vale 2004.

A40/2. ABCD es un cuadrilatero cualquiera, P

y Q los puntos medios de las diagonales BD y AC ,

respectivamente. Las paralelas por P y Q a la otra

diagonal se cortan en O . Si unimos O con los cu-

atro puntos medios de los lados X , Y , Z y T , se

forman cuatro cuadrilateros OXBY , OY CZ , OZDT

y OTAX .

Probar que los cuatro cuadrilateros tienen la misma

area.

B

A

D

C

Y

XT

Z

P

Q O

A40/3. Se representa por Z el conjunto de todos los enteros. Hallar todas las

funciones f : Z → Z tales que, para cualesquiera x, y enteros se cumple

f(x + f(y)

)= f(x) − y.

Page 89: COMPENDIUM OME - ToomatesEotvos de Hungría, que se iniciaron en 1894, precisamente durante la efervescencia de fin de siglo, consecuencia de la cual fue también el proceso iniciado

Segunda sesion. 27 de Marzo de 2004, de 9.00 a 13.00

A40/4. ¿Existe alguna potencia de 2, que al escribirla en el sistema decimal tenga

todos sus dıgitos distintos de cero y sea posible reordenar los mismos para formar con

ellos otra potencia de 2 distinta? Justificar la respuesta.

A40/5. Demostrar que la condicion necesaria y suficiente para que, en el triangulo

ABC , la mediana desde B sea dividida en tres partes iguales por la circunferencia

inscrita en el triangulo, esa

5=

b

10=

c

13.

A40/6. Colocamos, formando una circunferencia, 2004 fichas bicolores: blancas por

una cara y negras por la otra. Un movimiento consiste en elegir una ficha con la cara

negra hacia arriba y dar la vuelta a tres fichas: la elegida, la de su derecha, y la de

su izquierda. Supongamos que inicialmente hay una sola ficha con la cara negra hacia

arriba. ¿Sera posible, repitiendo el movimiento descrito, conseguir que todas las fichas

tengan la cara blanca hacia arriba? ¿Y si tuvieramos 2003 fichas, entre las cuales

exactamente una tiene al comienzo la cara negra hacia arriba?

Premiados: Joaquim Serra Montolı, Maite Pena Alcaraz, Elisa Lorenzo Garcıa, Miguel

Teixido Roman, Francisco J. Hernandez Heras, M. Isabel Cordero Marcos.

Page 90: COMPENDIUM OME - ToomatesEotvos de Hungría, que se iniciaron en 1894, precisamente durante la efervescencia de fin de siglo, consecuencia de la cual fue también el proceso iniciado

S E OME 1. Problema 1. Sugerencia

a) Expresar la parte imaginaria de las raıces de la ecuacion en funcion de a.

b) Buscar alguna relacion entre la parte real e imaginaria de las raıces, que no dependade a.

Page 91: COMPENDIUM OME - ToomatesEotvos de Hungría, que se iniciaron en 1894, precisamente durante la efervescencia de fin de siglo, consecuencia de la cual fue también el proceso iniciado

S E OME 1. Problema 2. Sugerencia

Expresar la recta de pendiente 0.14 en funcion de x y P .

Page 92: COMPENDIUM OME - ToomatesEotvos de Hungría, que se iniciaron en 1894, precisamente durante la efervescencia de fin de siglo, consecuencia de la cual fue también el proceso iniciado

S E OME 1. Problema 3. Sugerencia

a) Hay (n

2

)− n

diagonales y en cada vertice concurren n − 3.

b) Cuatro vertices determinan siempre un punto interior.

Page 93: COMPENDIUM OME - ToomatesEotvos de Hungría, que se iniciaron en 1894, precisamente durante la efervescencia de fin de siglo, consecuencia de la cual fue también el proceso iniciado

S E OME 1. Problema 4. Sugerencia

Construir un triangulo equilatero a partir de dicho segmento.

Page 94: COMPENDIUM OME - ToomatesEotvos de Hungría, que se iniciaron en 1894, precisamente durante la efervescencia de fin de siglo, consecuencia de la cual fue también el proceso iniciado

S E OME 1. Problema 5. Sugerencia

Hay cinco conjuntos de triangulos.

Page 95: COMPENDIUM OME - ToomatesEotvos de Hungría, que se iniciaron en 1894, precisamente durante la efervescencia de fin de siglo, consecuencia de la cual fue también el proceso iniciado

S E OME 1. Problema 6. Sugerencia

La funcion g(x) = |f(x)−k| consiste en desplazar la grafica de f(x) segun el vector (0,−k)y ademas simetrizar respecto del eje OX la parte de la grafica que se encuentra bajo esteeje.

Page 96: COMPENDIUM OME - ToomatesEotvos de Hungría, que se iniciaron en 1894, precisamente durante la efervescencia de fin de siglo, consecuencia de la cual fue también el proceso iniciado

S E OME 1. Problema 7. Sugerencia

Considerar el movimiento como una permutacion de los n elementos y escribirla con lanotacion habitual de ciclos.

Page 97: COMPENDIUM OME - ToomatesEotvos de Hungría, que se iniciaron en 1894, precisamente durante la efervescencia de fin de siglo, consecuencia de la cual fue también el proceso iniciado

S E OME 1. Problema 8. Sugerencia

Expresar la distancia del punto medio del segmento a la recta horizontal en funcion de ay α (angulo formado por dicho segmento y la horizontal) y encontrar el maximo de estafuncion.

Page 98: COMPENDIUM OME - ToomatesEotvos de Hungría, que se iniciaron en 1894, precisamente durante la efervescencia de fin de siglo, consecuencia de la cual fue también el proceso iniciado

S E OME 2. Problema 1. Sugerencia

Sea A′B′C′ el triangulo transformado y P , Q los puntos de corte de A′B′ con BC y BA,respectivamente. Demostrar que que el triangulo BPQ tiene angulos de 90, 60 y 30 yperımetro 4

√3.

Page 99: COMPENDIUM OME - ToomatesEotvos de Hungría, que se iniciaron en 1894, precisamente durante la efervescencia de fin de siglo, consecuencia de la cual fue también el proceso iniciado

S E OME 2. Problema 2. Sugerencia

Probar que si la cifra central es n > 2, existen n(n−1) numeros que cumplen las condicionesiniciales y (n − 1)2 numeros que cumplen las condiciones con cifras no repetidas.

Page 100: COMPENDIUM OME - ToomatesEotvos de Hungría, que se iniciaron en 1894, precisamente durante la efervescencia de fin de siglo, consecuencia de la cual fue también el proceso iniciado

S E OME 2. Problema 3. Sugerencia

Considerar la longitud de la pista como suma de circunferencias concentricas cuyos radiosestan en progresion aritmetica.

Page 101: COMPENDIUM OME - ToomatesEotvos de Hungría, que se iniciaron en 1894, precisamente durante la efervescencia de fin de siglo, consecuencia de la cual fue también el proceso iniciado

S E OME 2. Problema 4. Sugerencia

Interpretar geometricamente el problema. Tomando un punto P de la circunferencia deradio 1, centrada en el origen y siendo x el angulo que forma el semieje positivo OY conOP (medido en sentido antihorario).

Page 102: COMPENDIUM OME - ToomatesEotvos de Hungría, que se iniciaron en 1894, precisamente durante la efervescencia de fin de siglo, consecuencia de la cual fue también el proceso iniciado

S E OME 2. Problema 5. Sugerencia

Probar que con b = 0 no hay discrepancia y que en caso contrario la propiedad no puedeaplicarse porque quedarıa el denominador igual a cero.

Page 103: COMPENDIUM OME - ToomatesEotvos de Hungría, que se iniciaron en 1894, precisamente durante la efervescencia de fin de siglo, consecuencia de la cual fue también el proceso iniciado

S E OME 2. Problema 6. Sugerencia

Buscar los triangulos rectangulos adecuados y aplicar el teorema de Pitagoras.

Page 104: COMPENDIUM OME - ToomatesEotvos de Hungría, que se iniciaron en 1894, precisamente durante la efervescencia de fin de siglo, consecuencia de la cual fue también el proceso iniciado

S E OME 2. Problema 7. Sugerencia

Si x es el radio de la base pequena del tronco de cono, probar que la masa en funcion dex es

M(x) =(

13πr3m − 1

3πx3m

)d + πx2p

Page 105: COMPENDIUM OME - ToomatesEotvos de Hungría, que se iniciaron en 1894, precisamente durante la efervescencia de fin de siglo, consecuencia de la cual fue también el proceso iniciado

S E OME 2. Problema 8. Sugerencia

a) Buscar triangulos semejantes para demostrar que r2 = r1d − r1

d + r1.

b) Demostrar mediante induccion rn = r1

(d − r1

d + r2

)n−1

.

c) Esta es la suma de los terminos de una progresion geometrica.

Page 106: COMPENDIUM OME - ToomatesEotvos de Hungría, que se iniciaron en 1894, precisamente durante la efervescencia de fin de siglo, consecuencia de la cual fue también el proceso iniciado

S E OME 3. Problema 1. Sugerencia

Se debe resolver un sistema de tres variables y dos incognitas, teniendo en cuenta que lasvariables solo pueden tomar valores naturales y que la variable correspondiente al numerode productos de mayor precio debe ser la maxima posible.

Page 107: COMPENDIUM OME - ToomatesEotvos de Hungría, que se iniciaron en 1894, precisamente durante la efervescencia de fin de siglo, consecuencia de la cual fue también el proceso iniciado

S E OME 3. Problema 2. Sugerencia

El numero xyz en la base n, representa el numero x n2 + y n + z en base 10 y si n ≤ 10,cada una de sus cifras puede tomar valores entre 0 y n − 1.

Page 108: COMPENDIUM OME - ToomatesEotvos de Hungría, que se iniciaron en 1894, precisamente durante la efervescencia de fin de siglo, consecuencia de la cual fue también el proceso iniciado

S E OME 3. Problema 3. Sugerencia

Sea ABCDE el pentagono exterior y A′B′C′D′E′ el pentagono interior, nombrando losvertices en sentido horario y siendo A′ la interseccion de AC y BE. Probar que lostriangulos A′BA y CDE son semejantes y obtener alguna identidad que permita deducirel valor de la diagonal del pentagono en funcion del lado.

Page 109: COMPENDIUM OME - ToomatesEotvos de Hungría, que se iniciaron en 1894, precisamente durante la efervescencia de fin de siglo, consecuencia de la cual fue también el proceso iniciado

S E OME 3. Problema 4. Sugerencia

Expresar la funcion del coste de instalacion en funcion de x, siendo x la distancia entre elpunto M y el techo. Encontrar el mınimo (o los mınimos), distinguiendo los casos p < 2qy p ≥ 2q.

Page 110: COMPENDIUM OME - ToomatesEotvos de Hungría, que se iniciaron en 1894, precisamente durante la efervescencia de fin de siglo, consecuencia de la cual fue también el proceso iniciado

S E OME 3. Problema 5. Sugerencia

a) Aplicar el teorema del coseno a los triangulos AMB, AMC y de igual forma a lostriangulos ANC, ANB.

b) Los dos triangulos comparten una de sus alturas. M ′N ′ es un diametro de dichacircunferencia.

Page 111: COMPENDIUM OME - ToomatesEotvos de Hungría, que se iniciaron en 1894, precisamente durante la efervescencia de fin de siglo, consecuencia de la cual fue también el proceso iniciado

S E OME 3. Problema 6. Sugerencia

Si P (k) denota la probabilidad de que nazcan k chicos y 5− k chicas, la probabilidad quenos piden es P (2) + P (3) + P (4).

Page 112: COMPENDIUM OME - ToomatesEotvos de Hungría, que se iniciaron en 1894, precisamente durante la efervescencia de fin de siglo, consecuencia de la cual fue también el proceso iniciado

S E OME 3. Problema 7. Sugerencia

Resolver un sistema de ecuaciones cuyas variable sean la razon de la progresion y el terminocentral.

Page 113: COMPENDIUM OME - ToomatesEotvos de Hungría, que se iniciaron en 1894, precisamente durante la efervescencia de fin de siglo, consecuencia de la cual fue también el proceso iniciado

S E OME 3. Problema 8. Sugerencia

Demostrar que y(3) = 1, y′(3) = 1/4, y′′(3) = 0. A partir de esto plantear y resolver unsistema.

Page 114: COMPENDIUM OME - ToomatesEotvos de Hungría, que se iniciaron en 1894, precisamente durante la efervescencia de fin de siglo, consecuencia de la cual fue también el proceso iniciado

S E OME 4. Problema 1. Sugerencia

Podremos asegurar que y sera estrictamente creciente en un intervalo de valores de x si

i) 2x − x2 es estrictamente creciente en dicho intervalo.

ii) −8 ≤ 2x − x2 ≤ 8 para todo x de dicho intervalo.

Page 115: COMPENDIUM OME - ToomatesEotvos de Hungría, que se iniciaron en 1894, precisamente durante la efervescencia de fin de siglo, consecuencia de la cual fue también el proceso iniciado

S E OME 4. Problema 2. Sugerencia

Probar que el centro de inversion debe pertenecer a las circunferencias que tienen pordiametro AC y BD, respectivamente.

Page 116: COMPENDIUM OME - ToomatesEotvos de Hungría, que se iniciaron en 1894, precisamente durante la efervescencia de fin de siglo, consecuencia de la cual fue también el proceso iniciado

S E OME 4. Problema 3. Sugerencia

Representar mediante franjas las trayectorias de los coches que circulan en ambos sentidosy determinar durante que intervalo de tiempo no circulan coches a 400 m del primersemaforo.

Page 117: COMPENDIUM OME - ToomatesEotvos de Hungría, que se iniciaron en 1894, precisamente durante la efervescencia de fin de siglo, consecuencia de la cual fue también el proceso iniciado

S E OME 4. Problema 4. Sugerencia

El procedimiento sera medir la distancia entre el fondo de la botella y el nivel del vino;primero con la botella apoyada sobre su base y despues con la botella tambien vertical,pero invertida.

Page 118: COMPENDIUM OME - ToomatesEotvos de Hungría, que se iniciaron en 1894, precisamente durante la efervescencia de fin de siglo, consecuencia de la cual fue también el proceso iniciado

S E OME 4. Problema 5. Sugerencia

a) Demostrad que todos los arcos AkAk+1 son iguales.

b) Demostrad que si L es la longitud de la quebrada, AB = L cos α.

Page 119: COMPENDIUM OME - ToomatesEotvos de Hungría, que se iniciaron en 1894, precisamente durante la efervescencia de fin de siglo, consecuencia de la cual fue también el proceso iniciado

S E OME 4. Problema 6. Sugerencia

Encontrar el inverso de cada uno de los segmentos o semirrectas y determinar la regiontransformada correspondiente a cada una de las regiones iniciales.

Page 120: COMPENDIUM OME - ToomatesEotvos de Hungría, que se iniciaron en 1894, precisamente durante la efervescencia de fin de siglo, consecuencia de la cual fue también el proceso iniciado

S E OME 4. Problema 7. Sugerencia

Determinar la funcion correspondiente a la cantidad de coches que atraviesan un puntode control, en funcion de la velocidad de dichos coches y suponiendo un tiempo t fijado.Hallar el maximo de dicha funcion.

Page 121: COMPENDIUM OME - ToomatesEotvos de Hungría, que se iniciaron en 1894, precisamente durante la efervescencia de fin de siglo, consecuencia de la cual fue también el proceso iniciado

S E OME 4. Problema 8. Sugerencia

El nuevo algoritmo es practicamente identico al anterior, solo debe calcular el valor delpolinomio de coeficientes (n − i) ai para x = b.

Page 122: COMPENDIUM OME - ToomatesEotvos de Hungría, que se iniciaron en 1894, precisamente durante la efervescencia de fin de siglo, consecuencia de la cual fue también el proceso iniciado

S E OME 5. Problema 1. Sugerencia

Expresar la altura del hielo a las 0 h y a las 4 h en funcion de k y t, que seran respecti-vamente, la constante de crecimiento del hielo y el tiempo transcurrido desde que el aguaempezo a helarse y las 0 h.

Page 123: COMPENDIUM OME - ToomatesEotvos de Hungría, que se iniciaron en 1894, precisamente durante la efervescencia de fin de siglo, consecuencia de la cual fue también el proceso iniciado

S E OME 5. Problema 2. Sugerencia

Si dos sucesiones xn, yn tienden a un mismo valor k y las sucesiones f(xn), f(yn)no, entonces f no es continua en k. Para declarar que no puede decidirse si una funcioncon unas determinadas propiedades es continua en un punto o no, hay que encontrar dosfunciones que cumplan las condiciones, una de ellas continua en ese punto y otra no.

Page 124: COMPENDIUM OME - ToomatesEotvos de Hungría, que se iniciaron en 1894, precisamente durante la efervescencia de fin de siglo, consecuencia de la cual fue también el proceso iniciado

S E OME 5. Problema 3. Sugerencia

Recordar que dicha figura debe dibujarse a trozos. Son puntos importantes en la con-struccion, aquellos puntos interiores al cuadrado que uniendolos con alguna pareja devertices del mismo, forman un triangulo equilatero.

Page 125: COMPENDIUM OME - ToomatesEotvos de Hungría, que se iniciaron en 1894, precisamente durante la efervescencia de fin de siglo, consecuencia de la cual fue también el proceso iniciado

S E OME 5. Problema 4. Sugerencia

Si S(x, h) es la superficie del triangulo isosceles que resulta de cortar la figura por el puntox del segmento AB mediante un plano perpendicular a este mismo segmento; entonces

V = 2∫ r

0

S(x, h) dx.

Page 126: COMPENDIUM OME - ToomatesEotvos de Hungría, que se iniciaron en 1894, precisamente durante la efervescencia de fin de siglo, consecuencia de la cual fue también el proceso iniciado

S E OME 5. Problema 5. Sugerencia

Recordar que los puntos medios de los segmentos de paralelas a un lado del triangulo seencuentran sobre la mediana relativa a ese lado.

Page 127: COMPENDIUM OME - ToomatesEotvos de Hungría, que se iniciaron en 1894, precisamente durante la efervescencia de fin de siglo, consecuencia de la cual fue también el proceso iniciado

S E OME 5. Problema 6. Sugerencia

La perpendicular por el circuncentro de un triangulo coincide con el lugar geometrico de lospuntos que equidistan de los vertices. Los tetraedros con aristas sobre los ejes cartesianosson muy utiles para facilitar calculos.

Page 128: COMPENDIUM OME - ToomatesEotvos de Hungría, que se iniciaron en 1894, precisamente durante la efervescencia de fin de siglo, consecuencia de la cual fue también el proceso iniciado

S E OME 5. Problema 7. Sugerencia

El numero de cifras de 2n es 1+[n log 2] , donde [·] denota la parte entera inferior (maximoentero inferior o igual a n).

Page 129: COMPENDIUM OME - ToomatesEotvos de Hungría, que se iniciaron en 1894, precisamente durante la efervescencia de fin de siglo, consecuencia de la cual fue también el proceso iniciado

S E OME 5. Problema 8. Sugerencia

Suponer que el cuadrado es la celda de una cuadrıcula que ocupa todo el plano. En vezde estudiar las reflexiones sobre los lados del cuadrado, es mas facil estudiar los cortes conlos ejes de la semirrecta que tiene mismo origen y direccion que el rayo de luz.

Page 130: COMPENDIUM OME - ToomatesEotvos de Hungría, que se iniciaron en 1894, precisamente durante la efervescencia de fin de siglo, consecuencia de la cual fue también el proceso iniciado

S E OME 6. Problema 1. Sugerencia

Si A′ y B′ son los transformados de A y B, observar que la recta A′B′, es ortogonal a lacircunferencia inversa de γ.

Page 131: COMPENDIUM OME - ToomatesEotvos de Hungría, que se iniciaron en 1894, precisamente durante la efervescencia de fin de siglo, consecuencia de la cual fue también el proceso iniciado

S E OME 6. Problema 2. Sugerencia

Observar que el triangulo de vertices en O y en los afijos de z e iz es rectangulo en O eisosceles.

Page 132: COMPENDIUM OME - ToomatesEotvos de Hungría, que se iniciaron en 1894, precisamente durante la efervescencia de fin de siglo, consecuencia de la cual fue también el proceso iniciado

S E OME 6. Problema 3. Sugerencia

Tener en cuenta que, mediante rotaciones, dos coloraciones, en principio diferentes, enrealidad pueden ser la misma.

Page 133: COMPENDIUM OME - ToomatesEotvos de Hungría, que se iniciaron en 1894, precisamente durante la efervescencia de fin de siglo, consecuencia de la cual fue también el proceso iniciado

S E OME 6. Problema 4. Sugerencia

Calcular primero el valor de la distancia del centro de la circunferencia al punto medio deuno de los lados.

Page 134: COMPENDIUM OME - ToomatesEotvos de Hungría, que se iniciaron en 1894, precisamente durante la efervescencia de fin de siglo, consecuencia de la cual fue también el proceso iniciado

S E OME 6. Problema 5. Sugerencia

Distinguir los casos en que el corte vaya de vertice a vertice, de vertice a lado, o de lado alado. Despues estudia mas detenidamente los casos en que pueda ser posible.

Page 135: COMPENDIUM OME - ToomatesEotvos de Hungría, que se iniciaron en 1894, precisamente durante la efervescencia de fin de siglo, consecuencia de la cual fue también el proceso iniciado

S E OME 6. Problema 6. Sugerencia

Habiendo analizado las desigualdades primeras, aplica la o las que se cumplan para hallarM(x). Tengase en cuenta que el valor absoluto de un polinomio no es, en principio, unpolinomio.

Page 136: COMPENDIUM OME - ToomatesEotvos de Hungría, que se iniciaron en 1894, precisamente durante la efervescencia de fin de siglo, consecuencia de la cual fue también el proceso iniciado

S E OME 6. Problema 7. Sugerencia

Considerar los angulos centrales de cada lado y usa las desigualdades que se puedan extraerde estos para hallar las referidas a los angulos interiores.

Page 137: COMPENDIUM OME - ToomatesEotvos de Hungría, que se iniciaron en 1894, precisamente durante la efervescencia de fin de siglo, consecuencia de la cual fue también el proceso iniciado

S E OME 6. Problema 8. Sugerencia

Aplicar la teorıa de permutaciones. Comprobar si el producto de transformaciones “pinc-hazo” es una operacion interna.

Page 138: COMPENDIUM OME - ToomatesEotvos de Hungría, que se iniciaron en 1894, precisamente durante la efervescencia de fin de siglo, consecuencia de la cual fue también el proceso iniciado

S E OME 7. Problema 1. Sugerencia

El volumen desalojado al descender 1 cm el nivel coincide con el de un cilindro oblicuo debase elıptica (V = πabh, donde a y b son los semiejes de la elipse).

Page 139: COMPENDIUM OME - ToomatesEotvos de Hungría, que se iniciaron en 1894, precisamente durante la efervescencia de fin de siglo, consecuencia de la cual fue también el proceso iniciado

S E OME 7. Problema 2. Sugerencia

Siguiendo el metodo sugerido, comprueba que al sustituir una yema de alejamiento k, tantoel alejamiento total como la carga total aumentan en k + 2.

Page 140: COMPENDIUM OME - ToomatesEotvos de Hungría, que se iniciaron en 1894, precisamente durante la efervescencia de fin de siglo, consecuencia de la cual fue también el proceso iniciado

S E OME 7. Problema 3. Sugerencia

Suponiendo que PA = PB demuestra que existe un punto Q del lado AC tal que el areadel triangulo APQ sea la mitad del area inicial.

Page 141: COMPENDIUM OME - ToomatesEotvos de Hungría, que se iniciaron en 1894, precisamente durante la efervescencia de fin de siglo, consecuencia de la cual fue también el proceso iniciado

S E OME 7. Problema 4. Sugerencia

Descomponer los polinomios a partir de los datos del problema y trata de hallar las raıcesa partir de su mcm y su mcd.

Page 142: COMPENDIUM OME - ToomatesEotvos de Hungría, que se iniciaron en 1894, precisamente durante la efervescencia de fin de siglo, consecuencia de la cual fue también el proceso iniciado

S E OME 7. Problema 5. Sugerencia

Pensar en una clase con 100 alumnos y distribuir las notas de la manera menos favorableposible.

Page 143: COMPENDIUM OME - ToomatesEotvos de Hungría, que se iniciaron en 1894, precisamente durante la efervescencia de fin de siglo, consecuencia de la cual fue también el proceso iniciado

S E OME 7. Problema 6. Sugerencia

Notese que el lado MN (y el punto B) esta en el eje radical de las circunferencias γ y lacircunscrita al triangulo AMN .

Page 144: COMPENDIUM OME - ToomatesEotvos de Hungría, que se iniciaron en 1894, precisamente durante la efervescencia de fin de siglo, consecuencia de la cual fue también el proceso iniciado

S E OME 7. Problema 7. Sugerencia

Transformar la ecuacion en una que solo tenga cos x como incognita.

Page 145: COMPENDIUM OME - ToomatesEotvos de Hungría, que se iniciaron en 1894, precisamente durante la efervescencia de fin de siglo, consecuencia de la cual fue también el proceso iniciado

S E OME 7. Problema 8. Sugerencia

Aplicar el teorema del coseno en los triangulos OMA, OMB, OMC, OMD.

Page 146: COMPENDIUM OME - ToomatesEotvos de Hungría, que se iniciaron en 1894, precisamente durante la efervescencia de fin de siglo, consecuencia de la cual fue también el proceso iniciado

S E OME 8. Problema 1. Sugerencia

Recordar que 1331(k = k3 + 3k2 + 3k + 1.

Page 147: COMPENDIUM OME - ToomatesEotvos de Hungría, que se iniciaron en 1894, precisamente durante la efervescencia de fin de siglo, consecuencia de la cual fue también el proceso iniciado

S E OME 8. Problema 2. Sugerencia

Para el Teorema 1, tomar un punto E que no este en la recta AC. Ası puedo tomar unpunto F en la recta AE tal que E este entre A y F . Igualmente construid el punto D enla recta CF y estudiar la recta DE. Para el Teorema 2, usar el Teorema 1 y partir de unpunto D fuera de la recta que contiene a los tres puntos.

Page 148: COMPENDIUM OME - ToomatesEotvos de Hungría, que se iniciaron en 1894, precisamente durante la efervescencia de fin de siglo, consecuencia de la cual fue también el proceso iniciado

S E OME 8. Problema 3. Sugerencia

Dividir la inecuacion por la expresion px2 + qy2 (que es positiva, salvo el caso extremox = y = 0) y resolver la inecuacion que resulta.

Page 149: COMPENDIUM OME - ToomatesEotvos de Hungría, que se iniciaron en 1894, precisamente durante la efervescencia de fin de siglo, consecuencia de la cual fue también el proceso iniciado

S E OME 8. Problema 4. Sugerencia

Se cumple que (a − b) (A − B) = 0, y lo mismo con b − c y c − a.

Page 150: COMPENDIUM OME - ToomatesEotvos de Hungría, que se iniciaron en 1894, precisamente durante la efervescencia de fin de siglo, consecuencia de la cual fue también el proceso iniciado

S E OME 8. Problema 5. Sugerencia

Calcular el lımite que se pide demostrar mediante las igualdades anteriores, teniendo encuenta tambien que |z| < 1.

Page 151: COMPENDIUM OME - ToomatesEotvos de Hungría, que se iniciaron en 1894, precisamente durante la efervescencia de fin de siglo, consecuencia de la cual fue también el proceso iniciado

S E OME 8. Problema 6. Sugerencia

Estudiar las dos alternativas siguientes: ir directo de P a A o ir de P a B (siendo B algunpunto de la circunferencia) y de B a A por la circunferencia. Parametrizar B en funciondel angulo y estudia la funcion T (a) (que mide el tiempo tardado con la segunda opcionen funcion del angulo) para determinar la mejor alternativa en cada caso.

Page 152: COMPENDIUM OME - ToomatesEotvos de Hungría, que se iniciaron en 1894, precisamente durante la efervescencia de fin de siglo, consecuencia de la cual fue también el proceso iniciado

S E OME 8. Problema 7. Sugerencia

Calcular primero el radio de la circunferencia transformada distinguiendo los casos d > r,d < r y d = r (siendo d la distancia del centro de las circunferencia al centro de inversion).

Page 153: COMPENDIUM OME - ToomatesEotvos de Hungría, que se iniciaron en 1894, precisamente durante la efervescencia de fin de siglo, consecuencia de la cual fue también el proceso iniciado

S E OME 8. Problema 8. Sugerencia

Considera la descomposicion ai = 2bi pi , donde pi es impar.

Page 154: COMPENDIUM OME - ToomatesEotvos de Hungría, que se iniciaron en 1894, precisamente durante la efervescencia de fin de siglo, consecuencia de la cual fue también el proceso iniciado

S E OME 9. Problema 1. Sugerencia

Comprobar que (M, +) es un grupo y que cumple las propiedades necesarias para el pro-ducto. Utilizar la condicion que implica que una matriz sea invertible.

Page 155: COMPENDIUM OME - ToomatesEotvos de Hungría, que se iniciaron en 1894, precisamente durante la efervescencia de fin de siglo, consecuencia de la cual fue también el proceso iniciado

S E OME 9. Problema 2. Sugerencia

Calcular los maximos y mınimos en cada lado por separado.

Page 156: COMPENDIUM OME - ToomatesEotvos de Hungría, que se iniciaron en 1894, precisamente durante la efervescencia de fin de siglo, consecuencia de la cual fue también el proceso iniciado

S E OME 9. Problema 3. Sugerencia

Desarrollar el prisma sobre un plano.

Page 157: COMPENDIUM OME - ToomatesEotvos de Hungría, que se iniciaron en 1894, precisamente durante la efervescencia de fin de siglo, consecuencia de la cual fue también el proceso iniciado

S E OME 9. Problema 4. Sugerencia

El conjunto A es una semirrecta y el conjunto B una bola abierta.

Page 158: COMPENDIUM OME - ToomatesEotvos de Hungría, que se iniciaron en 1894, precisamente durante la efervescencia de fin de siglo, consecuencia de la cual fue también el proceso iniciado

S E OME 9. Problema 5. Sugerencia

Si llamamos PQR al triangulo buscado, existe un giro que transforma Q en R.

Page 159: COMPENDIUM OME - ToomatesEotvos de Hungría, que se iniciaron en 1894, precisamente durante la efervescencia de fin de siglo, consecuencia de la cual fue también el proceso iniciado

S E OME 9. Problema 6. Sugerencia

Considerar dos maneras de calcular el area del triangulo, una a partir del radio inscrito yotra a partir de los lados.

Page 160: COMPENDIUM OME - ToomatesEotvos de Hungría, que se iniciaron en 1894, precisamente durante la efervescencia de fin de siglo, consecuencia de la cual fue también el proceso iniciado

S E OME 9. Problema 7. Sugerencia

Intentese demostrarlo por induccion.

Page 161: COMPENDIUM OME - ToomatesEotvos de Hungría, que se iniciaron en 1894, precisamente durante la efervescencia de fin de siglo, consecuencia de la cual fue también el proceso iniciado

S E OME 9. Problema 8. Sugerencia

Las clases de equivalencia cumplen que la suma de las componentes de cada vector esconstante.

Page 162: COMPENDIUM OME - ToomatesEotvos de Hungría, que se iniciaron en 1894, precisamente durante la efervescencia de fin de siglo, consecuencia de la cual fue también el proceso iniciado

S E OME 10. Problema 1. Sugerencia

Considerar la funcion f(x) = 14

x4 − 10x2(x − 1) y buscar sus mınimos.

Page 163: COMPENDIUM OME - ToomatesEotvos de Hungría, que se iniciaron en 1894, precisamente durante la efervescencia de fin de siglo, consecuencia de la cual fue también el proceso iniciado

S E OME 10. Problema 2. Sugerencia

Solucionar primero el sistema de 3 ecuaciones.

Page 164: COMPENDIUM OME - ToomatesEotvos de Hungría, que se iniciaron en 1894, precisamente durante la efervescencia de fin de siglo, consecuencia de la cual fue también el proceso iniciado

S E OME 10. Problema 3. Sugerencia

Intentar hallar una expresion explıcita de la sucesion. Despues debes tener en cuenta que

la serie de potencias de termino generalxk

ktiene por valor − log(1 − x) para |x| < 1,

debido a las propiedades de la serie de Taylor.

Page 165: COMPENDIUM OME - ToomatesEotvos de Hungría, que se iniciaron en 1894, precisamente durante la efervescencia de fin de siglo, consecuencia de la cual fue también el proceso iniciado

S E OME 10. Problema 4. Sugerencia

Considerar el triangulo formado por el centro O de C y C′, el centro P de una de lascircunferencias Ci y el punto Q de tangencia entre Ci y Ci+1.

Page 166: COMPENDIUM OME - ToomatesEotvos de Hungría, que se iniciaron en 1894, precisamente durante la efervescencia de fin de siglo, consecuencia de la cual fue también el proceso iniciado

S E OME 10. Problema 5. Sugerencia

Considerar la condicion de independencia lineal y el polinomio de Taylor centrado en elpunto 2.

Page 167: COMPENDIUM OME - ToomatesEotvos de Hungría, que se iniciaron en 1894, precisamente durante la efervescencia de fin de siglo, consecuencia de la cual fue también el proceso iniciado

S E OME 10. Problema 6. Sugerencia

Considerar los triangulos ABP , ACP y BCP . Luego aplicar la desigualdad triangular yel lugar geometrico hallado en (b).

Page 168: COMPENDIUM OME - ToomatesEotvos de Hungría, que se iniciaron en 1894, precisamente durante la efervescencia de fin de siglo, consecuencia de la cual fue también el proceso iniciado

S E OME 10. Problema 7. Sugerencia

Tener en cuenta que los caminos sobre los ejes son de longitud constante. Por lo tanto,hay que minimizar el resto del camino.

Page 169: COMPENDIUM OME - ToomatesEotvos de Hungría, que se iniciaron en 1894, precisamente durante la efervescencia de fin de siglo, consecuencia de la cual fue también el proceso iniciado

S E OME 10. Problema 8. Sugerencia

Usar la ecuacion de una recta en el espacio.

Page 170: COMPENDIUM OME - ToomatesEotvos de Hungría, que se iniciaron en 1894, precisamente durante la efervescencia de fin de siglo, consecuencia de la cual fue también el proceso iniciado

S E OME 11. Problema 1. Sugerencia

Pensar en la disposicion espacial de los elementos de un dodecaedro.

Page 171: COMPENDIUM OME - ToomatesEotvos de Hungría, que se iniciaron en 1894, precisamente durante la efervescencia de fin de siglo, consecuencia de la cual fue también el proceso iniciado

S E OME 11. Problema 2. Sugerencia

Expresar el volumen del cono en funcion, unicamente, del angulo α y del radio del discometalico r, constante. Recuerda que el volumen del cono es

V =13

πρ2h

siendo ρ el radio de la circunferencia de la base y h la altura.

Page 172: COMPENDIUM OME - ToomatesEotvos de Hungría, que se iniciaron en 1894, precisamente durante la efervescencia de fin de siglo, consecuencia de la cual fue también el proceso iniciado

S E OME 11. Problema 3. Sugerencia

Comprobar que para dos elementos cualquier de Z(5) su resta y su producto tambienpertenecen al conjunto. Probar que Z(5) no es un cuerpo para responder negativamente ala ultima cuestion.

Page 173: COMPENDIUM OME - ToomatesEotvos de Hungría, que se iniciaron en 1894, precisamente durante la efervescencia de fin de siglo, consecuencia de la cual fue también el proceso iniciado

S E OME 11. Problema 4. Sugerencia

Construir el simetrico del triangulo respecto a cada lado en que se produce la reflexion, ycontinuar la progresion del rayo en el triangulo creado (como si no hubiese reflexion), paraque esta sea una lınea recta de la que puedes hallar su longitud.

Page 174: COMPENDIUM OME - ToomatesEotvos de Hungría, que se iniciaron en 1894, precisamente durante la efervescencia de fin de siglo, consecuencia de la cual fue también el proceso iniciado

S E OME 11. Problema 5. Sugerencia

Sean a, b ∈ G. Intentese demostrar que ab = ba, sabiendo que a2 = b2 = e.

Page 175: COMPENDIUM OME - ToomatesEotvos de Hungría, que se iniciaron en 1894, precisamente durante la efervescencia de fin de siglo, consecuencia de la cual fue también el proceso iniciado

S E OME 11. Problema 6. Sugerencia

Sea F el punto medio de BC. Prolonga la recta BC y elige en ella un punto G tal qued(F, G) = 3 d(F, B). Toma como D el punto en que la circunferencia corta a GA.

Page 176: COMPENDIUM OME - ToomatesEotvos de Hungría, que se iniciaron en 1894, precisamente durante la efervescencia de fin de siglo, consecuencia de la cual fue también el proceso iniciado

S E OME 11. Problema 7. Sugerencia

Calcular primero la altura del agua si el prisma esta en posicion vertical. Supongase queuna de las aristas consideradas esta apoyada en tierra. Entonces la diferencia de la alturacon el valor en posicion vertical de una arista se tiene que compensar con la altura de suopuesta.

Page 177: COMPENDIUM OME - ToomatesEotvos de Hungría, que se iniciaron en 1894, precisamente durante la efervescencia de fin de siglo, consecuencia de la cual fue también el proceso iniciado

S E OME 11. Problema 8. Sugerencia

Observar que si mas de la mitad de los lados son del mismo color, habra dos de ellos quesean contiguos.

Page 178: COMPENDIUM OME - ToomatesEotvos de Hungría, que se iniciaron en 1894, precisamente durante la efervescencia de fin de siglo, consecuencia de la cual fue también el proceso iniciado

S E OME 12. Problema 1. Sugerencia

Pensar en la definicion de integral Riemann de la funcion xk.

Page 179: COMPENDIUM OME - ToomatesEotvos de Hungría, que se iniciaron en 1894, precisamente durante la efervescencia de fin de siglo, consecuencia de la cual fue también el proceso iniciado

S E OME 12. Problema 2. Sugerencia

Calcular los lımites de f para x → 0, −1, ±∞. Comprobar ademas que la derivada espositiva.

Page 180: COMPENDIUM OME - ToomatesEotvos de Hungría, que se iniciaron en 1894, precisamente durante la efervescencia de fin de siglo, consecuencia de la cual fue también el proceso iniciado

S E OME 12. Problema 3. Sugerencia

Aplicar las definiciones de cuerpo, anillo y espacio vectorial.

Page 181: COMPENDIUM OME - ToomatesEotvos de Hungría, que se iniciaron en 1894, precisamente durante la efervescencia de fin de siglo, consecuencia de la cual fue también el proceso iniciado

S E OME 12. Problema 4. Sugerencia

Utilizar la desigualdad de medias aritmetica y geometrica.

Page 182: COMPENDIUM OME - ToomatesEotvos de Hungría, que se iniciaron en 1894, precisamente durante la efervescencia de fin de siglo, consecuencia de la cual fue también el proceso iniciado

S E OME 12. Problema 5. Sugerencia

Construir un triangulo con base en r y con A como vertice tal que B sea el incentro.

Page 183: COMPENDIUM OME - ToomatesEotvos de Hungría, que se iniciaron en 1894, precisamente durante la efervescencia de fin de siglo, consecuencia de la cual fue también el proceso iniciado

S E OME 12. Problema 6. Sugerencia

Comprobar como son las sucesiones modulo 8.

Page 184: COMPENDIUM OME - ToomatesEotvos de Hungría, que se iniciaron en 1894, precisamente durante la efervescencia de fin de siglo, consecuencia de la cual fue también el proceso iniciado

S E OME 12. Problema 7. Sugerencia

Estudiar la funcion en los intervalos (−8,−3], [−3,−1] , [−1, 2], [2, 5] y [5, 8), separada-mente.

Page 185: COMPENDIUM OME - ToomatesEotvos de Hungría, que se iniciaron en 1894, precisamente durante la efervescencia de fin de siglo, consecuencia de la cual fue también el proceso iniciado

S E OME 12. Problema 8. Sugerencia

En el cuadrado [0, 1] × [0, 1] del plano XY , determinar los puntos (x, y) que satisfacen lacondicion del enunciado y aplicar la regla de Laplace.

Page 186: COMPENDIUM OME - ToomatesEotvos de Hungría, que se iniciaron en 1894, precisamente durante la efervescencia de fin de siglo, consecuencia de la cual fue también el proceso iniciado

S E OME 13. Problema 1. Sugerencia

Suponiendo el cuadrado construido, ¿en que punto corta la diagonal con el cırculo dediametro AB? (donde A y B pertenecen a lados adyacentes).

Page 187: COMPENDIUM OME - ToomatesEotvos de Hungría, que se iniciaron en 1894, precisamente durante la efervescencia de fin de siglo, consecuencia de la cual fue también el proceso iniciado

S E OME 13. Problema 2. Sugerencia

Agrupar las r-plas en pares de suma 0.

Page 188: COMPENDIUM OME - ToomatesEotvos de Hungría, que se iniciaron en 1894, precisamente durante la efervescencia de fin de siglo, consecuencia de la cual fue también el proceso iniciado

S E OME 13. Problema 3. Sugerencia

El espejo realiza una simetrıa de eje vertical, y la lente una simetrıa de eje horizontal.

Page 189: COMPENDIUM OME - ToomatesEotvos de Hungría, que se iniciaron en 1894, precisamente durante la efervescencia de fin de siglo, consecuencia de la cual fue también el proceso iniciado

S E OME 13. Problema 4. Sugerencia

La expresion es igual a (n − 2)(n − 1)n(n + 1). Uno de estos terminos es multiplo de 4.

Page 190: COMPENDIUM OME - ToomatesEotvos de Hungría, que se iniciaron en 1894, precisamente durante la efervescencia de fin de siglo, consecuencia de la cual fue también el proceso iniciado

S E OME 13. Problema 5. Sugerencia

Realizar el cambio z = (1 − i) t. Resulta una ecuacion de cuarto grado con dos raıcescomplejas y dos reales; acotar sus argumentos.

Page 191: COMPENDIUM OME - ToomatesEotvos de Hungría, que se iniciaron en 1894, precisamente durante la efervescencia de fin de siglo, consecuencia de la cual fue también el proceso iniciado

S E OME 13. Problema 6. Sugerencia

Si M = A + B, con A simetrica y B antisimetrica, entonces M + M t = 2A.

Page 192: COMPENDIUM OME - ToomatesEotvos de Hungría, que se iniciaron en 1894, precisamente durante la efervescencia de fin de siglo, consecuencia de la cual fue también el proceso iniciado

S E OME 13. Problema 7. Sugerencia

El producto de dos numeros no negativos, con suma dada, es maximo cuando ambos soniguales.

Page 193: COMPENDIUM OME - ToomatesEotvos de Hungría, que se iniciaron en 1894, precisamente durante la efervescencia de fin de siglo, consecuencia de la cual fue también el proceso iniciado

S E OME 13. Problema 8. Sugerencia

Estudiar los lımites laterales en ese punto.

Page 194: COMPENDIUM OME - ToomatesEotvos de Hungría, que se iniciaron en 1894, precisamente durante la efervescencia de fin de siglo, consecuencia de la cual fue también el proceso iniciado

S E OME 14. Problema 1. Sugerencia

Intentar conseguir una fila con todos los terminos iguales para poder sacar el factor fueradel determinante. Luego, restando filas, transformarlo en un determinante cuyo valor seael producto de los elementos de la diagonal.

Page 195: COMPENDIUM OME - ToomatesEotvos de Hungría, que se iniciaron en 1894, precisamente durante la efervescencia de fin de siglo, consecuencia de la cual fue también el proceso iniciado

S E OME 14. Problema 2. Sugerencia

Para la primera parte, simplemente recordar la definicion de cuerpo conmutativo y aplicarlas propiedades de suma y producto de numeros reales. Para la segunda parte, plantear laecuacion en las matrices y resolver el sistema que aparece.

Page 196: COMPENDIUM OME - ToomatesEotvos de Hungría, que se iniciaron en 1894, precisamente durante la efervescencia de fin de siglo, consecuencia de la cual fue también el proceso iniciado

S E OME 14. Problema 3. Sugerencia

Suponer que todos dan un numero impar de apretones y estudiar la paridad de la sumade todos los numeros de apretones.

Page 197: COMPENDIUM OME - ToomatesEotvos de Hungría, que se iniciaron en 1894, precisamente durante la efervescencia de fin de siglo, consecuencia de la cual fue también el proceso iniciado

S E OME 14. Problema 4. Sugerencia

Expresar la suma como

(n − 2)2 + (n − 1)2 + n2 + (n + 1)2 + (n + 2)2

y desarrollar.

Page 198: COMPENDIUM OME - ToomatesEotvos de Hungría, que se iniciaron en 1894, precisamente durante la efervescencia de fin de siglo, consecuencia de la cual fue también el proceso iniciado

S E OME 14. Problema 5. Sugerencia

Plantear un sistema cuyas incognitas sean el numero de peldanos visibles y el numero depeldanos que se esconden en el tiempo que tardamos en bajar un escalon.

Page 199: COMPENDIUM OME - ToomatesEotvos de Hungría, que se iniciaron en 1894, precisamente durante la efervescencia de fin de siglo, consecuencia de la cual fue también el proceso iniciado

S E OME 14. Problema 6. Sugerencia

Si se llama O al centro de la circunferencia y M a la segunda interseccion de AO conla circunferencia circunscrita, intentar demostrar que AM es un diametro de esta ultimacircunferencia o, lo que es lo mismo, que el angulo ABM es recto. Puede ser util considerarel punto P , interseccion de AD con la circunferencia circunscrita, que ademas, es el puntomedio del arco BC.

Page 200: COMPENDIUM OME - ToomatesEotvos de Hungría, que se iniciaron en 1894, precisamente durante la efervescencia de fin de siglo, consecuencia de la cual fue también el proceso iniciado

S E OME 14. Problema 7. Sugerencia

Desarrollar los cuadrados y estudiar la funcion resultante. Recordar que el extremo de unafuncion parabolica se alcanza en el vertice.

Page 201: COMPENDIUM OME - ToomatesEotvos de Hungría, que se iniciaron en 1894, precisamente durante la efervescencia de fin de siglo, consecuencia de la cual fue también el proceso iniciado

S E OME 14. Problema 8. Sugerencia

Para encontrar la condicion, considerar que si el triangulo es equilatero, entonces ha de ser|z1 − z2| = |z2 − z3| = |z3 − z1|. para comprobar que la condicion es suficiente, consideresela condicion como ecuacion en z3 y deducir que

z3 − z2 = (z1 − z2)e±iπ3

Page 202: COMPENDIUM OME - ToomatesEotvos de Hungría, que se iniciaron en 1894, precisamente durante la efervescencia de fin de siglo, consecuencia de la cual fue también el proceso iniciado

S E OME 15. Problema 1. Sugerencia

El area de una elipse de semiejes a y b es S = πab, y su ecuacion centrada en (x0, y0) es

(x − x0)2

a2+

(y − y0)2

b2= 1;

una circunferencia de radio r se puede considerar una elipse de semiejes a = b = r.

Page 203: COMPENDIUM OME - ToomatesEotvos de Hungría, que se iniciaron en 1894, precisamente durante la efervescencia de fin de siglo, consecuencia de la cual fue también el proceso iniciado

S E OME 15. Problema 2. Sugerencia

Recordar que un movimiento conserva las distancias. Busca dos puntos que no quedena la misma distancia. Para la segunda parte, piensa en que letras quedan igual tras unasimetrıa.

Page 204: COMPENDIUM OME - ToomatesEotvos de Hungría, que se iniciaron en 1894, precisamente durante la efervescencia de fin de siglo, consecuencia de la cual fue también el proceso iniciado

S E OME 15. Problema 3. Sugerencia

Solucion algebraica. Observar que (1 + x)n(1 + x)n = (1 + x)2n y aplicar la formula delbinomio, fijandose en el termino de grado n.

Solucion combinatoria. Tomar X = a1, a2, . . . , am, b1, b2, . . . , bn y contar los subconjun-tos de r elementos que tiene X , teniendo en cuenta los ai o bi que contienen. Particularizarpara r = m = n.

Page 205: COMPENDIUM OME - ToomatesEotvos de Hungría, que se iniciaron en 1894, precisamente durante la efervescencia de fin de siglo, consecuencia de la cual fue también el proceso iniciado

S E OME 15. Problema 4. Sugerencia

Estudiar la naturaleza de z1 y z2 teniendo en cuanta que a y b son reales. Usar la formulade De Moivre.

Page 206: COMPENDIUM OME - ToomatesEotvos de Hungría, que se iniciaron en 1894, precisamente durante la efervescencia de fin de siglo, consecuencia de la cual fue también el proceso iniciado

S E OME 15. Problema 5. Sugerencia

Trasladar la funcion tres unidades a la izquierda.

Page 207: COMPENDIUM OME - ToomatesEotvos de Hungría, que se iniciaron en 1894, precisamente durante la efervescencia de fin de siglo, consecuencia de la cual fue también el proceso iniciado

S E OME 15. Problema 6. Sugerencia

Considerar primero que bolas puede haber en la caja y con que probabilidad se dara cadasituacion. Luego, para cada una de ellas, calcular la probabilidad de obtener las 4 bolasblancas. Usar las formulas de la probabilidad condicionada.

Page 208: COMPENDIUM OME - ToomatesEotvos de Hungría, que se iniciaron en 1894, precisamente durante la efervescencia de fin de siglo, consecuencia de la cual fue también el proceso iniciado

S E OME 15. Problema 7. Sugerencia

Intentar hallar el volumen del toro utilizando una integral. Recordar que el volumengenerado al girar la grafica de f(x) alrededor del eje OX en el intervalo [a, b] es

π

∫ b

a

(f(x)

)2dx .

Page 209: COMPENDIUM OME - ToomatesEotvos de Hungría, que se iniciaron en 1894, precisamente durante la efervescencia de fin de siglo, consecuencia de la cual fue también el proceso iniciado

S E OME 15. Problema 8. Sugerencia

Estudiar lo que ocurre con el termino independiente y con los de grado mayor que 2 de unpolinomio p(x) al derivarlo. Evaluar p′(0).

Page 210: COMPENDIUM OME - ToomatesEotvos de Hungría, que se iniciaron en 1894, precisamente durante la efervescencia de fin de siglo, consecuencia de la cual fue también el proceso iniciado

S E OME 16. Problema 1. Sugerencia

Fijando el lado conocido, hallar el lugar geometrico de los posibles vertices opuestos yrazonar como ha de ser el triangulo area maxima. Aplicar trigonometrıa a este triangulopara calcular los datos que se piden.

Page 211: COMPENDIUM OME - ToomatesEotvos de Hungría, que se iniciaron en 1894, precisamente durante la efervescencia de fin de siglo, consecuencia de la cual fue también el proceso iniciado

S E OME 16. Problema 2. Sugerencia

Abordar el problema mediante un diagrama de arbol. Razonando algunos detalles puedesimplificarse bastante su construccion.

Page 212: COMPENDIUM OME - ToomatesEotvos de Hungría, que se iniciaron en 1894, precisamente durante la efervescencia de fin de siglo, consecuencia de la cual fue también el proceso iniciado

S E OME 16. Problema 3. Sugerencia

Intentar aplicar alguna desigualdad conocida para obtener el enunciado.

Page 213: COMPENDIUM OME - ToomatesEotvos de Hungría, que se iniciaron en 1894, precisamente durante la efervescencia de fin de siglo, consecuencia de la cual fue también el proceso iniciado

S E OME 16. Problema 4. Sugerencia

Observar en primer lugar que si f(x) = 0 para todo x, podemos escribir

f ′(x)f(x)

= −x2.

Integrando cada miembro de la igualdad (el de la izquierda es de tipo logarıtmico) y usandoque f(1) = e, despejar el valor de la funcion y representarla, usando las tecnicas habituales.

Page 214: COMPENDIUM OME - ToomatesEotvos de Hungría, que se iniciaron en 1894, precisamente durante la efervescencia de fin de siglo, consecuencia de la cual fue también el proceso iniciado

S E OME 16. Problema 5. Sugerencia

Considerandoan = xn +

1xn

,

comenzar probando que an+1 = an a1 − an−1 y utilizar esta condicion para demostrar porinduccion completa el enunciado, utilizando la formula trigonometrica

cosA + cosB =12

(cos(A + B) + cos(A − B)

).

Page 215: COMPENDIUM OME - ToomatesEotvos de Hungría, que se iniciaron en 1894, precisamente durante la efervescencia de fin de siglo, consecuencia de la cual fue también el proceso iniciado

S E OME 16. Problema 6. Sugerencia

Observar que la expresion algebraica en que se traduce el enunciado es

n(n + 1)(n + 2)(n + 3) + 1

y desarrollarla. Para ver que siempre es un cuadrado perfecto, basta factorizar el polinomioresultante.

Page 216: COMPENDIUM OME - ToomatesEotvos de Hungría, que se iniciaron en 1894, precisamente durante la efervescencia de fin de siglo, consecuencia de la cual fue también el proceso iniciado

S E OME 16. Problema 7. Sugerencia

En primer lugar, observar que las ecuaciones parametricas de dicho punto son x = λ2,y = (2 − λ)2, con λ ∈ [0, 2]. Eliminando el parametro se puede calcular el area medianteuna integral. Por otro lado, aplicar el giro de 45 para obtener la ecuacion

y′ =√

24

x′2 +√

2

y deducir de esta formula como es la curva.

Page 217: COMPENDIUM OME - ToomatesEotvos de Hungría, que se iniciaron en 1894, precisamente durante la efervescencia de fin de siglo, consecuencia de la cual fue también el proceso iniciado

S E OME 16. Problema 8. Sugerencia

La formula de Heron para el area de un triangulo conocidos los lados es

S =√

p(p − a)(p − b)(p − c)

siendo p el semiperımetro. Suponiendo que los numeros sean n, n + 1, n + 2 y n + 3,sustituirlos por S, a, b, c segun las distintas ordenaciones posibles para obtener todas lassoluciones.

Page 218: COMPENDIUM OME - ToomatesEotvos de Hungría, que se iniciaron en 1894, precisamente durante la efervescencia de fin de siglo, consecuencia de la cual fue también el proceso iniciado

S E OME 17. Problema 1. Sugerencia

Escribir la suma como 7 (1 + 11 + 111 + · · ·+ 111 . . . 111) y descomponer cada uno de lossumandos del parentesis como potencias de diez.

Page 219: COMPENDIUM OME - ToomatesEotvos de Hungría, que se iniciaron en 1894, precisamente durante la efervescencia de fin de siglo, consecuencia de la cual fue también el proceso iniciado

S E OME 17. Problema 2. Sugerencia

Desarrollar el lateral del vaso como un rectangulo y dibujar sobre el las posiciones de lagota y la mosca. Ası el problema queda reducido a calcular la mınima distancia entre losdos puntos del plano tocando el borde del vaso.

Page 220: COMPENDIUM OME - ToomatesEotvos de Hungría, que se iniciaron en 1894, precisamente durante la efervescencia de fin de siglo, consecuencia de la cual fue también el proceso iniciado

S E OME 17. Problema 3. Sugerencia

Considerando α el angulo buscado, demostrar que las direcciones de r y u forman unangulo 2α, al igual que las de s y v, y deducir que las direcciones de u y v forman unangulo 3α. Concluir que α = 60.

Page 221: COMPENDIUM OME - ToomatesEotvos de Hungría, que se iniciaron en 1894, precisamente durante la efervescencia de fin de siglo, consecuencia de la cual fue también el proceso iniciado

S E OME 17. Problema 4. Sugerencia

Expresar sen (x − 2) = sen((x − 1) − 1

)y desarrollarlo como el seno de una diferencia

de angulos. Simplificar la expresion resultante y entonces aplicar el cambio de variablet = tan(x − 1). La integral resultante puede tratarse como una de tipo racional.

Page 222: COMPENDIUM OME - ToomatesEotvos de Hungría, que se iniciaron en 1894, precisamente durante la efervescencia de fin de siglo, consecuencia de la cual fue también el proceso iniciado

S E OME 17. Problema 5. Sugerencia

Representar la funcion para los primeros valores de n e indicar como se comporta parademas valores. Las integrales y el lımite no presentan problemas.

Page 223: COMPENDIUM OME - ToomatesEotvos de Hungría, que se iniciaron en 1894, precisamente durante la efervescencia de fin de siglo, consecuencia de la cual fue también el proceso iniciado

S E OME 17. Problema 6. Sugerencia

En primer lugar, descomponer la simetrıa respecto del punto (0, 0) como producto de dossimetrıas axiales de ejes perpendiculares x − y = 0 y x + y = 0. El movimiento quedaexpresado como producto de tres simetrıas axiales. Observar ahora que dos de los ejes sonparalelos resultando la traslacion buscada.

Page 224: COMPENDIUM OME - ToomatesEotvos de Hungría, que se iniciaron en 1894, precisamente durante la efervescencia de fin de siglo, consecuencia de la cual fue también el proceso iniciado

S E OME 17. Problema 7. Sugerencia

Aplicar directamente el teorema de Bayes (o teorema de la probabilidad inversa) a losdatos del problema.

Page 225: COMPENDIUM OME - ToomatesEotvos de Hungría, que se iniciaron en 1894, precisamente durante la efervescencia de fin de siglo, consecuencia de la cual fue también el proceso iniciado

S E OME 17. Problema 8. Sugerencia

Como a es impar, sera de la forma 2k+1. Sustituyendo este valor se obtiene otro polinomioen la indeterminada k. De esta expresion resulta inmediato que el resultado es multiplode 4. Para el primer apartado basta utilizar convenientemente la formula del cuadrado deuna suma.

Page 226: COMPENDIUM OME - ToomatesEotvos de Hungría, que se iniciaron en 1894, precisamente durante la efervescencia de fin de siglo, consecuencia de la cual fue también el proceso iniciado

S E OME 18. Problema 1. Sugerencia

Resolver la ecuacion diofantica en que se traduce el enunciado y observar que, con lasrestricciones que se tienen, la solucion no es unica.

Page 227: COMPENDIUM OME - ToomatesEotvos de Hungría, que se iniciaron en 1894, precisamente durante la efervescencia de fin de siglo, consecuencia de la cual fue también el proceso iniciado

S E OME 18. Problema 2. Sugerencia

Descomponer la rotacion como dos simetrıas axiales definidas por ejes que pasan por elpunto P y angulo 45 entre ellas. Trabajar entonces con tres simetrıas axiales.

Page 228: COMPENDIUM OME - ToomatesEotvos de Hungría, que se iniciaron en 1894, precisamente durante la efervescencia de fin de siglo, consecuencia de la cual fue también el proceso iniciado

S E OME 18. Problema 3. Sugerencia

El problema se reduce a calcular

limn→∞

1 − 1

2+

13− · · ·+ (−1)n

n

.

El criterio de Leibnitz asegura la convergencia de esta serie. Para calcular el lımite, bastadesarrollar en serie de potencias la funcion f(x) = log(1 + x) centrandose en el punto 0 ysustituir x = 1. Concluir que la altura final es 120 log 2.

Page 229: COMPENDIUM OME - ToomatesEotvos de Hungría, que se iniciaron en 1894, precisamente durante la efervescencia de fin de siglo, consecuencia de la cual fue también el proceso iniciado

S E OME 18. Problema 4. Sugerencia

La condicion es equivalente a que p(√

x2 + y2)≤ x4 + y4.

i) Tomando y = 0, x > 0 y p(x) = a1x + · · ·+ anxn, la desigualdad queda

a1

x3+

a2

x2+

a1

x+ a4 + a5x + · · ·+ anxn−4 ≤ 1.

Tomando lımites en 0 y +∞ de x, demostrar que han de ser a1 = a2 = a3 = a5 = · · · =an = 0.

ii) Deducir del apartado anterior y de la desigualdad que p(x) = ax4, a ≤ 1, con lo que ladesigualdad inicial queda a(ξ + η)2 ≤ ξ2 + η2 para ξ, η > 0.

iii) Transformar esta ultima desigualdad en

ξ η ≤ 2(1 − a)(

ξ + η

2

)2

y deducir de la desigualdad entre media aritmetica y geometrica que 0 ≤ a ≤ 1/2.

Page 230: COMPENDIUM OME - ToomatesEotvos de Hungría, que se iniciaron en 1894, precisamente durante la efervescencia de fin de siglo, consecuencia de la cual fue también el proceso iniciado

S E OME 18. Problema 5. Sugerencia

Construir un cuadrado cualquiera, hallando la suma del lado y la diagonal y despues pasara una figura semejante que tenga esta longitud igual a la dada.

Page 231: COMPENDIUM OME - ToomatesEotvos de Hungría, que se iniciaron en 1894, precisamente durante la efervescencia de fin de siglo, consecuencia de la cual fue también el proceso iniciado

S E OME 18. Problema 6. Sugerencia

Distinguir ν = 0 y ν = 0. En el segundo caso, u = λν y la desigualdad se traduce enλa ≤ λ a + 1 − a. Estudiar la funcion f(λ) = λ a + 1 − a − λa.

Page 232: COMPENDIUM OME - ToomatesEotvos de Hungría, que se iniciaron en 1894, precisamente durante la efervescencia de fin de siglo, consecuencia de la cual fue también el proceso iniciado

S E OME 18. Problema 7. Sugerencia

Expresar la suma y el producto de dos fraccionesa

by

c

dde S y observar como son sus

denominadores. Para la tercera pregunta, probar con varias fracciones.

Page 233: COMPENDIUM OME - ToomatesEotvos de Hungría, que se iniciaron en 1894, precisamente durante la efervescencia de fin de siglo, consecuencia de la cual fue también el proceso iniciado

S E OME 18. Problema 8. Sugerencia

Empezar por probar que para un punto M = (x, y) se cumple

d(M, C) =

d(M, A) si x < 32

d(M, B) si x ≥ 32

; d(M, C ′) =

d(M, A′) si y < 4d(M, B′) si y ≥ 4

y discutir los posibles casos en que se da la igualdad para obtener como la grafica de unafuncion definida a trozos el conjunto buscado, y estudiar su derivabilidad.

Page 234: COMPENDIUM OME - ToomatesEotvos de Hungría, que se iniciaron en 1894, precisamente durante la efervescencia de fin de siglo, consecuencia de la cual fue también el proceso iniciado

S E OME 19. Problema 1. Sugerencia

Considerar el angulo α que forman el perro P , el centro C de la columna y un punto detangencia T , es decir, PCT . Si la longitud de la cuerda es constante , expresar la distanciadel perro al centro de la columna como funcion de α usando trigonometrıa. Observar ahoraque la distancia a la que se rompe el nudo se alcanza en el punto en que se maximiza dichadistancia del perro al centro de la columna, justamente en π/6.

Page 235: COMPENDIUM OME - ToomatesEotvos de Hungría, que se iniciaron en 1894, precisamente durante la efervescencia de fin de siglo, consecuencia de la cual fue también el proceso iniciado

S E OME 19. Problema 2. Sugerencia

Encontrar primero la forma de dibujar el angulo y la circunferencia inscrita. Despuesdibujar sobre los lados dos longitudes que satisfagan la proporcion y aplicar el teorema deThales para dibujar el verdadero lado.

Page 236: COMPENDIUM OME - ToomatesEotvos de Hungría, que se iniciaron en 1894, precisamente durante la efervescencia de fin de siglo, consecuencia de la cual fue también el proceso iniciado

S E OME 19. Problema 3. Sugerencia

Comenzar probando que

Ak = r2 senkπ

n + 1

y a partir de esa formula expresar el lımite de las medias aritmeticas como

limSn =r2

πlim

n + 1

n∑k=1

n + 1

)

¿Que relacion existe entre este lımite y

∫ π

0

senx dx ?

Page 237: COMPENDIUM OME - ToomatesEotvos de Hungría, que se iniciaron en 1894, precisamente durante la efervescencia de fin de siglo, consecuencia de la cual fue también el proceso iniciado

S E OME 19. Problema 4. Sugerencia

Observar que el enunciado equivale a hallar el numero de puntos de corte de la curvay = 16x5 − 20x3 + 5x con la recta y = −m. Esto puede hacerse de forma especialmentefacil al calcular los extremos de la funcion cuya grafica describe la primera curva.

Page 238: COMPENDIUM OME - ToomatesEotvos de Hungría, que se iniciaron en 1894, precisamente durante la efervescencia de fin de siglo, consecuencia de la cual fue también el proceso iniciado

S E OME 19. Problema 5. Sugerencia

Utilizando el hecho de que al girar el punto A un angulo de 90 o −90 con centro en elpunto B tiene que coincidir con el punto C, calcular las posibles posiciones del punto A.Cuando se obtiene A no es difıcil calcular los demas vertices.

Page 239: COMPENDIUM OME - ToomatesEotvos de Hungría, que se iniciaron en 1894, precisamente durante la efervescencia de fin de siglo, consecuencia de la cual fue también el proceso iniciado

S E OME 19. Problema 6. Sugerencia

Intentar expresar los menus de los que queremos hallar el precio como combinacion linealde los que ya sabemos cuanto cuestan.

Page 240: COMPENDIUM OME - ToomatesEotvos de Hungría, que se iniciaron en 1894, precisamente durante la efervescencia de fin de siglo, consecuencia de la cual fue también el proceso iniciado

S E OME 19. Problema 7. Sugerencia

Observar que el hueco que queda vacıo en el tetraedro despues de echar el agua es otrotetraedro y calcular su altura. La altura que alcanza el agua puede hallarse como ladiferencia entre la altura total y la de este tetraedro.

Page 241: COMPENDIUM OME - ToomatesEotvos de Hungría, que se iniciaron en 1894, precisamente durante la efervescencia de fin de siglo, consecuencia de la cual fue también el proceso iniciado

S E OME 19. Problema 8. Sugerencia

Expresar ordenadamente las edades x, y, z en 1960 de los hermanos. De la primeracondicion x = y + z en 1960 y la segunda unos anos despues se puede llegar a que han deser y = 2z, x = 3z. De la tercera condicion pueden deducirse los valores exactos.

Page 242: COMPENDIUM OME - ToomatesEotvos de Hungría, que se iniciaron en 1894, precisamente durante la efervescencia de fin de siglo, consecuencia de la cual fue también el proceso iniciado

S E OME 20. Problema 1. Sugerencia

Girando la direccion AB, reducir el problema a construir un trapecio isosceles conociendola suma de las bases , la altura (distancia entre AB girada y CD) y que esta inscrito enuna circunferencia centrada en O.

Page 243: COMPENDIUM OME - ToomatesEotvos de Hungría, que se iniciaron en 1894, precisamente durante la efervescencia de fin de siglo, consecuencia de la cual fue también el proceso iniciado

S E OME 20. Problema 2. Sugerencia

Averiguar que valores puede tomar la ultima cifra del numero para que esto ocurra. Dis-cutir cuales pueden ser las demas cifras para cada uno de estos valores.

Page 244: COMPENDIUM OME - ToomatesEotvos de Hungría, que se iniciaron en 1894, precisamente durante la efervescencia de fin de siglo, consecuencia de la cual fue también el proceso iniciado

S E OME 20. Problema 3. Sugerencia

Para las dos primeras desigualdades es suficiente usar las desigualdades (√

x − √y)2 ≥ 0

y (x − y)2 ≥ 0. Para la tercera, tomando m = p + 1p

y n = q + 1q, probar primero que

p q ≤ 14 y deducir que m+n ≥ 5 y m2 +n2 ≥ 25−2mn. Con ello demostrar la desigualdad

buscada.

Page 245: COMPENDIUM OME - ToomatesEotvos de Hungría, que se iniciaron en 1894, precisamente durante la efervescencia de fin de siglo, consecuencia de la cual fue también el proceso iniciado

S E OME 20. Problema 4. Sugerencia

Transformar la expresion a la que hay que calcularle el lımite usando reiteradamente laformula del seno del angulo mitad.

Page 246: COMPENDIUM OME - ToomatesEotvos de Hungría, que se iniciaron en 1894, precisamente durante la efervescencia de fin de siglo, consecuencia de la cual fue también el proceso iniciado

S E OME 20. Problema 5. Sugerencia

Situando unos ejes de coordenadas en el punto medio del segmento que une los centros,comprobar que podemos expresar

A = (−d + cosa, sena), A′ = (d + cos b, sen b)B =

(− d + cos(a + x), sen (a + x)

), B′ =

(d + cos(a + x), sen (a + x)

).

Calcular el punto medio M de coordenadas (Mx, My) y probar que en el primer supuestoM2

x +M2y es constante mientras que en el segundo

−−→OM tiene siempre la misma direccion.

Page 247: COMPENDIUM OME - ToomatesEotvos de Hungría, que se iniciaron en 1894, precisamente durante la efervescencia de fin de siglo, consecuencia de la cual fue también el proceso iniciado

S E OME 20. Problema 6. Sugerencia

La recta a distancia 3 del eje OX es la y = 3 y tomando como parametro el angulo t queforma la recta variable con el eje OX , ver que las coordenadas parametricas de un puntodel lugar geometrico buscado son

(3

tan t, 3 sen 2t

)

y calcular a partir de ellas la ecuacion de la curva que describe dicho punto.

Page 248: COMPENDIUM OME - ToomatesEotvos de Hungría, que se iniciaron en 1894, precisamente durante la efervescencia de fin de siglo, consecuencia de la cual fue también el proceso iniciado

S E OME 20. Problema 7. Sugerencia

Si el numero en cuestion es de p+1 cifras, expresarlo como a ·10p+x, donde a es la ultimacifra. Observar que con esta expresion, el numero queda dividido por k al eliminar laultima cifra cuando a ·10p+x = k x. Transformar esta expresion para obtener la condicionbuscada.

Page 249: COMPENDIUM OME - ToomatesEotvos de Hungría, que se iniciaron en 1894, precisamente durante la efervescencia de fin de siglo, consecuencia de la cual fue también el proceso iniciado

S E OME 20. Problema 8. Sugerencia

Deducir que si el polinomio definido por el determinante es P (x), existe otro polinomioC(x) tal que P (x) = (x2 − 1)C(x) + (ax + b). Calcular a y b dandole valores convenientesa x.

Page 250: COMPENDIUM OME - ToomatesEotvos de Hungría, que se iniciaron en 1894, precisamente durante la efervescencia de fin de siglo, consecuencia de la cual fue también el proceso iniciado

S E OME 21. Problema 1. Sugerencia

Estudiar por separado los casos: f no tiene ningun punto fijo o f tiene un punto fijo. ¿Queocurre si f tiene dos o mas puntos fijos?

Page 251: COMPENDIUM OME - ToomatesEotvos de Hungría, que se iniciaron en 1894, precisamente durante la efervescencia de fin de siglo, consecuencia de la cual fue también el proceso iniciado

S E OME 21. Problema 2. Sugerencia

Tomar un semiplano del plano y estudiar las propiedades del subconjunto de Z × Z con-tenido en el.

Page 252: COMPENDIUM OME - ToomatesEotvos de Hungría, que se iniciaron en 1894, precisamente durante la efervescencia de fin de siglo, consecuencia de la cual fue también el proceso iniciado

S E OME 21. Problema 3. Sugerencia

Hay varias soluciones posibles, como intentar expresar una de las razones trigonometricasde la ecuacion en funcion de otra razon trigonometrica para luego resolver la nuevaecuacion, mas facil. Otra solucion consiste en sustituir la tangente por la razon del seno ycoseno para luego usar seno de sumas de angulos, y de angulos dobles.

Page 253: COMPENDIUM OME - ToomatesEotvos de Hungría, que se iniciaron en 1894, precisamente durante la efervescencia de fin de siglo, consecuencia de la cual fue también el proceso iniciado

S E OME 21. Problema 4. Sugerencia

Para el primer apartado intentar factorizar la expresion usando el cubo de tres sumandos.Para el segundo, piensa en dos casos posibles de k, y busca las ternas que cumplan lopedido.

Page 254: COMPENDIUM OME - ToomatesEotvos de Hungría, que se iniciaron en 1894, precisamente durante la efervescencia de fin de siglo, consecuencia de la cual fue también el proceso iniciado

S E OME 21. Problema 5. Sugerencia

Intenta encontrar una raız de forma sencilla.

Page 255: COMPENDIUM OME - ToomatesEotvos de Hungría, que se iniciaron en 1894, precisamente durante la efervescencia de fin de siglo, consecuencia de la cual fue también el proceso iniciado

S E OME 21. Problema 6. Sugerencia

Probar que el punto P es el centro del cırculo exinscrito al triangulo AMN correspondienteal lado AN .

Page 256: COMPENDIUM OME - ToomatesEotvos de Hungría, que se iniciaron en 1894, precisamente durante la efervescencia de fin de siglo, consecuencia de la cual fue también el proceso iniciado

S E OME 21. Problema 7. Sugerencia

Expresar el polinomio como producto del segundo polinomio y de otro, e igualar coeficientespara obtener un sistema del que obtener la solucion.

Page 257: COMPENDIUM OME - ToomatesEotvos de Hungría, que se iniciaron en 1894, precisamente durante la efervescencia de fin de siglo, consecuencia de la cual fue también el proceso iniciado

S E OME 21. Problema 8. Sugerencia

Intentar resolver las ecuaciones que provienen de la suma y productos constantes depen-diendo del valor de algunos coeficientes.

Page 258: COMPENDIUM OME - ToomatesEotvos de Hungría, que se iniciaron en 1894, precisamente durante la efervescencia de fin de siglo, consecuencia de la cual fue también el proceso iniciado

S E OME 22. Problema 1. Sugerencia

Buscar una funcion cuya imagen en el plano corresponde a la distancia del enunciado ytrabajar con ella.

Page 259: COMPENDIUM OME - ToomatesEotvos de Hungría, que se iniciaron en 1894, precisamente durante la efervescencia de fin de siglo, consecuencia de la cual fue también el proceso iniciado

S E OME 22. Problema 2. Sugerencia

En el apartado b), se puede pensar en que pasarıa si se iterase el proceso de dividir el ladodel pentagono interior, teniendo en cuenta la relacion que hay entre la diagonal y el ladodel pentagono, ademas de la relacion entre el lado del pentagono y el lado del pentagonointerior.

Page 260: COMPENDIUM OME - ToomatesEotvos de Hungría, que se iniciaron en 1894, precisamente durante la efervescencia de fin de siglo, consecuencia de la cual fue también el proceso iniciado

S E OME 22. Problema 3. Sugerencia

Estudiar por tanteo primero la ecuacion 5n + 3 = 2m para m = 7. Posteriormente, param > 7, trabajar con congruencias 5n + 3 ≡ 0 mod 28.

Page 261: COMPENDIUM OME - ToomatesEotvos de Hungría, que se iniciaron en 1894, precisamente durante la efervescencia de fin de siglo, consecuencia de la cual fue también el proceso iniciado

S E OME 22. Problema 4. Sugerencia

Si la primera y segunda derivada son positivas, la funcion es convexa creciente.

Page 262: COMPENDIUM OME - ToomatesEotvos de Hungría, que se iniciaron en 1894, precisamente durante la efervescencia de fin de siglo, consecuencia de la cual fue también el proceso iniciado

S E OME 22. Problema 5. Sugerencia

Elegir un punto convenientemente en el eje de ordenadas y su opuesto. Hacer pasar unarecta por ese punto y de pendiente t arbitraria. Determinar t para que el punto de inter-seccion de la recta con la curva tenga coordenadas racionales.

Page 263: COMPENDIUM OME - ToomatesEotvos de Hungría, que se iniciaron en 1894, precisamente durante la efervescencia de fin de siglo, consecuencia de la cual fue también el proceso iniciado

S E OME 22. Problema 6. Sugerencia

Se puede usar la factorizacion de z15−1 considerando las 15 raıces complejas de la unidadpara el calcular el producto de los 7 primeros factores. Despues se debe dar un valor a laz para que los calculos del producto sean faciles de resolver.

Page 264: COMPENDIUM OME - ToomatesEotvos de Hungría, que se iniciaron en 1894, precisamente durante la efervescencia de fin de siglo, consecuencia de la cual fue también el proceso iniciado

S E OME 23. Problema 1. Sugerencia

Tracese una tangente y dibujese las otras calculando angulos de 60. Para calcular las areasse puede tener en cuenta que si desde un punto P del plano se trazan perpendiculares a lostres lados de un triangulo equilatero, se obtienen 3 segmentos cuya suma (con los signosque convenga) es igual a la altura del triangulo.

Page 265: COMPENDIUM OME - ToomatesEotvos de Hungría, que se iniciaron en 1894, precisamente durante la efervescencia de fin de siglo, consecuencia de la cual fue también el proceso iniciado

S E OME 23. Problema 2. Sugerencia

Usa una conocida desigualdad de vectores y normas.

Page 266: COMPENDIUM OME - ToomatesEotvos de Hungría, que se iniciaron en 1894, precisamente durante la efervescencia de fin de siglo, consecuencia de la cual fue también el proceso iniciado

S E OME 23. Problema 3. Sugerencia

Se debe pensar en un caso base y como van aumentando las variables cuando se incrementael numero de vertices.

Page 267: COMPENDIUM OME - ToomatesEotvos de Hungría, que se iniciaron en 1894, precisamente durante la efervescencia de fin de siglo, consecuencia de la cual fue también el proceso iniciado

S E OME 23. Problema 4. Sugerencia

Usar el metodo de sustitucion en ambos sistemas. El primero es inmediato, mientras queel segundo se debe intentar expresar la inecuacion como factores en los que algunos nocambian de signo.

Page 268: COMPENDIUM OME - ToomatesEotvos de Hungría, que se iniciaron en 1894, precisamente durante la efervescencia de fin de siglo, consecuencia de la cual fue también el proceso iniciado

S E OME 23. Problema 5. Sugerencia

Dibuja DF , paralela a BC. El triangulo GDF sera equilatero. Probar ademas que eltriangulo GEF es isosceles.

Page 269: COMPENDIUM OME - ToomatesEotvos de Hungría, que se iniciaron en 1894, precisamente durante la efervescencia de fin de siglo, consecuencia de la cual fue también el proceso iniciado

S E OME 23. Problema 6. Sugerencia

Se puede usar teoremas conocidos del analisis matematico, usando puntos como 0 y 3/4.

Page 270: COMPENDIUM OME - ToomatesEotvos de Hungría, que se iniciaron en 1894, precisamente durante la efervescencia de fin de siglo, consecuencia de la cual fue también el proceso iniciado

S E OME 24. Problema 1. Sugerencia

Para un usar el principio del palomar con los pares 1, k + 1, 2, k + 2, . . . , k, k + kcomo agujeros.

Page 271: COMPENDIUM OME - ToomatesEotvos de Hungría, que se iniciaron en 1894, precisamente durante la efervescencia de fin de siglo, consecuencia de la cual fue también el proceso iniciado

S E OME 24. Problema 2. Sugerencia

Razona de forma inductiva.

Page 272: COMPENDIUM OME - ToomatesEotvos de Hungría, que se iniciaron en 1894, precisamente durante la efervescencia de fin de siglo, consecuencia de la cual fue también el proceso iniciado

S E OME 24. Problema 3. Sugerencia

Probar la equivalencia de 25x + 31y y 3x + 7y modulo 41.

Page 273: COMPENDIUM OME - ToomatesEotvos de Hungría, que se iniciaron en 1894, precisamente durante la efervescencia de fin de siglo, consecuencia de la cual fue también el proceso iniciado

S E OME 24. Problema 4. Sugerencia

Hay que intentar demostrar an+m = an−1am + anam+1 mediante induccion en una de lasvariables.

Otra solucion serıa pensar en otra recurrencia (forma de subir escaleras si se pueden subirlos peldanos de uno en uno o de dos en dos) y pensar en la solucion para subir 2n + 1peldanos, y despues intentar asociar la solucion a la sucesion de Fibonacci.

Page 274: COMPENDIUM OME - ToomatesEotvos de Hungría, que se iniciaron en 1894, precisamente durante la efervescencia de fin de siglo, consecuencia de la cual fue también el proceso iniciado

S E OME 24. Problema 5. Sugerencia

Pensar en embaldosado de cruces.

Page 275: COMPENDIUM OME - ToomatesEotvos de Hungría, que se iniciaron en 1894, precisamente durante la efervescencia de fin de siglo, consecuencia de la cual fue también el proceso iniciado

S E OME 24. Problema 6. Sugerencia

Reescribir la ecuacion como producto de polinomios y cuadrado de una expresion de t paracalcular mas facilmente las soluciones para x.

Page 276: COMPENDIUM OME - ToomatesEotvos de Hungría, que se iniciaron en 1894, precisamente durante la efervescencia de fin de siglo, consecuencia de la cual fue también el proceso iniciado

S E OME 25. Problema 1. Sugerencia

Estudiar primero los primeros casos y expresar la probabilidad de forma general.

Page 277: COMPENDIUM OME - ToomatesEotvos de Hungría, que se iniciaron en 1894, precisamente durante la efervescencia de fin de siglo, consecuencia de la cual fue también el proceso iniciado

S E OME 25. Problema 2. Sugerencia

Utilizar el siguiente hecho: si dos triangulos tienen la misma altura, entonces la relacionde areas es igual a la de las bases; o bien que si tienen la misma base, la relacion de areases la de las alturas.

Page 278: COMPENDIUM OME - ToomatesEotvos de Hungría, que se iniciaron en 1894, precisamente durante la efervescencia de fin de siglo, consecuencia de la cual fue también el proceso iniciado

S E OME 25. Problema 3. Sugerencia

Tomar el cuadrado de la fraccion del centro. Sustituir convenientemente factores de estafraccion para obtener una nueva fraccion mayor (respectivamente, menor) que otra que sepuede simplificar.

Page 279: COMPENDIUM OME - ToomatesEotvos de Hungría, que se iniciaron en 1894, precisamente durante la efervescencia de fin de siglo, consecuencia de la cual fue también el proceso iniciado

S E OME 25. Problema 4. Sugerencia

Comprobarlo para n = 1 e intentar usar esta expresion para potencias pares e impares.

Page 280: COMPENDIUM OME - ToomatesEotvos de Hungría, que se iniciaron en 1894, precisamente durante la efervescencia de fin de siglo, consecuencia de la cual fue también el proceso iniciado

S E OME 25. Problema 5. Sugerencia

Expresar 14 como el producto de 2 elementos de D y obtener la solucion dependiendo delvalor de alguna de las variables.

Page 281: COMPENDIUM OME - ToomatesEotvos de Hungría, que se iniciaron en 1894, precisamente durante la efervescencia de fin de siglo, consecuencia de la cual fue también el proceso iniciado

S E OME 25. Problema 6. Sugerencia

Suponer a y b como razones trigonometricas.

Page 282: COMPENDIUM OME - ToomatesEotvos de Hungría, que se iniciaron en 1894, precisamente durante la efervescencia de fin de siglo, consecuencia de la cual fue también el proceso iniciado

S E OME 26. Problema 1. Sugerencia

a) Comparar A2 con B2.

b) Expresar el termino que esta debajo de la raız del numero M como un cuadrado.Simplificar y comparar los dos nuevos numeros elevando al cuadrado cada uno de ellos.

Page 283: COMPENDIUM OME - ToomatesEotvos de Hungría, que se iniciaron en 1894, precisamente durante la efervescencia de fin de siglo, consecuencia de la cual fue también el proceso iniciado

S E OME 26. Problema 2. Sugerencia

Razonar en base a la siguiente figura, en la que cada lado es de longitud 1.

A

B

C

E

D

F G

1

Page 284: COMPENDIUM OME - ToomatesEotvos de Hungría, que se iniciaron en 1894, precisamente durante la efervescencia de fin de siglo, consecuencia de la cual fue también el proceso iniciado

S E OME 26. Problema 3. Sugerencia

Considerar el numeroE =

(4 +

√11

)n +(4 −

√11

)n.

Desarrollar segun la formula del binomio de Newton.

Page 285: COMPENDIUM OME - ToomatesEotvos de Hungría, que se iniciaron en 1894, precisamente durante la efervescencia de fin de siglo, consecuencia de la cual fue también el proceso iniciado

S E OME 26. Problema 4. Sugerencia

Considerar los siguientes cambios de variable

x =a + 1

2+

a + 36

√4a + 3

3

y =a + 1

2− a + 3

6

√4a + 3

3z = x + y.

dada la suma inicial, elevar al cubo y obtener un polinomio en Z. Encontrar la unica raızreal de ese polinomio y demostrar que condicion necesaria y suficiente para que sea unica

es que a ≥ −34.

Page 286: COMPENDIUM OME - ToomatesEotvos de Hungría, que se iniciaron en 1894, precisamente durante la efervescencia de fin de siglo, consecuencia de la cual fue también el proceso iniciado

S E OME 26. Problema 5. Sugerencia

a) y b) Expresar los lados en funcion de los lados originales y del parametro p.

c) Aplicar el teorema de Menelao para una eleccion conveniente de puntos.

Page 287: COMPENDIUM OME - ToomatesEotvos de Hungría, que se iniciaron en 1894, precisamente durante la efervescencia de fin de siglo, consecuencia de la cual fue también el proceso iniciado

S E OME 26. Problema 6. Sugerencia

Considerese un punto A y el conjunto de puntos que estan unidos con el. Estudiar enton-cesuno a uno los triangulos con vertices A y un par de dichos puntos. El angulo en A debeser mayor que 60 y no puede haber mas de cinco putos unidos con A.

Page 288: COMPENDIUM OME - ToomatesEotvos de Hungría, que se iniciaron en 1894, precisamente durante la efervescencia de fin de siglo, consecuencia de la cual fue también el proceso iniciado

S E OME 27. Problema 1. Sugerencia

a) Demostrar por contradiccion. Tomando tres puntos que cumpliesen las condiciones delenunciado, por el teorema del coseno resultarıa que cos 45 serıa un numero racional.

b) Razonamiento identico.

Page 289: COMPENDIUM OME - ToomatesEotvos de Hungría, que se iniciaron en 1894, precisamente durante la efervescencia de fin de siglo, consecuencia de la cual fue también el proceso iniciado

S E OME 27. Problema 2. Sugerencia

Calcular el resultado de multiplicar la primera fila por λ, la segunda por 1 − λ y sumar-las. Demostrar que, para n ≥ 3, podemos conseguir la fila que queramos, simplementemodificando el valor de λ.

Page 290: COMPENDIUM OME - ToomatesEotvos de Hungría, que se iniciaron en 1894, precisamente durante la efervescencia de fin de siglo, consecuencia de la cual fue también el proceso iniciado

S E OME 27. Problema 3. Sugerencia

Usar las formulas de Cardano-Vieta. La condicion necesaria y suficiente para que a, b, csean los lados de un triangulo, es que

(a + b− c)(b + c − a)(c + a − b) > 0 .

Page 291: COMPENDIUM OME - ToomatesEotvos de Hungría, que se iniciaron en 1894, precisamente durante la efervescencia de fin de siglo, consecuencia de la cual fue también el proceso iniciado

S E OME 27. Problema 4. Sugerencia

Utilizando angulos y observando que AC ′DB′ es simetrico respecto de AD, demostrar queel angulo ICB vale C/2. Los puntos I, B′, C y D son concıclicos.

Page 292: COMPENDIUM OME - ToomatesEotvos de Hungría, que se iniciaron en 1894, precisamente durante la efervescencia de fin de siglo, consecuencia de la cual fue también el proceso iniciado

S E OME 27. Problema 5. Sugerencia

Probar por induccion que σ(k) = 2k−1 + 1.

Observacion: La funcion σ esta extendida a los naturales, no a las potencias de 2.

Page 293: COMPENDIUM OME - ToomatesEotvos de Hungría, que se iniciaron en 1894, precisamente durante la efervescencia de fin de siglo, consecuencia de la cual fue también el proceso iniciado

S E OME 27. Problema 6. Sugerencia

Observar que1√k

>2√

k +√

k + 1,

1√k

<2√

k +√

k − 1.

Page 294: COMPENDIUM OME - ToomatesEotvos de Hungría, que se iniciaron en 1894, precisamente durante la efervescencia de fin de siglo, consecuencia de la cual fue también el proceso iniciado

S E OME 28. Problema 1. Sugerencia

Probar todos los casos sabiendo que el numero de divisores de un numero n = pα11 pα2

2 · · · pαrr

es d(n) = (α1 + 1)(α2 + 1) · · · (αr + 1)

Page 295: COMPENDIUM OME - ToomatesEotvos de Hungría, que se iniciaron en 1894, precisamente durante la efervescencia de fin de siglo, consecuencia de la cual fue también el proceso iniciado

S E OME 28. Problema 2. Sugerencia

Trasladar una de las circunferencias paralelamente a lo largo de la direccion dada unadistancia conveniente para que la interseccion con la otra circunferencia determine el puntosolucion.

Page 296: COMPENDIUM OME - ToomatesEotvos de Hungría, que se iniciaron en 1894, precisamente durante la efervescencia de fin de siglo, consecuencia de la cual fue también el proceso iniciado

S E OME 28. Problema 3. Sugerencia

Expresar las formulas del enunciado en funcion de los numeros triangulares.

Page 297: COMPENDIUM OME - ToomatesEotvos de Hungría, que se iniciaron en 1894, precisamente durante la efervescencia de fin de siglo, consecuencia de la cual fue también el proceso iniciado

S E OME 28. Problema 4. Sugerencia

Suponer que hay un numero finito de primos de la forma 4n + 3. Considerar el numeroq = 22 · 3 · 5 · · ·p − 1, siendo p el ultimo primo de la forma 4n + 3.

Page 298: COMPENDIUM OME - ToomatesEotvos de Hungría, que se iniciaron en 1894, precisamente durante la efervescencia de fin de siglo, consecuencia de la cual fue también el proceso iniciado

S E OME 28. Problema 5. Sugerencia

Construccion del punto P :

a) Trazar la circunferencia que pasa por A y es tangente en B al lado BC.

b) La paralela por B al lado AC corta en M a la circunferencia anterior.

c) La interseccion de la recta CM con la circunferencia determina P .

Usar el teorema de los senos en APC y BPC.

Page 299: COMPENDIUM OME - ToomatesEotvos de Hungría, que se iniciaron en 1894, precisamente durante la efervescencia de fin de siglo, consecuencia de la cual fue también el proceso iniciado

S E OME 28. Problema 6. Sugerencia

Hacer el cambio z = cos t + i sin t y utilizar la formula de De Moivre.

Page 300: COMPENDIUM OME - ToomatesEotvos de Hungría, que se iniciaron en 1894, precisamente durante la efervescencia de fin de siglo, consecuencia de la cual fue también el proceso iniciado

S E OME 29. Problema 1. Sugerencia

Aplicar directamente el principio del palomar.

Page 301: COMPENDIUM OME - ToomatesEotvos de Hungría, que se iniciaron en 1894, precisamente durante la efervescencia de fin de siglo, consecuencia de la cual fue también el proceso iniciado

S E OME 29. Problema 2. Sugerencia

Expresar la suma de los numeros de cada fila en funcion de la fila anterior primero, y enfuncion de la primera fila despues.

Page 302: COMPENDIUM OME - ToomatesEotvos de Hungría, que se iniciaron en 1894, precisamente durante la efervescencia de fin de siglo, consecuencia de la cual fue también el proceso iniciado

S E OME 29. Problema 3. Sugerencia

Expresar, en funcion de las longitudes de los lados y de los radios, los valores del area. Senecesita despues la desigualdad aritmetico-geometrica.

Page 303: COMPENDIUM OME - ToomatesEotvos de Hungría, que se iniciaron en 1894, precisamente durante la efervescencia de fin de siglo, consecuencia de la cual fue también el proceso iniciado

S E OME 29. Problema 4. Sugerencia

Considerar el conjunto 1, 10, 102, . . . , 10n−1 y sus restos modulo p . Aplicar el principiodel palomar.

Page 304: COMPENDIUM OME - ToomatesEotvos de Hungría, que se iniciaron en 1894, precisamente durante la efervescencia de fin de siglo, consecuencia de la cual fue también el proceso iniciado

S E OME 29. Problema 5. Sugerencia

Estudiar todas las posibilidades (tres) teniendo en cuenta las simetrıas determinadas porlas diagonales.

Page 305: COMPENDIUM OME - ToomatesEotvos de Hungría, que se iniciaron en 1894, precisamente durante la efervescencia de fin de siglo, consecuencia de la cual fue también el proceso iniciado

S E OME 29. Problema 6. Sugerencia

a) Estudiar la probabilidad de que la bola llegue a C o a D.

b) La duracion del juego es un numero par de movimientos. Observar que la duracionmedia es una serie aritmetico-geometrica, y sumarla.

Page 306: COMPENDIUM OME - ToomatesEotvos de Hungría, que se iniciaron en 1894, precisamente durante la efervescencia de fin de siglo, consecuencia de la cual fue también el proceso iniciado

S E OME 30. Problema 1. Sugerencia

Observese que (a + d)2 = a2 + d(2a + d).

Page 307: COMPENDIUM OME - ToomatesEotvos de Hungría, que se iniciaron en 1894, precisamente durante la efervescencia de fin de siglo, consecuencia de la cual fue también el proceso iniciado

S E OME 30. Problema 2. Sugerencia

Resolverlo por geometrıa analıtica.

Page 308: COMPENDIUM OME - ToomatesEotvos de Hungría, que se iniciaron en 1894, precisamente durante la efervescencia de fin de siglo, consecuencia de la cual fue también el proceso iniciado

S E OME 30. Problema 3. Sugerencia

Considerar la suma de dıas, modulo 4.

Page 309: COMPENDIUM OME - ToomatesEotvos de Hungría, que se iniciaron en 1894, precisamente durante la efervescencia de fin de siglo, consecuencia de la cual fue también el proceso iniciado

S E OME 30. Problema 4. Sugerencia

Expresar las proporciones entre los triangulos semejantes.

Page 310: COMPENDIUM OME - ToomatesEotvos de Hungría, que se iniciaron en 1894, precisamente durante la efervescencia de fin de siglo, consecuencia de la cual fue también el proceso iniciado

S E OME 30. Problema 5. Sugerencia

Primero descartar las combinaciones Negro-Negro-Negro y Blanco-Blanco-Blanco y luegoaplicar el principio del palomar.

Page 311: COMPENDIUM OME - ToomatesEotvos de Hungría, que se iniciaron en 1894, precisamente durante la efervescencia de fin de siglo, consecuencia de la cual fue también el proceso iniciado

S E OME 30. Problema 6. Sugerencia

a) Contar el numero de triangulos y de lados.

b) Demostrar por induccion sobre el numero de vertices, que dicho numero esm − n + 2

2.

Page 312: COMPENDIUM OME - ToomatesEotvos de Hungría, que se iniciaron en 1894, precisamente durante la efervescencia de fin de siglo, consecuencia de la cual fue también el proceso iniciado

S E OME 31. Problema 1. Sugerencia

¿Cuantas veces aparece cada lado en la suma total S(A)? ¿Cual es el valor mınimo quepuede tener el menor elemento del conjunto A?

Page 313: COMPENDIUM OME - ToomatesEotvos de Hungría, que se iniciaron en 1894, precisamente durante la efervescencia de fin de siglo, consecuencia de la cual fue también el proceso iniciado

S E OME 31. Problema 2. Sugerencia

Dibujar la situacion del enunciado en el espacio con ejes coordenados X , Y , Z. Recordarque dados dos puntos en una esfera, el centro se encuentra en el plano mediatriz de dichospuntos.

Page 314: COMPENDIUM OME - ToomatesEotvos de Hungría, que se iniciaron en 1894, precisamente durante la efervescencia de fin de siglo, consecuencia de la cual fue también el proceso iniciado

S E OME 31. Problema 3. Sugerencia

El producto en cuestion se puede expresar como funcion de una sola variable. Con estefin, utilizar vectores o triangulos semejantes.

Page 315: COMPENDIUM OME - ToomatesEotvos de Hungría, que se iniciaron en 1894, precisamente durante la efervescencia de fin de siglo, consecuencia de la cual fue también el proceso iniciado

S E OME 31. Problema 4. Sugerencia

Si mcd (a, a′) = 1 y a x = a′ x′, entonces a x′ es multiplo de a. (Se supone que todas estasvariables son numeros enteros).

Page 316: COMPENDIUM OME - ToomatesEotvos de Hungría, que se iniciaron en 1894, precisamente durante la efervescencia de fin de siglo, consecuencia de la cual fue también el proceso iniciado

S E OME 31. Problema 5. Sugerencia

Despejar n de la primera ecuacion y sustituir en la segunda.

Page 317: COMPENDIUM OME - ToomatesEotvos de Hungría, que se iniciaron en 1894, precisamente durante la efervescencia de fin de siglo, consecuencia de la cual fue también el proceso iniciado

S E OME 31. Problema 6. Sugerencia

a) Considerar las circunferencias circunscritas a los tres triangulos y sus puntos de inter-seccion.

b) ¿Cuanto vale el angulo APB?

c) Puede resolverse mediante geometrıa analıtica. Alternativamente, observese que queun lado del triangulo formado por los circuncentros es perpendicular a CP y el otro esperpendicular a AP .

d) Notese que los angulos B′′PA′′ y B′′CA′′ son iguales.

Page 318: COMPENDIUM OME - ToomatesEotvos de Hungría, que se iniciaron en 1894, precisamente durante la efervescencia de fin de siglo, consecuencia de la cual fue también el proceso iniciado

S E OME 32. Problema 1. Sugerencia

Demostrar que en las condiciones del problema, a + b es un multiplo de(mcd(a, b)

)2

Page 319: COMPENDIUM OME - ToomatesEotvos de Hungría, que se iniciaron en 1894, precisamente durante la efervescencia de fin de siglo, consecuencia de la cual fue también el proceso iniciado

S E OME 32. Problema 2. Sugerencia

Usar el teorema del coseno para calcular la longitud de las medianas, y expresar la relaciondel enunciado de tal manera que se pueda sacar c − b como factor comun.

Page 320: COMPENDIUM OME - ToomatesEotvos de Hungría, que se iniciaron en 1894, precisamente durante la efervescencia de fin de siglo, consecuencia de la cual fue también el proceso iniciado

S E OME 32. Problema 3. Sugerencia

Escribir el polinomio en la forma f(x) = Ax(x + 1) + Bx(x − 1) + C(x − 1)(x + 1) paraciertas constantes A, B, C.

Page 321: COMPENDIUM OME - ToomatesEotvos de Hungría, que se iniciaron en 1894, precisamente durante la efervescencia de fin de siglo, consecuencia de la cual fue también el proceso iniciado

S E OME 32. Problema 4. Sugerencia

Aislar un radical y elevar al cuadrado.

Page 322: COMPENDIUM OME - ToomatesEotvos de Hungría, que se iniciaron en 1894, precisamente durante la efervescencia de fin de siglo, consecuencia de la cual fue también el proceso iniciado

S E OME 32. Problema 5. Sugerencia

Para un agente dado, ¿cuantos de los demas no le espıan ni son espiados por el? ¿Quesucede si el numero de estos agentes es cero?

Page 323: COMPENDIUM OME - ToomatesEotvos de Hungría, que se iniciaron en 1894, precisamente durante la efervescencia de fin de siglo, consecuencia de la cual fue también el proceso iniciado

S E OME 32. Problema 6. Sugerencia

Se trata de calcular el volumen de una piramide pentagonal truncada.

Page 324: COMPENDIUM OME - ToomatesEotvos de Hungría, que se iniciaron en 1894, precisamente durante la efervescencia de fin de siglo, consecuencia de la cual fue también el proceso iniciado

S E OME 33. Problema 1. Sugerencia

Recordar que 12 + 22 + · · ·+ n2 =n(n + 1)(2n + 1)

6.

Page 325: COMPENDIUM OME - ToomatesEotvos de Hungría, que se iniciaron en 1894, precisamente durante la efervescencia de fin de siglo, consecuencia de la cual fue también el proceso iniciado

S E OME 33. Problema 2. Sugerencia

Empezar dividiendo el cuadrado en conjuntos disjuntos de puntos de tal manera que comomucho dos de los puntos elegidos esten en cada conjunto.

Page 326: COMPENDIUM OME - ToomatesEotvos de Hungría, que se iniciaron en 1894, precisamente durante la efervescencia de fin de siglo, consecuencia de la cual fue también el proceso iniciado

S E OME 33. Problema 3. Sugerencia

Calcular la ecuacion de la circunferencia. ¿Que debe ocurrir con los coeficientes de p y qpara que haya un punto (x, y) tal que, al sustituir en la ecuacion, el resultado no dependade p ni de q?

Page 327: COMPENDIUM OME - ToomatesEotvos de Hungría, que se iniciaron en 1894, precisamente durante la efervescencia de fin de siglo, consecuencia de la cual fue también el proceso iniciado

S E OME 33. Problema 4. Sugerencia

Considerar la condicion del enunciado como una ecuacion cuadratica en k. ¿Cuando seranenteras sus raıces?

Page 328: COMPENDIUM OME - ToomatesEotvos de Hungría, que se iniciaron en 1894, precisamente durante la efervescencia de fin de siglo, consecuencia de la cual fue también el proceso iniciado

S E OME 33. Problema 5. Sugerencia

Demostrar, por separado, que la suma de los lados es mayor o igual que 4, y que la sumade las diagonales es mayor o igual que

√2. Recordar que es posible acotar la suma de dos

numeros a partir de su producto.

Page 329: COMPENDIUM OME - ToomatesEotvos de Hungría, que se iniciaron en 1894, precisamente durante la efervescencia de fin de siglo, consecuencia de la cual fue también el proceso iniciado

S E OME 33. Problema 6. Sugerencia

Dibujar la grafica del combustible restante en cada momento de un recorrido completo, em-pezando en un deposito cualquiera. ¿Cual serıa la grafica empezando desde otro deposito?

Page 330: COMPENDIUM OME - ToomatesEotvos de Hungría, que se iniciaron en 1894, precisamente durante la efervescencia de fin de siglo, consecuencia de la cual fue también el proceso iniciado

S E OME 34. Problema 1. Sugerencia

Demostrar que AP = PA′ y que A′M = MB.

Page 331: COMPENDIUM OME - ToomatesEotvos de Hungría, que se iniciaron en 1894, precisamente durante la efervescencia de fin de siglo, consecuencia de la cual fue también el proceso iniciado

S E OME 34. Problema 2. Sugerencia

Hay solo 12 cubos de 4 cifras, y un sencillo argumento de divisibilidad nos deja son solotres posibilidades.

Page 332: COMPENDIUM OME - ToomatesEotvos de Hungría, que se iniciaron en 1894, precisamente durante la efervescencia de fin de siglo, consecuencia de la cual fue también el proceso iniciado

S E OME 34. Problema 3. Sugerencia

Los triangulos ABC, ADC y ABE son semejantes.

Page 333: COMPENDIUM OME - ToomatesEotvos de Hungría, que se iniciaron en 1894, precisamente durante la efervescencia de fin de siglo, consecuencia de la cual fue también el proceso iniciado

S E OME 34. Problema 4. Sugerencia

¿Cuanto vale la tangente de la suma de tres angulos?

Page 334: COMPENDIUM OME - ToomatesEotvos de Hungría, que se iniciaron en 1894, precisamente durante la efervescencia de fin de siglo, consecuencia de la cual fue también el proceso iniciado

S E OME 34. Problema 5. Sugerencia

Sabemos que si y > x es f(y)− f(x) ≥ y − x. Demostrar que siempre se tiene la igualdad.

Page 335: COMPENDIUM OME - ToomatesEotvos de Hungría, que se iniciaron en 1894, precisamente durante la efervescencia de fin de siglo, consecuencia de la cual fue también el proceso iniciado

S E OME 34. Problema 6. Sugerencia

Colorear cada casilla como en un tablero de ajedrez. ¿De que color son las casillas ocupadaspor las piezas?

Page 336: COMPENDIUM OME - ToomatesEotvos de Hungría, que se iniciaron en 1894, precisamente durante la efervescencia de fin de siglo, consecuencia de la cual fue también el proceso iniciado

S E OME 35. Problema 1. Sugerencia

Para calcular el area, expresar esta como suma de las areas de dos triangulos, ambos conbase m.

Page 337: COMPENDIUM OME - ToomatesEotvos de Hungría, que se iniciaron en 1894, precisamente durante la efervescencia de fin de siglo, consecuencia de la cual fue también el proceso iniciado

S E OME 35. Problema 2. Sugerencia

Dado cualquier k entero, encontrar un entero x tal que k2 + x2 sea cuadrado perfecto.

Page 338: COMPENDIUM OME - ToomatesEotvos de Hungría, que se iniciaron en 1894, precisamente durante la efervescencia de fin de siglo, consecuencia de la cual fue también el proceso iniciado

S E OME 35. Problema 3. Sugerencia

¿Cuantas veces se la ha dado la vuelta a la ultima ficha al retirarla?

Page 339: COMPENDIUM OME - ToomatesEotvos de Hungría, que se iniciaron en 1894, precisamente durante la efervescencia de fin de siglo, consecuencia de la cual fue también el proceso iniciado

S E OME 35. Problema 4. Sugerencia

¿Cuantas cartas pueden aberse sacado si todas las sumas aparecen dos veces o menos?

Page 340: COMPENDIUM OME - ToomatesEotvos de Hungría, que se iniciaron en 1894, precisamente durante la efervescencia de fin de siglo, consecuencia de la cual fue también el proceso iniciado

S E OME 35. Problema 5. Sugerencia

a) Calcular ga, gb, gc en terminos del area del triangulo y las longitudes de los lados.

b) Utilizar la desigualdad entre las medias aritmetica y armonica.

Page 341: COMPENDIUM OME - ToomatesEotvos de Hungría, que se iniciaron en 1894, precisamente durante la efervescencia de fin de siglo, consecuencia de la cual fue también el proceso iniciado

S E OME 35. Problema 6. Sugerencia

Una vez anadidas las rectas de los dos primeros tipos, ¿como aumento el numero de regionesal anadir cada recta del tercer tipo? ¿Es posible acotar el numero de regiones en funcionunicamente del numero total de rectas?

Page 342: COMPENDIUM OME - ToomatesEotvos de Hungría, que se iniciaron en 1894, precisamente durante la efervescencia de fin de siglo, consecuencia de la cual fue también el proceso iniciado

S E OME 36. Problema 1. Sugerencia

Las raıces comunes a dos polinomios son tambien raıces de su diferencia.

Page 343: COMPENDIUM OME - ToomatesEotvos de Hungría, que se iniciaron en 1894, precisamente durante la efervescencia de fin de siglo, consecuencia de la cual fue también el proceso iniciado

S E OME 36. Problema 2. Sugerencia

Tras una breve observacion de la figura se deduce que las dos personas solo se puedenencontrar en cuatro segmentos; por tanto, resta calcular las respectivas probabilidades deencuentro en cada segmento.

Page 344: COMPENDIUM OME - ToomatesEotvos de Hungría, que se iniciaron en 1894, precisamente durante la efervescencia de fin de siglo, consecuencia de la cual fue también el proceso iniciado

S E OME 36. Problema 3. Sugerencia

Sea O el punto medio del segmento que une los centros de las dos circunferencias. El puntoM es el simetrico del punto B respecto de O.

Page 345: COMPENDIUM OME - ToomatesEotvos de Hungría, que se iniciaron en 1894, precisamente durante la efervescencia de fin de siglo, consecuencia de la cual fue también el proceso iniciado

S E OME 36. Problema 4. Sugerencia

Si z = E(N/3), entonces z es uno de los 9 multiplos de 111 con tres cifras y ademas cumple

que existe un natural n tal que z =n(n + 1)

2.

Page 346: COMPENDIUM OME - ToomatesEotvos de Hungría, que se iniciaron en 1894, precisamente durante la efervescencia de fin de siglo, consecuencia de la cual fue también el proceso iniciado

S E OME 36. Problema 5. Sugerencia

Considera el cuadrilatero formado por los cuatro puntos y distingue dos casos, segun quesea concavo o convexo.

Page 347: COMPENDIUM OME - ToomatesEotvos de Hungría, que se iniciaron en 1894, precisamente durante la efervescencia de fin de siglo, consecuencia de la cual fue también el proceso iniciado

S E OME 36. Problema 6. Sugerencia

Intenta demostrar por induccion que f(n) = f(0) + n.

Page 348: COMPENDIUM OME - ToomatesEotvos de Hungría, que se iniciaron en 1894, precisamente durante la efervescencia de fin de siglo, consecuencia de la cual fue también el proceso iniciado

S E OME 37. Problema 1. Sugerencia

Primero comprueba que que si existe el polinomio Q, entonces P es simetrico respectode A.

Para la otra implicacion ayudate de un polinomio R tal que R(h) = P (a + h).

Page 349: COMPENDIUM OME - ToomatesEotvos de Hungría, que se iniciaron en 1894, precisamente durante la efervescencia de fin de siglo, consecuencia de la cual fue también el proceso iniciado

S E OME 37. Problema 2. Sugerencia

Observa que los triangulos ACB, PBQ y APR son semejantes.

Page 350: COMPENDIUM OME - ToomatesEotvos de Hungría, que se iniciaron en 1894, precisamente durante la efervescencia de fin de siglo, consecuencia de la cual fue también el proceso iniciado

S E OME 37. Problema 3. Sugerencia

¿Que relacion cumplen los lados de un triangulo para que este sea obtusangulo? Utilizaestas relaciones obtenidas por medio del teorema del coseno junto con la desigualdadtriangular.

Page 351: COMPENDIUM OME - ToomatesEotvos de Hungría, que se iniciaron en 1894, precisamente durante la efervescencia de fin de siglo, consecuencia de la cual fue también el proceso iniciado

S E OME 37. Problema 4. Sugerencia

La suma de los numeros 1 al 9 es multiplo de 9, pero 2001 no lo es.

Page 352: COMPENDIUM OME - ToomatesEotvos de Hungría, que se iniciaron en 1894, precisamente durante la efervescencia de fin de siglo, consecuencia de la cual fue también el proceso iniciado

S E OME 37. Problema 5. Sugerencia

Intenta expresar la condicion de que el cuadrilatero admite circunferencia inscrita comouna ecuacion que tenga solamente como incognitas la longitud del lado CD y las razonestrigonometricas de un angulo.

Page 353: COMPENDIUM OME - ToomatesEotvos de Hungría, que se iniciaron en 1894, precisamente durante la efervescencia de fin de siglo, consecuencia de la cual fue también el proceso iniciado

S E OME 37. Problema 6. Sugerencia

Haz una tabla de valores y encuentra una relacion entre f(n) y la representacion de n enbase 2.

Page 354: COMPENDIUM OME - ToomatesEotvos de Hungría, que se iniciaron en 1894, precisamente durante la efervescencia de fin de siglo, consecuencia de la cual fue también el proceso iniciado

S E OME 38. Problema 1. Sugerencia

Calcula P (0) y, a partir de sus posibles valores, los polinomios P .

Page 355: COMPENDIUM OME - ToomatesEotvos de Hungría, que se iniciaron en 1894, precisamente durante la efervescencia de fin de siglo, consecuencia de la cual fue también el proceso iniciado

S E OME 38. Problema 2. Sugerencia

a) Utiliza que los angulos BA′H y CA′H son rectos para obtener relaciones metricas apartir del seno y del coseno de los angulos B y C.

b) Toma unos ejes con centro el punto medio de BC e intenta expresar la condicion quehas obtenido en el parrafo anterior.

Page 356: COMPENDIUM OME - ToomatesEotvos de Hungría, que se iniciaron en 1894, precisamente durante la efervescencia de fin de siglo, consecuencia de la cual fue también el proceso iniciado

S E OME 38. Problema 3. Sugerencia

Haz una tabla de valores y encuentra una relacion entre f(n) y la representacion de n enbase 2.

Page 357: COMPENDIUM OME - ToomatesEotvos de Hungría, que se iniciaron en 1894, precisamente durante la efervescencia de fin de siglo, consecuencia de la cual fue también el proceso iniciado

S E OME 38. Problema 4. Sugerencia

Sea n = abc. Sustituye en la expresion del enunciado y utiliza los modulos adecuados paraimponer condiciones sobre a, b, c.

Page 358: COMPENDIUM OME - ToomatesEotvos de Hungría, que se iniciaron en 1894, precisamente durante la efervescencia de fin de siglo, consecuencia de la cual fue también el proceso iniciado

S E OME 38. Problema 5. Sugerencia

Cada segmento determina dos vectores de sentidos opuestos. Une los 4004 vectores paraobtener un polıgono.

Page 359: COMPENDIUM OME - ToomatesEotvos de Hungría, que se iniciaron en 1894, precisamente durante la efervescencia de fin de siglo, consecuencia de la cual fue también el proceso iniciado

S E OME 38. Problema 6. Sugerencia

Cada lado o diagonal del polıgono pertenece exactamente a tres triangulos isosceles distin-tos. Divide tanto los lados y diagonales com los triangulos segun su coloracion, e intentabuscar expresiones que relacionen la cantidad de unos con la de otros.

Page 360: COMPENDIUM OME - ToomatesEotvos de Hungría, que se iniciaron en 1894, precisamente durante la efervescencia de fin de siglo, consecuencia de la cual fue también el proceso iniciado

S E OME 39. Problema 1. Sugerencia

Considera los p primeros numeros compuestos solo por nueves. ¿Existiran dos que den elmismo resto al dividir por 9?

Page 361: COMPENDIUM OME - ToomatesEotvos de Hungría, que se iniciaron en 1894, precisamente durante la efervescencia de fin de siglo, consecuencia de la cual fue también el proceso iniciado

S E OME 39. Problema 2. Sugerencia

Al ser M finito, estara acotado, M ⊂ [x, y]. Aplica la propiedad cel conjunto a los elementosx e y hasta llegar a una contradiccion con la existencia de M .

Page 362: COMPENDIUM OME - ToomatesEotvos de Hungría, que se iniciaron en 1894, precisamente durante la efervescencia de fin de siglo, consecuencia de la cual fue también el proceso iniciado

S E OME 39. Problema 3. Sugerencia

Considera primero el caso C < 90. Sea A′ el pie de la altura por A. Intenta demostrarque los triangulos CHA′ y A′AB son congruentes.

Page 363: COMPENDIUM OME - ToomatesEotvos de Hungría, que se iniciaron en 1894, precisamente durante la efervescencia de fin de siglo, consecuencia de la cual fue también el proceso iniciado

S E OME 39. Problema 4. Sugerencia

Prueba primero que x no puede ser entero y luego, haciendo x = p/q, que tampoco puedeser racional.

Page 364: COMPENDIUM OME - ToomatesEotvos de Hungría, que se iniciaron en 1894, precisamente durante la efervescencia de fin de siglo, consecuencia de la cual fue también el proceso iniciado

S E OME 39. Problema 5. Sugerencia

Prolonga los lados para formar un triangulo equilatero.

Page 365: COMPENDIUM OME - ToomatesEotvos de Hungría, que se iniciaron en 1894, precisamente durante la efervescencia de fin de siglo, consecuencia de la cual fue también el proceso iniciado

S E OME 39. Problema 6. Sugerencia

Considera la diferencia entre bolas blancas y negras de las 2n primeras bolas, y despues, delas 2n ultimas. A medida que te desplazas por la cadena, ¿como puede variar la diferencia?

Page 366: COMPENDIUM OME - ToomatesEotvos de Hungría, que se iniciaron en 1894, precisamente durante la efervescencia de fin de siglo, consecuencia de la cual fue también el proceso iniciado

S E OME 40. Problema 1. Sugerencia

Halla primero la suma de cada fila en funcion de su primer y ultimo elemento, y luegosuma todas las filas.

Page 367: COMPENDIUM OME - ToomatesEotvos de Hungría, que se iniciaron en 1894, precisamente durante la efervescencia de fin de siglo, consecuencia de la cual fue también el proceso iniciado

S E OME 40. Problema 2. Sugerencia

Los cuadrilateros TPZD y TOZD tienen la misma area.

Page 368: COMPENDIUM OME - ToomatesEotvos de Hungría, que se iniciaron en 1894, precisamente durante la efervescencia de fin de siglo, consecuencia de la cual fue también el proceso iniciado

S E OME 40. Problema 3. Sugerencia

Prueba por induccion que para todo n entero se cumple f(x + n f(y)

)= f(x) − n y.

Page 369: COMPENDIUM OME - ToomatesEotvos de Hungría, que se iniciaron en 1894, precisamente durante la efervescencia de fin de siglo, consecuencia de la cual fue también el proceso iniciado

S E OME 40. Problema 4. Sugerencia

Observa que si un numero se forma reordenando los dıgitos de otro, entonces los dos dejanel mismo resto al dividirlos por 9.

Page 370: COMPENDIUM OME - ToomatesEotvos de Hungría, que se iniciaron en 1894, precisamente durante la efervescencia de fin de siglo, consecuencia de la cual fue también el proceso iniciado

S E OME 40. Problema 5. Sugerencia

Utiliza el teorema de Steward (formula de la longitud de la mediana) y la potencia de Brespecto de la circunferencia inscrita.

Page 371: COMPENDIUM OME - ToomatesEotvos de Hungría, que se iniciaron en 1894, precisamente durante la efervescencia de fin de siglo, consecuencia de la cual fue también el proceso iniciado

S E OME 40. Problema 6. Sugerencia

Primer caso: 2004 fichas. No se puede llegar a la configuracion pedida. Notese que cadamovimiento posible cambia en un numero impar la cantidad de fichas negras, por lo queel numero total de movimientos debe ser impar. Ahora busquese un razonamiento quedemuestre lo contrario, esto es, que el numero de movimientos debe ser par.

Segundo caso: 2003 fichas. Sı se puede.

Page 372: COMPENDIUM OME - ToomatesEotvos de Hungría, que se iniciaron en 1894, precisamente durante la efervescencia de fin de siglo, consecuencia de la cual fue también el proceso iniciado

E OME 1. Problema 1. Solucion

El discriminante de la ecuacion es a2 − 4, que tiene que ser estrictamente menor que cero;por lo tanto el intervalo pedido para a es

−2 < a < 2.

En esas condiciones, las raıces de la ecuacion son

−a

2± i

√4 − a2

2,

ası que las ecuaciones parametricas del lugar geometrico buscado, tomando como parame-tro a, son

x = −a

2

y = ±√

4 − a2

2.

Eliminando el parametro resulta

x2 + y2 = 1.

Cuando −2 < a < 2 resulta −1 < x < 1, ası que el lugar geometrico es la circunferenciaunidad, de ecuacion compleja |z| = 1, de la que se han eliminado los puntos (1, 0) y (0,−1).

Page 373: COMPENDIUM OME - ToomatesEotvos de Hungría, que se iniciaron en 1894, precisamente durante la efervescencia de fin de siglo, consecuencia de la cual fue también el proceso iniciado

E OME 1. Problema 2. Solucion

En los tres intervalos la derivada es constante, luego la funcion es lineal y podemos expre-sarla

f (x) =

0 0 ≤ x < 60000x − 60000 60000 ≤ x < P

P − 60000 + 0.14 (x − P ) x ≥ P.

Imponiendo la condicion f (140000) = 14000, resulta14000 = P − 60000 + 0.14 (140000− P ) =⇒ P = 63255.81.

O 60000 P 140000 X

P−60000

14000

Y

Page 374: COMPENDIUM OME - ToomatesEotvos de Hungría, que se iniciaron en 1894, precisamente durante la efervescencia de fin de siglo, consecuencia de la cual fue también el proceso iniciado

E OME 1. Problema 3. Solucion

Claramente n ≥ 3. Calculemos en primer lugar el numero de diagonales dn de un polıgonode n lados. Una diagonal es un segmento que une dos vertices de un polıgono, que noes un lado. Por tanto, como cada segmento diagonal o lado queda determinado por suspuntos extremos, el numero total de segmentos determinados por n puntos (vertices), esel numero de posibles pares de puntos (sin importar el orden) que se pueden formar conestos n puntos, es decir

(n2

)Entonces

dn =(

n

2

)− n =

n(n − 3)2

.

a) Para calcular el numero total de puntos donde se cortan las rectas diagonales (rectasdeterminadas por los extremos de cada segmento diagonal), observamos que el numeromaximo de intersecciones de k rectas en el plano es

(k2

), (k entero positivo). Ası las dn

rectas diagonales determinarıan(dn

2

)puntos, si no fuera por la restriccion de que por

cada vertice del polıgono pasan n − 3 rectas diagonales; es decir que concurren en cadavertice

(n−3

2

)puntos interseccion de estas rectas diagonales y en total en los n vertices

n(n−3

2

)puntos interseccion. Por tanto el numero total de puntos donde se cortan las

rectas diagonales es

P =(

dn

2

)− n

(n − 3

2

)=

n(n − 3)(n2 − 7n + 14)8

.

b) El numero PI de puntos interiores al polıgono donde se cortan las rectas diagonaleso segmentos diagonales simplemente, se obtiene con facilidad observando que cada puntointerseccion en el interior del polıgono queda determinado por los extremos de los dossegmentos diagonales que son cuatro vertices del polıgono dado. Recıprocamente, cadaeleccion de 4 vertices entre los n vertices del polıgono (ahora n ≥ 4) determina un unicopunto interior de interseccion de dos diagonales. Por tanto

PI =(

n

4

).

Para un triangulo obviamente d3 = 0 y PI = 0.Llamando PE al numero de puntos exteriores al polıgono dado donde se cortan las diago-nales resulta que

PE = P − PI =(n(n−3)

2

2

)− n

(n − 3

2

)−

(n

4

)=

n(n − 3)(n2 − 9n + 20)12

, para n ≥ 4.

Para un triangulo, PE = 0.

Page 375: COMPENDIUM OME - ToomatesEotvos de Hungría, que se iniciaron en 1894, precisamente durante la efervescencia de fin de siglo, consecuencia de la cual fue también el proceso iniciado

E OME 1. Problema 4. Solucion

Denotemos con B′C ′ a uno cualquiera de esos segmentos, y completemos, con un punto A′

sobre la cuerda AB, el triangulo A′C ′B′ (equilatero) de lados paralelos a los del trianguloABC. La longitud B′C ′ sera maxima cuando la altura del A′C ′B′ sea maxima, lo quecorresponde evidentemente al caso en que el punto B′ es justamente B0, el punto mediodel arco AB. La longitud maxima sera entonces (vease la figura) B0A0 = a

3.

B

A C

B0A0

B′ A′C0

C ′

a/3

Page 376: COMPENDIUM OME - ToomatesEotvos de Hungría, que se iniciaron en 1894, precisamente durante la efervescencia de fin de siglo, consecuencia de la cual fue también el proceso iniciado

E OME 1. Problema 5. Solucion

5 5 10

5 10

En cada figura se ha dibujado en color gris un representante de cada clase de triangulos yen el centro el numero de los que hay en esa clase.

Page 377: COMPENDIUM OME - ToomatesEotvos de Hungría, que se iniciaron en 1894, precisamente durante la efervescencia de fin de siglo, consecuencia de la cual fue también el proceso iniciado

E OME 1. Problema 6. Solucion

Primera solucion

Tomemos en primer lugar la variable independiente auxiliar ξ = x − 1. Se trata de repre-sentar la funcion

y(ξ) =∣∣∣ ∣∣|ξ| − 2

∣∣ − 3∣∣∣

en el intervalo −9 ≤ ξ ≤ 7.Pero la funcion y(ξ) es par, ası que basta trazar su grafica en el intervalo [0, 9], para tener,por una simetrıa hacia la izquierda respecto del eje ξ = 0, y una truncacion a la derechaque omita la parte correspondiente al intervalo [7, 9], la grafica completa en el intervalo[−9, 7].Nos planteamos, pues, representar

y(ξ) =∣∣|ξ − 2| − 3

∣∣en el intervalo 0 ≤ ξ ≤ 9.Consideremos que

y(ξ) =

|ξ − 5| si ξ ≥ 2 =

ξ − 5 si ξ ≥ 5,5 − ξ si 2 ≤ ξ ≤ 5,

| − ξ − 1| = ξ + 1 si 0 ≤ ξ ≤ 2.

Con lo que la grafica de la funcion y(x) = y(ξ + 1), compuesta de segmentos rectilıneos,queda ası:

−8 −4 −1 1 3 6 8

1

4

Segunda solucion

Basta partir de la conocida grafica de y = |x – 1| y seguir las siguientes transformaciones:- Traslacion de vector (0,−2) y simetrıa respecto de OX de la parte de la grafica situadabajo el eje X (de puntos en la figura), ası obtenemos la grafica de y = ||x − 1| − 2|.

Page 378: COMPENDIUM OME - ToomatesEotvos de Hungría, que se iniciaron en 1894, precisamente durante la efervescencia de fin de siglo, consecuencia de la cual fue también el proceso iniciado

- Traslacion de vector (0,−3) y simetrıa respecto de OX de la parte de la grafica situadabajo el eje X (de puntos en la figura), ası obtenemos la grafica de y = |||x − 1| − 2| − 3|.

−8 −4 −1 1 3 6 8

1

4

|x − 1|∣∣|x − 1| − 2

∣∣∣∣∣∣∣|x − 1| − 2

∣∣ − 3∣∣∣

Page 379: COMPENDIUM OME - ToomatesEotvos de Hungría, que se iniciaron en 1894, precisamente durante la efervescencia de fin de siglo, consecuencia de la cual fue también el proceso iniciado

E OME 1. Problema 7. Solucion

La operacion efectuada sobre n fichas es una permutacion del conjunto

N = 1, 2, 3, . . . , n− 2, n − 1, n ,

es decir una funcion biyectiva definida ası:

f(k) = k + 2 si k ≤ n − 3f (n − 2) = 1f (n − 1) = n

f (n) = 2.

Distinguiremos dos casos.

- n es par, y entonces podemos escribir f con la notacion habitual de ciclos como

f : (1, 3, 5, . . . , n − 1, n, 2, 4, 6, . . . , n − 2)

donde cada elemento tiene por imagen el de su derecha y el ultimo el primero.Es un ciclo de longitud n y por tanto de orden n, es decir la aplicacion sucesiva n vecesde f es la identidad y por tanto en el caso de 1000 fichas despues de 1000 ejecuciones delproceso descrito el fichero queda como estaba.

- n es impar, entonces f es producto de dos ciclos:

f : (1, 3, 5, . . . , n − 2) (2, 4, 6, . . . , n − 1, n)

de ordenesn − 1

2y

n + 12

respectivamente, que son primos entre sı al ser consecutivos.

Por tanto el orden de f en este caso esn2 − 1

4= n para cualquier natural n.

Page 380: COMPENDIUM OME - ToomatesEotvos de Hungría, que se iniciaron en 1894, precisamente durante la efervescencia de fin de siglo, consecuencia de la cual fue también el proceso iniciado

E OME 1. Problema 8. Solucion

Sea C el extremo del segmento que recorre la semicircunferencia dada, M el punto mediodel mismo y α el angulo que forma AC con la horizontal AB, (0 < α < π

2).

Sea N la proyeccion ortogonal del punto medio M sobre el segmento AB. Ponemos y =MN . Se desea hallar el valor de cosα para el que M ocupe el punto mas bajo, es decirpara el que la distancia y sea maxima.

A B

C

M

N

y

α

En el triangulo MNA, rectangulo en N , es y = AM senα y en el triangulo rectanguloCBA, es AC = a cosα. Pero AC = AM + MC = AM + a

2 , porque M es el punto mediodel segmento uno de cuyos extremos es C.Entonces AM = AC − MC = a cosα − a

2y

y =(a cosα − a

2

)senα =

a

2(2 cosα − 1) senα =

a

2(sen 2α − senα).

Esta funcion es continua y derivable en todos los numeros reales α.Derivando esta funcion respecto de α, obtenemos y′ =

a

2(2 cos 2α − cosα) e igualando a 0

la derivada y′, llegamos a la ecuacion 4 cos2 α − cosα − 2 = 0. De aquı encontramos dosposibles soluciones cosα1 = 1+

√33

8y cosα2 = 1−

√33

8, porque

∣∣∣1+√

338

∣∣∣ < 1 y∣∣∣1−

√33

8

∣∣∣ < 1.La derivada segunda de y es

y′′ =a

2(−4 sen 2α + senα) =

a

2senα (1 − 8 cosα)

y entonces y′′(α1) = −a2

√33 senα1 < 0, pues

sen α1 = ±

√1 − (1 +

√33)2

64= ±

√15 −

√33

32

y elegimos la solucion positiva para que α1 este en el primer cuadrante. Ası senα1 > 0.Entonces y alcanza un maximo en α = α1.Rechazamos la segunda solucion α2 que no esta en el primer cuadrante porque cosα2 esnegativo.Ası el valor maximo de y es:

Page 381: COMPENDIUM OME - ToomatesEotvos de Hungría, que se iniciaron en 1894, precisamente durante la efervescencia de fin de siglo, consecuencia de la cual fue también el proceso iniciado

y(α1) =a

2sen α1 (2 cos α1 − 1) =

a

2

√15 −

√33

32

(21 +

√33

8− 1

)=

=a

32(√

33 − 3)

√15 −

√33

2.

La respuesta es cos α1 =1 +

√33

8.

Page 382: COMPENDIUM OME - ToomatesEotvos de Hungría, que se iniciaron en 1894, precisamente durante la efervescencia de fin de siglo, consecuencia de la cual fue también el proceso iniciado

E OME 2. Problema 1. Solucion

B

A′

B′

C

C′

AO

x

2x √3x

P

Q

El area pedida es la diferencia entre el area del triangulo dado ABC y los tres triangulosgrises de la figura.Tenemos

[ABC] = 312

16√

32

= 12√

3, BC = 4√

3.

Para calcular el area de uno de los triangulos grises (BPQ de la figura), basta tener encuenta que sus angulos valen 90, 60 y 30 y si ponemos x = BQ, entonces PQ =

√3x

y BP = 2x.

[BPQ] =√

32

x2.

El area pedida S es:

S = 12√

3 − 3√

32

x2.

Solo nos queda calcular x; para ello basta tener en cuenta que:

x + 2x +√

3x = 4√

3 ⇐⇒ x = 2(√

3 − 1)

.

Sustituyendo en la expresion anterior y operando, resulta:

S = 12√

3 − 3√

32

x2 = 12√

3 − 3√

32

8(2 −

√3)

= 12(3 −

√3)

.

Page 383: COMPENDIUM OME - ToomatesEotvos de Hungría, que se iniciaron en 1894, precisamente durante la efervescencia de fin de siglo, consecuencia de la cual fue también el proceso iniciado

E OME 2. Problema 2. Solucion

a) Las cifras pueden repetirse. Sea n > 1 la cifra central. La de la izquierda puede ser unacifra cualquiera entre 1 y n− 1. La de la derecha puede ser una cualquiera entre 0 y n− 1.En total hay

9∑2

n(n − 1) =8∑1

n(n + 1) =8∑1

n2 +8∑1

n =8 · 9 · 17

6+

9 · 82

= 204 + 36 = 240.

b) Las cifras no pueden repetirse. Sea n > 1 la cifra central. La de la izquierda puede seruna cifra cualquiera entre 1 y n − 1. La de la derecha puede ser una cualquiera entre 0 yn − 1, excepto la utilizada a la izquierda. En total hay

9∑2

(n − 1)2 =8∑1

n2 =8 · 9 · 17

6= 204.

Page 384: COMPENDIUM OME - ToomatesEotvos de Hungría, que se iniciaron en 1894, precisamente durante la efervescencia de fin de siglo, consecuencia de la cual fue también el proceso iniciado

E OME 2. Problema 3. Solucion

Primera solucion

Adoptemos las siguientes notaciones: R0 = 29/2, radio exterior del surco; R1 = 11.5/2,radio interior del mismo; ∆R = R1 − R0 = 17.5/2; T = 1470 s, tiempo de audicion;ω = 33 1

3 rpm = 10π/9 rad/s, velocidad angular del disco.En un instante t de la reproduccion del disco (0 ≤ t ≤ T ), la aguja esta a una distanciaR(t) del centro que viene dada evidentemente (supuesta la regularidad de la espiral queforma el surco) por la formula

R(t) = R0 −∆R

Tt =

292

− t

168.

Y en ese mismo instante, la velocidad lineal V (t) a la que el surco esta pasando por debajode la aguja es

V (t) = ωR(t) =145π

9− 5π

756t .

El elemento diferencial de “longitud de surco” en el instante t es ds = V (t) dt. Luego lalongitud del surco, mediante una sencilla integracion, es

S =∫ T

0

ds =∫ 1470

0

(145π

9− 5π

756t

)dt =

[145π

9t − 5π

756t2

2

]1470

0

= 16 537.5π.

Segunda solucion

El surco de un disco forma una espiral que suponiendo constante la anchura del surco y laseparacion entre surcos es una espiral de Arquımedes de ecuacion polar:

ρ = aϕ

Con los datos que nos proporciona podemos hallar a y la longitud de la espiral.Llamando R y r a los radios exterior e interior de la corona que forma la parte grabada,tenemos:

R = aϕ2

r = aϕ1

⇒ (R − r) = a (ϕ2 − ϕ1) ⇒ a =

R − r

ϕ2 − ϕ1

siendo ϕ2 − ϕ1 el angulo descrito por la espiral en la zona grabada que podemos calcularsabiendo que en total se han dado 33 1

3 ·24.5 = 816.66 vueltas que corresponden a un angulode 2 · 816.66π = 5131.268 radianes. Entonces

a =R − r

ϕ2 − ϕ1=

14.5 − 5.755131.27

= 0.0017

Page 385: COMPENDIUM OME - ToomatesEotvos de Hungría, que se iniciaron en 1894, precisamente durante la efervescencia de fin de siglo, consecuencia de la cual fue también el proceso iniciado

la longitud s de arco ente los valores inicial ϕ1 = 5.75a = 3371.976 y final ϕ2 = 14.5

a =8503.244 es:

s = a

∫ ϕ2

ϕ1

√1 + ϕ2dϕ =

a

2

[ϕ√

1 + ϕ2 + ln(ϕ +

√1 + ϕ2

)]ϕ2

ϕ1

= 519.540893 m.

Tercera solucion

Tanto por el tipo de ecuacion en polares como por la artillerıa usada (la integral para lalongitud del arco no es precisamente inmediata) no parece que esta sea la solucion pensadapor el autor del problema.Podemos hacerlo con una aproximacion “razonable” consistente en considerar cırculosconcentricos cuyos radios estan en progresion aritmetica:

Longitud primera vuelta = π · 29 = 36.1283155Longitud ultima vuelta = π · 11.5 = 91.106187

Numero de vueltas = 3313· 24.5 = 816.66

Longitud total =(36.1283155 + 91.106187)816.66

2= 519.5408851m.

La aproximacion es mas que razonable ya que las soluciones difieren en menos de unacentesima de milımetro.

Page 386: COMPENDIUM OME - ToomatesEotvos de Hungría, que se iniciaron en 1894, precisamente durante la efervescencia de fin de siglo, consecuencia de la cual fue también el proceso iniciado

E OME 2. Problema 4. Solucion

1. Suponemos primero que 0 ≤ x ≤ 2π.Consideramos una circunferencia de radio unidad centrada en el origen de coordenadas O.Sea P un punto de la misma y x el angulo medido en sentido antihorario que forma elsemieje positivo de abscisas con OP . Entonces P tiene de coordenadas (cosx, sen x). Seaahora Q la proyeccion del punto P sobre el eje de abscisas. Entonces la desigualdadprimera cos x + sen x > 1 puede interpretarse geometricamente en el triangulo rectanguloOPQ como PQ+OQ > OP , es decir que la suma de las longitudes de los catetos es mayorque la longitud de la hipotenusa. Esta relacion se cumple obviamente para los angulos xdel primer cuadrante: 0 < x < π

2 . En otro cuadrante cualquiera o bien el senx o bien elcos x o los dos son negativos y como ademas sen x < 1 y cos x < 1 no se puede tener estadesigualdad primera.

La segunda desigualdad cos x + |sen x| > 1, requiere al igual que la primera que sus dossumandos sean estrictamente positivos y esta condicion solo se cumple si cos x > 0 y

sen x = 0 es decir si 0 < x <π

2o si

2< x < 2π.

2. Supongamos que el angulo x es cualquier numero real.Entonces la primera desigualdad se cumple para 2πn < x < π

2 + 2πn siendo n un entero

cualquiera. Y la segunda desigualdad para −π

2+2πm < x <

π

2+2πm siendo m cualquier

entero positivo, negativo o nulo.

Page 387: COMPENDIUM OME - ToomatesEotvos de Hungría, que se iniciaron en 1894, precisamente durante la efervescencia de fin de siglo, consecuencia de la cual fue también el proceso iniciado

E OME 2. Problema 5. Solucion

Sucede que operando en la hipotesis tenemos

3x − b

3x − 5b=

3a − 4b

3a − 8b⇐⇒ b (x − a + b) = 0

y por tanto para que se cumpla la hipotesis se debe dar necesariamente uno de estos casos:

a) b = 0. Entonces la hipotesis se reduce a3x

3x=

3a

3a= 1 y no hay discrepancia con la

tesis.

b) x−a+b = 0, en cuyo caso no podemos aplicar la propiedad pues quedarıa el denominador(y el numerador) cero.

Page 388: COMPENDIUM OME - ToomatesEotvos de Hungría, que se iniciaron en 1894, precisamente durante la efervescencia de fin de siglo, consecuencia de la cual fue también el proceso iniciado

E OME 2. Problema 6. Solucion

El cırculo inscrito en el triangulo es el cırculo menor de la esfera sobre el que reposa eltriangulo; sea I su centro, que pertenece al plano del triangulo, a cuyos vertices llamamosA, B, C. Sea O el centro de la esfera y P el punto de tangencia del cırculo inscrito con ellado AB (por ejemplo) del triangulo. P es un punto de la superficie de la esfera.El triangulo IPA es rectangulo en P ; llamando ρ al radio del cırculo inscrito, se tiene

1√3

= tan 30 =ρ2

=2ρ

,

ası que ρ =

2√

3.

Como ρ < r, debe ser

2√

3< r, es decir < 2

√3r para que se cumplan las condiciones

del enunciado y la esfera no pase a traves del triangulo.El teorema de Pitagoras en IPA permite escribir

IA2 = IP 2 + PA2 =(

2

)2

+(

2√

3

)2

=2

3;

en el triangulo rectangulo IPO nuevamente el teorema de Pitagoras da

IO2 = r2 − ρ2 = r2 − 2

12;

y finalmente, de nuevo, el teorema de Pitagoras en IAO da la distancia del centro de laesfera a los vertices del triangulo:

OA2 = IO2 + IA2 = r2 − 2

12+

2

3= r2 +

2

4,

luego

OA =

√r2 +

2

4.

Page 389: COMPENDIUM OME - ToomatesEotvos de Hungría, que se iniciaron en 1894, precisamente durante la efervescencia de fin de siglo, consecuencia de la cual fue también el proceso iniciado

E OME 2. Problema 7. Solucion

El volumen de un cono de radio de la base R y altura H es V = 13πR2H. Dado que

H = mR (m > 0 fijo), tendremos

V =13πR3m.

La masa del tronco de cono del problema, en funcio del radio 0 ≤ x < r de su base menores

M(x) =(

13πr3m − 1

3πx3m

)d + πx2p .

Derivando, se tieneM ′(x) = −πx2md + 2πpx,

yM ′′(x) = −2πmdx + 2pπ.

La derivada se anula para x0 = 0 y para x1 = 2pmd . El punto x0 es el extremo izquierdo del

intervalo de variacion de x, donde se va a alcanzar el valor mınimo M(0) = 0, y el puntox1 corresponde a un maximo relativo de M(x), pues M ′′(x1) = M ′′ ( 2p

md

)= −2pπ < 0.

Cuando este segundo punto crıtico sea interior al intervalo, es decir, si

2p

md< r,

en el se alcanzara el valor maximo de la funcion M(x) cuando x ∈ [0, r), siendo en estecaso la altura del tronco de cono igual a

h = m(r − x1) = m

(r − 2p

md

)= mr − 2

p

d.

En caso contrario, es decir,

si2p

md≥ r,

el valor maximo de M(x) serıa el

limx→r−

M(x) = πpr2,

que no se alcanza, pues corresponderıa a un tronco de cono de altura h = m(r − r) = 0.

Page 390: COMPENDIUM OME - ToomatesEotvos de Hungría, que se iniciaron en 1894, precisamente durante la efervescencia de fin de siglo, consecuencia de la cual fue también el proceso iniciado

E OME 2. Problema 8. Solucion

Por conveniencia, llamaremos r1 = r, O1 a su centro, T1 al punto de tangencia de una delas rectas con γ1 y T2 al de γ2 con esta misma recta.

γ1

γ2

γ3

O1 O2 O3P

T1

T2

T3

Los triangulos PT2O2 y PT1O1 son semejantes; la proporcionalidad existente entre suslados nos permite calcular r2, el radio de γ2

a

a − r1 − r2=

r1

r2⇐⇒ r2 = r1 ·

a − r1

a + r1.

Calculando de manera similar r3 resulta

r3 = r1 ·(

a − r1

a + r1

)2

,

y eso nos hace conjeturar que los radios de las circunferencias formaran una progresiongeometrica de razon

a − r1

a + r1< 1,

conjetura que se prueba por induccion, utilizando la semejanza entre los triangulos POnTn

y POn−1Tn−1.Entonces la expresion general para el radio de γn sera

rn = r1 ·(

a − r1

a + r1

)n−1

.

Y por ultimo, el lımite de la suma de las longitudes de las circunferencias es

limn→∞

2π (r1 + r2 + · · ·+ rn) = 2π · r1

1 − a−r1a+r1

= π (a + r1) .

Page 391: COMPENDIUM OME - ToomatesEotvos de Hungría, que se iniciaron en 1894, precisamente durante la efervescencia de fin de siglo, consecuencia de la cual fue también el proceso iniciado

E OME 3. Problema 1. Solucion

Pongamos x, y, z para las unidades de precios 50, 70 y 65 respectivamente. Tenemosobviamente

50x + 70y + 65z = 6850x + y + z = 100

Se trata de resolver el sistema anterior en los naturales haciendo y maximo.Simplificando y eliminando z, resulta:

y = 70 + 3x

perox + y ≤ 100 =⇒ 70 + 4x ≤ 100 =⇒ x ≤ 7.

Dado que debemos hacer y maximo, debemos tomar el valor maximo posible de x (ya quey = 70 + 3x). Por tanto

x = 7, z = 2, y = 91.

Page 392: COMPENDIUM OME - ToomatesEotvos de Hungría, que se iniciaron en 1894, precisamente durante la efervescencia de fin de siglo, consecuencia de la cual fue también el proceso iniciado

E OME 3. Problema 2. Solucion

En el sistema de base 7 solo se utilizan los numeros enteros del 0 al 6, y en el de base 9,del 0 al 8. Entonces los numeros x, y, z estan entre 0 y 6.El numero xyz en base 7, representa el numero x 72 + y 7 + z en base 10. Y el numero zxyen el sistema de numeracion de base 9, representa en base 10 el numero z 92 + y 9 + x.Por tanto x 72 + y 7 + z = z 92 + y 9 + x, que es equivalente a 8(3x − 5z) = y, con x, y, znumeros enteros comprendidos entre 0 y 6 inclusive. Entonces y debe ser 0 y 3x = 5z, dedonde x = z = 0 o x = 5 y z = 3. La primera conduce al numero 000 que no es de trescifras y la segunda al numero 503 en base 7 y al numero 305 en base 9. Ambos numerosson en base 10 el numero 248, y esta es la respuesta.

Page 393: COMPENDIUM OME - ToomatesEotvos de Hungría, que se iniciaron en 1894, precisamente durante la efervescencia de fin de siglo, consecuencia de la cual fue también el proceso iniciado

E OME 3. Problema 3. Solucion

a) Podemos considerar sin perdida de generalidad que el lado mide 1 con lo que la diagonald se calcula mediante la semejanza entre los triangulos sombreados:

d

1=

1d − 1

=⇒ d2 − d − 1 = 0 (y d ≥ 1) =⇒ d =1 +

√5

2.

1

d

Este numero es el llamado numero aureo y se suele designar por Φ. Cumple Φ2 = Φ + 1.La relacion r de semejanza del pentagono grande al pequeno es:

r =1a

=1

2 − Φ=

3 +√

52

.

b) La relacion de las areas es

r2 =7 − 3

√5

2.

c) Los dos pentagonos son homoteticos en una homotecia de centro el centro del pentagono

y razon −3 +√

52

Page 394: COMPENDIUM OME - ToomatesEotvos de Hungría, que se iniciaron en 1894, precisamente durante la efervescencia de fin de siglo, consecuencia de la cual fue también el proceso iniciado

E OME 3. Problema 4. Solucion

A B

M

P

x

7−x

La funcion que expresa el costo es:

f(x) = p (7 − x) + 2q√

x2 + 4.

Tenemos que buscar su mınimo cuando 0 ≤ x ≤ 7.Derivando e igualando a cero resulta:

f ′ (x) = −p +2qx√x2 + 4

= 0 ⇐⇒ x =2p√

4q2 − p2.

Distinguiremos dos casos:

a) p <14√53

q. Como14√53

< 2, en este caso existe el punto crıtico

x0 =2p√

4q2 − p2

que ademas es un mınimo ya que volviendo a derivar resulta:

f ′′(x) =8q

(x2 + 4)3/2> 0

y se tiene que 0 < x0 < 7, como se puede comprobar de forma inmediata. En este casolas longitudes del cable y de la cadena pa el precio mas economico son, respectivamente,7 − x0 y 2

√x2

0 + 4.

b) p ≥ 14√53

q. En este caso no hay ningun punto crıtico interior al intervalo [0, 7]. Ademas

f ′(x) = −p +2qx√x2 + 4

< −p + 2q7√53

≤ 0,

la funcion es decreciente y el maximo se alcanza para x0 = 7. En esta caso la longitud delcable es 0 y la de la cadena es 2

√53.

Page 395: COMPENDIUM OME - ToomatesEotvos de Hungría, que se iniciaron en 1894, precisamente durante la efervescencia de fin de siglo, consecuencia de la cual fue también el proceso iniciado

E OME 3. Problema 5. Solucion

A N ′ Bc

M

N

C

M ′

b s

t

Denominamos AB = c, AC = b, AM = t, AN = s, BM = k y MC = a − k.a) Sean los angulos α = AMB, 180 − α = AMC. Aplicamos el teorema del coseno alos triangulos ABM y ACM utilizando respectivamente en cada triangulo los angulos αy 180 − α, resultando

c2 = k2 + t2 − 2kt cosα, b2 = (a − k)2 + t2 − 2(a − k)t cos(180o − α);

eliminando ahora cosα en estas dos ecuaciones, reduciendo terminos y simplificando lle-gamos a

kb2 + (a − k)c2 = at2 + ak(a− k)

y procediendo de manera similar con los triangulos ANC y ANB, tenemos

kc2 + (a − k)b2 = as2 + ak(a− k).

Sumando estas dos ecuaciones y teniendo en cuenta que b2 + c2 = a2 obtenemos que

a3 = a(t2 + s2) + 2ak(a − k),

con lo que AM2 + AN2 = t2 + s2 = a2 − 2k(a − k).b) La base del triangulo AMN correspondiente al vertice A mide a− 2k.Los triangulos ABC y AMN tienen la misma altura correspondiente al vertice A. Portanto

Area(AMN )Area(ABC)

=a − 2k

a.

c) Los puntos A, M ′, N ′ estan en una circunferencia de diametro M ′N ′ porque el trianguloM ′N ′A es rectangulo en A. Se observa que BC es paralelo a M ′N ′ por la semejanza de lostriangulos rectangulos ABC y AN ′M ′. La proyeccion del segmento BM , que es paraleloa M ′N ′, sobre AC es AM ′ y la proyeccion de M ′N ′ sobre AC es tambien AM ′. Por tantoM ′N ′ = k. Entonces el area del cırculo de diametro M ′N ′ = k es

π

4k2.

Page 396: COMPENDIUM OME - ToomatesEotvos de Hungría, que se iniciaron en 1894, precisamente durante la efervescencia de fin de siglo, consecuencia de la cual fue también el proceso iniciado

E OME 3. Problema 6. Solucion

Hay que calcular de cuantas maneras se han podido producir los 5 nacimientos, en lascondiciones del problema, y cual es la probabilidad de cada una.La probabilidad asociada al nacimiento de los 5 hijos es

(12

)5

=132

.

En las condiciones del problema, el suceso 4V1M se puede producir de P 54,1 = 5 maneras

diferentes; el suceso 3V2M de P 52,3 = 10 maneras distintas, y el suceso 2V3M de P 5

3,2 = 10maneras distintas. En total, 25; por lo tanto la probabilidad pedida es

p =2532

.

Page 397: COMPENDIUM OME - ToomatesEotvos de Hungría, que se iniciaron en 1894, precisamente durante la efervescencia de fin de siglo, consecuencia de la cual fue también el proceso iniciado

E OME 3. Problema 7. Solucion

Llamando a al termino central y r a la razon, tenemos el sistema:

a( 1

r3+

1r2

+1r

)= 7

a(r + r2 + r3

)= 112

cuya solucion es r = 2 ; a = 8 y la progresion es: 1, 2, 4, 8, 16, 32, 64.

Page 398: COMPENDIUM OME - ToomatesEotvos de Hungría, que se iniciaron en 1894, precisamente durante la efervescencia de fin de siglo, consecuencia de la cual fue también el proceso iniciado

E OME 3. Problema 8. Solucion

Tenemos:

y′(x) = 3ax2 + 2bx + c; y′′ (x) = 6ax + 2b

y como la tangente pasa por el punto P (3, 1) y la pendiente de la recta vale 14, las condi-

ciones del enunciado se traducen en:

y(3) = 1, y′(3) =14, y′′(3) = 0,

que corresponde a

27a + 9b + 3c = 127a + 6b + c = 1

4

18a + 2b = 0.

Este sistema nos lleva a la solucion:

y =x3

108− x2

12+

x

2

cuya grafica es

1

3

Page 399: COMPENDIUM OME - ToomatesEotvos de Hungría, que se iniciaron en 1894, precisamente durante la efervescencia de fin de siglo, consecuencia de la cual fue también el proceso iniciado

E OME 4. Problema 1. Solucion

Primeramente indicamos que el enunciado hace notar el crecimiento de la funcion f en losextremos del intervalo [−8, 8], de tal modo que f es monotona creciente en −8 si existeun semiintervalo [−8,−8 + δ), con δ > 0, donde f es monotona creciente y analogamentef es monotona creciente en 8 si existe un semiintervalo (8 − δ′, 8], con δ′ > 0, donde f esmonotona creciente.Sea g(x) = 2x − x2. Esta funcion es creciente estrictamente en (−∞, 1). Se puede llegara este resultado observando que la grafica de g es una parabola de vertice (1, 1) que pasapor los puntos (0, 0) y (2, 0). O tambien calculando g′(x) = 2(1 − x), que es positiva parax < 1. Y entonces g es monotona creciente en (−∞, 1], considerando que g sea crecienteen 1.Entonces la funcion y = f(2x−x2) es la composicion de f y g, es decir, y(x) =

(f g

)(x) =

f(g(x)

). Sabemos que la composicion de dos funciones monotonas crecientes es monotona

creciente.Como f es monotona creciente en el intervalo cerrado [−8, 8] y g en (−∞, 1], entoncesse puede asegurar que la funcion y = f(2x − x2) sera monotona creciente en el conjuntoD ∩ (−∞, 1], siendo

D = x ∈ R | − 8 ≤ g(x) ≤ 8 =

x ∈ R | − 8 ≤ 2x − x2 ≤ 8

.

Entonces D queda determinado por las dos desigualdades siguientes:

x2 − 2x + 8 ≥ 0 (I) y x2 − 2x − 8 ≤ 0. (II)

(I) es equivalente a (x − 1)2 + 7 ≥ 0, que se cumple para todo x ∈ R.(II) es equivalente a (x + 2)(x − 4) ≤ 0, que se cumpe en el intervalo cerrado [−2, 4].Por tanto D es el conjunto de numeros reales que cumple (I), es decir que es cualquiernumero real y que cumple (II), es decir que pertenece al intervalo cerrado [−2, 4].Ası D = R ∩ [−2, 4] = [−2, 4] = x ∈ R| − 2 ≤ x ≤ 4.Y el conjunto D ∩ (−∞, 1] es el intervalo [−2, 1] = x ∈ R | − 2 ≤ x ≤ 1.

Page 400: COMPENDIUM OME - ToomatesEotvos de Hungría, que se iniciaron en 1894, precisamente durante la efervescencia de fin de siglo, consecuencia de la cual fue también el proceso iniciado

E OME 4. Problema 2. Solucion

Supongamos el problema resuelto.

P Q O

A

B

C

D

A′

B′

C′

D′

r

Evidentemente el polo buscado P no puede estar en la recta r que contiene a los puntosdados. Sea O el centro de la circunferencia circunscrita al rectangulo A′B′C ′D′ que es lainversa de r.Las rectas A′C ′ y B′D′ pasan por O, luego sus figuras inversas pasaran por el inverso de O.Sus figuras inversas son dos circunferencias que pasan por el polo P de inversion. Ademas,siendo ambas rectas ortogonales a la circunferencia de centro O, sus inversas deben serortogonales a r, es decir su centro ha de estar en r.La construccion es clara: Se trazan las circunferencias de diametro AC y BD, que secortaran en dos puntos P y Q. Uno de ellos es el polo buscado y el otro el inverso de O.Hay dos posibles polos de inversion P y Q (en la figura se ha representado la de polo P ).La potencia puede ser cualquiera.

Page 401: COMPENDIUM OME - ToomatesEotvos de Hungría, que se iniciaron en 1894, precisamente durante la efervescencia de fin de siglo, consecuencia de la cual fue también el proceso iniciado

E OME 4. Problema 3. Solucion

Por las franjas grises de la figura marchan, a lo largo de rectas paralelas a las que definenlas franjas de pendiente ±1000 m/min, los coches que van a 60 Km/h y pasan el semaforoprincipal en verde, y la recta correspondiente a cada uno define su movimiento durante los60 segundos que dura un perıodo de ese semaforo:

1500

1000

500400→

0

−500

−1000

−1500

6 24 t (segundos)

Si nos fijamos en la recta horizontal a la altura de 400 m (lugar del semaforo secundario),veremos que el unico intervalo en el que no pasa ninguno de esos coches ocurre entre lossegundos 6 y 24 desde que se ha puesto verde el semaforo principal. De modo que elsemaforo secundario puede permanecer en rojo 18 segundos como maximo para no obligara detenerse a ninguno de los coches que pasan en verde el semaforo principal.

Page 402: COMPENDIUM OME - ToomatesEotvos de Hungría, que se iniciaron en 1894, precisamente durante la efervescencia de fin de siglo, consecuencia de la cual fue también el proceso iniciado

E OME 4. Problema 4. Solucion

Debemos suponer que la botella no es opaca.Primeramente se pone de pie la botella sobre una mesa apoyada sobre su base.Con el doble decımetro medimos la distancia desde la base de la botella hasta el nivelindicado por el vino. Sea esta distancia a. Giramos 180 la botella hasta que ocupe denuevo la posicion vertical y observamos si ahora el nivel indicado por el vino llega hastala parte cilındrica. En este caso medimos la distancia desde la base de la botella, que estaarriba hasta el nivel de vino. Sea esta distancia b.A continuacion determinamos el radio del cırculo del fondo de la botella marcando unpunto A sobre la circunferencia y colocando la botella horizontalmente de modo que elpunto marcado este sobre la mesa. En esta senalamos un punto P . Hacemos rodar labotella sobre la mesa horizontalmente hasta que de un giro completo de 360 y el punto Aocupe su posicion mas baja. Senalamos de nuevo ese punto B sobre la mesa y la distanciaAB es la longitud de la circunferencia de la base de la botella.Entonces si el volumen de la botella es V se tiene

V

2= πR2

(a + b

2

),

siendo R =AB

2π; de donde se deduce

V =AB2

(a + b

2

).

El problema no podrıa resolverse si el volumen de vino no llena al menos toda la parte nocilındrica de la botella cuando a esta se le da la vuelta.Por otra parte el metodo indicado no tiene en cuenta el espesor del cristal o del materialde que esta hecha la botella.

Page 403: COMPENDIUM OME - ToomatesEotvos de Hungría, que se iniciaron en 1894, precisamente durante la efervescencia de fin de siglo, consecuencia de la cual fue también el proceso iniciado

E OME 4. Problema 5. Solucion

A BO

α

β

β

A1

A2

A3

a) Los triangulos con vertices Ak y lado opuesto sobre el diametro son isosceles y semejantestodos ellos al AA1O al tener todos los mismo angulos iguales α.En consecuencia, todos los arcos AkAk+1 son iguales y su valor comun es β = 180 − 2α.La quebrada pasara por B despues de n subidas y bajadas, cuando

βn =180

n⇐⇒ αn =

180 − βn

2= 90o − 90

n

con n = 2, 3, . . . .

b) Llamando Ln a la longitud de la quebrada al formar todos los segmentos el mismoangulo con el diametro, su proyeccion sobre el diametro es d y cumple:

d = Ln cos αn ⇒ Ln =d

cos αn=

d

sen(

90

n

) .

Page 404: COMPENDIUM OME - ToomatesEotvos de Hungría, que se iniciaron en 1894, precisamente durante la efervescencia de fin de siglo, consecuencia de la cual fue también el proceso iniciado

E OME 4. Problema 6. Solucion

La circunferencia de centro O y radio R es la autoinversa; los puntos A, B y C son dobles.El angulo marcado con vertice A se transforma en la lunula con vertices A y O.La recta BC se transforma en la circunferencia que pasa por B y C , marcada en la siguientefigura; en particular, el segmento BC se transforma en el arco BC . La parte comprendidaentre el segmento BC y el arco de circunferencia es globalmente invariante en la inversion;el resto de la region marcada en la primera figura se transforma en la zona limitada por elsegmento BC y los arcos BO y OC .

A

B

C

O

Page 405: COMPENDIUM OME - ToomatesEotvos de Hungría, que se iniciaron en 1894, precisamente durante la efervescencia de fin de siglo, consecuencia de la cual fue también el proceso iniciado

E OME 4. Problema 7. Solucion

A partir de un punto de la carretera y en un tiempo t, el espacio recorrido por un cochees e = vt y en ese espacio se ha de ubicar el mayor numero de coches.

Como cada coche ocupa un trozo de carretera a = 2, 98 +v2

100, el numero de coches n(v)

que se pueden colocar es:

n(v) =vt

1000

2, 98 + v2

100

=100vt1000

298 + v2

cuya derivada es:

n′(v) =1001000

t298− v2

(298 + v2)2.

Anulando la derivada y resolviendo en v sale v = 17, 26 Km/h. que es claramente unmaximo ya que en ese punto la derivada pasa de positiva a negativa.

Page 406: COMPENDIUM OME - ToomatesEotvos de Hungría, que se iniciaron en 1894, precisamente durante la efervescencia de fin de siglo, consecuencia de la cual fue también el proceso iniciado

E OME 4. Problema 8. Solucion

Un posible organigrama serıa simplemente ası:

Hacer A igual a na0

Hacer i igual a 1

Hacer P igual a Ab

Hacer A igual a

P + (n − i)ai

¿Es i = n − 1?

si

noIncrementar i

en 1 unidad

El valor es A

Page 407: COMPENDIUM OME - ToomatesEotvos de Hungría, que se iniciaron en 1894, precisamente durante la efervescencia de fin de siglo, consecuencia de la cual fue también el proceso iniciado

E OME 5. Problema 1. Solucion

Sea t el tiempo (en horas) transcurrido desde que comenzo a helarse el estanque hasta las0 horas. De acuerdo con el enunciado,

3 = k√

t.

El agua aumenta de volumen al helarse; su densidad disminuye (la del agua es 1), y supeso se conserva.Si S es la superficie del cilindro, el peso del agua es

10S × 1;

el peso del hielo sera

Phielo = Vhielo × 0, 9 = S × hhielo × 0, 9.

Igualando el peso del hielo y el del agua tenemos

S × 10 = S × hhielo × 0, 9

y de aquı hhielo = 10/0, 9 = 100/9 al terminar de helarse el estanque.Entonces

1009

= k√

t + 4,

y eliminando la constante de proporcionalidad

10027

=√

t + 4√t

.

Resolviendo esta ecuacion irracional se obtiene

t(1002 − 272

)= 272 × 4

es decir

t =29169271

= 0, 31 horas, aproximadamente,

lo que significa, aproximadamente, que 18 minutos y 52 segundos antes de las 0 horascomenzo a helarse el estanque.

Page 408: COMPENDIUM OME - ToomatesEotvos de Hungría, que se iniciaron en 1894, precisamente durante la efervescencia de fin de siglo, consecuencia de la cual fue también el proceso iniciado

E OME 5. Problema 2. Solucion

a) En este caso la funcion no es continua en x = 0, ya que xn = 1/(2n) e yn = 1/(2n + 1)son dos sucesiones de numeros reales que tienden a 0 cuando n → ∞ y se tiene lim f(xn) =lim f(yn).

b) Se tiene evidentemente limx→0−

f(x) = 0 = limx→0+

f(x) = f(0), luego la funcion f(x) es

continua en x = 0.

c) No puede decidirse. Por ejemplo, la funcion f(x) = 1 es continua en x = 0 y cumplela condicion, pero la funcion g(x) = 1 si x ∈ Q y 0 en otro caso, cumple la condicion y esdiscontinua en x = 0.

Page 409: COMPENDIUM OME - ToomatesEotvos de Hungría, que se iniciaron en 1894, precisamente durante la efervescencia de fin de siglo, consecuencia de la cual fue también el proceso iniciado

E OME 5. Problema 3. Solucion

La figura de la izquierda muestra la construccion de una cuarta parte de la figura pedida.Por giros de 90 con centro en O se completa (figura de la derecha).Cuando P recorre el tercio central del arco BD (centro A, radio a, amplitud 30 y extremosM y N ), la circunferencia de centro P y radio a pasa por A y el punto Q (imagen de P enla homotecia de centro A y razon 2) describe el arco de circunferencia M ′N ′ de amplitud30 y radio 2a.Cuando P esta sobre N el perfil de la figura pedida lo forman los arcos N ′S′ y AB, ambosde centro N , radio a y amplitud 60 .La figura de la derecha muestra el contorno completo. La longitud total L del contornocorresponde a cuatro arcos de radio a y amplitud 60, otros cuatro de radio 2a y amplitud30 para el contorno exterior y cuatro arcos mas de radio a y amplitud 60 para el contornointerior; el valor es:

L = 42πa

6+ 4

4πa

12+ 4

2πa

6=

16πa

3.

A B

CD

S′ N ′

P

M ′

NP

M

O

Page 410: COMPENDIUM OME - ToomatesEotvos de Hungría, que se iniciaron en 1894, precisamente durante la efervescencia de fin de siglo, consecuencia de la cual fue también el proceso iniciado

E OME 5. Problema 4. Solucion

Tomemos el origen de coordenadas en el espacio en el centro O del diametro AB; el eje xen la direccion AB; el eje y en la direccion del diametro perpendicular a AB; y el eje z enla perpendicular por O al plano del pavimento.Cortando la figura por planos paralelos al yz, se determinan triangulos isosceles de lamisma altura h.Calcularemos el area S(x, h) de estos triangulos en funcion de la distancia x entre el planoOyz y el plano del triangulo considerado. El volumen pedido sera entonces

2∫ r

0

S(x, h) dx.

Si PQ es la base de uno cualquiera de esos triangulos, a distancia x de O, se tiene

PQ = 2√

r2 − x2,

ası que

V = 2∫ r

0

S(x, h) dx = 2∫ r

0

√r2 − x2hdx

= 2h

∫ r

0

√r2 − x2 dx,

y como la integral representa la cuarta parte del area del cırculo de radio r, sera

V = 2hπr2

4=

πr2h

2.

Page 411: COMPENDIUM OME - ToomatesEotvos de Hungría, que se iniciaron en 1894, precisamente durante la efervescencia de fin de siglo, consecuencia de la cual fue también el proceso iniciado

E OME 5. Problema 5. Solucion

Dos de los vertices del rectangulo han de estar sobre un mismo lado del triangulo; supon-gamos que este lado es el BC.Sea AN la altura relativa al vertice A y sean E y F los puntos medios de los lados PS yQR respectivamente. El punto O cuyo lugar buscamos es el punto medio de EF .Sabemos que los puntos medios de los segmentos de paralelas a un lado de un trianguloestan sobre la mediana relativa a ese lado, por tanto E esta sobre la mediana AM de ABCy por la misma causa O esta en la mediana MT del triangulo AMN relativa al lado AN .El lugar pedido es el segmento que une el punto medio M de BC con el punto medio T dela altura AN .Haciendo lo mismo con los otro lados, el lugar pedido esta formado por tres segmentos queunen el punto medio de cada altura con el punto medio del lado que contiene el pie de esaaltura.

E SP

A

B Q N F M R C

O

T

Page 412: COMPENDIUM OME - ToomatesEotvos de Hungría, que se iniciaron en 1894, precisamente durante la efervescencia de fin de siglo, consecuencia de la cual fue también el proceso iniciado

E OME 5. Problema 6. Solucion

a) La recta perpendicular a una cara en su circuncentro es evidentemente el lugar geome-trico de los centros de las esferas que pasan por los tres vertices de dicha cara, de maneraque las cuatro perpendiculares se van a cortar en el centro de la (unica) esfera circunscritaal tetraedro.

b) No es cierto: considerese el tetraedro (en unos ejes cartesianos ortonormales OXY Z)de vertices O(0, 0, 0), A(1, 1, 0), B(0, 1, 0) y C(0, 0, 1). El ortocentro de la cara COB es elpunto O, y la perpendicular correspondiente es el eje OX . El ortocentro de la cara OABes el punto B, y la perpendicular correspondiente es la recta x = 0, y = 1, que se cruzacon el eje OX .

c) No es cierto: considerese el tetraedro (en unos ejes cartesianos ortonormales OXY Z) devertices O(0, 0, 0), A(3, 0, 0), B(0, 4, 0 y C(0, 0, 3). Son sencillas de calcular las coordenadasde los incentros de las caras OAB, OBC y OCA, resultando ser estos, respectivamente,los puntos P (1, 1, 0), Q(0, 1, 1) y R(r, 0, r) con r = 1

2 (6−3√

2). Las rectas perpendiculares,a la primera de estas tres caras en P y a la segunda en Q, se cortan en el punto (1, 1, 1),que no pertenece (pues r = 1) a la perpendicular a la tercera cara en R.

Page 413: COMPENDIUM OME - ToomatesEotvos de Hungría, que se iniciaron en 1894, precisamente durante la efervescencia de fin de siglo, consecuencia de la cual fue también el proceso iniciado

E OME 5. Problema 7. Solucion

El numero de cifras cn de 2n viene dado por cn = 1 + [n · log 2] = 1 + [n · 0.303103..] donde[x] designa la parte entera de x como es habitual.

0 1 2

1 2 3 4 5 6 7 8

Puede contestarse a todas las preguntas ayudandonos de una recta con doble graduacion,por la parte inferior con los numeros naturales (marcas mayores) y por la parte superiorcon la unidad u = log 2 (marcas mas cortas).El numero de terminos con p cifras coincide con el numero de marcas uk que estan com-prendidas entre p − 1 y p.Como log 2 es irracional estamos seguros de que no pueden coincidir en el mismo puntouna marca superior y una inferior.a) Para que hubiera solo dos terminos con un cierto numero de cifras tendrıa que cumplirse3u > 1 cosa que es falsa. La respuesta es negativa.b) La respuesta tambien es negativa puesto que la existencia de cinco terminos consecutivoscon el mismo numero de cifras exigirıa 4u < 1, que tambien es falso.c) De nuevo respuesta negativa al ser 7u > 2 mientras que el enunciado exige 7u < 2.

n n + 1 n + 2

0 1 2 3 4 5 6 7 8

d) Por lo probado en los dos primeros apartados entre dos marcas mayores consecutivassolo puede haber tres o cuatro marcas menores.En el caso mas favorable despues de un intervalo (n, n + 1) con cuatro marcas menores(0, 1, 2 y 3) seguido de un intervalo (n + 1, n + 2) con tres marcas menores (4, 5 y 6), ladistancia desde la primera marca menor hasta la marca mayor de su izquierda para el cason + 1, 4 vale:

4u − 1 = 0.20411998

va disminuyendo en los intervalos siguientes, para n + 2, 7 vale 7u − 2 = 0.10720997,para n + 3, 10 la distancia es 10u − 3 = 0.01029996, y en la siguiente ya es negativa:13u − 4 = −0.38764005.De lo anterior se deduce que existen un maximo de 3 intervalos mayores consecutivos contres menores cada uno. De esta forma, el numero maximo de potencias de 2 consecutivassin que entre ellas haya 4 con el mismo numero de cifras es como mucho 15 (9 de los

Page 414: COMPENDIUM OME - ToomatesEotvos de Hungría, que se iniciaron en 1894, precisamente durante la efervescencia de fin de siglo, consecuencia de la cual fue también el proceso iniciado

intervalos consecutivos con tres menores cada uno, mas tres del intervalo anterior y tresdel posterior). La cota superior 15 es inalcanzable. Por ejemplo,

291, 292, 293 tienen 28 cifras,294, 295, 296 tienen 29 cifras,297, 298, 299 tienen 30 cifras,

2100, 2101, 2102 tienen 31 cifras,2103, 2104, 2105 tienen 28 cifras,

Ası queda demostrado que el numero buscado es 15.

Page 415: COMPENDIUM OME - ToomatesEotvos de Hungría, que se iniciaron en 1894, precisamente durante la efervescencia de fin de siglo, consecuencia de la cual fue también el proceso iniciado

E OME 5. Problema 8. Solucion

a

a

Supongamos mcd(m, n) = 1. En la figura hemos representado el caso m = 2, n = 3. Engeneral anadirıamos un cuadriculado de 2n cuadros hacia la derecha (E) y 2m− 1 cuadroshacia abajo (S). El rayo de luz debe salir, por ejemplo, hacia la cara E, formando un angulo

cuya tangente sea2m

2n=

m

n.

Si mcd(m, n) = d, sean m = m1d y n = n1d. La trayectoria es como la del caso (m1, n1)repetida d veces, de modo que el rayo pasa d − 1 veces por el punto de partida antes delregreso definitivo.Es sencillo ver, a partir de la figura anterior, que la longitud total de la trayectoria es, encualquier caso, 2

√m2 + n2 veces el lado del cuadrado.

Page 416: COMPENDIUM OME - ToomatesEotvos de Hungría, que se iniciaron en 1894, precisamente durante la efervescencia de fin de siglo, consecuencia de la cual fue también el proceso iniciado

E OME 6. Problema 1. Solucion

El lugar geometrico pedido es la circunferencia ortogonal a la dada que pasa por A y B(en la figura, la de trazo grueso). La clave del problema esta en la recta A′B′, que esortogonal a la circunferencia inversa de la dada. Por tanto la inversa de la recta A′B′ quees la circunferencia que pasa por A, B y P , tiene que ser ortogonal a γ,y su centro es elpunto O interseccion de las tangentes a γ en A y B.

γ

O

B A

P

A′

B′

Page 417: COMPENDIUM OME - ToomatesEotvos de Hungría, que se iniciaron en 1894, precisamente durante la efervescencia de fin de siglo, consecuencia de la cual fue también el proceso iniciado

E OME 6. Problema 2. Solucion

Primera solucion.

Si z = a + ib, entonces iz = −b + ia.Los puntos (a, b) , (−b, a) y (0, 1) deben estar alineados, lo cual significa que

0 − a

−b − a=

1 − b

a − b;

haciendo operaciones y simplificando, resulta

a2 + b2 − a − b = 0,

que es la ecuacion de una circunferencia de centro en el punto(

12 , 1

2

)y radio

√2

2 , esdecir, el lugar pedido es la circunferencia circunscrita al cuadrado unidad, de vertices(0, 0) , (1, 0) , (1, 1) y (0, 1).

Segunda solucion.

Basta observar que el triangulo que tiene vertices en O y en los afijos de z e iz es rectanguloen O y ademas isosceles. Si z esta en el primer cuadrante iz esta en el segundo y i estaentre z e iz por lo que el angulo de vertice z mide constantemente 45 y sus lados pasanpor dos puntos fijos: (0, 0) y (1, 0). Por tanto el lugar pedido es una parte del arco capazde 45 construido sobre el segmento que une esos puntos, es decir el arco contenido en elprimer cuadrante de la circunferencia de la figura.Si z esta en el segundo cuadrante se puede hacer un razonamiento parecido, y desde z seve el segmento de extremos 0 e i bajo angulo de 135, de forma que el lugar es el arco dela misma circunferencia de antes, pero contenido esta vez en el segundo cuadrante.El complejo z no puede estar en el tercer cuadrante, y si esta en el cuarto, se ve facilmenteque ve los puntos 0 e i bajo angulo de 45, de forma que se trata del arco del cuartocuadrante de la circunferencia de siempre. En conclusion, el lugar geometrico buscado esdicha circunferencia.

iz

iz

x

y

O

Page 418: COMPENDIUM OME - ToomatesEotvos de Hungría, que se iniciaron en 1894, precisamente durante la efervescencia de fin de siglo, consecuencia de la cual fue también el proceso iniciado

E OME 6. Problema 3. Solucion

Si consideramos el desarrollo plano del cubo, el numero de formas de distribuir 6 coloresen las 6 casillas o caras es 6! = 720.Pero el cubo admite 24 movimientos directos que lo dejan invariante, de forma que paracada configuracion de colores, habra 24 configuraciones equivalentes.Entonces el numero de cubos distinguibles es

6!6 × 4

=72024

= 30.

Para ver que hay 24 movimientos directos que dejan invariante el cubo, podemos razonarcomo sigue: primero observamos que una cara cualquiera puede llevarse a una posicionpredeterminada, digamos la inferior, es decir, hay 6 formas de llevar las caras a la partebaja del cubo. Una vez hecho esto, todavıa podemos girar alrededor del eje vertical (per-pendicular a la cara inferior) segun 4 movimientos distintos. En total tendremos 6 · 4 = 24movimientos distintos.

Page 419: COMPENDIUM OME - ToomatesEotvos de Hungría, que se iniciaron en 1894, precisamente durante la efervescencia de fin de siglo, consecuencia de la cual fue también el proceso iniciado

E OME 6. Problema 4. Solucion

Sean P , Q, R y S los vertices del cuadrado. Supongamos que P y S estan sobre la cuerdaCH, de la que M es su punto medio, y sea O el centro de la circunferencia.Es claro que el angulo COM , que es la mitad del COH, es de 3π/8 radianes.

Ası resulta OM = R cos3π

8, que es precisamente la mitad del lado del cuadrado cuya area

se pide.Entonces, si indicamos por [PQRS] el area pedida,

[PQRS] = (2 · OM)2 = 4R2 cos23π

8.

Podemos expresar este valor sin mencion explıcita de las razones trigonometricas, porque

2 cos23π

8= 1 + cos

4= 1 −

√2

2=

2 −√

22

y finalmente,

[PQRS] =(2 −

√2)

R2.

Page 420: COMPENDIUM OME - ToomatesEotvos de Hungría, que se iniciaron en 1894, precisamente durante la efervescencia de fin de siglo, consecuencia de la cual fue también el proceso iniciado

E OME 6. Problema 5. Solucion

Sea n el numero de lados del polıgono (n ≥ 3). Distinguiremos tres casos segun lasposiciones de los extremos del corte:a) De vertice a vertice, es decir la recta de corte pasa por dos vertices del polıgono.Si llamamos a y b al numero de lados completos del polıgono inicial que forman partede cada uno de los polıgonos descompuestos y teniendo en cuenta que estos tambien sonsemejantes resulta:

a = b; a + b = n; a + 1 = n

y eliminando a y b se deduce que n = 2. Esto es imposible pues n ≥ 3.

b) De vertice a lado; la recta de corte pasa por un vertice y corta a otro lado.Con la misma notacion que antes se debe cumplir:

a + 1 = b + 1; 2a + 1 = n; a + 2 = n

eliminando a y b se deduce que n = 3.

c) De lado a lado, la recta de corte no pasa por ningun vertice.Ahora las condiciones son:

a + 2 = b + 2; a + b + 2 = n; a + 3 = n

Page 421: COMPENDIUM OME - ToomatesEotvos de Hungría, que se iniciaron en 1894, precisamente durante la efervescencia de fin de siglo, consecuencia de la cual fue también el proceso iniciado

eliminando a y b se deduce que n = 4.Luego solo es posible para triangulos cuando la recta de corte solo pasa por un vertice ypara cuadrilateros cuando no pasa por ninguno. Analicemos estos casos.

1.- Triangulo.Por la semejanza de los triangulos ABD y ABC, debe cumplirse A = α. Analogamente,por la semejanza de los triangulos ADC y ABC, debe cumplirse: A = β.

A

B

CD

α β

De donde 2A = α + β = 180 ⇒ A = α = β = 90. Esto que exige que el triangulo searectangulo e isosceles.

2.- Cuadrilatero.Por la semejanza de los cuadrilateros ABFE y ABCD se debe cumplir γ = C, α = D.Por la semejanza de los cuadrilateros EFCD y ABCD se debe cumplir β = A, δ = B.

A

B

C

D

F

Eα β

γ δB F

A

ademas (figura de la izquierda), α + β = γ + δ = 180, es decir, A + D = B + C = 180

o de modo equivalente AB es paralelo a CD y AD lo es a BC. En otras palabras, elcuadrilatero es un paralelogramo y la lınea de corte es paralela a dos lados opuestos.Finalmente, como

AE

AB=

ED

AB⇒ AE = ED

entonces E y F son los punto medios de AD y BC respectivamente. Solo nos queda deter-minar la proporcion entre los lados del paralelogramo. Por la semejanza de los cuadrilaterosABFE y ABCD y llamando a = AD, b = AB, tenemos:

a2

b=

a

b⇐⇒ a2 = 2b2 ⇐⇒ a

b=

√2,

en resumen, los cuadrilateros buscados son todos los paralelogramos cuyos lados esten enrazon

√2.

Page 422: COMPENDIUM OME - ToomatesEotvos de Hungría, que se iniciaron en 1894, precisamente durante la efervescencia de fin de siglo, consecuencia de la cual fue también el proceso iniciado

E OME 6. Problema 6. Solucion

1. Tenemos la cadena de equivalencias

P (x) ≤ P (x)2 ⇔ P (x) − P (x)2 ≤ 0 ⇔ P (x)2 − P (x) ≥ 0 ⇔ P (x)[P (x)− 1] ≥ 0,

pero si 0 < P (x) < 1 para algunos valores de x, la ultima desigualdad no es cierta. Asıpor ejemplo tomando P (x) = 1

3x esta ultima desigualdad no se verifica para 0 < x < 3.

2. P (x) < 1 + P (x)2 ⇐⇒ P (x)2 − P (x) + 1 > 0, lo que es cierto pues,

P (x)2 − P (x) + 1 =[P (x) − 1

2

]2

+

(√3

2

)2

> 0,

para todo x real.

3. P (x) ≤ 12 + 1

2P (x)2 ⇔ P (x)2 − 2P (x) + 1 ≥ 0 ⇔ [P (x) − 1]2 ≥ 0, lo que es ciertopara todo numero real x.

4. Las condiciones son equivalentes a |P (x)| < M(x) y a |Q(x)| < M(x) para todo numeroreal x.Pero por ejemplo |P (x)| < 1+

(P (x)

)2, ası que basta tomar M(x) =(P (x)

)2+(Q(x)

)2+1.

Page 423: COMPENDIUM OME - ToomatesEotvos de Hungría, que se iniciaron en 1894, precisamente durante la efervescencia de fin de siglo, consecuencia de la cual fue también el proceso iniciado

E OME 6. Problema 7. Solucion

Consideramos los angulos centrales α1, α2, α3, ..., αn correspondientes respectivamente alos lados AnA1, A1A2, A2A3, ..., An−1An.Entonces la desigualdad de la hipotesis implica que α1 > α2 > α3 > ... > αn. Por tanto,

α1 +α2 > α2 +α3, α2 +α3 > α3 +α4, α3 +α4 > α4 +α5, . . . , αn−2 +αn−1 > αn−1 +αn.

Por otra parte,

A1 = 180 − α1 + α2

2, A2 = 180 − α2 + α3

2, · · · , An−1 = 180 − αn−1 + αn

2.

De donde se deduce que

180 − α1 + α2

2< 180 − α2 + α3

2< 180 − α3 + α4

2< · · · < 180 − αn−1 + αn

2,

que es equivalente a A1 < A2 < A3 < ... < An−1.Como αn−1 < α1, entonces

180 − αn−1 + αn

2> 180 − αn + α1

2,

es decir An−1 > An.

Analogamente, al ser αn < α2, 180 − αn + α1

2> 180 − α1 + α2

2,

es decir, An > A1, y se cumple que An−1 > An > A1.

Page 424: COMPENDIUM OME - ToomatesEotvos de Hungría, que se iniciaron en 1894, precisamente durante la efervescencia de fin de siglo, consecuencia de la cual fue también el proceso iniciado

E OME 6. Problema 8. Solucion

a) Indicamos por 1, 2, 3, 4, R las posiciones de las ruedas. Si G es la transformacionR →3 →2 →1 →4 → R, la transformacion G2 sera R →2 →4 →3 →1 → R, la G3 seraR →1 →3 →4 →4 → R, la G4 sera R →4 →1 →2 →3 → R, finalmente G5 = I.Es evidente que estas cinco transformaciones forman un grupo cıclico de 5 elementos.

b) Indicamos por Pi la transformacion intercambio R ↔ i, correspondiente al intercambiode la rueda del lugar i con la del lugar recambio.Entonces por comprobacion directa resulta ser

G = P4P3P2P1.

Las cuatro transformaciones Pi no forman grupo ya que el producto de dos de ellas no esotra de las cuatro. Por ejemplo, el producto P2P1 es el intercambio R →1 →2 →R queno es ninguna Pi.

Observacion:En el lenguaje de la teorıa de permutaciones, podemos decir que

G = (R,3,2,1,4)

que es un ciclo de orden 5 dentro del grupo simetrico S5, y que

Pi = (i,R)

son transposiciones (ciclos de orden 2). La teorıa tambien dice que todo ciclo es productode transposiciones, y que estas se pueden tomar incluso con un elemento comun, R ennuestro caso. Ası

(R,3,2,1,4) = (R,4) (R,1) (R,2) (R,3).

Page 425: COMPENDIUM OME - ToomatesEotvos de Hungría, que se iniciaron en 1894, precisamente durante la efervescencia de fin de siglo, consecuencia de la cual fue también el proceso iniciado

E OME 7. Problema 1. Solucion

α

α

El lıquido desalojado forma un cilindro oblicuo de base elıptica y altura 1 cm.Si llamamos r al radio de la base del cilindro, el semieje mayor a de la elipse para unainclinacion α mide:

a =r

cosα.

El semieje menor coincide con el radio r y el volumen del cilindro es

V = πab · 1 =πr2

cosα

llamando d a la densidad, tenemos para la inclinacion de 30,

40 =πr2d

cos 30=⇒ πr2d = 20

√3

y para la de 45, si llamamos p al peso del lıquido extraıdo:

p =πr2d

cos 45=

2πr2d√2

=40

√3√

2= 20

√6 gr.

Page 426: COMPENDIUM OME - ToomatesEotvos de Hungría, que se iniciaron en 1894, precisamente durante la efervescencia de fin de siglo, consecuencia de la cual fue también el proceso iniciado

E OME 7. Problema 2. Solucion

Si una yema que tiene alejamiento k es sustituida por una bifurcacion en dos ramas,terminadas en sendas yemas:a) El numero de ramas aumenta en 2.b) El numero de yemas aumenta en 1.c) El alejamiento total aumenta en k + 1 + k + 1 − k = k + 2.d) Cada una de las k ramas por las que pasaba la savia para alimentar a la primitiva yemasoporta ahora una yema mas, y hay dos nuevas ramas de 1 yema, luego la carga totalaumenta en k + 2.Respondemos ahora al problema:1) El numero de ramas es igual a 2n. Para n = 1 hay dos ramas. Si para n bifurcacioneshay 2n ramas, teniendo en cuenta a), para n + 1 bifurcaciones habra 2n + 2 = 2(n + 1)ramas.2)El numero de yemas es igual a n+1. Para n = 1 hay dos yemas. Si para n bifurcacioneshay n + 1 yemas teniendo en cuenta b), para n + 1 bifurcaciones hay (n + 1) + 1 yemas.3) Para n = 1 la carga total es 2, es decir la suma de los alejamientos de las dos yemas.Supuesto cierto para n, es cierto para n + 1, ya que por c) y d) se ve que el aumento de lacarga es igual al aumento de alejamientos.

Page 427: COMPENDIUM OME - ToomatesEotvos de Hungría, que se iniciaron en 1894, precisamente durante la efervescencia de fin de siglo, consecuencia de la cual fue también el proceso iniciado

E OME 7. Problema 3. Solucion

C

A BP

Q

Suponemos que PB = γc, 0 < γ < 1 y que QB = αa siendo 0 ≤ α ≤ 1. En este casoAP = c(1 − γ).Puesto que el area del triangulo ABC es doble que la del triangulo PQB, se deduce que

2αγ = 1. En este caso Q debe ser tal que BQ = a(1− 12γ

) ≥ 0, es decir γ ≥ 12. Obviamente

si γ =12, entonces Q = B.

En resumen, siempre que γ ≥ 12 es posible encontrar Q sin mas que elegirlo sobre el lado

BC de modo queQB

QC=

a(1 − α)aα

=1α− 1 = 2γ − 1 siendo γ la proporcion

PB

AB.

Si γ <12, entonces Q estarıa sobre AC y se razonarıa analogamnte.

Page 428: COMPENDIUM OME - ToomatesEotvos de Hungría, que se iniciaron en 1894, precisamente durante la efervescencia de fin de siglo, consecuencia de la cual fue también el proceso iniciado

E OME 7. Problema 4. Solucion

Puesto que tienen dos raıces dobles comunes, podemos poner que

x5 − 4x4 − 13x3 + 28x2 + 85x + 50 = (x − x1)2(x − x2)2(x − α)

y que2x5 − 13x4 + 4x3 + 61x2 + 20x − 25 = 2(x − x1)2(x − x2)2(x − β).

Por lo tanto

2(x−β)(x5 −4x4 −13x3 +28x2 +85x+50) = (x−α)(2x5 −13x4 +4x3 +61x2 +20x−25).

De aquı podemos obtener que α = −2 y que β =12.

Por lo tanto,

x5 − 4x4 − 13x3 + 28x2 + 85x + 50x + 2

= x4 − 6x3 − x2 + 30x + 25 = (x − x1)2(x − x2)2.

De aquı se deduce que

x1 + x2 = 3x2

1 + x22 + 4x1x2 = −1

x1x22 + x2x

21 = −15

x22x

21 = 25.

Ademas. x1 y x2 deben ser reales, con lo que, necesariamente, se tiene el sistema

x1 + x2 = 3x1 x2 = 5

y las raıces son x1 =3 +

√29

2y x2 =

3 −√

292

.

Page 429: COMPENDIUM OME - ToomatesEotvos de Hungría, que se iniciaron en 1894, precisamente durante la efervescencia de fin de siglo, consecuencia de la cual fue también el proceso iniciado

E OME 7. Problema 5. Solucion

Primera solucion

Para clarificar el resultado tomaremos una poblacion de referencia de 100 alumnos. De-notaremos por F , M , I y PH , respectivamente, los conjuntos de alumnos que apruebanFısica, Matematicas, Idioma y Filosofıa.Denotaremos con Ac el complementario del conjunto A y con N (A) el cardinal del conjuntoA. A partir del enunciado se obtiene directamente

N (F c) ≤ 30, N (M c) ≤ 25, N (PHc) ≤ 10 y N (Ic) ≤ 15.

Por lo tantoN (F c ∪ M c ∪ PHc ∪ Ic) ≤ 30 + 25 + 10 + 15 = 80.

Ahora bien, (F ∩ M ∩ PH ∩ I)c = F c ∪ M c ∪ PHc ∪ Ic, luego

N (F ∩ M ∩ PH ∩ I) = 100 − N (F c ∪ M c ∪ PHc ∪ Ic) ≥ 100 − 80 = 20.

De modo que al menos el 20% de los alumnos aprueban esas cuatro asignaturas.

Segunda solucion

Supongamos que el numero de alumnos es 100. El numero total de notas es 400, mientrasque el numero total de aprobados es, como mınimo, 320. Por lo tanto, por el principio deDirichlet hay como mınimo 20 alumnos con las cuatro asignaturas aprobadas. Es decir,que como mınimo, el 20% de los alumnos aprueban todas las asignaturas.

Page 430: COMPENDIUM OME - ToomatesEotvos de Hungría, que se iniciaron en 1894, precisamente durante la efervescencia de fin de siglo, consecuencia de la cual fue también el proceso iniciado

E OME 7. Problema 6. Solucion

Primera solucion

(0,c)

O

A(0,0)

MN B(a,b)

γ

El lado MN del triangulo esta en el eje radical de ambas circunferencias, siendo M y Nlos puntos de interseccion de la circunferencia y la secante variable desde el punto B.Elegimos un sistema de referencia en el que la circunferencia tenga ecuacion x2 +(y−c)2 =r2 y sea A(0, 0).La ecuacion de la circunferencia que pasa por A y tiene por centro (α, β) esx2 + y2 − 2αx − 2βy = 0.El eje radical de las dos circunferencias es

2αx − 2(c − β)y + c2 − r2 = 0.

La condicion sobre α y β es que el eje radical pase por B(a, b); es decir,2aα − 2(c − β)b + c2 − r2 = 0, y la ecuacion del lugar geometrico es

2ax + 2by − 2bc + c2 − r2 = 0.

Segunda solucion

La potencia de B respecto de la circunferencia γ es constante y de valor BM · BN , y lapotencia de B respecto de las circunferencias circunscritas tambien vale BM · BN , y porlo tanto es constante. Podemos entonces hallar el punto P de la recta AB tal que BA ·BPsea la potencia de B respecto de γ, que es conocida. Entonces conocemos dos puntos fijos(A y P ) de las circunferencias circunscritas y el lugar geometrico del centro esta sobre lamediatriz del segmento AP .

Page 431: COMPENDIUM OME - ToomatesEotvos de Hungría, que se iniciaron en 1894, precisamente durante la efervescencia de fin de siglo, consecuencia de la cual fue también el proceso iniciado

E OME 7. Problema 7. Solucion

Tenemos sen 2x − 2 cos2 x +12

sen2x = 1 − cos2 x − 2 cos2 x +122 senx cosx = 0,

es decir 1 − 3 cos2 x +√

1 − cos2 x · cosx = 0;hacemos operaciones y tenemos que resolver la ecuacion

10 cos4 x − 7 cos2 x + 1 = 0,

cuyas soluciones son cosx = ±√

22

y cosx = ±√

55

.

Page 432: COMPENDIUM OME - ToomatesEotvos de Hungría, que se iniciaron en 1894, precisamente durante la efervescencia de fin de siglo, consecuencia de la cual fue también el proceso iniciado

E OME 7. Problema 8. Solucion

A

C

D

B

O

M

r

α

a c

e

bd

La suma de las areas de los triangulos AMC y BMD es

S =ac + be

2≥

√acbe = r2 − d2,

pues ab = ce = r2 − d2 es la potencia del punto M respecto de la circunferencia. Luegoel valor mınimo de S es r2 − d2, que se alcanza cuando ac = be, es decir cuando a = e y

b = c; entonces el angulo α es de 45 y cos 45 =√

22

.

Page 433: COMPENDIUM OME - ToomatesEotvos de Hungría, que se iniciaron en 1894, precisamente durante la efervescencia de fin de siglo, consecuencia de la cual fue también el proceso iniciado

E OME 8. Problema 1. Solucion

Se tiene11(k = k + 1,

1331(k = k3 + 3k2 + 3k + 1 = (k + 1)3.

Entonces11(k

23√

1331(k

=k + 12k+1

,

ası que la suma pedida es

S =k=49∑k=5

1 + k

21+k=

k=50∑k=6

k

2k.

Ahora bien,

2S =k=49∑k=5

2 + 2k

21+k=

k=49∑k=5

k

2k+

k=49∑k=5

12k

= S +525

− 50250

+

125

− 1250

12

,

luego,

S =7 · 243 − 13

248.

Page 434: COMPENDIUM OME - ToomatesEotvos de Hungría, que se iniciaron en 1894, precisamente durante la efervescencia de fin de siglo, consecuencia de la cual fue también el proceso iniciado

E OME 8. Problema 2. Solucion

Veamos el Teorema 1. De acuerdo con AII, sea E un punto que no esta en la recta (AC),y sea, por AIV, F un punto de la recta (AE) tal que E esta entre A y F , es decir, elpunto E esta en el segmento [AF ].Sea, otra vez por AIV, D un punto de (FC) tal que C esta entre F y D, y consideremosla recta (ED). Dicha recta no pasa ni por A ni por F ni por C, pues E ∈ (AC); y pasapor el punto E del segmento [AF ]. Entonces por AVI debe pasar por un punto de [AC]o por un punto de [FC].Pero si suponemos que (ED) pasa por un punto G de [FC], entonces por AV, G = D, ypor AI, (ED) = (FD); luego (AF ) = (FD) y (AC) = (EC). Esto es absurdo. Entonces(ED) pasa por un punto B del segmento [AC], ası que existe B ∈ [AC], como se querıaprobar.

A

C

D

B

E

F

A BC

IJG

D

A continuacion veamos el Teorema 2. Sean A, B, C tres puntos distintos situados sobreuna recta r. Por AII existe un punto D no situado en r; y por el Teorema 1 existe unpunto G que esta entre C y D. Pero por AI, G no esta situado en r. Y por AVI en eltriangulo CDB, la recta GA corta a [CB] en A (entonces A esta situado entre C y B) o a[DB] en I (1). Analogamente por AVI en el triangulo CDA la recta GB corta a [CA] en B(entonces B esta entre C y A) o a [DA] en J (2). Si se cumplen (1) y (2) simultaneamenteentonces en el triangulo DJB se tiene que I esta entre D y B y que la recta (AI) corta ala recta (JD) en A, pero como J esta entre A y D, por AV se tiene que A no esta entreJ y D. Por lo tanto por AVI, (AI) corta a [JB] en un punto K.Observese que la recta (AI) es la recta (AG) y que la recta (JB) es la recta (BG). (AG)y (BG) tienen un punto comun K, y como (AG) y (BG) son distintas, por AI debe serK = G. Luego G esta entre J y B y tambien esta entre C y D. En el triangulo AJB setiene que la recta (DC) corta a la (AJ) en D y como J esta entre A y D, por AV, D noesta entre A y J , y (DC) corta a [JB] en G. Por AVI, (DC) corta a [AB] en C; por lotanto C esta entre A y B.

Page 435: COMPENDIUM OME - ToomatesEotvos de Hungría, que se iniciaron en 1894, precisamente durante la efervescencia de fin de siglo, consecuencia de la cual fue también el proceso iniciado

E OME 8. Problema 3. Solucion

Se tienep2x2 + q2y2 + 2pqxy

px2 + qy2= p + q − pq(x − y)2

px2 + qy2≤ p + q < 1.

Page 436: COMPENDIUM OME - ToomatesEotvos de Hungría, que se iniciaron en 1894, precisamente durante la efervescencia de fin de siglo, consecuencia de la cual fue también el proceso iniciado

E OME 8. Problema 4. Solucion

Se puede probar algo mas de lo pedido en el enunciado

π

3≤ aA + bB + cC

a + b + c<

π

2.

Supongamos a ≥ b ≥ c. Entonces A ≥ B ≥ C y por lo tanto

(a − b)(A − B) ≥ 0, (b − c)(B − C) ≥ 0, (c − a)(C − A) ≥ 0. (1)

Sumando estas desigualdades se obtiene

(a − b) (A − B) + (b − c) (B − C) + (c − a) (C − A) ≥ 0,

y desarrollando se llega a

2 (aA + bB + cC) ≥ (b + c)A + (c + a)B + (a + b)C;

sumando a los dos miembros aA + bB + cC resulta

3 (aA + bB + cC) ≥ (a + b + c) (A + B + C) ,

y puesto que A + B + C = π, ya se obtiene

π

3≤ aA + bB + cC

a + b + c,

con igualdad si y solo si a = b = c.Para probar la segunda desigualdad, como a, b, c son los lados de un triangulo, se cumplenlas desigualdades triangulares

a + b + c > 2a,

a + b + c > 2b

a + b + c > 2c

ası que, multiplicando respectivamente por A, B, C y sumando, resulta

(a + b + c) (A + B + C) > 2(aA + bB + cC),

luego

aA + bB + cC

a + b + c<

π

2.

La solucion esta tomada del libro Geometric Inequalities, de Bottema - Djordjevic - Janic- Mitrinovic - Vasic, prob.3.3, p.36; Wolters-Noordhoff, 1968. Segun dicho libro, estademostracion sigue la idea de J.Kurschak dada en Aufgaben und Lehrsatze aus der Analysisde Polya-Szego, vol.II, Berlin 1925, pp.166 y 393.

Page 437: COMPENDIUM OME - ToomatesEotvos de Hungría, que se iniciaron en 1894, precisamente durante la efervescencia de fin de siglo, consecuencia de la cual fue también el proceso iniciado

E OME 8. Problema 5. Solucion

Obviamente (1 + z2n

)(1 − z2n

) = 1 − (z2n

)2 = 1 − z2n+1para todo valor de n.

Entonces, si |z| = 1 se tiene

(1 + z2)(1 + z22) . . . (1 + z2k

) =1 − z22

1 − z2· 1 − z23

1 − z22 · · · · · 1 − z2k+1

1 − z2k =1 − z2k+1

1 − z2,

y esta claro que

limk−→∞

(1 − z2k+1) = 1 cuando |z| < 1.

Por lo tanto el lımite del producto es1

1 − z2.

Page 438: COMPENDIUM OME - ToomatesEotvos de Hungría, que se iniciaron en 1894, precisamente durante la efervescencia de fin de siglo, consecuencia de la cual fue también el proceso iniciado

E OME 8. Problema 6. Solucion

C P O A

B(60 cos α,60senα)

α

Podemos describir los posibles caminos que puede seguir el submarino de la siguientemanera: primero va en lınea recta hasta un punto B de la circunferencia y, a continuacion,del punto B al punto A a lo largo de la circunferencia.Puesto que

d(P, B) =√

(60 cosα + 30)2 + (60 senα)2 = 60

√54

+ cos α ,

el tiempo empleado, en funcion del angulo α, es

T (α) =60v

√54

+ cos α +60kv

α .

La funcion T (α) es continua y derivable en [0, π], luego su mınimo absoluto se alcanza enalgun punto crıtico interior o en un extremo del intervalo.

En primer lugar, se tiene T (0) =90π

y T (π) =30v

(1 +2π

k), luego T (π) ≥ T (0) si k ≤ π, y

T (π) < T (0) si k > π.Por otra parte,

T ′(α) =60v

(−sen α√

5 + 4 cosα+

1k

),

luego T ′(α) = 0 si y solo si−sen α√

5 + 4 cosα=

1k

. (1)

Si k < 2, la ecuacion (1) no se satisface para ningun valor de α; en este caso el tiempooptimo es T (0), que corresponde a la trayectoria directa PA.

Page 439: COMPENDIUM OME - ToomatesEotvos de Hungría, que se iniciaron en 1894, precisamente durante la efervescencia de fin de siglo, consecuencia de la cual fue también el proceso iniciado

Si k = 2, la ecuacion (1) se transforma en (2 cosα+1)2 = 0, cuya solucion unica cos α = −12

corresponde a α =2π

3. Como T ( 2π

3 ) > T (0), la trayectoria optima sigue siendo PA.Cuando k > 2 hay dos puntos crıticos,

cos α1 =−2 +

√4 − 5k2 + k4

k2y cos α2 =

−2 −√

4 − 5k2 + k4

k2,

0 < α1 < α2 < π. Del estudio del signo de T ′(α) se deduce que α1 corresponde a unmaximo relativo y α2 a un mınimo relativo.Llamaremos B2 al punto correspondiente al angulo α2 sobre la circunferencia.Si k ≥ π, resulta evidente (dado que T (0) ≥ T (π)) que el tiempo optimo es T (α2), quecorresponde a la trayectoria mixta PB2A.(Observese que,si k → ∞,entonces α2 → π, y la trayectoria optima es el camino PCA.)Todavıa queda por estudiar el problema en el intervalo 2 < k < π. En este caso, comoT (π) ≥ T (0), hay que comparar los valores T (0) y T (α2). Con ayuda de una calculadorase puede encontrar la unica solucion de la ecuacion T (0) = T (α2) en dicho intervalo,

k0 = 2, 95918....

y podemos comprobar que:Si 2 < k < k0, T (0) < T (α2) y la trayectoria optima es PA.Si k = k0,T (0) = T (α2) y hay dos trayectorias igualmente optimas, PA y PB2A.Si k0 < k < π,T (0) > T (α2) y la trayectoria optima es PB2A.En el caso particular pedido k =

√5, como

√5 < k0, la trayectoria optima sera PA.

Page 440: COMPENDIUM OME - ToomatesEotvos de Hungría, que se iniciaron en 1894, precisamente durante la efervescencia de fin de siglo, consecuencia de la cual fue también el proceso iniciado

E OME 8. Problema 7. Solucion

En una inversion de centro O y potencia k2, una circunferencia cuyo centro diste d de Oy cuyo radio sea r = d, se transforma en otra circunferencia de radio

r′ =

k2r

r2 − d2si d > r,

k2r

d2 − r2si d < r.

Sean r1 y r2 los radios de las circunferencias concentricas dadas. Supongamos r1 < r2.

Si d < r1 o r2 < d la igualdad de radios transformadosk2r1

r21 − d2

=k2r2

r22 − d2

conduce a la

relacion d2 = −r1r2 que no tiene solucion real.

Si r1 < d < r2, la igualdadk2r1

d2 − r21

=k2r2

r22 − d2

conduce a la relacion d =√

r1r2.

Toda inversion cuyo centro O este a una distancia del centro de las circunferencias con-centricas igual a la media geometrica de los radios de dichas circunferencias transformaraestas circunferencias en otras dos iguales. La potencia de inversion puede ser cualquiera.

Page 441: COMPENDIUM OME - ToomatesEotvos de Hungría, que se iniciaron en 1894, precisamente durante la efervescencia de fin de siglo, consecuencia de la cual fue también el proceso iniciado

E OME 8. Problema 8. Solucion

Sean a1, a2, · · · , an+1 los numeros enteros elegidos.Todo numero entero se puede escribir como producto de una potencia de 2 por un factorimpar

ar = 2br · pr, con pr impar.

Ya que todos los ai son menores o iguales que 2n, lo mismo ocurre con los factores imparesrespectivos, pi. Ahora bien, como el numero de enteros impares menores que 2n es n, ytenemos n + 1 numeros impares pi, necesariamente al menos dos de ellos seran iguales:pj = pk. Entonces es claro que de los numeros aj = 2bj · pj y ak = 2bk · pj , uno es divisordel otro (el que tenga menor exponente de 2).

(Problema original de Paul Erdos, propuesto en The American Mathematical Monthly en1937.)

Page 442: COMPENDIUM OME - ToomatesEotvos de Hungría, que se iniciaron en 1894, precisamente durante la efervescencia de fin de siglo, consecuencia de la cual fue también el proceso iniciado

E OME 9. Problema 1. Solucion

a) Evidentemente (M, +) es un grupo abeliano por serlo K. El producto es asociativo,distributivo respecto de + y el elemento unidad es la matriz

(1 00 1

),

donde e es el elemento unidad de K.No es conmutativo, por ejemplo

(e 00 e

)·(

e 0e e

)=

(e 0e e

)·(

e e0 e

), ya que e = 0.

b) Sea (a bc d

)

y su determinante |A| = ad − bc ∈ K. Si ad − bc = 0, la matriz

A−1 =1|A|

(d −b−c a

)

y se tiene que A · A−1 = I.Si ad− bc = 0 y existiera A−1 inversa de A, se tendrıa que (ad− bc) · |A−1| = |I| = e, luegoe = 0, que es absurdo.c) Evidente, pues dadas dos matrices A, B tales que |A| = 0, |B| = 0, se tiene

|A · B| = |A| · |B| = 0,

luego A · B tambien tiene inversa.

Page 443: COMPENDIUM OME - ToomatesEotvos de Hungría, que se iniciaron en 1894, precisamente durante la efervescencia de fin de siglo, consecuencia de la cual fue también el proceso iniciado

E OME 9. Problema 2. Solucion

C

A B5

43

O

El triangulo es rectangulo en C.Por lo tanto si consideramos los puntos del segmento AC, el mınimo se alcanza en C yvale 7, y el maximo se alcanza en A y vale 8.Si consideramos los puntos del segmento CB, el mınimo se alcanza en C, vale 7 y el maximose alcanza en B y vale 9. Y por ultimo si consideramos los puntos del segmento AB , elmınimo se alcanza en O, siendo O el centro de la circunferencia circunscrita y vale 7.4; elmaximo se alcanza en B y vale 9.En resumen, el mınimo absoluto esta en C y vale 7 y el maximo absoluto se alcanza en By vale 9.

Page 444: COMPENDIUM OME - ToomatesEotvos de Hungría, que se iniciaron en 1894, precisamente durante la efervescencia de fin de siglo, consecuencia de la cual fue también el proceso iniciado

E OME 9. Problema 3. Solucion

A

B

h

a

Desarrollando el prisma, se tiene un rectangulo y la poligonal que tiene longitud mınimaes la diagonal del rectangulo. Si la altura es h y el lado de la base es a, la longitud seraL =

√h2 + 36a2.

Page 445: COMPENDIUM OME - ToomatesEotvos de Hungría, que se iniciaron en 1894, precisamente durante la efervescencia de fin de siglo, consecuencia de la cual fue también el proceso iniciado

E OME 9. Problema 4. Solucion

2+i

P=2+3i

A

B

A es una semirrecta de origen P = (2 + 3i); B es el interior del cırculo de centro (2 + i)y radio 2 excluida la circunferencia. El punto (2 + 3i) ∈ B. Por tanto A ∩ B = ∅ yconsecuentemente la proyeccion sera ∅.

Page 446: COMPENDIUM OME - ToomatesEotvos de Hungría, que se iniciaron en 1894, precisamente durante la efervescencia de fin de siglo, consecuencia de la cual fue también el proceso iniciado

E OME 9. Problema 5. Solucion

P

r

r′

r′′

A

B

Con centro P y un angulo de 60 se gira la recta r, obteniendose la recta r′′, que no puedeser paralela a r′, cortandola en el punto A. En el giro opuesto el transformado de A esun punto B de la recta r. El triangulo PAB es el pedido. Si el giro se hace en sentidocontrario se obtiene otra solucion distinta.

Page 447: COMPENDIUM OME - ToomatesEotvos de Hungría, que se iniciaron en 1894, precisamente durante la efervescencia de fin de siglo, consecuencia de la cual fue también el proceso iniciado

E OME 9. Problema 6. Solucion

Sean O, O′, O′′ los centros de las tres circunferencias dadas. Las longitudes de los ladosdel triangulo OO′O′′ son

OO′ = r + r′, O′O′′ = r′ + r′′, O′′O = r′′ + r.

Por lo tanto el perımetro 2p de OO′O′′ es

2 (r + r′ + r′′) ,

de donde

p − (r + r′) = r′′, p − (r′ + r′′) = r, p − (r + r′′) = r′.

El area de cualquier triangulo es igual a su semiperımetro por el radio de la circunferenciainscrita, por una parte, y por otra a la expresion dada por la formula de Heron:

p ρ =√

p rr′r′′,

donde ρ es el radio buscado; ası se tiene

ρ =√

prr′r′′

r + r′ + r′′=

√rr′r′′

r + r′ + r′′.

Page 448: COMPENDIUM OME - ToomatesEotvos de Hungría, que se iniciaron en 1894, precisamente durante la efervescencia de fin de siglo, consecuencia de la cual fue también el proceso iniciado

E OME 9. Problema 7. Solucion

Si razonamos por induccion tenemos que para n = 1, es A1 = 8; supuesto que An = k · 8para algun k ∈ N, entonces An+1 = 5n+1 +2 · 3n +1 = 4 · 5n +5n +2 · 3n−1 +4 · 3n−1 +1 =4 · 5n + 4 · 3n−1 + k · 8 = 4(5n + 3n−1) + k · 8, que es multiplo de 8 pues 5n + 3n−1 es unnumero par.

Page 449: COMPENDIUM OME - ToomatesEotvos de Hungría, que se iniciaron en 1894, precisamente durante la efervescencia de fin de siglo, consecuencia de la cual fue también el proceso iniciado

E OME 9. Problema 8. Solucion

a) Consideremos dos elementos cualesquiera de L,

(x1, x2, x3); (y1, y2, y3) y x1 + x2 + x3 = 0, y1 + y2 + y3 = 0.

Dados α, β ∈ R, la combinacion lineal

α(x1, x2, x3) + β(y1, y2, y3) = (αx1 + βy1, αx2 + βy2, αx3 + βy3)

cumple que:

αx1 + βy1 + αx2 + βy2 + αx3 + βy3 = α(x1 + x2 + x3) + β(y1 + y2 + y3) = 0

con lo cual tenemos que L es un subespacio vectorial.

b) xRx, ∀x ∈ R3 ya que x − x = 0 ∈ L, es decir R es reflexiva.

∀x, y ∈ R3, si xRy ⇒ x − y ∈ L ⇒ y − x ∈ L ⇒ yRx; es decir R es simetrica.

∀x, y, z ∈ R3, si xRy y yRz entonces x − y ∈ L y tambien y − z ∈ L. Por lo tanto

x− y + y − z ∈ L, es decir x− z ∈ L y esto quiere decir que xRz, es decir R es transitiva.

c)

[(−1, 3, 2)] = (x1, x2, x3) ∈ R3/(−1, 3, 2)R (x1, x2, x3) =

= (x1, x2, x3) ∈ R3/(1 + x1, x2 − 3, x3 − 2) ∈ L =

= (x1, x2, x3) ∈ R3/x1 + x2 + x3 = 4

Por ejemplo, (4, 0, 0) ∈ [(−1, 3, 2)] y (0, 4, 0) ∈ [(−1, 3, 2)].

Page 450: COMPENDIUM OME - ToomatesEotvos de Hungría, que se iniciaron en 1894, precisamente durante la efervescencia de fin de siglo, consecuencia de la cual fue también el proceso iniciado

E OME 10. Problema 1. Solucion

Sea f(x) = 14x4 − 10x3 + 10x2. Se cumple f ′(x) = x3 − 30x2 + 20x; si f ′(x) = 0, se tiene

que x = 0 y x = 15 ±√

205. Estudiamos la derivada segunda f ′′(x) = 3x2 − 60x + 20;f ′′(0) = 20 > 0, y f ′′(15 +

√205) = 20 > 0; por lo tanto x = 0 y x = 15 +

√205 son

puntos de mınimo relativo.Como f(0) = 0 y f(15 +

√205) < 0, la funcion alcanza su valor mınimo absoluto en

x = 15 +√

205 = 29, 3178....Se tiene f(29) = −58659, 75 y f(30) = −58500, luego el menor termino de la sucesiones a29.

Page 451: COMPENDIUM OME - ToomatesEotvos de Hungría, que se iniciaron en 1894, precisamente durante la efervescencia de fin de siglo, consecuencia de la cual fue también el proceso iniciado

E OME 10. Problema 2. Solucion

Las tres primeras ecuaciones forman un sistema no homogeneo, cuyas soluciones son

x = 47z − 22y = 21z − 10.

Para que se cumpla la inecuacion tiene que suceder:

3(47z − 22) + 11(21z − 10) − z + 9 > 0371z − 167 > 0,

es decir,

z >167371

.

Page 452: COMPENDIUM OME - ToomatesEotvos de Hungría, que se iniciaron en 1894, precisamente durante la efervescencia de fin de siglo, consecuencia de la cual fue también el proceso iniciado

E OME 10. Problema 3. Solucion

La sucesion Re (an) es convergente, pero el lımite no esta comprendido entre las cotasindicadas. Sabemos que

1 +∞∑

k=1

xk

k= 1 − log(1 − x),

que es convergente si |x| < 1.Por otra parte, la sucesion dada es:

an = 1 +n∑

k=1

1k

eik π4 ,

luego

limn→∞

an = 1 − log(1 − eiπ4 ) = 1 − log

√2 −

√2 + i

8.

Por lo tanto el lımite buscado es:

limn→∞

Re (an) = Re ( limn→∞

an) = 1 − log√

2 −√

2,

cuyo valor aproximado es 1, 2674.

Page 453: COMPENDIUM OME - ToomatesEotvos de Hungría, que se iniciaron en 1894, precisamente durante la efervescencia de fin de siglo, consecuencia de la cual fue también el proceso iniciado

E OME 10. Problema 4. Solucion

O

P

Q

OP =r + r′

2

PQ =r′ − r

2π/8

En la figura vemos que

senπ

8=

r′ − r

2r + r′

2

=

r′

r− 1

1 +r′

r

de donde se tiene que

r′

r=

1 + senπ

81 − sen

π

8

.

Page 454: COMPENDIUM OME - ToomatesEotvos de Hungría, que se iniciaron en 1894, precisamente durante la efervescencia de fin de siglo, consecuencia de la cual fue también el proceso iniciado

E OME 10. Problema 5. Solucion

1) Es obvio

2) Si consideramos

λ0 · 1 + λ1(x − 2) + λ2(x − 2)2 + λ3(x − 2)3 + λ4(x − 2)4 = 0; λi ∈ Q i = 0, . . . , 4

se tiene que λi = 0 ∀i, es decir los polinomios 1, x − 2, (x − 2)2, (x − 2)3 y (x − 2)4 sonlinealmente independientes y como se cumple que

P (x) =4∑

i=0

P (i)(2)i!

(x − 2)i,

tambien son un sistema generador. (En esta formula, P (i) denota la derivada i-esima de P ,y P (0) = P .

3)

P (x) = −1 + 38(x − 2) +1142!

(x − 2)2 +1443!

(x − 2)3 +724!

(x − 2)4 =

= −1 + 38(x − 2) + 57(x − 2)2 + 24(x − 2)3 + 3(x − 2)4.

Esta relacion corresponde a P (x) = 7 + 2x − 15x2 + 3x4. El resultado es

P (x) = −121 − 82(x − 2) + 27(x − 2)2 + 24(x − 2)3 + 3(x − 2)4.

Page 455: COMPENDIUM OME - ToomatesEotvos de Hungría, que se iniciaron en 1894, precisamente durante la efervescencia de fin de siglo, consecuencia de la cual fue también el proceso iniciado

E OME 10. Problema 6. Solucion

a) Sea a el lado del triangulo.

B C

A

P

y

x

z

Dividiendo el triangulo ABC en los tres triangulos PAC, PAB y PBC, se tiene la igualdad

a · 12

=ax

2+

ay

2+

az

2luego x + y + z = 1.

b) Son los puntos en los que x >12

o y >12

o z >12

c) Debe serx ≤ y + z, y ≤ x + z, z ≤ x + y

y la probabilidad sera14.

Page 456: COMPENDIUM OME - ToomatesEotvos de Hungría, que se iniciaron en 1894, precisamente durante la efervescencia de fin de siglo, consecuencia de la cual fue también el proceso iniciado

E OME 10. Problema 7. Solucion

Como los trayectos a lo largo de cada eje son de longitud constante solo debemos preocu-parnos de minimizar el resto del trayecto. Para ello se halla P ′ simetrico de P respectodel eje OX y se transforma en P ′′ por el vector (−1, 0).Analogamente, Q′ es el simetrico de Q respecto del eje OY y se transforma en Q′′ por elvector (0,−2).

Q′(−5,11) Q(5,11)

Q′′(−5,9)

D

C

B A

P (8,2)

P ′(8,−2)

P ′′(7,−2)

O

x

y

Las intersecciones de la recta P ′′Q′′ con los ejes nos dan los puntos B y C en los que elmovil abandona el eje OX y entra en el eje OY respectivamente.El resto es evidente. Con B y C se determinan A y D y la justificacion de longitud mınimaes obvia.La longitud del camino mınimo es:

L = 3 + P ′′Q′′ = 3 +√

122 + 112 = 19, 279....

Page 457: COMPENDIUM OME - ToomatesEotvos de Hungría, que se iniciaron en 1894, precisamente durante la efervescencia de fin de siglo, consecuencia de la cual fue también el proceso iniciado

E OME 10. Problema 8. Solucion

a) Se trata de la recta que pasa por el punto (1, 2,−1) y tiene como vector director(−1,−3, 2).

b) Una semirrecta contenida en L, de origen (1, 2,−1) y que pasa por el punto (0, 5, 1).

c) (−1,−3, 2).

d) 2(1 − t) + 3(2 − 3t) + 2(2t − 1) + 1 = 0, de donde t = 1.

e) P (3) = (−2,−7, 5), el plano pedido es 2(x + 2) + 3(y + 7) + 2(z − 5) = 0, es decir2x + 3y + 2z + 15 = 0.

f) P (2) = (−1,−4, 3), el plano pedido es −(x + 1) − 3(y + 4) + 2(z − 3) = 0, es decir−x − 3y + 2z − 19 = 0.

Page 458: COMPENDIUM OME - ToomatesEotvos de Hungría, que se iniciaron en 1894, precisamente durante la efervescencia de fin de siglo, consecuencia de la cual fue también el proceso iniciado

E OME 11. Problema 1. Solucion

1. Numero de vertices:12 · 5

3= 20.

2. Numero de aristas:12 · 5

2= 30.

3. Numero de diagonales de todas las caras

12((

52

)− 5

)= 60.

4. Numero de segmentos rectilıneos determinados por cada dos vertices

(202

)= 190.

5. Numero de diagonales del dodecaedro: 190 − 30 − 60 = 100.

Page 459: COMPENDIUM OME - ToomatesEotvos de Hungría, que se iniciaron en 1894, precisamente durante la efervescencia de fin de siglo, consecuencia de la cual fue también el proceso iniciado

E OME 11. Problema 2. Solucion

Sea r el radio del disco y α la medida en radianes del angulo del sector. El arco AB dedicho sector tiene una longitud igual a αr.La longitud de la circunferencia de la base del cono es 2πr − rα = r(2π − α), ası que elradio ρ de esta circunferencia sera

2πρ = r(2π − α) , o sea ρ =r(2π − α)

2π.

Llamando h a la altura del cono, cuya generatriz es r, se tendra

h2 = r2 − ρ2 =r2

4π2(4π − α)α,

h =r

√(4π − α)α .

Entonces el volumen del cono es:

V =13πρ2h =

r3

24 π2(2π − α)2

√(4π − α)α ,

y debemos buscar el maximo relativo en el intervalo (0, 2π), de la funcion

f(α) = (2π − α)2√

(4π − α)α .

Calculando f ′(α), simplificando e igualando a cero obtenemos la ecuacion

(2π − α)2 − 2α(4π − α)√(4π − α)α

= 0 .

Por lo tanto debe ser3α2 − 12πα + 4π2 = 0,

cuyas soluciones son

α = 2π ± 2√

63

π ,

y como hay que desechar la correspondiente al signo +; resulta entonces

α = 2π − 2√

63

π radianes.

Page 460: COMPENDIUM OME - ToomatesEotvos de Hungría, que se iniciaron en 1894, precisamente durante la efervescencia de fin de siglo, consecuencia de la cual fue también el proceso iniciado

E OME 11. Problema 3. Solucion

Sean α ∈ Z(5), con α =p

qy q = 5; y β ∈ Z(5) con β =

p′

q′y q′ = 5.

Entonces:

α − β =p

q− p′

q′=

pq′ − qp′

qq′siendo qq′ = 5. Luego Z(5) es subgrupo de Q.

α · β =p

q· p′

q′=

pp′

qq′siendo qq′ = 5. Luego Z(5) es subanillo de Q.

52∈ Z(5) y

25

/∈ Z(5). Luego Z5 no es subcuerpo de Q. Por tanto, no puede ser espaciovectorial sobre Z5 ya que Z(5) no es cuerpo.

Page 461: COMPENDIUM OME - ToomatesEotvos de Hungría, que se iniciaron en 1894, precisamente durante la efervescencia de fin de siglo, consecuencia de la cual fue también el proceso iniciado

E OME 11. Problema 4. Solucion

Sin mas palabras:

1

52

√3

2

l =

√254

+34

=√

7.

Page 462: COMPENDIUM OME - ToomatesEotvos de Hungría, que se iniciaron en 1894, precisamente durante la efervescencia de fin de siglo, consecuencia de la cual fue también el proceso iniciado

E OME 11. Problema 5. Solucion

Sean a, b ∈ G. Entonces a · b ∈ G y (a · b)(a · b) = e. Esta igualdad se puede escribir, por lapropiedad asociativa, a(b · a)b = e, y sale, multiplicando a la derecha por b, a(b · a)(b · b) =e · b = b. Volviendo a aplicar la propiedad asociativa, queda (a · a)(b · a) = a · b, de dondea · b = b · a.

Page 463: COMPENDIUM OME - ToomatesEotvos de Hungría, que se iniciaron en 1894, precisamente durante la efervescencia de fin de siglo, consecuencia de la cual fue también el proceso iniciado

E OME 11. Problema 6. Solucion

Construimos la recta BC perpendicular a OA, y elegimos el punto G de modo qued(F, G) = 3d(F, B).La recta GA corta a la circunferncia en D. Construimos DE perpendicular a OA.La cuerda pedida es DE.

ADE

O

F

I

C B G

Si d(A, B) =√

2, como el radio de la circunferencia es 1, resulta que AB es el lado delcuadrado inscrito y el punto F coincide con el centro O.En este caso si llamamos a a la altura AI del triangulo isosceles pequeno semejante al

triangulo AOB, se tiene ID = 3a y OI = 1 − a; y por tanto a =15, por lo que los

segmentos pedidos miden, uno√

25

y el otro4√

25

.

Page 464: COMPENDIUM OME - ToomatesEotvos de Hungría, que se iniciaron en 1894, precisamente durante la efervescencia de fin de siglo, consecuencia de la cual fue también el proceso iniciado

E OME 11. Problema 7. Solucion

Calculemos primero la altura que alcanzarıa el agua con el deposito en posicion vertical.

9 =32

√3 · h, luego h = 2

√3 > 2.

Cuando el deposito se inclina, los 9 metros cubicos de agua se reparten entre el volumende un prisma recto de altura 2 metros y el de medio prisma recto de altura h1.En el primero caben 3

√3

2 · 2 = 3√

3 metros cubicos, ası que el volumen restante cumple

9 − 3√

3 =12· 3

√3

2· h1,

h1 = 4(√

3 − 1)

.

Por lo tanto, la altura del agua en la arista opuesta a la indicada en el enunciado es

2 + h1 = 2(2√

3 − 1)

.

Este numero es efectivamente mayor que 2√

3.

Page 465: COMPENDIUM OME - ToomatesEotvos de Hungría, que se iniciaron en 1894, precisamente durante la efervescencia de fin de siglo, consecuencia de la cual fue también el proceso iniciado

E OME 11. Problema 8. Solucion

Sea K = L + M + N .Si K es par debe ser:

L ≤ K

2; M ≤ K

2y N ≤ K

2.

Es decir: L + M ≥ N ; L + N ≥ M y M + N ≥ L.Si K es impar debe ser:

0 < L ≤ K − 12

; 0 < M ≤ K − 12

y 0 < N ≤ K − 12

.

Es decir: L + M > N > 0; L + N > M > 0 y M + N > L > 0.

Estas condiciones necesarias, tambien son suficientes: supongamos, sea cual sea la paridadde K y sin perder generalidad, que L ≥ M ≥ N . Comenzamos coloreando de rojo enun orden circular, un lado sı, uno no, hasta completar los L lados rojos. Todos los ladosrojos quedan separados. Quedan por colorear L − 1 lados desconectados y un tramo deK − (2L − 1) = M + N − L + 1 ≥ 1 lados consecutivos.Como L ≥ M , es M +N −L+1 ≤ N +1, luego este tramo de lados consecutivos se puedecolorear alternativamente amarillo-azul-amarillo-etc. sin que haya dos lados consecutivosdel mismo color. Y para terminar, los L− 1 lados desconectados se colorean de amarillo oazul hasta terminar con los M lados amarillos y los N azules.

Page 466: COMPENDIUM OME - ToomatesEotvos de Hungría, que se iniciaron en 1894, precisamente durante la efervescencia de fin de siglo, consecuencia de la cual fue también el proceso iniciado

E OME 12. Problema 1. Solucion

Para el calculo del lımite se puede seguir el procedimiento de construccion de la integral.

limk−→∞

1n

(1nk

+2k

nk. . . +

(n − 1)k

nk+

nk

nk

)=

∫ 1

0

xk dx =1

k + 1.

Page 467: COMPENDIUM OME - ToomatesEotvos de Hungría, que se iniciaron en 1894, precisamente durante la efervescencia de fin de siglo, consecuencia de la cual fue también el proceso iniciado

E OME 12. Problema 2. Solucion

Si analizamos su comportamiento tenemos que

limx−→−∞

(1 +

1x

)x

= e; limx−→−1x<−1

(1 +

1x

)x

= ∞

limx−→0x>0

(1 +

1x

)x

= 1; limx→∞

(1 +

1x

)x

= e.

Si analizamos su derivada

f ′(x) =(

1 +1x

)x [log

(1 +

1x

)+

−1x + 1

]=⇒ f ′ > 0.

e

1

5

−5 5

Page 468: COMPENDIUM OME - ToomatesEotvos de Hungría, que se iniciaron en 1894, precisamente durante la efervescencia de fin de siglo, consecuencia de la cual fue también el proceso iniciado

E OME 12. Problema 3. Solucion

Sean α ∈ Z(5), con α =p

qy q = 5; y β ∈ Z(5) con β =

p′

q′y q′ = 5.

Entonces:

α − β =p

q− p′

q′=

pq′ − qp′

qq′siendo qq′ = 5. Luego Z(5) es subgrupo de Q.

α · β =p

q· p′

q′=

pp′

qq′siendo qq′ = 5. Luego Z(5) es subanillo de Q.

52∈ Z(5) y

25

/∈ Z(5). Luego Z5 no es subcuerpo de Q. Por tanto, no puede ser espaciovectorial sobre Z5 ya que Z(5) no es cuerpo.

Observacion. Este problema es identico al problema 3 de la OME 11.

Page 469: COMPENDIUM OME - ToomatesEotvos de Hungría, que se iniciaron en 1894, precisamente durante la efervescencia de fin de siglo, consecuencia de la cual fue también el proceso iniciado

E OME 12. Problema 4. Solucion

Primera solucionConsideremos a1 · a2 · . . . · an = 1 y S = a1 + a2 + · · ·+ an.Suponemos que no todos son 1.Sea ar el mayor y as el menor de los elementos ai, ar ≥ 1 y as ≤ 1. Se consideran losn numeros obtenidos al cambiar ar por 1 y as por (ar · as). El producto sigue siendo 1.Haciendo esto la suma S′ no aumenta pues

S′ − S = 1 + ar · as − ar − as = (1 − ar)(1 − as) ≤ 0

repitiendo el proceso se consigue tener los n numeros iguales a 1 y su suma sera nEn el proceso la suma no aumenta, luego S ≥ n

Segunda solucionSe tiene

a1 · a2 · · ·an = 1.

Aplicando la desigualdad de las medias aritmetica y geometrica,

a1 + ... + an

n≥ n

√a1 · · ·an = n

√1 = 1,

que es lo que querıamos demostrar.

Page 470: COMPENDIUM OME - ToomatesEotvos de Hungría, que se iniciaron en 1894, precisamente durante la efervescencia de fin de siglo, consecuencia de la cual fue también el proceso iniciado

E OME 12. Problema 5. Solucion

M ′

M

B

A

r

Se traza con centro B la circunferencia tangente a r. Se trazan por A las tangentes a lacircunferencia anterior que cortan a r en M y M ′. B es el incentro del triangulo AM ′M .

Page 471: COMPENDIUM OME - ToomatesEotvos de Hungría, que se iniciaron en 1894, precisamente durante la efervescencia de fin de siglo, consecuencia de la cual fue también el proceso iniciado

E OME 12. Problema 6. Solucion

Primera solucion

Las sucesiones dadas son

1, 1, 3, 5, 11, 21, . . . que modulo 8 es 1, 1, 3, 5, 3, 5, . . .1, 7, 17, 55, 127, . . . que modulo 8 es 1, 7, 1, 7, 1, 7, . . .

Segunda solucion

(Solucion de Murray Klamkin, publicada en su libro USA Mathematical Olympiad 1972-1986 and forty supplementary problems).

Resolviendo la ecuacion en diferencias para la sucesion (xn) se obtiene como ecuacioncaracterıstica

(t − 2) (t + 1) = 0,

con lo cual

xn =13

(2n+1 + (−1)n)

;

haciendo lo mismo con la segunda sucesion, obtenemos

yn = 2 · 3n − (−1)n.

Entonces, para que xn = xm, debe ser

3m+1 − 2n =12

(3 (−1)m + (−1)n) .

Si n = 0 o n = 1, se ve que la unica solucion es m = 0.Sea n ≥ 2. Si m y n son de la misma paridad, el segundo miembro de la anterior igualdades par y el primero es impar.Si m y n son de distinta paridad, tomando modulo 4 se comprueba que la ecuacion no secumple.

(Origen del problema : Olimpiada U.S.A. 1973).

Page 472: COMPENDIUM OME - ToomatesEotvos de Hungría, que se iniciaron en 1894, precisamente durante la efervescencia de fin de siglo, consecuencia de la cual fue también el proceso iniciado

E OME 12. Problema 7. Solucion

La funcion dada es la siguiente pepet pepet

f(x) =

1−4x + 3

si x ≤ −3,

1−2x + 9

si − 3 < x ≤ −1,

111

si − 1 < x ≤ 2

12x + 7

si 2 < x ≤ 5

14x − 3

si 5 < x

El valor maximo es111

y se alcanza en −1 ≤ x ≤ 2.

−8 −4 4 8

110

120

Page 473: COMPENDIUM OME - ToomatesEotvos de Hungría, que se iniciaron en 1894, precisamente durante la efervescencia de fin de siglo, consecuencia de la cual fue también el proceso iniciado

E OME 12. Problema 8. Solucion

Casos favorables Casos posibles

p =2

∫ 1

0x2 dx

1=

23.

Page 474: COMPENDIUM OME - ToomatesEotvos de Hungría, que se iniciaron en 1894, precisamente durante la efervescencia de fin de siglo, consecuencia de la cual fue también el proceso iniciado

E OME 13. Problema 1. Solucion

A

B

E

I

J

C

D

G

L

K

γ2

γ1

γ1, γ2 son las circunferencias de diametros los segmentos AB y CD, respectivamente.Los puntos E, G son los centros de γ1 y γ2. La recta perpendicular a AB en E cortaa γ1 en I. La recta perpendicular a CD en G corta a γ2 en J . La recta IJ es unadiagonal del cuadrado pedido. Sus otras intersecciones con γ1 y γ2 dan los vertices K yL respectivamente. Las rectas AK, BK, CL y LD contienen a los lados del cuadradorequerido.

Page 475: COMPENDIUM OME - ToomatesEotvos de Hungría, que se iniciaron en 1894, precisamente durante la efervescencia de fin de siglo, consecuencia de la cual fue también el proceso iniciado

E OME 13. Problema 2. Solucion

La suma de todas las componentes de todas las r−plas sera 0, ya que en cada componentehay la misma cantidad de 1 como de −1.Si excluimos la r-upla (1, 1, 1, . . . , 1), la suma sera −r.

Page 476: COMPENDIUM OME - ToomatesEotvos de Hungría, que se iniciaron en 1894, precisamente durante la efervescencia de fin de siglo, consecuencia de la cual fue también el proceso iniciado

E OME 13. Problema 3. Solucion

El grafico siguiente ilustra el comportamiento de la lente y el espejo sobre la matrıcula.Las dos transformaciones conmutan.

CS-3965-EN

CS-3965-EN

CS-3965-EN

CS-3965-EN

CS-3965-EN

↓ ↓

ESPEJO LENTE

LENTE ESPEJO

Page 477: COMPENDIUM OME - ToomatesEotvos de Hungría, que se iniciaron en 1894, precisamente durante la efervescencia de fin de siglo, consecuencia de la cual fue también el proceso iniciado

E OME 13. Problema 4. Solucion

Se tiene

n5 − 5n3 + 4n

n + 2=

n(n4 − 5n2 + 4

)n + 2

=n

(n2 − 1

) (n2 − 4

)n + 2

= (n − 2) (n − 1)n (n + 1) ,

que es el producto de 4 enteros consecutivos; de ellos, uno es multiplo de 4, otro lo es de 3y ademas hay otro numero par entre ellos : 4 × 3 × 2 = 24.

Observacion:El producto de n enteros consecutivos siempre es divisible por n!, y en este caso, 4! = 24.

Page 478: COMPENDIUM OME - ToomatesEotvos de Hungría, que se iniciaron en 1894, precisamente durante la efervescencia de fin de siglo, consecuencia de la cual fue también el proceso iniciado

E OME 13. Problema 5. Solucion

Sea z = (1 − i) t. Con este cambio de variable queda la ecuacion −4t4 + 8t + 1 = 0 concoeficientes reales. La funcion f(t) = −4t4 + 8t + 1 cambia de signo en los intervalos[−1

8, −1

16] y [1,

√2] y su derivada mantiene signo constante en dichos intervalos ası que la

ecuacion tiene dos raıces reales, x ∈ (−18 , −1

16 ), y ∈ (1,√

2) y dos complejas conjugadas quedesignaremos por α y α. Por las relaciones de Cardano

x · y · α · α = xy|α|2 =−14

=⇒ |α|2 =1

4|x|y

x + y + α + α = x + y + 2 Re(α) = 0 =⇒ Re(α) = −x + y

2.

En particular, Re(α) < 0, es decir α y α estan en el segundo y tercer cuadrante.

Ahora bien |Re(α)| =∣∣∣∣x + y

2

∣∣∣∣ <

√2 − 1/16

2<

√2

2y |α|2 =

14|x|y > 1 ⇒ |α| > 1.

De ambas desigualdades podemos deducir que

| cos(Arg(α)

)| =

|Re(α)||α| <

√2/21

=√

22

⇒ π

2< Arg(α) <

4.

Luego los afijos de las soluciones en t estan en el eje OX , en el segundo y en el tercercuadrante. Y por tanto los afijos de las soluciones de la ecuacion en z estan cada uno enun cuadrante, pues se tiene que

Arg(zj) = Arg(1 − i) + Arg(tj) = Arg(tj) −π

4.

α

α

x y

t1 = y

t2 = x

t3 = α

t4 = α

Page 479: COMPENDIUM OME - ToomatesEotvos de Hungría, que se iniciaron en 1894, precisamente durante la efervescencia de fin de siglo, consecuencia de la cual fue también el proceso iniciado

E OME 13. Problema 6. Solucion

Consideramos las matrices

A =M + MT

2y B =

M − MT

2;

obviamente A + B = M y ademas A es simetrica pues

AT =MT + M

2= A

y B es antisimetrica ya que

BT =MT − M

2= −B.

Page 480: COMPENDIUM OME - ToomatesEotvos de Hungría, que se iniciaron en 1894, precisamente durante la efervescencia de fin de siglo, consecuencia de la cual fue también el proceso iniciado

E OME 13. Problema 7. Solucion

Sea p el peso del diamante entero. Su precio es entonces k p2. Si lo dividimos en dos partes,de pesos x y p − x, el precio conjunto de los dos diamantes es

k x2 + k (p − x)2 = k p2 − 2kx (p − x),

Por lo tanto, el diamente se deprecia, ya que suponemos x < p.La depreciacion es 2kx(p − x), y sera maxima cuando lo sea el producto x(p − x) de dosfactores positivos cuya suma es constante e igual a p; por lo tanto sera maxima cuando losdos sean iguales, es decir, cuando el diamante se divide en dos partes iguales.

Page 481: COMPENDIUM OME - ToomatesEotvos de Hungría, que se iniciaron en 1894, precisamente durante la efervescencia de fin de siglo, consecuencia de la cual fue también el proceso iniciado

E OME 13. Problema 8. Solucion

Las raıces de x2 − 4x + 3 = 0 son x = 1, x = 3; por lo tanto

f(x) =

x2 − 4x + 3 si x ≤ 1,

−x2 + 4x − 3 si 1 < x ≤ 3,

x2 − 4x + 3 si x > 3

limx→1−

f(x) = 0, limx→1+

f(x) = 0 =⇒ f(x) es continua en 1.

f ′(x) =

2x − 4 si x < 1,

−2x + 4 si 1 < x < 3,

2x − 4 si x > 3

f ′(1−) = −2, f ′(1+) = 2 =⇒ f(x) no es derivable en 1.

El area pedida es ∫ 3

1

(−x2 + 4x − 3)dx =43

4

0

1

31−1 4

x

y

Page 482: COMPENDIUM OME - ToomatesEotvos de Hungría, que se iniciaron en 1894, precisamente durante la efervescencia de fin de siglo, consecuencia de la cual fue también el proceso iniciado

E OME 14. Problema 1. Solucion

Hacemos las siguientes transformaciones y calculos.∣∣∣∣∣∣∣∣∣∣

8 3 3 . . . 33 8 3 . . . 33 3 8 . . . 3...

......

. . ....

3 3 3 . . . 8

∣∣∣∣∣∣∣∣∣∣=(I)

∣∣∣∣∣∣∣∣∣∣

8 3 3 . . . 33 8 3 . . . 33 3 8 . . . 3...

......

. . ....

3(n − 1) + 8 3(n − 1) + 8 . . . 3(n − 1) + 8

∣∣∣∣∣∣∣∣∣∣=

= (3n + 5)

∣∣∣∣∣∣∣∣∣∣

8 3 3 . . . 33 8 3 . . . 33 3 8 . . . 3...

......

. . ....

1 1 1 . . . 1

∣∣∣∣∣∣∣∣∣∣=(II) (3n + 5)

∣∣∣∣∣∣∣∣∣∣

5 0 0 . . . 3−5 5 0 . . . 30 −5 5 . . . 3...

......

. . ....

0 0 0 . . . 1

∣∣∣∣∣∣∣∣∣∣= (3n + 5) · 5n−1.

Para que (3n + 5) · 5n−1 sea multiplo de 10 debe ser 3n + 5 multiplo de 2 y n ≥ 2, es decirn = 2k + 1, k ≥ 1.En (I) sumamos a la ultima fila las demas.En (II) restamos a cada columna la siguiente.

Page 483: COMPENDIUM OME - ToomatesEotvos de Hungría, que se iniciaron en 1894, precisamente durante la efervescencia de fin de siglo, consecuencia de la cual fue también el proceso iniciado

E OME 14. Problema 2. Solucion

La suma y multiplicacion de matrices 2 × 2 son operaciones asociativas, y la segunda esdistributiva respecto de la primera, ası que lo seran en particular cuando nos restrinjamosa las matrices del problema.Por lo tanto, basta ver que:

La matriz nula pertenece a C. En efecto(

0 0−0 0

)∈ C .

La matriz identidad pertenece a C. En efecto,(

1 0−0 1

)∈ C.

C contiene al opuesto de cada elemento. En efecto,(a b−b a

)∈ C =⇒

(−a −bb −a

)∈ C.

El producto es conmutativo :

(a b−b a

) (c d−d c

)=

(ac − bd ad + bc−ad − bc ac − bd

)(

c d−d c

) (a b−b a

)=

(ca − db cb + da−da − cb −db + ca

)

y las propiedades de las operaciones con numeros reales justifican la igualdad.Existencia de inverso para las matrices del conjunto no nulas :Si (

a b−b a

)

no es la matriz nula, esto significa que a y b no son ambos cero. Entonces:

(a b−b a

) (x y−y x

)=

(1 00 1

)

que nos da el sistema ax − by = 1, bx + ay = 0 que, una vez resuelto nos da la matriz

(x y−y x

)=

1a2 + b2

(a −bb a

).

Por lo tanto, el conjunto de esas matrices es un cuerpo conmutativo.Dada A ∈ C , vamos a hallar dos matrices X ∈ C tales que X2 = A.Ponemos

A =(

a b−b a

), X =

(x y−y x

).

Page 484: COMPENDIUM OME - ToomatesEotvos de Hungría, que se iniciaron en 1894, precisamente durante la efervescencia de fin de siglo, consecuencia de la cual fue también el proceso iniciado

Entonces X2 = A se escribe como(x2 − y2 2xy−2xy x2 − y2

)=

(a b−b a

).

Esto conduce al sistema x2 − y2 = a, 2xy = b.Supongamos x distinto de cero; entonces

y =b

2x

y al sustituir en la primera ecuacion obtenemos la ecuacion bicuadrada 4x4−4ax2−b2 = 0,con lo cual

x2 =a +

√a2 + b2

2,

y delante del radical hay que poner signo + para que el valor resultante de x2 sea positivo.Por lo tanto

x = ±

√a +

√a2 + b2

2

Sustituyendo en la primera ecuacion, se obtiene para y2 el valor

y2 =−a +

√a2 + b2

2,

donde nuevamente hay que elegir signo + para que y2 resulte positivo. Entonces

y = ±

√−a +

√a2 + b2

2.

Los signos de x e y estan determinados por la ecuacion 2xy = b; si b > 0, los signos debenser iguales; si b < 0, opuestos.Por ejemplo, para el caso b > 0 las dos matrices buscadas son

√a+

√a2+b2

2

√−a+

√a2+b2

2

−√

−a+√

a2+b2

2

√a+

√a2+b2

2

y

√a+

√a2+b2

2−

√−a+

√a2+b2

2√−a+

√a2+b2

2 −√

a+√

a2+b2

2

y analogamente en el caso b < 0.

Observacion : El cuerpo de matrices consideradas es isomorfo al cuerpo de los numeroscomplejos.

Page 485: COMPENDIUM OME - ToomatesEotvos de Hungría, que se iniciaron en 1894, precisamente durante la efervescencia de fin de siglo, consecuencia de la cual fue también el proceso iniciado

E OME 14. Problema 3. Solucion

Como en cada apreton intervienen dos personas, la suma de los numeros de apretonesdados por cada una de esas 285 personas es par. Si los 285 numeros fuesen impares, susuma serıa impar, luego entre las 285 debe haber al menos una que ha dado un numeropar de apretones de mano.

Page 486: COMPENDIUM OME - ToomatesEotvos de Hungría, que se iniciaron en 1894, precisamente durante la efervescencia de fin de siglo, consecuencia de la cual fue también el proceso iniciado

E OME 14. Problema 4. Solucion

Expresamos la suma de cuadrados de cinco numeros consecutivos en la forma

(n − 2)2 + (n − 1)2 + n2 + (n + 1)2 + (n + 2)2 = 5n2 + 10 = 5(n2 + 2).

Para que sea un cuadrado perfecto debe ser (n2 + 2) = 5k2.Los restos obtenidos al dividir un cuadrado perfecto entre 5 son:

n n2 n2 + 2

0 0 21 1 32 4 13 4 14 1 3

Por lo tanto, n2+2 no puede ser multiplo de 5. Y en consecuencia 5(n2+2) no es cuadradoperfecto.

Page 487: COMPENDIUM OME - ToomatesEotvos de Hungría, que se iniciaron en 1894, precisamente durante la efervescencia de fin de siglo, consecuencia de la cual fue también el proceso iniciado

E OME 14. Problema 5. Solucion

En el tiempo que tardamos en bajar un escalon se esconden a peldanos de la escalera. Seax el numero de peldanos visibles de la escalera:

50a + 50 = x

125a

5+ x = 125

cuya solucion es a = 1 y x = 100.

Page 488: COMPENDIUM OME - ToomatesEotvos de Hungría, que se iniciaron en 1894, precisamente durante la efervescencia de fin de siglo, consecuencia de la cual fue también el proceso iniciado

E OME 14. Problema 6. Solucion

A

C

B

D

O

O′

P

M

Llamamos O al centro de la circunferencia que pasa por A y D, y O′ al centro de la cir-cunferencia circunscrita al triangulo. Trazamos la recta AO, esta corta a la circunferenciacircunscrita en M . Veamos que AM es un diametro de esta circunferencia, con lo cual lascircunferencias necesariamente son tangentes en A,pues AO pasa por O′.Evidentemente, se tienen las siguientes relaciones:

APB = C , CBP =A

2, ADC = B +

A

2y AOD =

π

2−B − A

2= OAD, pues el triangulo

AOD es isosceles.En esta situacion arco(AM) = arco(AC) + arco(CP ) + arco(PM), pero segun lo anteriorarco(AC) = 2 B, arco(CP ) = A y arco(PM) = π − 2 B − A, por lo tanto arco(AM) = π.

Page 489: COMPENDIUM OME - ToomatesEotvos de Hungría, que se iniciaron en 1894, precisamente durante la efervescencia de fin de siglo, consecuencia de la cual fue también el proceso iniciado

E OME 14. Problema 7. Solucion

Solucion de Niven : Maxima and Minima without Calculus, M.A.A. 1981, Ejercicio B31)Sea

A =a1 + · · ·+ an

ny pongamos f(x) = (x − a1)

2 + (x − a2)2 + · · ·+ (x − an)2

Como se tiene, evidentemente,

f(x) = (a1 − x)2 + · · · + (an − x)2 ,

desarrollando tenemos

f(x) − a21 − · · · − a2

n = nx2 − 2x (a1 + · · ·+ an) = n(x2 − 2Ax

),

y el mınimo valor de x2 − 2Ax ocurre cuando x = A, ya que

x2 − 2Ax = (x − A)2 − A2.

Page 490: COMPENDIUM OME - ToomatesEotvos de Hungría, que se iniciaron en 1894, precisamente durante la efervescencia de fin de siglo, consecuencia de la cual fue también el proceso iniciado

E OME 14. Problema 8. Solucion

Supongamos en primer lugar que el triangulo es equilatero. Poniendo

α = z1 − z2, β = z2 − z3, γ = z3 − z1,

el hecho de ser equilatero se traduce en

|z1 − z2| = |z2 − z3| = |z3 − z1| ,

es decir,

|α| = |β| = |γ| ⇐⇒ αα = ββ = γγ.

Pero α + β + γ = 0, ası que α + β + γ = 0, lo cual se escribe como

+1β

+1γ

= 0 ⇐⇒ 1z1 − z2

+1

z2 − z3+

1z3 − z1

= 0,

y desarrollando resulta en definitiva la condicion

z21 + z2

2 + z23 = z1z2 + z2z3 + z3z1.

Esta condicion es tambien suficiente. En efecto, consideremosla como una ecuacion cua-dratica en z3 y resolvamos la ecuacion

z23 − z3 (z1 + z2) + z2

1 + z22 − z1z2 = 0.

Entonces

2z3 = z1 + z2 ±√

(z1 + z2)2 − 4 (z2

1 + z22 − z1z2) =

= z1 + z2 ±√

(−3) (z1 − z2)2 = z1 + z2 ± i

√3 (z1 − z2) ,

es decir

z3 − z2 = (z1 − z2) e±iπ/3,

de donde

|z3 − z2| = |z1 − z2|

yArg(z3 − z2) − Arg(z1 − z2) = ±π

3y el triangulo es equilatero.

Page 491: COMPENDIUM OME - ToomatesEotvos de Hungría, que se iniciaron en 1894, precisamente durante la efervescencia de fin de siglo, consecuencia de la cual fue también el proceso iniciado

E OME 15. Problema 1. Solucion

B(0,2)

A(4,0)

(2,1)

O

S1 =π · 2 · 4

4= 2π, es el area de un cuadrante de la elipse, S2 =

12· 4 · 2 es el area del

triangulo AOB, y S3 =π · 52

es el area del semicırculo de diametro AB.

El area pedida es S = S1 + S3 − S2 = 2π +5π

2− 4 =

2− 4.

Page 492: COMPENDIUM OME - ToomatesEotvos de Hungría, que se iniciaron en 1894, precisamente durante la efervescencia de fin de siglo, consecuencia de la cual fue también el proceso iniciado

E OME 15. Problema 2. Solucion

Recordando que los movimientos conservan las distancias vemos que la distancia entre lospuntos marcados en las palabras no coincide .SE.BASTIAN, NAITSABE..S. Por lo tanto,no existe un movimiento que transforme una en otra.Para que pueda existir tal movimiento, las letras que intervienen en la palabra deben sersimetricas respecto de un eje vertical; estas son A H I M O T U V W X Y. Ası, por ejemplo,la palabra AVITO se transforma en OTIVA por medio de una simetrıa axial.

Page 493: COMPENDIUM OME - ToomatesEotvos de Hungría, que se iniciaron en 1894, precisamente durante la efervescencia de fin de siglo, consecuencia de la cual fue también el proceso iniciado

E OME 15. Problema 3. Solucion

Primera solucion

Es suficiente observar que el primer miembro de esta igualdad es el coeficiente de xn delpolinomio

(1 + x)n · (1 + x)n =[(

n

0

)+

(n

1

)x + . . . +

(n

n

)xn

] [(n

n

)+

(n

n − 1

)x + . . . +

(n

0

)xn

].

siendo el segundo miembro el coeficiente de xn del polinomio

(1 + x)2n =(

2n

0

)+

(2n

1

)x + . . . +

(2n

n

)xn + . . . +

(2n

2n

)x2n.

Segunda solucion

Sea X = a1, a2, · · · , am, b1, b2, · · · , bn un conjunto de m+n elementos. Vamos a contarel numero de combinaciones r-arias de X .Suponiendo que una combinacion r-aria contenga i ”aes”, donde i = 0, 1, 2, · · · , r, entonceslos restantes r − i elementos son ”bes”; y en este caso, el numero de maneras de formar lacombinacion considerada es

(mi

)(n

r−i

). Por lo tanto,

r∑i=0

(m

i

)(n

r − i

)=

(m + n

r

).

Haciendo m = n = r y utilizando la identidad(nr

)=

(n

n−r

)se obtiene la igualdad propuesta.

Page 494: COMPENDIUM OME - ToomatesEotvos de Hungría, que se iniciaron en 1894, precisamente durante la efervescencia de fin de siglo, consecuencia de la cual fue también el proceso iniciado

E OME 15. Problema 4. Solucion

Por ser la ecuacion de coeficientes reales, las soluciones seran, o ambas reales, o complejasconjugadas. Si las dos son reales, zn

1 + zn2 es un numero real; si son complejas conjugadas,

expresadas en forma trigonometrica seran

z1 = ρ(cosα + isen α) =⇒ zn1 = ρn(cos nα + i sen nα)

z2 = ρ(cosα − isen α) =⇒ zn2 = ρn(cosnα − i sen nα).

Por lo tanto zn1 + zn

2 = 2ρn cos nα.

Las soluciones son z1 = 1 + i =√

2π4

y z2 = 1 − i =√

2−π4

.

En consecuencia, zn1 + zn

2 = 2√

2n cosnπ

4.

Page 495: COMPENDIUM OME - ToomatesEotvos de Hungría, que se iniciaron en 1894, precisamente durante la efervescencia de fin de siglo, consecuencia de la cual fue también el proceso iniciado

E OME 15. Problema 5. Solucion

Haciendo el cambio de variable x − 3 = t, los extremos de integracion son −1 y 1.

∫ 4

2

sen (x − 3)3dx =∫ 1

−1

sen t3dt = 0

por ser sin t3 una funcion impar.

Page 496: COMPENDIUM OME - ToomatesEotvos de Hungría, que se iniciaron en 1894, precisamente durante la efervescencia de fin de siglo, consecuencia de la cual fue también el proceso iniciado

E OME 15. Problema 6. Solucion

Sean U1, U2, U3, U4 los sucesos:

U1: la urna contiene 3 bolas blancas y p(U1) =18,

U2: la urna contiene 2 bolas blancas y 1 negra y p(U2) =38,

U3: la urna contiene 1 bola blanca y 2 negras y p(U3) =38,

U4: la urna contiene 3 bolas negras y p(U4) =18.

Estos sucesos son incompatibles dos a dos, siendo la union el suceso seguro. Sea B elsuceso “sacar cuatro bolas blancas de la urna”.Se tiene,

p(B|U1) = 1, p(B|U2) =1681

, p(B|U3) =181

, p(B|U4) = 0.

Luego, por el teorema de la probabilidad total,

P (B) =18· 1 +

1681

· 38

+181

· 38

+ 0 · 18

=1154

.

Page 497: COMPENDIUM OME - ToomatesEotvos de Hungría, que se iniciaron en 1894, precisamente durante la efervescencia de fin de siglo, consecuencia de la cual fue también el proceso iniciado

E OME 15. Problema 7. Solucion

Consideramos el solido obtenido al girar alrededor de OX la circunferencia de centro (0, a)y radio r: x2 + (y − a)2 = r2, a ≥ r.

V = V1 − V2 = π

∫ r

−r

(a +√

r2 − x2)2 dx − π

∫ r

−r

(a −√

r2 − x2)2 dx =

= π

∫ r

−r

4a√

r2 − x2 dx = 4πa

∫ r

−r

√r2 − x2 dx = 4πa

πr2

2= 2πaπr2

pues∫ r

−r

√r2 − x2 dx es el area de un semicırculo de radio r.

Page 498: COMPENDIUM OME - ToomatesEotvos de Hungría, que se iniciaron en 1894, precisamente durante la efervescencia de fin de siglo, consecuencia de la cual fue también el proceso iniciado

E OME 15. Problema 8. Solucion

El polinomio es de la forma

1 + 3x + 5x2 + 7x3 + ... + (2n + 1)xn,

es decir, a0 = 1, a1 = 3 = 2 × 1 + 1, a2 = 5 = 2 × 2 + 1, etc.Cuando este polinomio se deriva por primera vez, desaparece el termino independiente a0;al derivarlo por segunda vez, desaparece (modificado por la derivacion) el termino en a1;y ası sucesivamente. Por lo tanto, al derivar por 325-esima vez, desaparece el termino ena324. Por lo tanto interesara saber en que se transforma a325x

325 despues de derivarlo 325veces: se convertira en una constante, que sera el valor pedido, porque los terminos degrado mayor contienen potencias de x que se anularan cuando x = 0.Como an = 2n + 1, a325 = 2 × 325 + 1 = 651, ası que derivaremos

651x325

trescientas veinticinco veces. El resultado es

651 · 325! .

Page 499: COMPENDIUM OME - ToomatesEotvos de Hungría, que se iniciaron en 1894, precisamente durante la efervescencia de fin de siglo, consecuencia de la cual fue también el proceso iniciado

E OME 16. Problema 1. Solucion

Supongamos BC = 5. El vertice A del triangulo esta situado en el arco capaz desde el quese ve el segmento BC bajo angulo de 30. Por lo tanto, el triangulo de area maxima serael triangulo isosceles, cuyo vertice A esta lo mas alto posible.En este caso, los otros dos angulos son de 75 cada uno.Si h es la altura de este triangulo isosceles, se tiene

tan 15 =52h

.

Calculemos tan 15.

tan2 15 =1 − cos 30

1 + cos 30=

2 −√

32 +

√3

= (2 −√

3)2;

entonces

h =5

2(2 −

√3) =

5(2 +

√3)

2

y el area del triangulo isosceles es

Smax =25

(2 +

√3)

4.

Page 500: COMPENDIUM OME - ToomatesEotvos de Hungría, que se iniciaron en 1894, precisamente durante la efervescencia de fin de siglo, consecuencia de la cual fue también el proceso iniciado

E OME 16. Problema 2. Solucion

Un escrutinio puede representarse por un camino en la cuadrıcula de la figura. Dichocamino va siempre hacia arriba (un voto para B) y hacia la derecha (un voto para A). Sien un escrutinio el candidato A va siempre por delante del candidato B, el camino que lorepresenta debe estar siempre estrictamente por debajo de la diagonal representada. Elnumero total de caminos de (0, 0) a (9, 6) es conocido y vale(

9 + 66

)=

(156

)=

15!9! 6!

.

El numero de caminos desde (1, 0) a (9, 6) que cortan a la diagonal se puede calcular de laforma siguiente:El primer voto debe ser para el candidato A, luego el camino debe ir del punto (0, 0) alpunto (1, 0) y luego de el al punto (9, 6). Si un camino sale de (1, 0) y llega a (9, 6) cortandoo tocando la diagonal, se puede considerar el punto de corte mas a la izquierda. La parte dela izquierda de dicho camino, es decir la parte que va desde (1, 0) al primer punto de corte,tiene una figura simetrica que va del punto (0, 1) al punto de corte. En otras palabras,hay una correspondencia biyectiva entre los caminos de (1, 0) a (9, 6) cortando o tocandola diagonal y todos los caminos que van de (0, 1) y (9, 6). Estos son(

9 + 55

)=

(145

)=

14!9! 5!

.

Los que van de (1, 0) hasta (9, 6) sin tocar la diagonal son

14!8! 6!

− 14!9! 5!

=3 · 14!9! 6!

.

La probabilidad buscada es

p =3·14!9! 6!15!9! 6!

=15.

Page 501: COMPENDIUM OME - ToomatesEotvos de Hungría, que se iniciaron en 1894, precisamente durante la efervescencia de fin de siglo, consecuencia de la cual fue también el proceso iniciado

E OME 16. Problema 3. Solucion

Primera solucion

Comentario de Francisco Bellot Rosado: No es facil saber la intencion del Tribunal de laOlimpiada de ese ano al proponer este problema ; en cualquier caso, se trata de la de-sigualdad de las medias aritmetica y armonica, porque la desigualdad propuesta se escribeen la forma

a1 + · · ·+ an

n≥ n(

1a1

+ 1a2

+ · · ·+ 1an

) .

Si se da por establecida esa desigualdad, el problema parece trivial; si no es ası, hay quebasarse en la desigualdad de las medias aritmetica y geometrica, y geometrica y armonica,para llegar a la conclusion.Si no se admite ninguna de las dos desigualdades, habra que probarlas. Puede verse, porejemplo, Niven, Maxima and Minima without Calculus, M.A.A., 1981, p.36.

Se puede demostrar directamente la desigualdad de la siguiente forma.

(a1 + a2 + · · ·+ an)(

1a1

+1a2

+ · · ·+ 1an

)=

=n∑1

ai

ai+

∑i =j

ai

aj=

= n +∑i<j

(ai

aj+

aj

ai

)≥ n + 2

n(n − 1)2

= n2.

En la ultima desigualdad se ha utilizado que

x +1x≥ 2

para todo x real positivo. Como que en este caso solo vale la igualdad si x = 1, podemosdeducir que la desigualdad principal del problema es una igualdad si y solo si

a1 = a2 = · · · = an.

Segunda solucion

Aplicamos la desigualdad de Cauchy-Schwartz a los vectores

x = (√

a1,√

a2, . . . ,√

an), y = (1

√a1

,1

√a2

, . . . ,1

√an

).

La igualdad se da si y solo si los dos vectores son linealmente dependientes, es decir, sia1 = a2 = · · · = an.

Page 502: COMPENDIUM OME - ToomatesEotvos de Hungría, que se iniciaron en 1894, precisamente durante la efervescencia de fin de siglo, consecuencia de la cual fue también el proceso iniciado

E OME 16. Problema 4. Solucion

Dividiendo por f(x) tenemos f ′(x)f(x)

= −x2, e integrando resulta∫f ′(x)f(x)

dx =∫

−x2 dx ⇔ log(f(x)

)= −x3

3+ k.

Haciendo x = 1 y despejando k, queda 1 = k − 1/3 de donde k = 4/3 y la expresion finalpara la funcion es:

f(x) = e4−x3

3 .

Para representarla basta tener en cuenta que existe, es positiva y continua para todo x, yademas

limx→+∞

= e4−x3

3 = 0, limx→−∞

= e4−x3

3 = +∞.

Derivando:f ′(x) = −x2e

4−x33 ≤ 0 y se anula solo para x = 0. La funcion es decreciente con tangente

horizontal en x = 0.Volviendo a derivar resulta:

f ′′(x) =(x4 − 2x

)e

4−x3

3 = x(x3 − 2

)e

4−x3

3

que es negativa en (−∞, 0) ∪(

3√

2,∞), es

decir, es una funcion concava; y es positivaen

(0, 3

√2), es decir, es una funcion convexa

y se anula en x = 0 y en x = 3√

2, dondetiene puntos de inflexion.En x = 0 la funcion es decreciente y tiene unpunto de inflexion con tangente horizontalde ecuacion y = e

43 . Todo se resume en la

grafica que se dibuja a la derecha.

6

1

−2 3

Page 503: COMPENDIUM OME - ToomatesEotvos de Hungría, que se iniciaron en 1894, precisamente durante la efervescencia de fin de siglo, consecuencia de la cual fue también el proceso iniciado

E OME 16. Problema 5. Solucion

Lo demostraremos por induccion sobre n.Supongamos que para todo k = 1, 2, . . . , n se cumple xk + 1

xk = 2 cos kα.Veamos que tambien se cumple para k = n + 1.

(xn +1xn

)(x +1x

) = xn+1 +1

xn+1+ xn−1 +

1xn−1

de dondexn+1 +

1xn+1

= (xn +1xn

)(x +1x

) − (xn−1 +1

xn−1)

y por la hipotesis de induccion:

xn+1 +1

xn+1= 2 cosnα · 2 cosα − 2 cos(n − 1)α = 2

(2 cosnα cosα − cos(n − 1)α

)y usando la formula trigonometrica:

cosA + cosB =12(cos(A + B) + cos(A − B)

),

queda:

xn+1 +1

xn+1= 2

(22

cos (n + 1)α +22

cos (n − 1)α − cos (n − 1)α

)= 2 cos (n + 1)α.

Page 504: COMPENDIUM OME - ToomatesEotvos de Hungría, que se iniciaron en 1894, precisamente durante la efervescencia de fin de siglo, consecuencia de la cual fue también el proceso iniciado

E OME 16. Problema 6. Solucion

Calculamos

N = n(n + 1)(n + 2)(n + 3) + 1 = n4 + 6n3 + 11n2 + 6n + 1,

y trataremos de identificar este polinomio en n con el cuadrado de un polinomio de segundogrado (tambien en n):

(n2 + an + b

)2= n4 + 2an3 +

(a2 + 2b

)n2 + 2abn + b2;

identificando coeficientes se obtiene el sistema

2a = 6; a2 + 2b = 11; 2ab = 6; b2 = 1

que resuelto da la solucion a = 3, b = 1, con lo cual

N = n(n + 1)(n + 2)(n + 3) + 1 =(n2 + 3n + 1

)2.

Page 505: COMPENDIUM OME - ToomatesEotvos de Hungría, que se iniciaron en 1894, precisamente durante la efervescencia de fin de siglo, consecuencia de la cual fue también el proceso iniciado

E OME 16. Problema 7. Solucion

Las ecuaciones parametricas son claramente x = λ2, y = (2 − λ)2, 0 ≤ λ ≤ 2, de las queeliminando λ queda:

y = (2 −√

x)2 = 4 + x − 4√

x ⇔ 4√

x = x − y + 4. (1)

Elevando al cuadrado resulta:

16x = x2 + y2 + 16 + 8x − 8y − 2xy ⇔ (x − y)2 − 8(x + y) + 16 = 0. (2)

Aplicando un giro de 45 en sentido de las agujas del reloj cuyas ecuaciones son:

x =√

22

x′ +√

22

y′

y = −√

22

x′ +√

22

y′de donde resulta

x − y =

√2 x′

x + y =√

2 y′

que sustituidas en (2) dan lugar a

2x′2 − 8√

2y′ + 16 = 0 ⇔ y′ =√

24

x′2 +√

2.

Se trata claramente de una parabola cuya grafica se muestra abajo.

4

3

2

1

1 2 3 4

Para hallar el area podemos usar la ecuacion (1) y resulta:

A =∫ 4

0

(4 + x − 4

√x)dx =

[4x +

x2

2− 8

3

√x3

]4

0

=83

m2.

Page 506: COMPENDIUM OME - ToomatesEotvos de Hungría, que se iniciaron en 1894, precisamente durante la efervescencia de fin de siglo, consecuencia de la cual fue también el proceso iniciado

E OME 16. Problema 8. Solucion

Sean los lados n, n + 1, n + 2 y el area n + 3. Si se aplica la formula de Heron tendremos

n + 3 =

√3n + 3

2· n + 3

2· n + 1

2· n − 1

2.

Elevando al cuadrado y dividiendo por n + 3,

n + 3 =(n + 1)2 · (n − 1) · 3

16;

lo que, operando convenientemente, nos lleva a

3n3 + 3n2 − 19n − 51 = 0,

ecuacion cuya unica solucion real es n = 3.Por tanto el unico triangulo con los tres lados y el area numeros enteros consecutivos, eneste orden, es el de lados 3, 4 y 5, cuya area es 6.Veamos ahora que si el area no es el mayor de los numeros, no existe ninguna solucion.Si el area fuese n + 2 tendrıamos

n + 2 =

√3n + 4

2· n + 4

2· n + 2

2· n − 2

2.

Una solucion de esta ecuacion es n = −2, y las restantes cumplen 3n3+10n2−32n−64 = 0,donde las unicas soluciones enteras positivas que puede haber son los divisores de 64, ypor simple inspeccion directa se comprueba que ninguno de ellos es solucion.Si el area fuese n + 1 tendrıamos

n + 1 =

√3n + 5

2· n + 5

2· n + 1

2· n − 1

2.

Una solucion es n = −1, y las restantes cumplen 3n3 + 17n2 − 11n − 41 = 0; las unicasposibles soluciones enteras positivas que puede tener esta ecuacion son los divisores de 41,y por simple inspeccion directa se comprueba que ninguno de los dos es solucion.Si el area fuese n tendrıamos

n =

√3n + 6

2· n + 4

2· n + 2

2· n

2.

Una solucion de esta ecuacion es n = 0, y las restantes cumplen 3n3 +24n2 +44n+48 = 0,ecuacion sin soluciones positivas.

Page 507: COMPENDIUM OME - ToomatesEotvos de Hungría, que se iniciaron en 1894, precisamente durante la efervescencia de fin de siglo, consecuencia de la cual fue también el proceso iniciado

E OME 17. Problema 1. Solucion

Escribamos la suma en la forma

7(1 + 11 + 111 + · · ·+ 11...11

),

y vamos a calcular la suma del parentesis. Para eso la escribimos adecuadamente

11 + 101 + 10 + 1001 + 10 + 100 + 1000· · ·

1 + 10 + 100 + 1000 + · · ·+ 1000...0

Y sumamos en columnas, obteniendo

(n + 10(n − 1) + 100(n − 2) + 1000(n − 3) + · · ·+ 10n−1

)=

= n + n(10 + 102 + · · ·+ 10n−1)−−

(10 × 1 + 2 × 102 + 3 × 103 + · · ·+ (n − 1) × 10n−1

)La primera suma entre parentesis es una progresion geometrica; la segunda es una pro-gresion aritmetico-geometrica, que se suma con el mismo procedimiento que las progre-siones geometricas. Sean S y S′ las dos sumas, respectivamente. Entonces

S =10

(10n−1 − 1

)9

.

Para calcular S′ procedemos ası:S′ = 1× 10 + 2× 102 + 3× 103 + · · ·+ (n − 1)× 10n−1, y multiplicando por la razon de laprogresion geometrica,

10S′ = 1 × 102 + 2 × 103 + · · ·+ (n − 2) × 10n−1 + (n − 1) × 10n

ası que restando obtenemos

S′(1−10) = 1×10+1×102+· · ·+1×10n−1−(n − 1)×10n =10n−1 × 10 − 10

9−(n − 1) 10n.

Llevando estos valores a la suma que queremos calcular, resulta, tras hacer operaciones ymultiplicar por 7,

7 (n + nS − S′) =781

(10n+1 − 9n − 10

).

Page 508: COMPENDIUM OME - ToomatesEotvos de Hungría, que se iniciaron en 1894, precisamente durante la efervescencia de fin de siglo, consecuencia de la cual fue también el proceso iniciado

E OME 17. Problema 2. Solucion

Desarrollando la superficie lateral del vaso cortando por la generatriz que pasa por el puntoen el que esta la gota de miel (A en la figura), el problema queda reducido a ir desde elpunto en el que se encuentra la mosca B hasta A tocando el borde superior.

A

C

A′

BB′

Una reflexion de uno de los puntos respecto del borde superior resuelve facilmente elproblema.La distancia se obtiene del triangulo rectangulo A′B′B y vale:

√102 + 62 =

√136.

Page 509: COMPENDIUM OME - ToomatesEotvos de Hungría, que se iniciaron en 1894, precisamente durante la efervescencia de fin de siglo, consecuencia de la cual fue también el proceso iniciado

E OME 17. Problema 3. Solucion

Primera solucionDado que las rectas r y s se cruzan, tienen una perpendicular comun que designamos por t.Toda simetrıa respecto de un eje se puede descomponer en el producto de dos simetrıasrespecto de dos planos perpendiculares que se cortan segun dicho eje.La simetrıa respecto de s se puede descomponer en el producto de la simetrıa respecto delplano definido por s y t y la simetrıa respecto del plano que contiene a s y es perpendicu-lar a t.Si α es el angulo que forman las direcciones definidas por r y s, la recta r′ simetrica der por la simetrıa respecto del plano que contiene a s y t forma con r un angulo 2α, y larecta u simetrica de r′ respecto al plano que contiene a s y es perpendicular a t tiene unadireccion que forma con la direccion de r un angulo 2α.La simetrıa respecto a r se puede descomponer en el producto de la simetrıa respecto delplano definido por r y t y la simetrıa respecto del plano que contiene a r y es perpendiculara t.La recta s′ simetrica de s por la simetrıa respecto del plano que contiene a r y t formacon s un angulo −2α, y la recta v simetrica de s′ respecto del plano que contiene a r y esperpendicular a t tiene una direccion que forma con la direccion de s un angulo −2α.Por lo tanto las direcciones de u y v forman un angulo igual a 3α. Para que sean coplanarias,debe ser 3α = 180, luego α = 60.

Segunda solucionEste problema puede resolverse tambien por geometrıa analıtica, pudiendose tomar paraello, para facilitar los calculos y sin perder generalidad,

r :

x

yz

=

λ

00

, s :

x

yz

=

µ cosα

µsenαk

,

con k = 0, obteniendose entonces para las rectas simetricas u y v las ecuaciones

u :

x

yz

=

λ cos 2α

λsen2α2k

, v :

x

yz

=

µ cosα

−µsenα−k

,

y la condicion de coplanaridad de u y v es sen 3α = 0, luego α = 60.

Page 510: COMPENDIUM OME - ToomatesEotvos de Hungría, que se iniciaron en 1894, precisamente durante la efervescencia de fin de siglo, consecuencia de la cual fue también el proceso iniciado

E OME 17. Problema 4. Solucion

Antes de utilizar el cambio sugerido, ponemos

sen (x − 2) = sen((x − 1) − 1

)= sen (x − 1) cos 1 + sen1 cos(x − 1).

Sustituyendo, multiplicando por sen (x − 1) y dividiendo numerador y denominador porcos2(x − 1) se llega a

I =∫ dx

cos2(x − 1)cos 1 · tan2(x − 1) + sen 1 · tan(x − 1)

=

=∫

dt

t2 cos 1 + tsen1=

∫dt

t(t cos 1 + sen1)

y descomponiendo en fracciones simples, resulta

1t(t cos 1 + sen1)

=

1sen 1

t+

− cot 1t cos 1 + sen1

,

ası que

I =1

sen1

∫dt

t− cot 1

∫dt

t cos 1 + sen1=

1sen 1

ln |t| − 1sin 1

· ln |t cos 1 + sen1| + C

y deshaciendo el cambio resulta

I =1

sen 1ln | tan(x − 1)| − 1

sin 1· ln

∣∣ cos 1 · tan(x − 1) + sen1∣∣ + C =

=1

sin 1ln

∣∣∣∣ sin(x − 1)sin(x − 2)

∣∣∣∣ + C.

La ultima igualdad sale de

1sin 1

ln∣∣∣∣ tan(x − 1)cos 1 tan(x − 1) + sin 1

∣∣∣∣ =

=1

sin 1ln

∣∣∣∣ sen (x − 1)cos 1 sen (x − 1) + sin 1 cos(x − 1)

∣∣∣∣ .

Page 511: COMPENDIUM OME - ToomatesEotvos de Hungría, que se iniciaron en 1894, precisamente durante la efervescencia de fin de siglo, consecuencia de la cual fue también el proceso iniciado

E OME 17. Problema 5. Solucion

a) Todas las graficas se componen de dos segmentos OPn y QnRn, siendo sus coordenadas:

Pn

(1n

, n

); Qn

(1n

,3n

); Rn

(1,

3n

)

4

3

2

1

112

13

14

(

(

(

f1f2

f3

f4

f2

f3

f4

Se ha representado la funcion para n = 1, 2, 3, 4 tal como esta indicado en al figura.b) Se tiene

An =∫ 1

0

fn(x) dx =∫ 1/n

0

n2x dx +∫ 1

1/n

3n

dx =

=n2

2[x2

]1/n

0+

3n

[x]11/n =n2

2· 1n2

+3n

=12

+3n

.

c) Por el apartado anterior

limn→∞

An =12.

Page 512: COMPENDIUM OME - ToomatesEotvos de Hungría, que se iniciaron en 1894, precisamente durante la efervescencia de fin de siglo, consecuencia de la cual fue también el proceso iniciado

E OME 17. Problema 6. Solucion

La simetrıa de centro (0, 0) es un giro de 180 que se puede descomponer en producto dedos simetrıas de ejes e1 y e2 concurrentes en (0, 0) y formando entre sı angulo de 90.

−3 −2 −1 1 2 3

1

2

3

−1

−2

−3

e

s

e1v

Por otra parte, el producto de dos simetrıas es una traslacion cuando los ejes de simetrıason paralelos y el vector de la traslacion producto es perpendicular a ambos ejes y demodulo doble que la distancia entre los ejes paralelos.Llamando g a la simetrıa central dada y s a la simetrıa axial de eje x = y + 1, resulta:

s g = s (e1 e) = (s e1) e = t e

siendo e y e1 las simetrıas axiales de ejes del mismo nombre trazadas en la figura y t latraslacion de vector v = (1,−1).La recta e1 esta unıvocamente determinada pues ha de ser paralela a s y pasar por elorigen; e tambien lo esta por pasar por el origen y ser perpendicular a s .Finalmente v esta unıvocamente determinado por los datos que fijan su direccion (perpen-dicular a s), sentido (de e1 a s) y modulo (doble de la distancia entre las paralelas e1 y s).Por tanto e y v son unicos.

Page 513: COMPENDIUM OME - ToomatesEotvos de Hungría, que se iniciaron en 1894, precisamente durante la efervescencia de fin de siglo, consecuencia de la cual fue también el proceso iniciado

E OME 17. Problema 7. Solucion

Es una aplicacion tıpica del teorema de Bayes.Designaremos por d el suceso “ser defectuosa” y por mk el suceso “estar fabricada en lamaquina k-esima”.Loas datos del problema son

p(m1) = 0.1, p(m2) = 0.2, p(m3) = 0.3 p(m4) = 0.4

p(d|m1) = 0.01, p(d|m2) = 0.02, p(d|m3) = 0.04 p(d|m4) = 0.15 .

Se puede calcular ahora p(d)

p(d) = p(d|m1)p(m1) + p(d|m2)p(m2) + p(d|m3)p(m3) + p(d|m4)p(m4) = 0.077 .

La probabilidad pedida es:

p(m3|d) =p(m3 ∩ d)

p(d)=

p(d|m3)p(m3)p(d)

=0.0120.077

= 0.155844156

Page 514: COMPENDIUM OME - ToomatesEotvos de Hungría, que se iniciaron en 1894, precisamente durante la efervescencia de fin de siglo, consecuencia de la cual fue también el proceso iniciado

E OME 17. Problema 8. Solucion

Empezaremos por probar la segunda parte.Poniendo a = 2k + 1 y sustituyendo, se obtiene que el numero propuesto se puede escribircomo

N = 16k4 + 64k3 + 116k2 + 88k + 24= 4(4k4 + 16k3 + 29k2 + 22k + 6).

Observando los dos primeros terminos de la expresion de N nos podemos dar cuenta deque

(4k2 + 8k + 4)2 = 16k4 + 64k3 + 96k2 + 64k + 16,

ası que el problema estara resuelto si conseguimos expresar como suma de dos cuadradosel numero

20k2 + 24k + 8.

Esto es mas sencillo, porque

20k2 + 24k + 8 = 16k2 + 4k2 + 16k + 8k + 4 + 4 == (4k + 2)2 + (2k + 2)2,

de manera que en definitiva se tiene

N =(4(k2 + 2k + 1)

)2

+(2(2k + 1)

)2

+(2(k + 1)

)2

.

Page 515: COMPENDIUM OME - ToomatesEotvos de Hungría, que se iniciaron en 1894, precisamente durante la efervescencia de fin de siglo, consecuencia de la cual fue también el proceso iniciado

E OME 18. Problema 1. Solucion

Si designamos por x el numero de tebeos de Antonio, el numero de tebeos de Jose sera160 − x. Deberan existir dos numeros enteros λ y µ tales que

x = 7λ + 4160 − x = 8µ + 4

de donde160− 7λ − 4 = 8µ + 4

y operando convenientemente resulta

7λ = 8(19 − µ).

Para que las soluciones de esta ultima ecuacion sean enteras debe haber un numero enteroλ1 tal que se tenga λ = 8λ1. Sustituyendo y dividiendo la ecuacion por 8, resulta ahora

7λ1 = 19 − µ

o bienµ = 19 − 7λ1.

Las unicas posibilidades en las que x y 160−x sean ambos positivos son las tres siguientes

λ1 λ µ x 160 − x

0 0 19 4 1561 8 12 60 1002 16 5 116 44

siendo la solucion del periodico solamente una de ellas.

Page 516: COMPENDIUM OME - ToomatesEotvos de Hungría, que se iniciaron en 1894, precisamente durante la efervescencia de fin de siglo, consecuencia de la cual fue también el proceso iniciado

E OME 18. Problema 2. Solucion

No se contempla que el movimiento sea en el plano o en el espacio. Suponemos que tienelugar en el plano.M es un movimiento, pues todo giro de centro C y angulo α se puede descomponer comoproducto de dos simetrıas axiales (dos rectas que pasan por el centro de giro, y forman unangulo α/2).No existen rectas fijas. Si las hubiera, el centro de giro deberıa estar sobre r.En definitiva el resultado final es el producto de tres simetrıas axiales respecto a ejes queno se cortan en un mismo punto, que es una simetrıa axial con deslizamiento (traslacion)a lo largo de su eje.

Page 517: COMPENDIUM OME - ToomatesEotvos de Hungría, que se iniciaron en 1894, precisamente durante la efervescencia de fin de siglo, consecuencia de la cual fue también el proceso iniciado

E OME 18. Problema 3. Solucion

Llamando h a la altura requerida, podemos considerar, a partir del enunciado, que se tiene

h = 120 ·(

1 − 12

+13− 1

4+

15− 1

6+ · · ·

)= 120 ·

∞∑n=1

(−1)n−1

n.

Esta serie numerica es convergente segun el criterio de Leibniz para series alternadas.Ademas, el desarrollo en serie de potencias de la funcion ln(1 + x), convergente para|x| < 1, es

ln(1 + x) =∞∑

n=1

(−1)n−1

nxn.

Aplicando el teorema del lımite de Abel se tiene entonces

∞∑n=1

(−1)n−1

n= lim

x→1−ln(1 + x) = ln2,

luegoh = 120 ln 2.

Page 518: COMPENDIUM OME - ToomatesEotvos de Hungría, que se iniciaron en 1894, precisamente durante la efervescencia de fin de siglo, consecuencia de la cual fue también el proceso iniciado

E OME 18. Problema 4. Solucion

Puesto que p(0) = 0, podemos poner p(ξ) = a1ξ + a2ξ2 + . . . + anξn, con que ai ≥ 0 para

i = 1, . . . , n. Ya que debe ser

p(√

x2 + y2)≤ x4 + y4 para todos los x e y reales, (1)

en particular tomando aquı y = 0, para todo x > 0 debe ser

a1x + a2x2 + . . . + anxn ≤ x4,

o biena1

x3+

a2

x2+

a3

x+ a4 + a5x + . . . + anxn−4 ≤ 1, (2)

luego a1 = a2 = a3 = 0, pues de lo contrario serıa limx→0+

p(x)x4

= +∞, y existirıa un

ξ0 > 0 tal que p(ξ0) > ξ40 . Tambien a5 = . . . = an = 0 pues de lo contrario serıa

limx→+∞

p(x)x4

= +∞, y existirıa un ξ1 > 0 tal que p(ξ1) > ξ41 . Entonces la inecuacion (2) da

a4 ≤ 1.Pongamos ası que sea p(ξ) = aξ4 con 0 ≤ a ≤ 1. La desigualdad (1) se reduce a

a(x2 + y2)2 ≤ x4 + y4 para todos los x e y reales,

o bien, equivalentemente, a

a(ξ + η)2 ≤ ξ2 + η2 para todos los ξ y η reales positivos.

Anadiendo a ambos lados de esta desigualdad el termino 2ξη, reescribamosla como

a(ξ + η)2 + 2ξη ≤ (ξ + η)2,

o bien2ξη ≤ (1 − a)(ξ + η)2,

o bien, finalmente,

ξη ≤ 2(1 − a)(

ξ + η

2

)2

. (3)

La desigualdad entre las medias geometrica y aritmetica de dos numeros positivos, ξη ≤(ξ+η2

)2

(con igualdad si y solo si ξ = η), da que (3) se cumple cuando 1 ≤ 2(1 − a), estoes, cuando a ≤ 1/2. Y que no se cumple cuando a > 1/2, pues en este caso, al tomar ξ = ηresulta

2(1 − a)(

ξ + η

2

)2

= 2(1 − a)ξ2 < ξ2 = ξη.

Las (unicas) soluciones para el problema son pues los polinomios de la forma p(ξ) = aξ4

con 0 ≤ a ≤ 1/2.

Page 519: COMPENDIUM OME - ToomatesEotvos de Hungría, que se iniciaron en 1894, precisamente durante la efervescencia de fin de siglo, consecuencia de la cual fue también el proceso iniciado

E OME 18. Problema 5. Solucion

Si la suma de la diagonal y el lado es r, se construye un cuadrado y su diagonal de cualquiermedida, y despues se aplica el teorema de Thales.Sean d, la diagonal y el lado del cuadrado, y sea AB = d + la distancia conocida.Trazamos una recta auxiliar BD; sobre ella construimos un cuadrado BB1C1C cualquieray tomamos el punto D tal que BC1 = CD.Trazamos la recta AD y por el punto C trazamos una recta paralela a la recta AD. Estaultima recta corta a AB en el punto E tal que AE = d y EB = .

A E B

C

D

d0

C1

B1

d0

Page 520: COMPENDIUM OME - ToomatesEotvos de Hungría, que se iniciaron en 1894, precisamente durante la efervescencia de fin de siglo, consecuencia de la cual fue también el proceso iniciado

E OME 18. Problema 6. Solucion

Si v = 0 entonces 0 ≤ au y la desigualdad es cierta. Si v > 0, sea u = λv con λ ≥ 0; ladesigualdad a probar es

λavavb ≤ λav + bv,

o bien, dado que a + b = 1 y v = 0,

λa ≤ λa + 1 − a.

Consideremos la funcion derivable f(λ) = λa + 1 − a − λa para λ ∈ [0, +∞). Se tienef(0) = 1 − a > 0. La derivada es f ′(λ) = a− aλa−1 y solo se anula para λ = 1. Para estevalor de λ, la derivada segunda f ′′(λ) = −a(a−1)λa−2 toma el valor f ′′(1) = (1−a)a > 0.Como f(1) = 0, f(λ) alcanza su valor mınimo absoluto igual a 0 en λ = 1. Luego f(λ) ≥ 0para todo λ ≥ 0, como debıamos probar.

Page 521: COMPENDIUM OME - ToomatesEotvos de Hungría, que se iniciaron en 1894, precisamente durante la efervescencia de fin de siglo, consecuencia de la cual fue también el proceso iniciado

E OME 18. Problema 7. Solucion

Sean a/b y c/d dos fracciones con b y d numeros impares. Entonces su suma

ad + bc

bd

tiene el denominador impar bd (o un divisor suyo, que sera tambien impar). Igualmentesu producto

ac

bdtambien tendra denominador impar. Las inversas de las fracciones de

numerador y denominador impar, tienen denominador impar. Las tres contestaciones sonpues afirmativas.

Page 522: COMPENDIUM OME - ToomatesEotvos de Hungría, que se iniciaron en 1894, precisamente durante la efervescencia de fin de siglo, consecuencia de la cual fue también el proceso iniciado

E OME 18. Problema 8. Solucion

Si d (A, B) indica la distancia de A a B y (x, y) son las coordenadas de M se tendra

d (MC) = min√

(x − 1)2 + y2,√

(x − 2)2 + y2 =

√(x − 1)2 + y2 si x < 3

2√(x − 2)2 + y2 si x ≥ 3

2

d (MC ′) = min√

x2 + (y − 1)2,√

x2 + (y − 7)2 = √

x2 + (y − 1)2 si y < 4√x2 + (y − 7)2 si x ≥ 4.

En el primer caso la igualdad se tiene si x =32, y en el segundo caso si y = 4.

Ası pues, la igualdad se da de la siguiente forma:

Si x <32; y < 4 al igualar las distancias obtenemos que y = x,

Si x <32; y ≥ 4, tenemos y =

x + 247

,

Si x ≥ 32; y < 4, se tiene y =

4x − 32

.

Y por ultimo, si x ≥ 32; y ≥ 4 obtenemos y =

4x + 4514

.

La segunda parte no sirve, pues la recta que se obtiene no tiene interseccion con la regionen la que esta definida.

Recomponiendo los datos anteriores tenemos que:

f(x) =

x si x <32,

2x − 32

si32≤ x <

114

,

2x

7+

4514

si x ≥ 114

.

Page 523: COMPENDIUM OME - ToomatesEotvos de Hungría, que se iniciaron en 1894, precisamente durante la efervescencia de fin de siglo, consecuencia de la cual fue también el proceso iniciado

La grafica es

32

114

x

2x − 32

27x + 45

14

Evidentemente la funcion no es derivable ni en x = 32

ni en x = 114

, pues su derivada es:

f(x) =

1 si x < 32 ,

2 si 32 < x < 11

4 ,27

si x > 114

.

Page 524: COMPENDIUM OME - ToomatesEotvos de Hungría, que se iniciaron en 1894, precisamente durante la efervescencia de fin de siglo, consecuencia de la cual fue también el proceso iniciado

E OME 19. Problema 1. Solucion

Sea la longitud total de la cuerda y t la longitud de las tangentes.

OB D

T

T ′

N P

t

α

d (O,P)=y

d (O,N)=x

Se tiene

= y − x + 2t + T ′BT = y − x + 2t + 2πr − T ′T ⇒ y = + x − 2t − 2πr + T ′T .

Expresando x, t y el arco T ′DT en funcion del angulo α, resulta:

OT

x= cosα ⇒ x =

r

cosα;

t

r= tanα ⇒ t = r tanα

El arco TT ′ mide 2rα, y por tanto y = f(α) = +r

cosα− 2r tanα − 2πr + 2αr.

La cuerda se rompe cuando el punto P esta lo mas alejado posible de C, es decir, cuandof(α) es maxima.

f ′(α) =r senα

cos2 α− 2r

cos2 α+ 2r

f ′(α) = 0 ⇔ senα (1 − 2senα) = 0.

El maximo se alcanza en α =π

6, y por tanto x =

2r√3

=2r√

33

.

Observacion. Una simple consideracion fısica permite resolver mas facilmente el problema.Si el nudo no produce fuerzas de rozamiento, la tension a lo largo de la cuerda es la mismaen todos los puntos. En particular, en el punto N tenemos tres fuerzas de igual moduloque deben sumar 0. Esta condicion obliga a que formen angulos de 120. Se deduce queTNO = 60 y que α = 30.

Page 525: COMPENDIUM OME - ToomatesEotvos de Hungría, que se iniciaron en 1894, precisamente durante la efervescencia de fin de siglo, consecuencia de la cual fue también el proceso iniciado

E OME 19. Problema 2. Solucion

A

C

C′

I

B

B′

AB′=p

AC′=q

Dado el angulo, se trazan paralelas a sus lados a una distancia igual al radio r de lacircunferencia inscrita. Su punto de corte es el incentro I. Con centro en I y radio r setraza la circunferencia inscrita. Si la razon de los lados es

p

q, sobre los lados del angulo a

partir del vertice, se toman las distancias p y q determinando el triangulo AB′C ′. Se trazaahora la recta paralela a B′C ′, tangente a la circunferencia inscrita que deja en el mismosemiplano a A y a la circunferencia inscrita.

Page 526: COMPENDIUM OME - ToomatesEotvos de Hungría, que se iniciaron en 1894, precisamente durante la efervescencia de fin de siglo, consecuencia de la cual fue también el proceso iniciado

E OME 19. Problema 3. Solucion

Primera solucion

Mn =r

n(r sen

π

n + 1+ r sen

n + 1+ · · ·+ r sen

n + 1)

limn→∞

Mn = r2 limn→∞

senπ

n + 1+ sen

n + 1+ · · ·+ sen

n + 1n + 1

=

=r2

πlim

n→∞

senπ

n + 1+ sen

n + 1+ · · ·+ sen

n + 1n + 1

π

=r2

π

∫ π

0

senx dx =2r2

π

Segunda solucion

Sean P1, . . . , Pn los puntos de division de la semicircunferencia, en sentido antihorario.Uniendolos con el centro de la semicircunferencia se forman los angulos centrales

t1 =π

n + 1, t2 =

n + 1, . . . , tk =

n + 1, . . . , tn =

n + 1.

El area es Ak = r2 senkπ

n + 1, ası que debemos calcular

limn→∞

A1 + A2 + ... + An

n.

En primer lugar vamos a calcular la suma

senπ

n + 1+ ... + sen

n + 1. (1)

Para ello consideramos los numeros complejos

u = cos t + i sen t

u2 = cos 2t + i sen2t

· · ·un = cosnt + i sennt,

donde para escribir los segundos miembros hemos utilizado la formula de De Moivre.Entonces, sumando obtenemos

u + u2 + ... + un = (cos t + cos 2t + · · ·+ cosnt) + i (sen t + sen 2t + · · ·+ sennt)

ası que la suma que buscamos (de senos de angulos en progresion aritmetica) es la parteimaginaria del numero complejo del primer miembro, que por tratarse de una suma determinos de una progresion geometrica se puede escribir

Page 527: COMPENDIUM OME - ToomatesEotvos de Hungría, que se iniciaron en 1894, precisamente durante la efervescencia de fin de siglo, consecuencia de la cual fue también el proceso iniciado

u(un − 1)u − 1

=(cos t + i sen t)(cosnt + i sennt− 1)

cos t + i sen t − 1=

=(cos t + i sen t)(cosnt + i sennt − 1)(cos t − 1 − i sen t)

(cos t − 1)2 + sen 2t

El denominador de esta ultima fraccion es 2(1 − cos t) = 4sen 2 t

2.

Por su parte, del numerador solo nos interesa la parte imaginaria, que es

sennt− cos t sennt − sen t cosnt + sen t ==sennt + sen t− (sennt cos t + cosnt sen t) =

=2 sen(n + 1) t

2cos

(n − 1) t

2− sen (n + 1) t =

=2 sen(n + 1) t

2cos

(n − 1) t

2− 2 sen

(n + 1) t

2cos

(n + 1) t

2=

=2 sen(n + 1) t

2

(cos

(n − 1) t

2− cos

(n + 1) t

2

)=

=2 sen(n + 1) t

2

[−2 sen

nt

2sen

(− t

2

)]= 4 sen

(n + 1) t

2sen

nt

2sen

t

2

En definitiva queda

sen(n + 1) t

2sen

nt

2

sent

2

,

y poniendo t = πn+1

, se trata de calcular el lımite

limn→∞

sen(n + 1)π

2(n + 1)sen

n π

2(n + 1)

n senπ

2(n + 1)

.

En el numerador, el primer factor es sen π2

= 1; el segundo factor tiene lımite 1, porquelimn→∞ n/(n + 1) = 1 y, de nuevo, senπ/2 = 1.Para calcular el lımite del denominador, como π

2(n+1)tiende a cero cuando n tiende a

infinito, sustituimos el seno por el arco, con lo cual debemos hallar el lımite

limn→∞

2(n + 1)=

π

2,

ası que

limn→∞

A1 + A2 + · · ·+ An

n=

2 r2

π.

Page 528: COMPENDIUM OME - ToomatesEotvos de Hungría, que se iniciaron en 1894, precisamente durante la efervescencia de fin de siglo, consecuencia de la cual fue también el proceso iniciado

E OME 19. Problema 4. Solucion

Sea f(x) = 16x5 − 20x3 + 5x = x(16x4 − 20x2 + 5).Si f(x) = 0, tenemos que x = 0 y que 16x4 − 20x2 + 5 = 0 lo que nos proporciona

x = ±√

5 +√

58

y x = ±√

5 −√

58

.

Entonces f ′(x) = 80x4 − 60x2 + 5 y si f ′(x) = 0 debe ser x = ±√

3 ±√

58

; el mınimo se

alcanza en x =

√3 +

√5

8y en x = −

√3 −

√5

8, y en los otros dos puntos se alcanza el

maximo , x = −√

3 +√

58

y en x =

√3 −

√5

8.

Pero ademas

f

√3 −

√5

8

= f

√3 +

√5

8

= −1 y f

√3 −

√5

8

= f

√3 +

√5

8

= 1.

Finalmente:Si −1 < m < 1 se tienen 5 raıces reales.Si m = ±1 hay tres raıces reales.Si m > 1 o m < −1 hay una raız real.

Page 529: COMPENDIUM OME - ToomatesEotvos de Hungría, que se iniciaron en 1894, precisamente durante la efervescencia de fin de siglo, consecuencia de la cual fue también el proceso iniciado

E OME 19. Problema 5. Solucion

Inmediatamente se comprueba que a = 4.En un giro de centro B y amplitud 90, la recta x = 0 se transforma, dependiendo delsentido de giro, en y = 4 o en y = −4. Las posibles posiciones de A estan dadas por lainterseccion de estas dos rectas con y = 2x + 6.En el primer caso se trata del punto A1(−1, 4). La perpendicular a A1B por B corta aleje x = 0 en el punto (0,−5), que es C1. Con esto, el cuarto vertice es D1(−5,−1).En el segundo caso, el punto A2, interseccion de y = 2x + 6 con y = −4 es

A2(−5,−4);

El punto C2 se calcula de la misma manera que antes, dando C2(0, 9), y finalmente,D2(−9, 5).

Page 530: COMPENDIUM OME - ToomatesEotvos de Hungría, que se iniciaron en 1894, precisamente durante la efervescencia de fin de siglo, consecuencia de la cual fue también el proceso iniciado

E OME 19. Problema 6. Solucion

Llamaremos al precio de un vaso de limonada, s el precio de un sandwich y b al preciode un bizcocho. Nos preguntamos por el precio de + s + b, y por el precio de 2 + 3s +5bsabiendo que + 3s + 7b = 14 y que + 4s + 10b = 17. Restando estas dos igualdadestenemos que s + 3b = 3.Es decir + s+ b = +3s+7b−2s−6b = 14−6 = 8 y ademas tenemos que 2+3s+5b =2( + s + b) + s + 3b = 16 + 3 = 19.

Page 531: COMPENDIUM OME - ToomatesEotvos de Hungría, que se iniciaron en 1894, precisamente durante la efervescencia de fin de siglo, consecuencia de la cual fue también el proceso iniciado

E OME 19. Problema 7. Solucion

Supondremos que la unidad de longitud de la arista es el cm; para simplificar los calculosdaremos las medidas en dm. Ademas, por ”area de la superficie libre de agua” enten-deremos el area del triangulo equilatero formado por la capa superior de agua dentro deltetraedro.Primero resolveremos el problema sin datos numericos (pero midiendo todas las longitudesen dm).Entonces tenemos:Si b es la arista del tetraedro, V su volumen, y x es la arista del tetraedro libre de agua,se tiene (x

b

)3

=V − 2

V, de donde x = b

3

√V − 2

V.

Llamando Sb al area de la base del tetraedro de arista b y Sx a la del de arista x, se cumpletambien

Sx

Sb=

(x

b

)2

, luego Sx =x2 · Sb

b2.

Por ultimo, la altura que alcanza el agua es la diferencia entre las alturas de los dostetraedros, hb y hx, de modo que

b

x=

hb

hx, es decir,

b − x

x=

hb − hx

hx,

y por lo tanto

hb − hx =hx(b − x)

x.

El area de la base de un tetraedro de arista b es

Sb =b2√

34

;

y su altura,

hb =b√

63

.

Por tanto, su volumen es

V =b3√

212

.

En nuestro caso, b = 3 dm, ası que V =94√

2 > 2, y V − 2 =9√

2 − 84

.

Page 532: COMPENDIUM OME - ToomatesEotvos de Hungría, que se iniciaron en 1894, precisamente durante la efervescencia de fin de siglo, consecuencia de la cual fue también el proceso iniciado

Como

Sx =√

34

b2 · 3

√(V − 2

V

)2

,

poniendo x = 3 resulta

Sx =9√

34

· 3

√√√√(9 − 4

√2

9

)2

.

Por su parte,

hb − hx =√

6 · 3

√9 − 4

√2

9

1 − 3

√9 − 4

√2

9

.

Page 533: COMPENDIUM OME - ToomatesEotvos de Hungría, que se iniciaron en 1894, precisamente durante la efervescencia de fin de siglo, consecuencia de la cual fue también el proceso iniciado

E OME 19. Problema 8. Solucion

1960 1960 + d 1960 + 23 (x + y + z) 1960 + 4z

edad del hermano mayor x x + d x + 23(x + y + z) x + 4z

edad del hermano medio y y + d y + 23 (x + y + z) y + 4z

edad del hermano menor z z + d z + 23(x + y + z) z + 4z

Tenemos x = y + z. Si 2(x + d) = y + d + z + d, entonces 2x = y + z, y como x = y + zresulta x = 0, luego, 2(y + d) = x+ d+ z + d implica 2y = x+ z, y como x = y + z, resultay = 2z, x = 3z.

Ademas23(x + y + z) = 4z y como x + 4z = 21, queda 7z = 21, de donde z = 3

De z = 3 sale x = 9 y y = 6, y por tanto las edades son 21, 18, 15.

Page 534: COMPENDIUM OME - ToomatesEotvos de Hungría, que se iniciaron en 1894, precisamente durante la efervescencia de fin de siglo, consecuencia de la cual fue también el proceso iniciado

E OME 20. Problema 1. Solucion

Sean AB y CD las trayectorias rectilıneas seguidas por los tanques. Supongamos el prob-lema resuelto y MN + PQ = (MN sobre AB y PL sobre CD).

M

M ′

A

N

N ′

B

O

P QC D

R S

α

Girando la recta AB, con centro O el angulo que forman las rectas AB y CD, se tienela recta A′B′ y la figura PM ′N ′Q, (M ′N ′ = MN) es un trapecio; la paralela media RSmedira

12(PQ + M ′N ′) =

12(PQ + MN) =

l

2.

Por otra parte P y M ′ equidistan de O, luego RO es la mediatriz de M ′P y analogamenteSO lo es de N ′Q.La construccion resulta evidente: se gira AB para que sea paralela a CD, se dibuja unarecta equidistante de ambas y se traza la perpendicular por O; a partir del punto de

interseccion se toman a un lado y a otro dos segmentos de longitudl

4, hallandose los

puntos R y S. Las perpendiculares a RO por R y a SO por S, cortan, respectivamente,a A′B′ y a CD en M ′ y N ′ y en P y Q. La distancia de O a cualquiera de estos puntos(radio del cırculo circunscrito al trapecio PM ′N ′Q) es el alcance del canon.

Page 535: COMPENDIUM OME - ToomatesEotvos de Hungría, que se iniciaron en 1894, precisamente durante la efervescencia de fin de siglo, consecuencia de la cual fue también el proceso iniciado

E OME 20. Problema 2. Solucion

La forma mas simple de resolver el problema es tratar de encontrar un numero abcde talque abcde · abcde= . . .abcde, los posibles valores para e son 0, 1, 5 o 6. Si e= 0 tendrıamosel numero 00000; si e= 1 tenemos la solucion 00001.Si e= 5, entonces d= 2 y como 10c+6 termina en c, entonces c= 6. Para obtener btenemos que (a0000+b625) · (a0000+b625) = . . .b625, lo que nos dice que 10b+10 terminaen b, con lo cual b= 0. Finalmente a0625·a0625 =a0625, con lo cual 10a+9 termina en 9y por lo tanto a= 9. Entonces el numero solucion es:

906252 = 821890625.

Si e= 6, siguiendo el mismo procedimiento se obtiene que:

093762 = 87909376.

Page 536: COMPENDIUM OME - ToomatesEotvos de Hungría, que se iniciaron en 1894, precisamente durante la efervescencia de fin de siglo, consecuencia de la cual fue también el proceso iniciado

E OME 20. Problema 3. Solucion

1) Como x > 0, y > 0 existen√

x y√

y tales que

(√

x −√y)2 ≥ 0; x + y − 2

√xy ≥ 0, de donde:

x + y

2≥ √

xy .

2) Six2 + y2

2≥

(x + y

2

)2

, es cierto, haciendo operaciones se llega a la formula evidente

x2 + y2 − 2xy ≥ 0. Desarrollando los pasos al reves tendrıamos el resultado.

3) Como p + q = 1, tenemos que p · q ≤ 14

(el maximo del producto se obtiene cuando

ambos son iguales).

En este caso: p +1p

+ q +1q

= p + q +p + q

pq= 1 +

1pq

≥ 5, lo que equivale a

(p +

1p

)2

+(

q +1q

)2

≥ 25 − 2(

p +1p

)·(

q +1q

)

Pero

[(p +

1p

)−

(q +

1q

)]2

≥ 0 =⇒(

p +1p

)2

+(

q +1q

)2

≥ 2(

p +1p

)·(

q +1q

).

Y sumando estas dos desigualdades se obtiene

(p +

1p

)2

+(

q +1q

)2

≥ 252

.

Page 537: COMPENDIUM OME - ToomatesEotvos de Hungría, que se iniciaron en 1894, precisamente durante la efervescencia de fin de siglo, consecuencia de la cual fue también el proceso iniciado

E OME 20. Problema 4. Solucion

Puesto que se cumple

sen x = 2 senx

2cos

x

2=

= 22senx

22cos

x

22cos

x

2=

= · · ·

= 2nsenx

2ncos

x

2ncos

x

2n−1· · · cos

x

2,

y tambien que

limn→∞

2nsenx

2n= x,

porque senx

2nes equivalente a

x

2n, el lımite pedido es

limn→∞

cosx

2cos

x

22cos

x

23· · · cos

x

2n=

sen x

x.

Page 538: COMPENDIUM OME - ToomatesEotvos de Hungría, que se iniciaron en 1894, precisamente durante la efervescencia de fin de siglo, consecuencia de la cual fue también el proceso iniciado

E OME 20. Problema 5. Solucion

Podemos suponer sin perdida de generalidad que el radio de las circunferencias dadas es 1.Tomando unos ejes con origen en el punto medio O de los centros de las circunferencias,eje OX la recta que los une y llamando 2d a la distancia entre los centros, tenemos parael caso a)

aA

B

a+x

O

M

M ′A′

B′b+x

b

B′′

b−x

X

Y

A = (−d + cos a, sen a); A′ = (d + cos b, sen b)B =

(− d + cos(a + x), sen (a + x)

); B′ =

(d + cos(b + x), sen (b + x)

)

y para el punto medio M del segmento BB′

Mx =12(cos(a + x) + cos(b + x)

)

My =12(sen (a + x) + sen (b + x)

)

para eliminar x, elevando al cuadrado y sumando queda

M2x + M2

y =14

(1 + 1 + 2 cos(a + x) cos(b + x) + 2sen (a + x)sen (b + x)

)=

12

(1 + cos

((a + x) − (b + x)

))=

12

(1 + cos(a − b)

)

como a y b son fijos, el lugar de M es una circunferencia de centro el origen y radio

√12

(1 + cos(a − b)

).

Page 539: COMPENDIUM OME - ToomatesEotvos de Hungría, que se iniciaron en 1894, precisamente durante la efervescencia de fin de siglo, consecuencia de la cual fue también el proceso iniciado

Para el caso b) basta cambiar x por −x en las coordenadas de B′ resultando para el nuevopunto medio M ′

Mx =12(cos(a + x) + cos(b − x)

)

My =12(sen (a + x) + sen (b − x)

)y operando,

M ′x = 1

22 cos (a+b)

2cos x

M ′y = 1

22 sen (a+b)

2cos x

−−→OM ′ = cos x

(cos

a + b

2, sen

a + b

2

)

es decir, el lugar de M ′ es el segmento de extremos P

(cos

a + b

2, sen

a + b

2

)y Q, simetrico

de P respecto del origen.La figura siguiente muestra ambos lugares en trazo grueso.

aA

B

a+x

OM

M ′

A′

B′

b+x

b

B′′ b−x

X

Y

P

Q

Page 540: COMPENDIUM OME - ToomatesEotvos de Hungría, que se iniciaron en 1894, precisamente durante la efervescencia de fin de siglo, consecuencia de la cual fue también el proceso iniciado

E OME 20. Problema 6. Solucion

Consideramos la circunferencia γ y la recta r. Sea t el angulo que la recta variable formacon OX . Se tiene que

O

M

P

X

M ′ P ′x

y

OM = 6 cos t, MM ′ = 6 cos t sen t = 3 sen 2t; OP ′ =3

tan t. Las coordenadas del punto

son X =( 3

tan t, 3 sen 2t

), que son las ecuaciones parametricas del lugar. Eliminando t

tenemosy =

18x

x2 + 9

que es la ecuacion cartesiana del lugar.

Page 541: COMPENDIUM OME - ToomatesEotvos de Hungría, que se iniciaron en 1894, precisamente durante la efervescencia de fin de siglo, consecuencia de la cual fue también el proceso iniciado

E OME 20. Problema 7. Solucion

a) Escribimos el numero en la forma 10pa+x, donde x es el numero de p cifras que resulta

de suprimir las primeras. Del enunciado se obtiene 10pa + x = 5x, es decir, x =10pa

4.

Como x tiene p cifras, se cumple x < 10p, luego a < 4, de manera que la primera cifra apuede ser 1, 2 o 3. El menor numero es el 25, que se obtiene para p = 1 y a = 2. Losdemas numeros con esta propiedad tienen la forma 125a · 10p−2, con p ≥ 2 y a = 1, 2, 3.

b) En este caso, 10pa+x = 12x, de donde x =10pa

11; pero este numero no puede ser entero

ya que a es un entero comprendido entre 1 y 9.

c) De la misma forma, 10pa+x = kx, donde 1 ≤ a ≤ 9, y resulta x =10pa

k − 1, con a < k−1,

o biena

k − 1=

x

10p.

Esta formula permite formular el siguiente criterio general: El numero abc . . . h(10 quedadividido por k al suprimir la primera cifra (es decir, la cifra a), si y solo si la division de apor k − 1 da el decimal exacto 0.bc . . . h en base 10.En particular, el denominador de la fraccion irreducible correspondiente a la fraccion

a

k − 1solo puede tener los factores primos 2 y 5.Por ejemplo, el entero 31875 queda dividido por 17 al suprimir el 3, pues 3/16 = 0.1875 .

Page 542: COMPENDIUM OME - ToomatesEotvos de Hungría, que se iniciaron en 1894, precisamente durante la efervescencia de fin de siglo, consecuencia de la cual fue también el proceso iniciado

E OME 20. Problema 8. Solucion

Consideremos el polinomio de grado 5 que se obtiene al desarrollar el determinante

P (x) =

∣∣∣∣∣∣∣

x3 + 3x 2 1 0x2 + 5x 3 0 2

x4 + x2 + 1 2 1 3x5 + 1 1 2 3

∣∣∣∣∣∣∣.

Si hacemos la division por x2 − 1 podemos escribirP (x) = (x2 − 1)C(x) + ax + b, siendo C(x) el polinomio cociente, y ax + b el resto. Dandovalores a x saleP (1) = a + b y P (−1) = −a + b. Calculamos P (1) = 15 y P (−1) = −69; resolvemos elsistema y tenemos que b = −27 y a = 42. Por lo tanto el resto es

42x − 27.

Page 543: COMPENDIUM OME - ToomatesEotvos de Hungría, que se iniciaron en 1894, precisamente durante la efervescencia de fin de siglo, consecuencia de la cual fue también el proceso iniciado

E OME 21. Problema 1. Solucion

Vamos a denotar por (PQ) la recta determinada por dos puntos distintos P y Q, y abre-viaremos f(P ) por P ∗. De acuerdo con la condicion (a), (f es inyectiva), P = Q ⇒ P ∗ =Q∗, y de acuerdo con la condicion (b) se tiene, cuando P = Q que f

((PQ)

)= (P ∗Q∗). Se

puede senalar que la condicion de ser f suprayectiva es superflua cumpliendose las demas,pues suponiendo que haya un punto P sin preimagen, debe ser P ∗ = P , y entonces por lacondicion (c) se deduce que la recta f

((PP ∗)

)coincide con (PP ∗) (ya que comparte con

ella el punto P ∗), luego, usando (b), el punto P ha de tener preimagen, contra lo supuesto.Por otra parte, cuando sea P ∗ = P diremos que P es un punto fijo de f . Atendiendo alnumero de puntos fijos de f , los tres lemas siguientes cubren todas las posibilidades:Lema 1.– Si f no tiene ningun punto fijo, es una traslacion de vector no nulo.Demostracion.– Fijemos en nuestra consideracion un punto arbitrario P . Por hipotesis esP ∗ = P y ası

−−→P P ∗ = 0.

Sea ahora Q = P . En primer lugar, si Q /∈ (PP ∗), aplicando ademas la hipotesis (Q∗ =Q) y la condicion (c), la recta (P ∗Q∗) es paralela pero no coincidente con (PQ). Si(PP ∗) y (QQ∗) se cortasen en un punto O, se tendrıa O∗ = O (pues como entoncesP ∗ ∈ (OP ) ∩ (O∗P ∗) y (OP ) debe ser paralela o coincidente con (O∗P ∗), se sigue queestas dos rectas son coincidentes, luego O∗ ∈ (OP ); y de igual modo O∗ ∈ (OQ), asıque O∗ = (OP ) ∩ (OQ) = O), contra la hipotesis. Luego las rectas (PP ∗) y (QQ∗) sonparalelas y no coincidentes y se tiene

−−→QQ∗ =

−−→P P ∗.

P Q

P ∗ Q∗

En segundo lugar, si Q ∈ (PP ∗), podemos siempre considerar otro punto R /∈ (PQ);aplicando dos veces la argumentacion anterior, para el punto Q /∈ (RR∗) y para el puntoR /∈ (PP ∗), se obtiene

−−→QQ∗ =

−−→RR∗ =

−−→P P ∗, y el lema queda probado.

Lema 2.– Si f tiene exactamente un punto fijo O, es una homotecia de centro O y razonk = 1.Demostracion.– Sea P = O un punto arbitrario que fijamos en nuestra consideracion; comoO∗ = O, es (O∗P ∗) = (OP ∗) y, usando (c), se tiene que (OP ∗) coincide con (OP ), luego

P ∗ ∈ (OP ). Ademas es P ∗ = P por hipotesis. Sea entonces k =OP ∗

OP= 1 (consideramos

estas distancias orientadas desde O).

Page 544: COMPENDIUM OME - ToomatesEotvos de Hungría, que se iniciaron en 1894, precisamente durante la efervescencia de fin de siglo, consecuencia de la cual fue también el proceso iniciado

Y ahora sea en primer lugar Q /∈ (OP ); por el mismo razonamiento anterior se tieneque Q∗ ∈ (OQ) y, segun (c), la recta (P ∗Q∗) es paralela pero no coincidente con (PQ).

Aplicando el teorema de Thales se obtieneOQ∗

OQ=

OP ∗

OP= k.

P Q

P ∗ Q∗

En segundo lugar, si Q ∈ (OP ) y Q = O, considerando otro punto R /∈ (OP ) y aplicandodos veces lo ya demostrado, se tiene

OR∗

OR=

OP ∗

OPy

OQ∗

OQ=

OR∗

OR,

luego tambien en este caso esOQ∗

OQ=

OP ∗

OP= k.

Lema 3.– Si f tiene mas de un punto fijo, entonces todos los puntos del plano son fijos yf es la transformacion identidad.Demostracion.– Supongamos que P y Q son dos puntos distintos y fijos por f . Sea R unpunto cualquiera diferente de esos dos. En primer lugar, si R /∈ (PQ), aplicando (c) seobtiene

(P ∗R∗) = (PR∗) = (PR)

y(Q∗R∗) = (QR∗) = (QR).

Pero entonces R = (PR) ∩ (QR) = (P ∗R∗) ∩ (Q∗R∗) = R∗, de modo que R es un puntofijo de f .En segundo lugar, si R ∈ (PQ), se considera otro punto S /∈ (PQ) para el cual, por lamisma argumentacion anterior, se tendra S∗ = S. Ahora bien, segun (c) es R∗ ∈ (SR) ysegun (b) es R∗ ∈ (P ∗Q∗) = (PQ), luego R∗ = (SR) ∩ (PQ) = R, y tambien en este casoR es punto fijo de f .

Page 545: COMPENDIUM OME - ToomatesEotvos de Hungría, que se iniciaron en 1894, precisamente durante la efervescencia de fin de siglo, consecuencia de la cual fue también el proceso iniciado

E OME 21. Problema 2. Solucion

Cualquier recta que pase por el origen determina dos semiplanos. Si la recta tiene pendienteracional, contiene puntos de coordenadas enteras distintos del origen. Si la pendiente esirracional, solamente contiene al origen.Como ejemplos de conjuntos E pueden tomarse los puntos de coordenadas enteras de cadauno de los semiplanos determinados por tales rectas. En el caso de que la recta separadoratenga pendiente racional, debe elegirse una semirecta de ella desde el origen como partede E y la otra semirecta como parte de −E.Dado un irracional cualquiera k, los conjuntos

E = (x, y)| x < ky ∪ 0

o bienE′ = (x, y)| x > ky ∪ 0

cumplen trivialmente todo lo pedido en el enunciado.Dado un racional cualquiera k, los conjuntos del tipo

E = (x, y)| x < ky ∪ (x, y)| y > 0 ∧ x = ky ∪ 0

tambien cumplen las condiciones.

Se puede demostrar que los conjuntos E antes descritos son los unicos que cumplen lascondiciones del enunciado. (Solucion de Vıctor Gonzalez Alonso).1) En primer lugar demostraremos que −E cumple las mismas propiedades que E. Elhecho de que un par (x, y) es de −E si y solo si su opuesto es de E se desprende de laspropiedades de E, y −E es cerrado por la suma, porque de no serlo, existirıan dos paresde −E que sumados estarıan en E, pero tomando los opuestos (que seran de E), su suma(que sera la opuesta de la que habıamos obtenido) tambien serıa de E lo que contradice elhecho de que E no contiene simultaneamente un par y su opuesto.2) El siguiente paso es demostrar que si (x, y) ∈ E, todos los puntos de la semirrecta quepasa por el y sale del origen (es decir, los puntos (x′, y′) del mismo cuadrante que (x, y)tales que y

x= y′

x′ , tambien estan en E.Sea d el maximo comun divisor de x e y. Entonces el punto (x′, y′) = (x

d , yd ) ha de estar

en E o en −E, pero todos los puntos de la semirrecta de la que estamos hablando sonmultiplos positivos de este punto (que llamaremos generador de la semirrecta), por lo que(x, y) esta en el mismo subconjunto (E o −E) que (x′, y′), lo que indica que (x′, y′) ∈ E ytodos sus multiplos positivos (que forman toda la semirrecta).3) Puesto que el punto (1, 0) ha de estar en E o en −E, y −E tiene las mismas propiedadesque E, podemos suponer que (1, 0) ∈ E (y por tanto, todo el semieje positivo de abscisas).Una vez asumido esto, demostremos que si un punto (a, b) = (0, 0) (por el segundo punto

Page 546: COMPENDIUM OME - ToomatesEotvos de Hungría, que se iniciaron en 1894, precisamente durante la efervescencia de fin de siglo, consecuencia de la cual fue también el proceso iniciado

de la demostracion, podemos asumir que a y b son primos entre sı, y que si a es 0, |b| = 1)es de E, todos los puntos que quedan en el sector convexo determinado por la semirrectagenerada por (a, b) y el semieje positivo de abscisas estan tambien en E.Consideremos tres casos:• a = 0:Es evidente, puesto que si un punto (c, d) esta en el sector considerado, es porque c ≥ 0y d tiene el mismo signo que a, por lo que podemos poner (c, d) como combinacion linealentera positiva de (0, b) y (1, 0)

(c, d) = c(1, 0) + |d|(0, b).

• a > 0:En este caso, la condicion equivalente a que el punto este en el sector (y no en las semir-rectas, en cuyo caso el resultado es obvio) es:

0 <|d|c

<|b|a

donde d y b tienen el mismo signo (basta considerar las pendientes).Veamos que en estas condiciones, la ecuacion diofantica k(c, d) = α(a, b) + β(1, 0) tienesolucion positiva (notese que intentamos obtener un multiplo de (c, d), no necesariamenteel propio (c, d), pero por el segundo punto de la demostracion podremos decir que (c, d)tambien estara en E).Si tomamos k = |b|, α = |d| y β = kc − αa, es evidente que k y α son positivos, y que βtambien lo es se deduce de la relacion entre las pendientes.• a < 0:En este caso, notese primero que el semieje correspondiente de ordenadas tambien esta enE, puesto que podemos obtener el punto (0, b) como (a, b) + |a|(1, 0).Por tanto, si c ≥ 0 se reduce a uno de los dos casos anteriores, por lo que tan solo habraque considerar el caso en que c < 0; entonces, a y c tendran el mismo signo, ası como b yd, y ahora la condicion caracterıstica sera:∣∣∣∣dc

∣∣∣∣ >

∣∣∣∣ b

a

∣∣∣∣(de nuevo, basta considerar las pendientes).Puesto que un semieje de ordenadas esta contenido en el sector (el correspondiente alsigno de b y de d), tan solo hemos de ver que podemos encontrar soluciones enteras parak(c, d) = α(a, b) + β(0, 1), donde k y α son positivos, y β tiene el mismo signo que b. Sitomamos α = |c| y k = |a|, tenemos que β = |a|d−|c|b, pero como |a||d|− |c||b| es positivo,es evidente que β tiene el signo deseado.4) Una vez demostrado esto, tan solo queda ver que la frontera entre E y −E es una rectaque pasa por el origen; para ello definiremos el argumento de un par (a, b) = (0, 0) como elangulo entre π y −π que forma la semirrecta que lo contiene con el eje positivo de abscisas.Una expresion analıtica podrıa ser la siguiente:

Page 547: COMPENDIUM OME - ToomatesEotvos de Hungría, que se iniciaron en 1894, precisamente durante la efervescencia de fin de siglo, consecuencia de la cual fue también el proceso iniciado

arg(x, y) =

π + arctan yx

Si x < 0, y ≥ 0π2 Si x = 0, y > 0arctan y

xSi x > 0

−π2 Si x = 0, y < 0

−π + arctan yx Si x < 0, y < 0

Sean α = sup(x,y)∈E arg(x, y) y β = inf(x,y)∈E arg(x, y), y hemos de ver que α − β = π.Si asumimos que el semieje positivo de abscisas esta en E, es obvio que 0 ≤ α ≤ π y−π ≤ β ≤ 0.• Supongamos que α − β > π. Entonces obtenemos las desigualdades siguientes:

π ≥ α > π + β ≥ 00 ≥ α − π > β ≥ −π

En particular, de la primera desigualdad y de la continuidad del argumento, deducimosque existe un racional q

pirreducible con q positivo, tal que α > arg(p, q) > π + β > β,

(notese que (p, q) ∈ E).Ahora bien, de la definicion de argumento se desprende que el argumento de dos paresopuestos difiere en π, y si restamos π a la desigualdad anterior obtenemos: α > α − π >arg(p, q) − π = arg(−p,−q) > β, y comparandolo con la segunda desigualdad, obtenemosque (−p,−q) tambien pertenece a E, en contradiccion con las propiedades de E.• Supongamos entonces que α − β < π, lo que da las desigualdades

α < π + β

α − π < β

Analogamente al caso anterior, existe un racional irreducible qp , q > 0, tal que α <

arg(p, q) < π + β, por lo que (p, q) ∈ E, y de nuevo, restando π a toda la desigual-dad, obtenemos que en este caso, (−p,−q) tampoco es de E, lo que nuevamente contradicelas propiedades de E.Por tanto, podemos concluir que α− β = π, lo que indica que la frontera entre E y −E esuna recta de pendiente m = tan α.5) Tan solo queda estudiar el caso en que la pendiente m es racional, puesto que si esirracional no contendra ningun punto de Z×Z y E estara perfectamente separado de −E.Ahora bien, si la pendiente m es racional, la recta contendra puntos de Z × Z, algunos deE y otros de −E; pero, por lo demostrado en el segundo punto, si contiene un punto, hade contener toda la semirrecta, pero por las propiedades de E, la semirrecta opuesta hade estar en −E.

Conclusion:

Las unicas maneras de separar Z × Z en dos conjuntos con las propiedades mencionadasconsisten en dividir el plano en dos semiplanos mediante una recta que pase por el origen;y si la recta tiene pendiente racional, asignar una de las semirrectas a E y la otra a −E.

Page 548: COMPENDIUM OME - ToomatesEotvos de Hungría, que se iniciaron en 1894, precisamente durante la efervescencia de fin de siglo, consecuencia de la cual fue también el proceso iniciado

E OME 21. Problema 3. Solucion

Primera solucion (de M. Ascension Lopez Chamorro)

La ecuaciontan2 2x + 2 tan 2x tan 3x − 1 = 0

se puede escribir como

tan 3x =1 − tan2 2x

2 tan 2x= cot 4x,

ası que3x + 4x =

π

2+ kπ, k ∈ Z,

es decir

x =π

14+

7.

Segunda solucion

Escribimos la ecuacion como

sen 2x

cos2 2x+ 2

sen 2x

cos 2x

sen 3x

cos 3x− 1 = 0

o biensen 22x cos2 3x + 2 sin 2x sin 3x cos 2x − cos2 2x cos 3x = 0

que se puede escribir

−(cos2 2x − sen 22x) cos 3x + (2sen 2x cos 2x)sen 3x = 0.

Simplificando queda− cos 4x cos 3x + sin 4x sin 3x = 0

o bien− cos 7x = 0.

Las soluciones sonx =

π

14+ k

π

7.

Page 549: COMPENDIUM OME - ToomatesEotvos de Hungría, que se iniciaron en 1894, precisamente durante la efervescencia de fin de siglo, consecuencia de la cual fue también el proceso iniciado

E OME 21. Problema 4. Solucion

Tenemos la identidad

a3 + b3 + c3 =((a + b + c)2 − 3(ab + bc + ac)

)(a + b + c) + 3abc;

pero comoa + b + c

abc=

1k

=⇒ abc = k(a + b + c)

la igualdad anterior se puede escribir

a3 + b3 + c3 = (a + b + c)[(a + b + c)2 − 3(ab + bc + ac) + 3k

]y por lo tanto este numero no es primo, ya que a + b + c > 1 y a + b + c < a3 + b3 + c3 yaque la terna a = b = c = 1 no cumple las condiciones.

Si k es impar, la terna

a =k + 3

2, b = 3, c = k;

cumple las condiciones, ya que

k(a + b + c) = k(k + 3

2+ 3 + k

)= k

(3k + 92

)= 3k

k + 32

= abc.

Para cualquier natural k, la terna

a = k + 2, b = k, c = 2;

cumple las condiciones, ya que

k(a + b + c) = k(k + 2 + k + 2) = 2k(k + 2) = abc.

Page 550: COMPENDIUM OME - ToomatesEotvos de Hungría, que se iniciaron en 1894, precisamente durante la efervescencia de fin de siglo, consecuencia de la cual fue también el proceso iniciado

E OME 21. Problema 5. Solucion

Se comprueba sin dificultad que z = −i es una solucion de la ecuacion; como

z3 + (−1 + i) z2 + (1 − i) z + i = (z + i)(z2 − z + 1

)y las raıces de z2 − z + 1 son

1 ± i√

32

.

Las tres raıces tienen modulo 1, y es inmediato que la circunferencia pedida es la circun-ferencia unidad en el plano complejo, de ecuacion |z| = 1.

Page 551: COMPENDIUM OME - ToomatesEotvos de Hungría, que se iniciaron en 1894, precisamente durante la efervescencia de fin de siglo, consecuencia de la cual fue también el proceso iniciado

E OME 21. Problema 6. Solucion

Primera solucion

O A

M

N

P

α

β

β

y

x

r2

r1

P1

Por la construccion de P , este punto equidista de Oy y de AN , ademas tambien equidistade Oy y de r1, luego P es el centro del cırculo exinscrito al triangulo AMN correspondienteal lado AN .

Llamando α al angulo de las semirrectas Ox, Oy, y poniendo β = OAM = ONA, resulta

PAN =2β + α

2= β +

α

2,

de donde se deduce

PAx =α

2.

Luego P estara en la bisectriz exterior del triangulo AMN correspondiente al vertice Aque forma un angulo constante α/2 con Ox.

Al ser P exincentro, equidista de la recta Oy y MA y esta distancia comun es PP1. En eltriangulo APP1 rectangulo en P1 es AP > PP1 = dist(P, Oy) y por tanto, el lugar de Pes una semirrecta con origen en un punto que cumple

dist(P, Oy) ≤ PA.

Page 552: COMPENDIUM OME - ToomatesEotvos de Hungría, que se iniciaron en 1894, precisamente durante la efervescencia de fin de siglo, consecuencia de la cual fue también el proceso iniciado

O A x

M

R

S

N

Q

P

y

r1

r2

Los puntos para los que se cumple la igualdad

dist(P, Oy) = PA

forman una parabola de foco A y directriz Oy. Basta entonces determinar la interseccionde la parabola con la recta AP mas proxima a P ; para ello se traza la perpendicular porA a AP que corta en R a Oy, la circunferencia de centro R y radio RA corta a Oy en S.Finalmente, la perpendicular por S a Oy corta a AP en el punto Q buscado y el lugarpedido es la semirrecta con origen en Q en la direccion que forma angulo α/2 con Ox.

Segunda solucion (M. Ascension Lopez Chamorro).

Sean Ox, Oy las semirrectas dadas, y A ∈ Ox. Convenimos en llamar M al punto masproximo a O (de M y N). Conviene observar que, al ser AM y AN antiparalelas conrespecto a Ox, Oy, la manera de construir dos puntos M , N en las condiciones del problemaes trazar una circunferencia tangente a Ox en A: los dos puntos de interseccion de lacircunferencia con Oy son los puntos M y N .Sea M0 ∈ Oy tal que OA = OM0 (El punto R de la figura).Por A se traza una paralela a Oy (la llamaremos Ay′); sea s la bisectriz interior del anguloxOy , y sea s′ la paralela a s trazada por A.Los siguientes angulos son iguales:

AM0O = OAM0 = M0Ay′ = α0 (le llamamos ası).

Los puntos M y N son los extremos de la cuerda determinada por Oy sobre una circun-ferencia tangente a Ox en A, como se ha dicho. En el caso de que la circunferencia seatangente a las dos semirrectas, r1 = r2 y los tres puntos M , N y M0 coinciden.

Page 553: COMPENDIUM OME - ToomatesEotvos de Hungría, que se iniciaron en 1894, precisamente durante la efervescencia de fin de siglo, consecuencia de la cual fue también el proceso iniciado

Las bisectrices de los angulos AMy y ANy determinan un exincentro del triangulo AMN ,y por lo tanto son concurrentes con la bisectriz exterior del angulo MAN . Ahora bien,AM0 es la bisectriz interior de NAM , pues poniendo α = OAM , se tiene

MAM0 = OAM0 − OAM = α0 − α,

M0AN = M0Ay′ − NAy′ = α0 − α.

Luego la bisectriz exterior de MAN es la recta s′, independientemente de la posicion deM y N sobre Oy.Puesto que el caso extremo lo tenemos para M ≡ N ≡ M0, si P0 es el punto de interseccionde la bisectriz de AM0y con s′, el lugar geometrico pedido es la semirrecta de origen P0

contenida en s′, y que no contiene al punto A.

Page 554: COMPENDIUM OME - ToomatesEotvos de Hungría, que se iniciaron en 1894, precisamente durante la efervescencia de fin de siglo, consecuencia de la cual fue también el proceso iniciado

E OME 21. Problema 7. Solucion

De acuerdo con las condiciones del problema, debera ser

x5 − px − 1 = (x2 − ax + b)(x3 + cx2 + dx + e) == x5 + (c − a)x4 + (d − ac + b)x3 + (e − ad + bc)x2 + (bd − ae)x + be

ası que, identificando coeficientes, se obtiene el sistema

c − a = 0d − ac + b = 0

e − ad + bc = 0bd − ae = −p

be = −1

De la primera y quinta ecuaciones sale c = a, e = −1/b; de la segunda, d = a2 − b, ası que,sustituyendo en la cuarta,

p = −a + b(a2 − b)b

.

Ahora bien, como el sistema tiene que ser compatible, se debe cumplir la tercera ecuacioncon los valores anteriores, luego

−1b− a

(a2 − b

)+ ba = 0,

es decir 1 = ab (2b − a).Ya que a y b son enteros, esta igualdad solo es posible si los factores son +1 o −1; en elprimer caso, el sistema

ab = 12b − a = 1

equivale a la ecuacion cuadratica a2 + a − 2 = 0, cuyas raıces son a = 1 y a = −2; estaultima no es valida porque entonces b no serıa entero (b = −1/2).Por lo tanto, en este supuesto se tiene

a = 1, b = 1 y p = −1.

La otra posibilidad conduce al sistemaab = −1

2b − a = −1

Page 555: COMPENDIUM OME - ToomatesEotvos de Hungría, que se iniciaron en 1894, precisamente durante la efervescencia de fin de siglo, consecuencia de la cual fue también el proceso iniciado

equivalente a la ecuacion cuadratica a2 − a + 2 = 0, que no tiene soluciones reales. Por lotanto, la unica solucion del problema es

a = 1, b = 1 y p = −1,

de la que se obtiene c = 1, d = 0, e = −1, y la descomposicion factorial del polinomio

x5 + x − 1 = (x2 − x + 1)(x3 + x2 − 1) .

Las dos raıces mencionadas en el enunciado son

x1 =1 + i

√3

2, x2 =

1 − i√

32

.

Page 556: COMPENDIUM OME - ToomatesEotvos de Hungría, que se iniciaron en 1894, precisamente durante la efervescencia de fin de siglo, consecuencia de la cual fue también el proceso iniciado

E OME 21. Problema 8. Solucion

Las condiciones para que la matriz a d g

b e hc f i

(1)

sea de suma constante son

a + b + c = a + d + g = a + e + i = d + e + f = b + e + h = c + f + i = g +h + i = c + e + g.

Despejando convenientemente se obtiene

a = − i + 2e

b =2e − h

c =h + i − e

d =2i − 2e + h

f = − 2i + 4e − h

g = − i + 3e − h.

(2)

Para que la matriz (1) sea de producto constante se debe cumplir

abc = adg = aei = def = beh = cfi = ghi = ceg. (3)

Caso I.– Si e = 0 entonces (2) se reduce a

a = − i

b = − h

c =h + i

d =2i + h

f = − 2i − h

g = − i − h.

(4)

Sustituyendo (4) en (3) resulta el sistema de ecuaciones

ih(h + i) = i(h + i)(2i + h) = (h + i)(2i + h)i = −hi(i + h) = 0

cuyas soluciones (i, h) = (0, 0) son las mismas que las de la ecuacion i(h + i) = 0. Cuandoi = 0 resultan las matrices de suma y de producto constante

0 h −h−h 0 hh −h 0

, h ∈ R (5)

Page 557: COMPENDIUM OME - ToomatesEotvos de Hungría, que se iniciaron en 1894, precisamente durante la efervescencia de fin de siglo, consecuencia de la cual fue también el proceso iniciado

y cuando i = 0, i = −h, las matrices h −h 0

−h 0 h0 h −h

, h = 0. (6)

Caso II.– Si e = 0, la ecuacion aei = beh implica, utilizando (2),

(h − i)(h + i − 2e) = 0.

• Cuando h = i, la ecuacion aei = def implica, utilizando (2), (i − e)2 = 0, es decir, i = e.Y la matriz (1) que entonces resulta,

h h h

h h hh h h

, h = 0, (7)

es de suma y de producto constante.

• Si e =h + i

2, el sistema (3) es equivalente a la ecuacion (e = 0)

(h − i)2 = 0,

lo que remite al caso anterior.En resumen, las matrices de suma y producto constante deben ser de una de las formas(5), (6) o (7).

Page 558: COMPENDIUM OME - ToomatesEotvos de Hungría, que se iniciaron en 1894, precisamente durante la efervescencia de fin de siglo, consecuencia de la cual fue también el proceso iniciado

E OME 22. Problema 1. Solucion

Consideramos la funcion f : R → R2 dada por f(x) =

([x], x

)y observamos que la

distancia definida en el enunciado entre dos puntos de R, es la distancia euclıdea entre susimagenes por f en el plano. De hecho, el conjunto imagen f(R) queda descompuesto enun “peine” de segmentos verticales de altura 1 situados en los puntos del eje de abscisasde coordenadas enteras. El punto 3/2 ∈ R se transforma en el (1, 1/2) del plano. Lospuntos pedidos son los que estan dentro de la circunferencia de centro (1, 1/2) y radio202/100. Es evidente que los segmentos verticales sobre los puntos de abscisas 0, 1 y 2estan completamente contenidos en la circunferencia. En cambio, los que estan sobre lasabscisas −1 y 3, solo estan contenidos parcialmente.

( ) [ ) ( )2

ry

−2 −1 0 1 2 3 4

Para encontrar estos segmentos debemos resolver el triangulo rectangulo de la figura, dehipotenusa r = 202/100 = 2 + 2/100. Obtenemos

y =√

20150

<12.

El conjunto buscado es

(−1

2−

√20150

,−12

+√

20150

)∪

[0, 3

)∪

(72−

√20150

,72

+√

20150

).

Page 559: COMPENDIUM OME - ToomatesEotvos de Hungría, que se iniciaron en 1894, precisamente durante la efervescencia de fin de siglo, consecuencia de la cual fue también el proceso iniciado

E OME 22. Problema 2. Solucion

a) Si tenemos s = nd y s′ = n′d, con n, n′ numeros naturales, entonces evidentemente,s′ − s = (n′ − n) d.

b) Llamamos d a la diagonal AC del pentagono y s al lado. El paralelismo entre diagonalesy lados opuestos produce cinco rombos interiores al pentagono, formados, cada uno, pordos lados consecutivos y las diagonales respectivamente opuestas. Una diagonal cualquiera,digamos la AC, queda pues partida en tres segmentos con longitudes tal como aparecenen la figura. El segmento central, de longitud 2s − d, es el lado de un nuevo pentagonoregular interior al dado.

d−s d−s2s−d

s

A C

B

P

E D

La desigualdad triangular aplicada al triangulo APB nos dice (d− s) + (d− s) > s o bien2d > 3s , de donde 2s − d < 1/2 s.Si d y s fuesen conmensurables, es decir, existiera un segmento u que dividiese a ambos,tambien dividiria a 2s − d, que es el lado de un nuevo pentagono de lado menor que lamitad del primero. Iterando el proceso las veces que haga falta, resultarıa al final queobtendrıamos un pentagono de lado a la vez menor que u y multiplo de u, lo que esabsurdo. Por tanto s y d deben ser inconmensurables.Observacion:La semejanza de los triangulos APB y EPC nos da la relacion

d

s=

s

d − s

que da lugar a la ecuacion(d/s)2 − (d/s) − 1 = 0

de solucion positivad

s=

1 +√

52

, numero irracional.

Page 560: COMPENDIUM OME - ToomatesEotvos de Hungría, que se iniciaron en 1894, precisamente durante la efervescencia de fin de siglo, consecuencia de la cual fue también el proceso iniciado

E OME 22. Problema 3. Solucion

Primera solucionConsideremos la ecuacion

5α + 3 = 2β.

Calculemos los posibles valores de α dando valores bajos de β. Los valores β = 0, 1 no dansolucion. Para β = 2 nos sale α = 0. Los valores β = 4, 5, 6, 8, 9 tampoco dan solucion, ypara β = 7 sale α = 3.Nos queda por demostrar que no hay solucion para β ≥ 10. Como 210 = 1024, calculandomodulo 1024 nos queda 5α = −3 = 1021, con α = 163. pero las potencias de 5 modulo1024 tienen periodicidad 256, de forma que α = 163 + k256.En Z tendremos

5163 5k256 + 3 = 2β.

Si calculamos ahora modulo 257, tendremos 256i = 1 y 5163+3 = 2β , o sea, 246 = 2β. Perolas potencias de 2 modulo 257 tienen periodicidad 16 ya que 28 = 256 = −1. Calculandolas potencias de 2 nos sale 20 = 1, 21 = 2, 22 = 4, 23 = 8, 24 = 16, 25 = 32, 26 = 64,27 = 128, 28 = −1, 29 = −2, 210 = −4, 211 = −8, 212 = −16, 213 = −32, 214 = −64,215 = −128 y ninguno de ellos es 246 = −11.Las unicas soluciones posibles son pues

51 + 3 = 23 y 53 + 3 = 27.

Segunda solucionLas unicas soluciones con m ≤ 7 son 50 + 3 = 22, 51 + 3 = 23 y 53 + 3 = 27. Si hay otrasolucion, sera m > 7 y entonces n cumplira la condicion

5n + 3 ≡ 0 modulo 28.

De donde resulta (a partir de una tabla de restos potenciales de 5 modulo 28) que n =35 + 64k, con k = 0, 1, 2, . . . .Entonces,

5n + 3 = 535+64k + 3 ≡ 14 · 16k + 3 modulo 257,

pues se puede comprobar que 535 ≡ 14 y 564 ≡ 16 modulo 257.Ahora, se tiene 16k ≡ ±1,±16 modulo 257, luego

5n + 3 ≡ 17, −30, −11 o 36 modulo 257,

mientras que

2m ≡ ±1, ±2, ±4, ±8, ±16, ±32, ±64 o ± 128 modulo 257,

luego la coincidencia de valores entre 5n + 3 y 2m cuando m > 7 es imposible.

Page 561: COMPENDIUM OME - ToomatesEotvos de Hungría, que se iniciaron en 1894, precisamente durante la efervescencia de fin de siglo, consecuencia de la cual fue también el proceso iniciado

E OME 22. Problema 4. Solucion

Si f es una funcion real de variable real tal que f ′ > 0 y f ′′ > 0 estonces es una funcionconvexa, es decir, la grafica esta por debajo de cualquier cuerda, tal como esta representadoen la figura.

a bλa+(1−λ)b

f(b)

f(a)

f(λa+(1−λ)b)

λf(a)+(1−λ)f(b)

En el caso que λ = 12 , tendremos el resultado del enunciado.

Para demostrar la propiedad, consideremos la funcion

φ(x) = f(a) +f(b) − f(a)

b − a(x − a) − f(x)

cuyas derivadas son

φ′(x) =f(b) − f(a)

b − a− f ′(x) y φ′′(x) = −f ′′(x).

Es evidente que φ(a) = φ(b) = 0 y que φ′(ξ) = 0 para un cierto punto ξ ∈ (a, b), porel teorema de Rolle. Este mismo teorema nos dice que el punto ξ es unico. Como queφ′′(x) = −f ′′(x) < 0, resulta que φ′ es decreciente con un unico cero ξ. De ahı que φ(x)solamente pueda anularse en a y b y φ(x) > 0 para todo x ∈ (a, b).

a bξ a

b

ξ

φ(x) φ′(x)

Page 562: COMPENDIUM OME - ToomatesEotvos de Hungría, que se iniciaron en 1894, precisamente durante la efervescencia de fin de siglo, consecuencia de la cual fue también el proceso iniciado

E OME 22. Problema 5. Solucion

Primera solucion

Los puntos (0, b) y (0,−b) son dos puntos de coordenadas racionales de Γ; para hallar untercer punto hacemos la siguinte identificacion.

x3 + bx + b2 = y2 =(b +

x

2

)2

= b2 + bx +x2

4

de dondex =

14, y = ±

(b +

18

).

Se han obtenido los cuatro puntos

(0, b), (0,−b),(1

4, b +

18

),

(14,−b − 1

8

)

Puesto que elegidos tres de ellos no estan alineados, se obtiene un triangulo solucion. Portanto hemos obtenido 4 soluciones posibles.

Segunda solucion

Una vez hallado el punto (0, b), trazamos una recta variable por el de pendiente t, y deecuacion y = tx + b. Las intersecciones con la cubica, despues de eliminar la x = 0 quecorresponde al punto (0, b), vendran dadas por la ecuacion de segundo grado

x2 − t2x + b − 2bt = 0,

que tiene por soluciones

x =t2 ±

√t4 − 4(b − 2bt)

2.

Los posibles valores racionales de t que nos den un valor racional de x corresponderan apuntos sobre Γ de coordenadas racionales. Por ejemplo, es evidente que para t = 1/2 salenlos puntos obtenidos antes.

Page 563: COMPENDIUM OME - ToomatesEotvos de Hungría, que se iniciaron en 1894, precisamente durante la efervescencia de fin de siglo, consecuencia de la cual fue también el proceso iniciado

E OME 22. Problema 6. Solucion

Consideremos las 15-raıces complejas de la unidad, de las que solo una de ellas es real,z = 1, y las demas son 7 parejas complejas conjugadas. Tenemos

z15 − 1 = (z − 1)7∏1

(z − εk)(z − εk), con εk = ei 2kπ15

o bienz15 − 1z − 1

= 1 + z + z2 + · · · + z14 =7∏1

(z2 − 2z cos

2kπ

15+ 1

)y dando a z el valor −1

1 =7∏1

(2 + 2 cos

2kπ

15)

= 277∏1

(1 + cos

2kπ

15)

= 2147∏1

cos2kπ

15,

donde se ha utilizado la formula 1 + cos 2α = 2 cos2 α2 . De la ultima igualdad sale

7∏1

cos2kπ

15=

1214

de donde7∏1

coskπ

15= ± 1

27.

El signo debe ser positivo, ya que los 7 angulos son del primer cuadrante y los cosenos sonpositivos. En el producto del enunciado hay 14 angulos, 7 del primer cuadrante, con cosenopositivo, y 7 del segundo cuadrante, con cosenos negativos y en valor absoluto iguales alos 7 primeros ya que son suplementarios de ellos. Por lo tanto el producto de todos loscosenos sera negativo.

14∏1

coskπ

15=

7∏1

coskπ

15

14∏8

coskπ

15= − 1

27

127

= − 1214

Page 564: COMPENDIUM OME - ToomatesEotvos de Hungría, que se iniciaron en 1894, precisamente durante la efervescencia de fin de siglo, consecuencia de la cual fue también el proceso iniciado

E OME 23. Problema 1. Solucion

Si desde un punto P del plano se trazan perpendiculares a los tres lados de un trianguloequilatero, se obtienen 3 segmentos cuya suma (con los signos que convenga) es igual a laaltura del triangulo. La demostracion se obtiene simplemente calculando las areas de lostriangulos determinados por P y los vertices del triangulo equilatero, sumando o restandosegun los casos.

d1

d3

d2

d1

d2d3

d3

d2d1

h=d1+d2+d3 h=d1−d2+d3 h=d3−d2−d1

Caso I Caso II Caso III

Sean ahora a < b < c los tres lados dados. Dibujamos tres circunferencias concentricascon centro O y radios a, b y c. Trazamos una tangente cualquiera a la circunferencia deradio a. Trazamos tangentes paralelas a la circunferencia de radio b que formen angulo de60 con la primera. Sean s1 y s2. Analogamente, trazamos tangentes paralelas r1 y r2 queformen angulo de 60 con t y con s1 y s2.

A

D E F

Oa

b

c

G

B H C

t

r1

r2

s2

s1

Page 565: COMPENDIUM OME - ToomatesEotvos de Hungría, que se iniciaron en 1894, precisamente durante la efervescencia de fin de siglo, consecuencia de la cual fue también el proceso iniciado

De esta forma obtenemos los triangulos equilateros ABC, ADE, EFG y GCH que cumplenlas condiciones del enunciado, y ademas, son los unicos que tienen area diferente. Los girosalrededor de O producen figuras equivalentes. Respecto de cada uno de los triangulosequilateros, el centro O puede estar situado en el interior (Caso I de la primera figura) yen este caso h = a + b + c, o puede estar situado como en el Caso II, y en este caso puedeser h = a + b − c, h = b + c − a o h = c + a − b. El Caso III no puede darse ya que serıah = a− b− c < 0, o bien h = b− c− a < 0 o bien h = c− a− b < 0 por a, b, c los lados deun triangulo.En los cuatro casos que nos quedan, y llamando p al semiperımetro, a + b + c = 2p,a + b − c = 2(p − c), b + c − a = 2(p − a), c + a − b = 2(p − b), de donde las areas

S(ABC) =4√

3p2

3,

S(ADE) =4√

3(p − c)2

3,

S(EFG) =4√

3(p − b)2

3,

S(CGH) =4√

3(p − a)2

3.

Page 566: COMPENDIUM OME - ToomatesEotvos de Hungría, que se iniciaron en 1894, precisamente durante la efervescencia de fin de siglo, consecuencia de la cual fue también el proceso iniciado

E OME 23. Problema 2. Solucion

Pongamos

u = (1, 2, . . . , n) y v =((

n

1

),

(n

2

), . . . ,

(n

n

)).

Sabemos que |u · v| ≤ ‖u‖ · ‖v‖. En este caso

‖u‖ =√

12 + 22 + . . . + n2 =

√(2n + 1)n(n + 1)

6=

√(2n3 + 3n2 + n)

6

y

‖v‖ =

√(n

1

)+

(n

2

)+ . . . +

(n

n

)=

√2n − 1

luego

1 ·

√(n

1

)+ 2 ·

√(n

2

)+ . . . + n ·

√(n

n

)≤

≤√

2n − 1

√(2n3 + 3n2 + n)

6=

=

√2n−1

(2 − 1

2n−1

)· 12

(2n3 + 3n2 + n)3

=

=

√2n−1

(1 − 1

2n

)· (2n3 + 3n2 + n)

3.

Para tener el resultado solo queda probar que

(1 − 1

2n

)· (2n3 + 3n2 + n)

3< n3

lo que es equivalente a

n3

3− n2 − n

3+

12n

(2n3

3+ n2 +

n

3

)> 0

Para n = 1, 2, 3 se comprueba directamente y para n ≥ 4 tenemos quen3

3− n2 − n

3> 0

ya que la funcion y =x3

3− x2 − x

3es positiva y creciente para x > 4.

Page 567: COMPENDIUM OME - ToomatesEotvos de Hungría, que se iniciaron en 1894, precisamente durante la efervescencia de fin de siglo, consecuencia de la cual fue también el proceso iniciado

E OME 23. Problema 3. Solucion

En el triangulo ABC tenemos n = 1, s = 3, v = 3. Tomando un punto P en el interior deltriangulo unido con los vertices resultan 3 triangulos; se tiene ahora n = 3, s = 6, v = 4.

A B

C

P

De una forma general, tomemos un nuevo punto Q en el interior de los triangulos formados.El triangulo APC se sustituye por los triangulos AQP , AQC, y CQP ; esto es, al aumentaren una unidad el numero de vertices, el numero de triangulos lo hace en 2 y el de aristas en3. Despues de elegir h puntos en el interior del triangulo, el numero de triangulos se habraincrementado en 2h, y en 3h los lados; luego en total se tendra n = 1 + 2h, s = 3 + 3h,v = 3 + h, y se cumplira n + v = s + 1, formula parecida a la de Euler para los poliedros.

A B

C

P

Q

Es inmediato comprobar que como 1 + 2h es siempre impar, no existe ninguna descom-posicion en numero par de triangulos.Por otra parte, como cada lado, salvo los del triangulo original, es comun a dos triangulos,

se tiene que (s − 3) · 2 + 3 = 3n de donde s =3(n + 1)

2y v =

n + 52

.

Page 568: COMPENDIUM OME - ToomatesEotvos de Hungría, que se iniciaron en 1894, precisamente durante la efervescencia de fin de siglo, consecuencia de la cual fue también el proceso iniciado

E OME 23. Problema 4. Solucion

En el primer caso, de la primera ecuacion despejamos y = 1−x, sustituimos en la inecuaciony hacemos operaciones, lo que nos proporciona (a − b)2x2 ≤ (a − b)2x, cuya solucion es elintervalo [0, 1] y por lo tanto y recorre el mismo intervalo. Geometricamente es el segmentocuyos extremos tienen por coordenadas los puntos (0, 1) y (1, 0).En el segundo caso, trabajamos de la misma manera y con un poco mas de esfuerzo tenemosque:

x(x − 1)(a − b)2((a − b)2x2 + (a2 + 2ab − 3b2)x + (a2 + 2ab + 3b2

)≤ 0

y puesto que (a− b)2x2 +(a2 +2ab− 3b2)x+(a2 +2ab+3b2) es siempre positivo, tenemoslas mismas soluciones que antes: x ∈ [0, 1], e y ∈ [0, 1].

Page 569: COMPENDIUM OME - ToomatesEotvos de Hungría, que se iniciaron en 1894, precisamente durante la efervescencia de fin de siglo, consecuencia de la cual fue también el proceso iniciado

E OME 23. Problema 5. Solucion

A

D F

G

E

B C

En el triangulo de la figura se dibuja DFla paralela a BC, el triangulo BGC esequilatero, luego GC = BC, el trianguloBCE es isosceles, puesto que tiene dosangulos de 50, consecuentemente BC =CE. Luego CE = GC y el triangulo CGEtambien es isosceles;

EGC = GEC =12(180 − 20) = 80

y

FGE = 180 − DGF − EGC = 40

y

GFE = 180 − FBC − FCB = 40.

Por tanto, FGE es isosceles y como DFGes equilatero, los triangulos DEF y DEGson iguales, luego DE⊥GF , esto es DE esla altura del triangulo equilatero DFG; portanto

EDC =12FDC = 30.

Page 570: COMPENDIUM OME - ToomatesEotvos de Hungría, que se iniciaron en 1894, precisamente durante la efervescencia de fin de siglo, consecuencia de la cual fue también el proceso iniciado

E OME 23. Problema 6. Solucion

a) Tenemos Pn(0) = 1 y

Pn

(34

)=

(34

)n+2

− 32

+ 1 =(

34

)n+2

− 12

pero(

34

)n+2es decreciente y para n = 1 es

(34

)1+2= 27

64 < 12 , de donde resulta que

Pn( 34 ) < 1

2 .Por el teorema de Bolzano sabemos que existe un cn ∈ (0, 3/4) tal que Pn(cn) = 0 y P (x)tiene por lo menos un cero en (0, 1).Este cero debe ser unico. En efecto, la derivada P ′(x) = (n + 2)xn+1 − 2 se anula en ununico punto de (0, 1),

αn =(

2n + 2

) 1n+1

< 1.

Como que Pn(1) = 0 y Pn(cn) = 0, el teorema de Rolle nos asegura que cn < αn < 1 yque no puede existir otro cero de Pn(x) en (0, 1).

b) Como que 0 < cn < 34 , tenemos 0 < cn+2

n <(

34

)n+2 y

limn→∞

cn+2n = 0.

Pero de cn+2n − 2cn + 1 = 0 sale

cn =cn+2n + 1

2

y limn→∞ cn existe y vale 12 .

Page 571: COMPENDIUM OME - ToomatesEotvos de Hungría, que se iniciaron en 1894, precisamente durante la efervescencia de fin de siglo, consecuencia de la cual fue también el proceso iniciado

E OME 24. Problema 1. Solucion

En primer lugar x1 = 1, x1 < x2 ≤ 2 implica que x2 = 2 y x2 − x1 = 1.Sea ahora k cualquier numero natural mayor que 1; dispongamos los 2k numeros

1, 2, 3, . . . , 2k

agrupados en k bloques de la siguiente forma:

1, 1 + k 2, 2 + k . . . k − 1, 2k − 1 k, 2k

como xk+1 ≤ 2k, Los numeros x1, x2, . . . , xk+1 son k + 1 numeros entre 1 y 2k y habrados de ellos perteneciendo al mismo bloque, pues solo hay k bloques. Esos los que verificanque xr − xs = k.

Page 572: COMPENDIUM OME - ToomatesEotvos de Hungría, que se iniciaron en 1894, precisamente durante la efervescencia de fin de siglo, consecuencia de la cual fue también el proceso iniciado

E OME 24. Problema 2. Solucion

1

4

5

3

7

26

Ejemplo de configuracion

Vamos a unir con un segmento dos puntos relacionados. Veamos que en una configuracionde n puntos hay un segmento mas que en una configuracion de n − 1 puntos, lo queproporciona el resultado que se quiere probar junto con el hecho de que en una configuracionde tres puntos no hay ningun segmento dibujado.Supongamos que tenemos una configuracion de n − 1 puntos; anadimos una unidad atodas las marcas y entre dos cualesquiera de esos n−1 puntos colocamos una nuevo puntomarcado con 1; solo falta el segmento que une esos dos puntos elegidos para tener unaconfiguracion de n puntos.Por lo tanto si tenemos n puntos, el numero de pares de puntos relacionados es n − 3.

Page 573: COMPENDIUM OME - ToomatesEotvos de Hungría, que se iniciaron en 1894, precisamente durante la efervescencia de fin de siglo, consecuencia de la cual fue también el proceso iniciado

E OME 24. Problema 3. Solucion

Si a y b son enteros, modulo 41 tenemos las siguientes equivalencias:

25a + 31b ≡ 0 ⇔ (multiplicando por 2)⇔ 50a + 62b ≡ 0 ⇔⇔ 9a + 21b ≡ 0 ⇔

⇔ 3(3a + 7b) ≡ 0 ⇔ (dividiendo por 3)⇔ 3a + 7b ≡ 0 mod 41,

en las que se ha utilizado el hecho que 2 y 3 son primos con 41.

Page 574: COMPENDIUM OME - ToomatesEotvos de Hungría, que se iniciaron en 1894, precisamente durante la efervescencia de fin de siglo, consecuencia de la cual fue también el proceso iniciado

E OME 24. Problema 4. Solucion

Primera solucion (combinatoria)

Llamemos fn a la sucesion de Fibonacci propiamente dicha, es decir, la que cumple fn+1 =fn +fn−1, con f1 = 1 y f2 = 1, y pongamos an = fn+1 para n ≥ 1. La sucesion an cumpletambien la recurrencia an+1 = an + an−1 pero con las condiciones iniciales a1 = 1 ya2 = 2. La ventaja de la sucesion an es que tiene interpretaciones combinatorias masfaciles de establecer. Por ejemplo, an es el numero de maneras de subir una escalera den peldanos, si podemos subir un solo peldano o bien dos peldanos en un solo paso. Debecumplirse an = an−1 + an−2 ya que si el primer paso es de un solo peldano, nos quedarann− 1 por subir, y esto se podra hacer de an−1 maneras distintas; o bien en el primer pasosubimos 2 peldanos, nos quedan n − 2 por subir, y esto se puede hacer de an−2 formasdistintas. Ademas, a1 = 1, ya que una escalera de 1 peldano solo se puede subir de unaunica manera, y a2 = 2 ya que una escalera de 2 peldanos se puede subir de dos manerasdistintas.Supongamos ahora que queremos subir una escalera de 2n+1 peldanos. Pueden suceder 2cosas: o bien paramos en el peldano n y de el subimos hasta arriba los n + 1 peldanos quequedan, y esto se puede hacer de anan+1 maneras distintas; o bien subimos n−1 peldanos,luego un paso de 2 peldanos, y al final subimos otros n, y esto lo podemos hacer de an−1an

maneras, de donde a2n+1 = anan+1 + an−1an.De esta igualdad deducimos f2n+2 = fn+1fn+2 + fnfn+1 = f2

n+1 + 2fnfn+1, es decir

f2n = f2n + 2fnfn−1.

Segunda solucion (algebraica)

Los numeros de Fibonacci cumplen la relacion

fn+m = fn−1fm + fnfm+1.

En efecto, para m = 1 tenemos fn+1 = fn−1f1 + fnf2 = fn−1 + fn, (observese la impor-tancia de las condiciones iniciales en la demostracion de estas recurrencias). Si hacemosinduccion sobre m tendremos fn+m+1 = fn+m+fn+m−1 = fn−1fm+fnfm+1+fn−1fm−1+fnfm = fn−1(fm + fm−1) + fn(fm+1 + fm) = fn−1fm+1 + fnfm+2, y la formula es ciertapor induccion. Haciendo ahora n = m sale

f2n = fn−1fn + fnfn+1 = fn−1fn + fn(fn + fn−1) = f2n + 2fnfn−1.

Page 575: COMPENDIUM OME - ToomatesEotvos de Hungría, que se iniciaron en 1894, precisamente durante la efervescencia de fin de siglo, consecuencia de la cual fue también el proceso iniciado

E OME 24. Problema 5. Solucion

u

v

Consideremos el “embaldosado” del plano con cruces congruentes. Teniendo en cuentalas traslaciones de vectores −→u y −→v como generadores del embaldosado, observamos que elcuadrado de lados −→u y −→v aplicado en cualquier punto del plano es una region fundamental,es decir un motivo mınimo que por sucesivas traslaciones de vectores ±n−→u ±m−→v engendratambien el diseno. Si ponemos el cuadrado de modo que corte a 4 cruces, tendremos unaparticion de la cruz, de modo que las partes puedan formar el cuadrado.Hay pues una infinidad de soluciones para el problema, localizando el vertice del cuadradode modo que corte solamente a cuatro cruces. La figura de la izquierda muestra la divisiondel enunciado, y la de la derecha una division en una posicion general. La figura siguientepresenta la division de la cruz en 4 partes iguales y la forma en la que dichas partescomponen un cuadrado.

Page 576: COMPENDIUM OME - ToomatesEotvos de Hungría, que se iniciaron en 1894, precisamente durante la efervescencia de fin de siglo, consecuencia de la cual fue también el proceso iniciado

E OME 24. Problema 6. Solucion

Escribimos la expresion

y2 = x4 − 22x3 + 43x2 + 858x + t2 + 10452(t + 39)

en la siguiente forma:

y2 = (x2 − 11x − 5226)(x2 − 11x + 5148) + (t + 5226)2

La expresion x2 − 11x − 5226 se anula para x = −67 y para x = 78, lo que da solucionesenteras y = ±(t + 5226).

Page 577: COMPENDIUM OME - ToomatesEotvos de Hungría, que se iniciaron en 1894, precisamente durante la efervescencia de fin de siglo, consecuencia de la cual fue también el proceso iniciado

E OME 25. Problema 1. Solucion

Si n = 1, es p1 = 1, si n = 2, es p2 =12

+12· 12; si n = 3, tenemos p3 =

13

+23· 13

+23· 23· 13;

En general

pn =1n

+n − 1

n· 1n

+(

n − 1n

)2

· 1n

+ . . . +(

n − 1n

)n−1

· 1n

=

=1n

[1 +

n − 1n

+(

n − 1n

)2

+ . . . +(

n − 1n

)n−1]

= 1 −(

n − 1n

)n

limn→∞

pn = 1 − e−1

La sucesion 1−(

n − 1n

)n

es una sucesion decreciente y por lo tanto la mayor de las cotas

inferiores es el lımite.

Page 578: COMPENDIUM OME - ToomatesEotvos de Hungría, que se iniciaron en 1894, precisamente durante la efervescencia de fin de siglo, consecuencia de la cual fue también el proceso iniciado

E OME 25. Problema 2. Solucion

A B

C

B′

A′

C′

C1

B1

A1

Para resolver el problema utilizaremos varias veces que si dos triangulos tienen la mismaaltura, entonces la relacion de areas es igual a la de las bases; o bien que si tienen la mismabase, la relacion de areas es la de las alturas.Para empezar, llamaremos s al area del triangulo pequeno AB′C1. El area del trianguloB′C1C sera ks, y el del AC1C la suma s + ks.Los dos triangulos AC1C y AC1B comparten la base AC1, luego el AC1B tiene areak(s + ks) = ks + k2s. Simplemente sumando, nos queda que el triangulo ABB′ tiene areas + ks + k2s.

×k

×k

×k

Los triangulos BB′A y BB′C comparten altura, luego el BB′C tiene area ks(1 + k + k2).Y el triangulo total ABC tendra area S = s(1 + k)(1 + k + k2).El triangulo central A1B1C1 tendra un area S′ que se puede obtener restando al areatotal S la de los tres triangulos ABC1, BCA1 y CAB1, las tres iguales a s(k + k2). Esdecir,

S′ = S − 3s(k + k2) = s((1 + k)(1 + k + k2) − 3(k + k2

)= s(1 − k)2(1 + k)

y por lo tantoS′

S=

(1 − k)2

1 + k + k2.

Page 579: COMPENDIUM OME - ToomatesEotvos de Hungría, que se iniciaron en 1894, precisamente durante la efervescencia de fin de siglo, consecuencia de la cual fue también el proceso iniciado

E OME 25. Problema 3. Solucion

Primera solucion

Veamos la primera desigualdad(12

34

56

. . .99100

)2

=12

12

34

34

. . .99100

99100

>

>12

12

34

34

. . . 99100

98 99

>12

12

34

34

. . .97 98

98100

>12

12

34

34

. . .96 97

97100

> · · ·

· · · >12

1100

=⇒ 110

√2

<1 · 3 · 5 . . .992 · 4 · 6 . . .100

Y ahora veamos la otra(12

34

56

. . .99100

)2

=12

12

34

34

. . .99100

99100

<

<1 2

2 3

3 4

4 5 . . .

99100

< · · · <1

100=⇒ 1 · 3 · 5 . . . 99

2 · 4 · 6 . . . 100<

110

Segunda solucion

Tomada del libro The USSR Olympiad problem book, de D.O. Shklarsky, N.N. Chenzovy I.M. Yaglom, Ed. W.H. Freeman and company . San Francisco and London 1962.Pongamos:

A =12· 34· 56· . . . · 99

100; B =

23· 45· 67· . . . · 98

99.

Como23

>12,

45

>34,

67

>56, . . . ,

9899

>9798

se tiene B > A o, de forma equivalente, A2 < A · B. Pero,

A · B =12· 23· 34· . . . · 98

99· 99100

=1

100⇒ A2 < A · B =

1100

⇒ A <110

y tenemos probada la desigualdad de la derecha.Para probar la de la izquierda, teniendo en cuenta que

2 · A =34· 56· 78· . . . · 99

100.

y que23

<34,

45

<56,

67

<78, . . . ,

9899

<99100

resulta2 · A > B ⇔ 2A2 > A · B =

1100

de donde A >1

200=

110

√2.

Page 580: COMPENDIUM OME - ToomatesEotvos de Hungría, que se iniciaron en 1894, precisamente durante la efervescencia de fin de siglo, consecuencia de la cual fue también el proceso iniciado

E OME 25. Problema 4. Solucion

Comprobamos que

1989 = 9 · 221 = 9(102 + 112) = 9(142 + 52).

Por lo tanto

19892n+1 = 19892n1989 = ((1989)n)2 · 9(102 + 112) = ((1989)n)2 · 9(142 + 52)

y tambien

19892n+2 = 19892n19892 = ((1989)n)2 · (1952 + 1042) = ((1989)n)2 · (2042 + 852).

Page 581: COMPENDIUM OME - ToomatesEotvos de Hungría, que se iniciaron en 1894, precisamente durante la efervescencia de fin de siglo, consecuencia de la cual fue también el proceso iniciado

E OME 25. Problema 5. Solucion

Si a, b, c, d ∈ Z, entonces14 = (a+b

√−13)(c+d

√−13) = (ac−13bd)+(bc+ad)

√−13; y hay que resolver el sistema

ac − 13bd = 14bc + ad = 0

Si b = 0 puede ser a = 14, 7, 2, 1 y entonces c = 1, 7, 14, 2, y d = 0y las soluciones son (14 + 0 ·

√−13)(1 + 0 ·

√−13) y (7 + 0 ·

√−13)(2 + 0 ·

√−13)

Si b = 0, podemos poner c = −ad

by tambien, sustituyendo esta igualdad en la segunda

ecuacion−(a2 + 13b2) d = 14.

Entonces el valor de b solamente puede ser 1 o −1 y las soluciones son (1+√−13)(1−

√−13)

y (−1 +√−13)(−1 −

√−13).

Page 582: COMPENDIUM OME - ToomatesEotvos de Hungría, que se iniciaron en 1894, precisamente durante la efervescencia de fin de siglo, consecuencia de la cual fue también el proceso iniciado

E OME 25. Problema 6. Solucion

Pondremos a = tanα y b = tanβ, y de este modo tendremos

tan(α − β) =tanα − tan β

1 + tanα tanβ=

a − b

1 + ab, y | tan(α − β)| <

1√3

= tanπ

6.

Esto nos dice que dados siete numeros reales cualesquiera, existen 7 angulos tales quesu tangente es el numero dado. Como la funcion tangente es periodica de periodo π, es

equivalente a elegirlos en un intervalo de amplitud π, por ejemplo[−π

2,π

2

]; en dicho

intervalo, la funcion tangente es creciente, y la maxima distancia entre ellos esπ

7.

π

7<

π

6=⇒ tan

π

7< tan

π

6=

1√3.

Si tenemos 6 puntos se puede obtener la igualdad. Basta para ello elegir los numeros

−5π

12,−π

4,− π

12,

π

12,π

4,5π

12.

La maxima distancia entre ellos esπ

6y tan

π

6=

1√3

.

Page 583: COMPENDIUM OME - ToomatesEotvos de Hungría, que se iniciaron en 1894, precisamente durante la efervescencia de fin de siglo, consecuencia de la cual fue también el proceso iniciado

E OME 26. Problema 1. Solucion

Tenemos

B =√

x +√

y

√1 + x + 2

√x =

=√

x +√

y

√(1 +

√x)2 =

=√

x +√

y(1 +

√x)

==

√x +

√y +

√xy = A.

En lo que se refiere a la segunda parte, transformando el segundo radical de M mediantelo que acabamos de hacer, obtenemos

M =√

5 +√

22 +

√3 + 5 −

√22 + 2

√3√

5 −√

22

=√

5 +√

22 +√

3 +√

5 −√

22,

y si queremos compararlo con L resulta que debemos comparar√10 + 2

√3 con

√5 +

√22 +

√5 −

√22;

al elevar al cuadrado la suma de radicales se obtiene en efecto 10 + 2√

3, luego L = M .Observaciones.La primera parte del problema se puede resolver elevando al cuadrado A y B y viendo queA2 = B2.Otra solucion se puede obtener mediante la siguiente (y monstruosa) identidad entre radi-cales, publicada en 1976 en la revista canadiense CRUX MATHEMATICORUM:

√z +

√x + 2y =

√x

2+ 2

√(x2 − 4y)

16+

+

√√√√z +

x

2− 2

√(x2 − 4y)

16+ 2

√zx

2− 2z

√(x2 − 4y)

16,

de la que pueden obtenerse verdaderas preciosidades dandole a x, y, z valores convenientes.

Page 584: COMPENDIUM OME - ToomatesEotvos de Hungría, que se iniciaron en 1894, precisamente durante la efervescencia de fin de siglo, consecuencia de la cual fue también el proceso iniciado

E OME 26. Problema 2. Solucion

La respuesta es sı. Consideremos puntos A, B, C, D, E, F, G de manera que ABFD es unrombo, de lados AB, AD, BD, BF , y FD de longitud 1; y AEGC es tambien un rombo,de lados AE,AC,EC,EG y GC de longitud 1. Ademas unimos F con G por una arista delongitud 1.Razonaremos por reduccion al absurdo y supondremos que la propiedad no se cumple.Entonces A sera de color a, B del b, D del c; entonces F debe estar pintado de color a.Analogamente se prueba que G esta pintado de color a. Pero esto es contradictorio, puesF y G estan a distancia 1.

A

B

C

E

D

F G

1

(Solucion tomada del libro de Ioan Tomescu Problems in Combinatory and graph theory,ed. Wiley; el problema es original de Hadwiger, publicado en 1948 en la revista suizaElemente der Mathematik.)

Page 585: COMPENDIUM OME - ToomatesEotvos de Hungría, que se iniciaron en 1894, precisamente durante la efervescencia de fin de siglo, consecuencia de la cual fue también el proceso iniciado

E OME 26. Problema 3. Solucion

Como 0 < 4 −√

11 < 1, resulta que 0 <(4 −

√11

)n< 1.

Consideremos la suma

E =(4 +

√11

)n

+(4 −

√11

)n

;

al desarrollar el segundo miembro mediante el binomio de Newton se observa que se obtieneun numero par, 2k, y al haberse obtenido sumando a

(4 +

√11

)nun numero menor que 1,

se trata del primer entero mayor que(4 +

√11

)n. Luego el entero inmediatamente anterior

a este, que es justamente la parte entera de(4 +

√11

)n, es impar.

Observacion. Hay un problema similar en American Mathematical Monthly, 1909, p.123.

Page 586: COMPENDIUM OME - ToomatesEotvos de Hungría, que se iniciaron en 1894, precisamente durante la efervescencia de fin de siglo, consecuencia de la cual fue también el proceso iniciado

E OME 26. Problema 4. Solucion

Poniendo

x =a + 1

2+

a + 36

√4a + 3

3,

y =a + 1

2− a + 3

6

√4a + 3

3

se obtiene que

x + y = a + 1, xy = −a3

33.

Entonces, llamando z a la suma considerada, se tiene

z = 3√

x + 3√

y,

y elevando ambos miembros al cubo resulta

z3 = x + y + 3 3√

xy(

3√

x + 3√

y)

= a + 1 − az,

es decir,

z3 + az − (a + 1) = 0

que admite la raiz z = 1, con lo cual la ecuacion anterior se escribe como

(z − 1)(z2 + z + a + 1

)= 0.

Pero el discriminante de z2+z+a+1 es − (3 + 4a) < 0, ası que ese polinomio es irreduciblesobre los numeros reales, con lo cual z = 1, es decir,

3

√a + 1

2+

a + 36

√4a + 3

3+

3

√a + 1

2− a + 3

6

√4a + 3

3= 1.

Page 587: COMPENDIUM OME - ToomatesEotvos de Hungría, que se iniciaron en 1894, precisamente durante la efervescencia de fin de siglo, consecuencia de la cual fue también el proceso iniciado

E OME 26. Problema 5. Solucion

Si ponemos a = BC, b = CA, c = AB, se tiene :

AC′ = cp, BC′ = c(1 − p)BA′ = ap, CA′ = a(1 − p)CB′ = bp, AB′ = b(1 − p).

Entonces

SA′BC′ =12c (1 − p) ap sen B = p (1 − p)S,

y lo mismo ocurre con las areas de los triangulos A′B′C y B′AC′, ya que todas son iguales.Entonces

1)SA′B′C′ = S − 3p(1 − p)S = S [1 − 3p(1 − p)] .

2) Esta area sera mınima cuando p = 1/2, porque se trata de hacer maximo el producto delos dos factores positivos de suma es constante p + 1 − p = 1. Esto ocurre cuando ambosson iguales.

3) Si C′B′′ es paralela a BC (con B′′ sobre AC), los triangulos AC′B′′ y ABC sonsemejantes, y entonces AB′′ = pb, C′B′′ = pa.Si C′A′′ es paralela a AC (con A′′ sobre BC), entonces los triangulos C′A′′B y ABC sonsemejantes, y BA′′ = (1 − p)a.Sea P el punto de interseccion de B′′A′ y C′A′′ (estas rectas no pueden ser paralelas,porque son respectivamente paralelas a dos de los lados del triangulo original); entonceslos triangulos PC′B′′ y PA′A′′ son paralelos, y se tiene

PA′′

PC′ =A′A′′

C′B′′ =a (1 − 2p)

ap.

Sea AM la mediana correspondiente al vertice A del triangulo ABC. En los lados deltriangulo BC′A′′ consideramos los puntos A, M y P . Se cumple

AC′

AB· MB

MA′′ ·PA′′

PC′ =pc

c· a/2a

(12 − p

) · a (1 − 2p)pa

= 1,

luego por el recıproco del teorema de Menelao, esos puntos estan alineados. Por lo tanto,el lugar geometrico pedido es la mediana AM.

Page 588: COMPENDIUM OME - ToomatesEotvos de Hungría, que se iniciaron en 1894, precisamente durante la efervescencia de fin de siglo, consecuencia de la cual fue también el proceso iniciado

E OME 26. Problema 6. Solucion

Sea A0 uno cualquiera de los n puntos. Los demas seran representados por A1, A2, etc.Consideremos el triangulo A0A1A2. Podemos suponer que A1A2 es el mayor lado (ya queuno de los otros dos es el mas pequeno); eso significa que el angulo del triangulo con verticeen A0 es el mayor. Como no hay dos angulos iguales, este angulo debe ser mayor que 60.En efecto, si se cumplen las relaciones angulares A = A, A > B, A > C, sumandolas resulta3A > 180.Si A3 es otro punto del conjunto, considerando el triangulo A0A2A3 y razonando de lamisma manera llegaremos a la conclusion de que el angulo en A0 en este triangulo esmayor que 60.Es claro, entonces, que solo se pueden considerar 5 triangulos con angulos en el vertice A0

mayores que 60, pues 6 × 60 = 360.Ası, A0 estara como maximo unido a los puntos A1, A2, A3, A4 y A5.

Page 589: COMPENDIUM OME - ToomatesEotvos de Hungría, que se iniciaron en 1894, precisamente durante la efervescencia de fin de siglo, consecuencia de la cual fue también el proceso iniciado

E OME 27. Problema 1. Solucion

Supongamos que los puntos A, B y C son tales que AC y AB forman un angulo de 45,y AB y AC son enteros; entonces BC =

√v, con v entero. Segun el teorema del coseno,

√2

2= cos 45 =

AB2 + AC2 − v

2 · AB · AC∈ Q,

lo cual es una contradiccion.En el espacio, tomando vectores equipolentes con origen en el de coordenadas O, estarıamosen el caso anterior, pues O, con los extremos de los vectores, A y B , estan en un plano.

Page 590: COMPENDIUM OME - ToomatesEotvos de Hungría, que se iniciaron en 1894, precisamente durante la efervescencia de fin de siglo, consecuencia de la cual fue también el proceso iniciado

E OME 27. Problema 2. Solucion

Multiplicando la primera fila por λ, la segunda por 1 − λ, y sumando, resulta para unelemento cualquiera la relacion

λ(a + bk

)+ (1 − λ)

(a2 + bk

)= λa + a2 − λa2 + bk.

Por lo tanto, para que este sea un elemento cualquiera de la matriz, tiene que ser

λa + a2 − λa2 = a [λ + a (1 − λ)] = ai,

es decir

λ (1 − a) = ai−1 − a,

de donde

λ =a

(ai−2 − 1

)

1 − a= −a

(1 + a + a2 + ... + ai−3

).

Entonces si n = 3, basta tomar λ = −a.Si n > 3, tenemos λ = −a

(1 + a + a2 + ... + ai−3

), desde i = 3 hasta i = n.

El conjunto S esta formado, entonces, por las dos primeras filas de la matriz.

Page 591: COMPENDIUM OME - ToomatesEotvos de Hungría, que se iniciaron en 1894, precisamente durante la efervescencia de fin de siglo, consecuencia de la cual fue también el proceso iniciado

E OME 27. Problema 3. Solucion

Identificando coeficientes entre x3 + px2 + qx + r y (x − x1)(x − x2)(x − x3) se obtienenlas llamadas relaciones de Cardano-Vieta entre las raıces y los coeficientes del polinomio(los primeros miembros son las funciones simetricas elementales de las raıces)

−x1x2x3 = r,

x1x2 + x2x3 + x3x1 = q

− (x1 + x2 + x3) = p.

Ademas, para que los numeros positivos xi puedan ser las longitudes de los lados de untriangulo se tienen que cumplir las desigualdades triangulares

x1 + x2 − x3 > 0,

x2 + x3 − x1 > 0,

x3 + x1 − x2 > 0;

que pueden englobarse en una sola equivalente, multiplicandolas :

(x1 + x2 − x3)(x2 + x3 − x1)(x3 + x1 − x2) > 0.

Es evidente que si las tres desigualdades primeras son positivas, su producto tambien loes. Si el producto es positivo, puede haber o ninguno o dos factores negativos. Pero de lastres desigualdades anteriores, solo una puede ser negativa. El problema quedara resueltocuando consigamos expresar el primer miembro de la desigualdad anterior en terminos dep, q, r.Haciendo operaciones, se tiene

(x1 + x2 − x3)(x2 + x3 − x1)(x3 + x1 − x2) == (−p − 2x1)(−p − 2x2)(−p − 2x3) = −(p + 2x1)(p + 2x2)(p + 2x3) == −

(p3 + 2p2(x1 + x2 + x3) + 4p(x1x2 + x2x3 + x3x1) + 8x1x2x3 =

= −(p3 − 2p3 + 4pq − 8r) = p3 − 4pq + 8r > 0.

Para que se cumpla la condicion del enunciado, tienen que ser positivos los numeros si-guientes:

−p, q,−r y p3 − 4pq + 8r.

Como que los razonamientos son reversibles, las condiciones son tambien suficientes.

Observacion. En el libro de Ed Barbeau Polynomials, Ed.Springer, 1989, p.379 se incluyeuna version mas general de este problema.

Page 592: COMPENDIUM OME - ToomatesEotvos de Hungría, que se iniciaron en 1894, precisamente durante la efervescencia de fin de siglo, consecuencia de la cual fue también el proceso iniciado

E OME 27. Problema 4. Solucion

A

B C

I

A′

D

B′

C ′γ

β

δε

C2

A2

El triangulo BC ′A′ es isosceles, luego 2β + B = 180, de donde β = 90 − B2 . El angulo

γ es suplementario del β y vale γ = 180 − β = 90 + B2 . El triangulo DC ′A nos permite

calcular el angulo δ que vale

δ = 180 − A

2− β =

C

2.

La figura AC ′DB′ es simetrica respecto de la bisectriz AI, luego ε = δ = C2 .

El segmento IB′ se ve desde los puntos D y C bajo el mismo angulo ε = C2. Por lo tanto

los puntos I, C, B′ y D son concıclicos, con circunferencia de diametro IC. Luego elangulo IDC es recto. Obtenemos pues,

ADC = 90.

(La circunferencia que pasa por I, B′, C y D pasa tambien por el punto A′.)

Page 593: COMPENDIUM OME - ToomatesEotvos de Hungría, que se iniciaron en 1894, precisamente durante la efervescencia de fin de siglo, consecuencia de la cual fue también el proceso iniciado

E OME 27. Problema 5. Solucion

Escribiendo los numeros 1, 2, ..., 2k en base 2 tenemos:

0 = 0 0 ... 0 0; k ceros1 = 0 0 ... 0 1; k − 1 ceros2 = 0 0 ... 1 0; k − 1 ceros

... ... ... ... ... ...2k − 1 = 1 1 ... 1 1 ningun cero

2k = 1 0 0... 0 0 k ceros

La suma σ(k) es la suma total de cifras 1 que hay en el cuadro, mas la correspondiente a2k; en total se tiene

σ(k) = 2k−1 · k + 1.

Observacion. El enunciado es confuso ya que puede interpretarse la suma

σ(k) = s(1) + s(2) + · · ·+ s(2k)

como si estuviese extendida a las potencias de 2. En este caso el problema es trivial yaque σ(k) = k.

Page 594: COMPENDIUM OME - ToomatesEotvos de Hungría, que se iniciaron en 1894, precisamente durante la efervescencia de fin de siglo, consecuencia de la cual fue también el proceso iniciado

E OME 27. Problema 6. Solucion

Tenemos

1√k

=2

2√

k>

2√k +

√k + 1

= 2(√

k + 1 −√

k), k ≥ 1, luego

S = 1 +1√2

+ . . . +1√

10000>

> 2(√

2 −√

1 +√

3 −√

2 + . . . +√

10001 −√

10000) =

= 2(√

10001 − 1) > 2 · 99 = 198.

Por otro lado,

1√k

=2

2√

k<

2√k +

√k − 1

= 2(√

k −√

k − 1), k ≥ 2, luego

1√2

+ . . . +1√

10000<

< 2(√

2 −√

1 +√

3 −√

2 + . . . +√

10000 −√

9999) =

= 2(√

10000− 1) = 198.

Luego

S =1√1

+1√2

+ . . . +1√

10000< 1 + 198 = 199;

Por tanto tenemos que 198 < S < 199, es decir su parte entera es 198.

Page 595: COMPENDIUM OME - ToomatesEotvos de Hungría, que se iniciaron en 1894, precisamente durante la efervescencia de fin de siglo, consecuencia de la cual fue también el proceso iniciado

E OME 28. Problema 1. Solucion

Supongamos N = 2k · 3p · 5q · · · 83r. Entonces N2 = 22k · 32p · 52q · · · 832r, con r = 0.Debe ser 63 = (2k + 1)(2p + 1)(2q + 1) · · · (2r + 1), pero las unicas descomposiciones de 63son 63, 7 · 9 y 7 · 3 · 3.En el primer caso debe ser r = 31 y N = 8331.En el segundo caso debe ser k = 4, r = 3 y sale N = 24 833. (Si hacemos k = 3 y r = 4obtenemos un numero mayor).En el tercer caso es k = 3, p = 1, r = 1, de donde sale N = 23 31 831 (las demascombinaciones de exponentes dan numeros mayores, ya que 23 · 3 < 2 · 33).De los tres numeros obtenidos, el menor es el ultimo y por lo tanto el numero buscado esN = 23 · 3 · 83.

Page 596: COMPENDIUM OME - ToomatesEotvos de Hungría, que se iniciaron en 1894, precisamente durante la efervescencia de fin de siglo, consecuencia de la cual fue también el proceso iniciado

E OME 28. Problema 2. Solucion

dY X/2

A B A′ M C

O2

O1 PM ′

Podemos suponer que la direccion dada es horizontal y viene dada por la recta d.Pongamos O1 y O2 para los centros de las circunferencias de radios r y r’ respectivamente.Proyectamos O2 perpendicularmente sobre d en XTrasladamos X paralelamente a d una distancia

2 en el sentido O2O1 y ası determinamos Y .La perpendicular por Y a d y la paralela por O1 a d determinan P .Trasladamos la circunferencia de centro O1 mediante el vector O1P .La circunferencia trasladada (de puntos en la figura) corta eventualmente a la de centroO2 en dos puntos M y M ′.Por cada interseccion situada en el mismo semiplano que Y respecto de O2X tenemos unasolucion (en la figura el punto M) como se comprueba en la figura de modo inmediato.

Observacion. Si existe alguna interseccion en el mismo semiplano (M ’ en la figura), seobtienen cuerdas cuya diferencia mide .

Page 597: COMPENDIUM OME - ToomatesEotvos de Hungría, que se iniciaron en 1894, precisamente durante la efervescencia de fin de siglo, consecuencia de la cual fue también el proceso iniciado

E OME 28. Problema 3. Solucion

Sea Tn el n-esimo numero triangular

Tn = 1 + 2 + · · ·+ n =n(n + 1)

2.

Si n > m se cumple

Tn − Tm = (m + 1) + (m + 2) + · · ·+ (n + 1) + n ≥ m + 1

La igualdad(a + b)2 + 2a + b = (c + d)2 + 2c + d

la escribimos en la forma equivalente

(a + b)2 + (a + b) + a = (c + d)2 + (c + d) + c

o bien

(a + b)((a + b) + 1

)+ a = (c + d)

((c + d) + 1

)+ c o bien 2Ta+b + a = 2Tc+d + c,

que se puede escribir2(Ta+b − Tc+d) = c − a. (*)

Si suponemos a + c > c + d, tenemos

c − a = 2(Ta+b − Tc+d) ≥ 2(c + d + 1) o bienc − a

2≥ c + d + 1

lo que es absurdo si los enteros son positivos y a + b = c + d. El caso a + b < c + d saleanalogamente.

Supongamos ahora(a + b)2 + 3a + b = (c + d)2 + 3c + d.

Operando como antes, la igualdad (*) queda convertida en

Ta+b − Tc+d = c − a.

de dondec − a = Ta+b − Tc+d ≥ c + d + 1

que es absurdo tanto si a + b > c + d como si a + b < c + d.

Page 598: COMPENDIUM OME - ToomatesEotvos de Hungría, que se iniciaron en 1894, precisamente durante la efervescencia de fin de siglo, consecuencia de la cual fue también el proceso iniciado

Si ponemos(a + b)2 + 4a + b = (c + d)2 + 4c + d,

la igualdad (*) se convierte en

2(Ta+b − Tc+d) = 3(c − a)

o bien3(c − a) = 2(Ta+b − Tc+d) ≥ 2(c + d) o bien

32(c − a) ≥ c + d

que es una desigualdad que se puede cumplir en algunos casos. Por ejemplo,

2(T9 − T8) = 2(45 − 36) = 18 = 3 · 6 = 3(7 − 1)2 T8+1 + 3 · 1 = 2 T7+1 + 3 · 7

que nos dice que para a = 1, b = 8, c = 7 y d = 1 sale

(1 + 8)2 + 4 · 1 + 4 = (7 + 1)2 + 4 · 7 + 1.

Page 599: COMPENDIUM OME - ToomatesEotvos de Hungría, que se iniciaron en 1894, precisamente durante la efervescencia de fin de siglo, consecuencia de la cual fue también el proceso iniciado

E OME 28. Problema 4. Solucion

Demostrar que en la sucesion

3, 7, 11, 15, . . . , 4n + 3, ....

hay infinitos primos es lo mismo que probar que hay infinitos primos de la forma 4n + 3.Para demostrarlo, utilizamos el hecho de que el producto de dos numeros de la forma 4n+1es a su vez de la misma forma pues (4n1 + 1)(4n2 + 1) = (4n3 + 1).Sea q = 22 · 3 · 5 · . . . · p− 1, siendo 2, 3, 5, . . . , p todos los numeros primos hasta llegar a p.Entonces q es de la forma 4n− 1 o bien 4n+3, y no es divisible por ningun numero primomenor o igual que p.Si q es un numero primo ya hemos acabado.Si q es un numero compuesto quiere decir que hay primos mayores que p que dividen a q.Si todos fuesen de la forma 4n + 1, su producto tambien lo serıa, y q no lo es, de formaque alguno tiene que ser de la forma 4n + 3.En los dos casos demostramos que dado un conjunto finito de primos, existe un primomayor que ellos de la forma 4n + 3. Luego el conjunto de primos de esta forma es infinito.

Page 600: COMPENDIUM OME - ToomatesEotvos de Hungría, que se iniciaron en 1894, precisamente durante la efervescencia de fin de siglo, consecuencia de la cual fue también el proceso iniciado

E OME 28. Problema 5. Solucion

A

ϕ

ϕ ϕ

M

P

B C

Pongamos ϕ = PAB = PBC = PCA.La determinacion de P se hace en los siguientes pasos:1) Se traza la circunferencia pasando por A y tangente en B al lado BC.2) La paralela por B al lado AC corta en M a la circunferencia anterior.3) La interseccion de la recta CM con la circunferencia determina P .En efecto, PAB = PBC = PCA por angulos inscritos en el mismo arco y por ser MBparalela a AC.

Para la segunda parte, por el teorema de los senos en APC tenemos:

sen (A − ϕ)sen ϕ

=CP

AP

Por el mismo teorema en APC y BPC se cumple:

sen ϕ

AP=

sen A

bsen ϕ

CP=

sen C

a

, y dividiendo resultaCP

AP=

a sen A

b sen C

ademas

a

b=

sen A

sen Bimplica que

CP

AP=

sen 2A

sen B sen C

y sustituyendo en (*) resulta

sen (A − ϕ)sen ϕ

=sen 2A

sen B sen C

Page 601: COMPENDIUM OME - ToomatesEotvos de Hungría, que se iniciaron en 1894, precisamente durante la efervescencia de fin de siglo, consecuencia de la cual fue también el proceso iniciado

dividiendo por sen A, poniendo senA = sen (B + C) y desarrollando queda

sen A cosϕ − cos A sen ϕ

sen A sen ϕ=

sen B cos C + cos B sen C

sen B sen C⇐⇒ cot ϕ−cot A = cot C +cotB

y la relacion pedida escot ϕ = cotA + cotB + cotC.

Page 602: COMPENDIUM OME - ToomatesEotvos de Hungría, que se iniciaron en 1894, precisamente durante la efervescencia de fin de siglo, consecuencia de la cual fue también el proceso iniciado

E OME 28. Problema 6. Solucion

Como z = cos t + i sen t, sustituyendo en la igualdad resulta la ecuacion trigonometrica

cosn t = cos nt,

que para n = 2, 3, 4 tiene las soluciones

n = 2, S(n) = 1,−1n = 3, S(n) = 1,−1, i,−in = 4, S(n) = 1,−1, cos t + i sen t, t = arctan(±

√6)

S(4) tiene 6 elementos.Como para cada valor de cos t hay dos valores de sen t que resuelven la ecuacion, una cotasuperior del numero de elementos de S(n) es 2n − 2.

Page 603: COMPENDIUM OME - ToomatesEotvos de Hungría, que se iniciaron en 1894, precisamente durante la efervescencia de fin de siglo, consecuencia de la cual fue también el proceso iniciado

E OME 29. Problema 1. Solucion

Si en cada grupo de 6 personas, 2 son de la misma edad, solo puede haber 5 edadesdiferentes, ya que, si hubiese 6 edades diferentes, eligiendo una persona de cada edadtendrıamos 6 personas de edades distintas contra la hipotesis.Como 201 = 2 · 100 + 1, debe haber al menos 101 personas del mismo sexo.Como 101 = 5 · 20 + 1, debe haber al menos 21 personas de la misma edad y sexo.Y finalmente, como 21 = 4·5+1, debe haber al menos 5 personas de la misma nacionalidad,edad y sexo.

Tambien puede argumentarse de la manera siguiente: A cada persona le asignamos uncarnet de identidad con tres casillas, una para el sexo, una para la nacionalidad, y otrapara la edad. Como hay 2 sexos posibles, 5 edades posibles y 5 nacionalidades posibles,hay en total 50 carnets de identidad posibles. Puesto que en el conjunto hay 201 personas,por lo menos 5 de ellas tiene el mismo carnet de identidad, es decir, el mismo sexo, lamisma edad y la misma nacionalidad.

Page 604: COMPENDIUM OME - ToomatesEotvos de Hungría, que se iniciaron en 1894, precisamente durante la efervescencia de fin de siglo, consecuencia de la cual fue también el proceso iniciado

E OME 29. Problema 2. Solucion

Primera solucion

Vamos a sumar los elementos de cada fila y llamaremos Si a la suma de los elementos dela fila i.

S1 =0 + 1993

21994 = 997 · 1993

S2 = 2S1 − (0 + 1993) = ˙1993S3 = 2S2 − (0 + 1993) = ˙1993

· · ·S1994 = 2S1993 − (0 + 1993) = ˙1993

Podemos calcular cada suma recursivamente:

S2 = 2 S119931994

, S3 = 2 S219921993

= 22 S119921994

, . . . ,

. . . , S1994 = 21993 S11

1994= 21992 1993 = ˙1993

Segunda solucion

La suma total es

S1993 = 0(

19930

)+ 1

(1993

1

)+ 2

(1993

2

)+ · · ·+ 1993

(19931993

).

Si ponemos

f(x) = 1(

n

1

)+ 2x

(n

2

)+ · · · + nxn−1

(n

n

)

resulta que la primitiva de f(x) es

F (x) = x

(n

1

)+ x2

(n

2

)+ · · ·+ xn

(n

n

)= (1 + x)n − 1

de dondef(x) = n(1 + x)n−1 y f(1) = 2n−1n.

Page 605: COMPENDIUM OME - ToomatesEotvos de Hungría, que se iniciaron en 1894, precisamente durante la efervescencia de fin de siglo, consecuencia de la cual fue también el proceso iniciado

E OME 29. Problema 3. Solucion

Primera solucion

El area en funcion del radio de la circunferencia inscrita es S =(a + b + c)r

2, de donde

r =2S

a + b + c.

El area en funcion del radio de la circunferencia circunscrita es S =abc

4R, de donde R =

abc

4S.

Tenemos que probar que4S

a + b + c≤ abc

4S, es decir 16S2 ≤ (a + b + c)abc

De la formula de Heron tenemos que 16S2 = (a + b + c)(a + b − c)(a + c − b)(b + c − a).Tendremos el resultado si probamos que (a + b − c)(a + c − b)(b + c − a) ≤ abc, pero es

evidente que (a + b − c)(a + c − b)(b + c − a) ≤(

a + b + c

3

)3

y sin mas que utilizar la

desigualdad de las medias, y aplicando otra vez la misma desigualdada + b + c

3≤ 3

√abc

tenemos la desigualdad buscada.

Segunda solucion

Esta solucion es un razonamiento puramente geometrico que se formula en diversas fases.

Lema 1.- En un triangulo equilatero se cumple R = 2r.

Lema 2.- Si en un triangulo ABC fijamos la base AB y la recta que contiene el lado a , esdecir, el angulo B, y aumentamos el angulo A (o lo que es lo mismo, el lado a), el radio raumenta con A.En efecto, la bisectriz del angulo B se mantiene constante al mover C y el radio r aumentacon la longitud del lado a. El mismo razonamiento se puede hacer si el angulo B es obtuso.

A B

C

C′

II′

rr′

c

a

Page 606: COMPENDIUM OME - ToomatesEotvos de Hungría, que se iniciaron en 1894, precisamente durante la efervescencia de fin de siglo, consecuencia de la cual fue también el proceso iniciado

Lema 3.- Si en un triangulo ABC mantenemos la base AB fija y movemos el vertice Csobre una paralela a la base, el radio r sera maximo cuando el triangulo sea isosceles.En efecto, el area del triangulo se mantiene constante, y como es S = pr, el radio r seramaximo si p es mınimo, es decir, el triangulo es isosceles.

A B

C C′C′′

Lema 4.- Si dos triangulos comparten base y cırculo circunscrito, el mas cercano a isosceleses el que tiene mayor radio inscrito r. Trazamos la paralela a la base y obtenemos el puntoC′′. Podemos pasar del triangulo ABC al ABC′ en dos fases: primero al ABC′′, y porel Lema 3 el radio r aumenta, y despues al ABC′, y por el Lema 2 el radio r tambienaumenta.

A B

C

C′

C′′

Tercera solucion

Con la misma notacion de antes para los radios de las circunferencias inscrita y circunscrita,si d es la distancia entre el incentro y el circuncentro, la conocida formula de Euler nosdice que

d2 = R2 − 2Rr = r(R − 2r) ≥ 0, de donde R ≥ 2r.

Page 607: COMPENDIUM OME - ToomatesEotvos de Hungría, que se iniciaron en 1894, precisamente durante la efervescencia de fin de siglo, consecuencia de la cual fue también el proceso iniciado

E OME 29. Problema 4. Solucion

Si consideramos las clases de restos modulo p, entre las potencias 100, 101, 102, . . . ,10p−1,tiene que haber al menos 2 que pertenezcan a la misma clase. La clase 0 no puede aparecerpues p es distinto de 2 y de 5. Sean esas potencias 10a y 10b y supongamos b > a.10a = p · q + r, y 10b = p · q′ + r; por lo tanto: 10b − 10a = p(q′ − q) o bien 10a(10k − 1) =p(q′ − q). Como p es distinto de 2 y de 5, p es primo con 10a, y por lo tanto p|(10k − 1).Si p = 3, entonces p|111 . . .1, y si p = 3, existe el numero 111, que es multiplo de 3. Esdecir, al menos hay un multiplo que cumple las condiciones exigidas.Pero si el numero M formado por k unos es multiplo de p, tambien lo sera el numeroM ′ = M10k + M = M(10k + 1), y el numero M ′′ = M ′(102k + 1), y ası sucesivamente.

Page 608: COMPENDIUM OME - ToomatesEotvos de Hungría, que se iniciaron en 1894, precisamente durante la efervescencia de fin de siglo, consecuencia de la cual fue también el proceso iniciado

E OME 29. Problema 5. Solucion

Las posibles distancias son:1, 2, 3,

√2,√

5,√

8,√

10,√

13;las posibles distancias de B a A son 1, 2, 3,

√2,√

5,√

10,√

8,√

13,y las distancias de B a D son

√13,

√10, 3,

√8,√

5,√

2, 2, 1Las posibles coordenadas de B son:(1, 0), (0, 1), (2, 0), (0, 2), (1, 1), (2, 2), (3, 1), (1, 3), (3, 2), (2, 3); las simetrıas que nos pro-porcionan las dos diagonales nos permiten reducir el estudio a tres posiciones de B,(1, 0), (2, 0), (1, 1).Si B es (1, 0) entonces C pueda estar en (0, 2), (2, 2), (3, 1), (1, 3). En los otros casos noaparece ninguna figura distinta. Por tanto solo hay 4 figuras posibles. Teniendo en cuentalas dos simetrıas anteriores, por cada una de las figuras tenemos otras tres, y por tantohay 16 figuras que cumplen las condiciones.Observemos que A(0, 0) y D(3, 3) son siempre los mismos. Si utilizamos una relacionvectorial

−→AB +

−→AC +

−→AD +

−→BC +

−→BD +

−→CD =

−→AD +

−→AD +

−→AD +

−→BC−→

AD = (3, 3) y−→BC = (x, y). La suma pedida es: 9 + 9 + |x| + |y|, observando las cuatro

figuras anteriores se tiene que |x| + |y| = 3, que se mantiene en todas las simetrıas por loque la suma constante es 21.

Page 609: COMPENDIUM OME - ToomatesEotvos de Hungría, que se iniciaron en 1894, precisamente durante la efervescencia de fin de siglo, consecuencia de la cual fue también el proceso iniciado

E OME 29. Problema 6. Solucion

Partiendo de S, la probabilidad de llegar a C o a D es23; una vez en C o en D, la proba-

bilidad de ganar es igual que la de perder,12. Por tanto, partiendo de S, la probabilidad

de ganar es23· 12

=13.

Del analisis del juego se deduce que acabara en un numero par de movimientos.

La probabilidad que el juego acabe en dos tiradas es13, la de que acabe en cuatro tiradas

es23· 13, la de que acabe en seis

23· 23· 13, etc.

Y la duracion media sera

d = 2 · 23

+ 4 · 23· 13

+ 6 · 23· 23· 13

+ . . . =

=

[23

+(

23

)2

+(

23

)3

+ . . .

]+

[23

+(

23

)2

+(

23

)3

+ . . .

]+

+

[23

+(

23

)2

+(

23

)3

+ . . .

]+

[23

+(

23

)2

+(

23

)3

+ . . .

]+ . . . =

=2/31/3

+(2/3)2

1/3+

(2/3)3

1/3+ . . . =

21/3

= 6.

La probabilidad P de ganar tambien se puede calcular sumando las probabilidades de ganarsegun la duracion de la jugada, es decir

P =232

· 12

+22

33· 12

+23

34· 12

. . . =132

+233

+22

34. . . =

13.

Page 610: COMPENDIUM OME - ToomatesEotvos de Hungría, que se iniciaron en 1894, precisamente durante la efervescencia de fin de siglo, consecuencia de la cual fue también el proceso iniciado

E OME 30. Problema 1. Solucion

Bastara probar que a partir de un cuadrado perfecto podemos construir otro. Sea laprogresion:

a2, a2 + d, a2 + 2d, . . . , a2 + kd . . .

Como (a + d)2 = a2 + 2ad + d2 = a2 + (2a + d) d, basta tomar k = 2a + d para obtenerotro cuadrado en la progresion.

Page 611: COMPENDIUM OME - ToomatesEotvos de Hungría, que se iniciaron en 1894, precisamente durante la efervescencia de fin de siglo, consecuencia de la cual fue también el proceso iniciado

E OME 30. Problema 2. Solucion

En la figura se muestran con trazo discontinuo las circunferencias que resultan de intersecarla esfera con los planos coordenados. Las proyecciones del centro W de la esfera sobre estosplanos coinciden con los centros de estas circunferencias (denotados F, G y H en la figura)y al ser el triedro trirrectangulo, F , G y H estan en los puntos medio de los segmentosPQ, QC y CP que son diametros de sus circunferencias.Parametrizando con la distancia OP= λ tenemos trivialmente en la referencia OXY Z lasiguientes coordenadas:

P (λ, 0, 0) ; Q (0, k − λ, 0) ; C (0, 0, c)

X

Y

Z

C

Q

P

O

G

H

F

W

F

2,k − λ

2, 0

); G

(0,

k − λ

2,c

2

); H

2, 0,

c

2

); W

2,k − λ

2,c

2

)

El lugar de F es la recta x+ y = k2

del plano XOY . El lugar de W es una recta paralela ala anterior situada en el plano z = c

2 , y mas concretamente es la interseccion de los planos:

x + y =k

2z =

c

2

Page 612: COMPENDIUM OME - ToomatesEotvos de Hungría, que se iniciaron en 1894, precisamente durante la efervescencia de fin de siglo, consecuencia de la cual fue también el proceso iniciado

E OME 30. Problema 3. Solucion

Al suprimir una region, la suma de dıas soleados o lluviosos de las restantes ha de sermultiplo de 4. Esta suma vale 1994 para las seis regiones, valor que dividido entre 4da resto 2. El unico dato de esta columna que da resto 2 al dividirlo entre 4 es 330correspondiente a la region F. Suprimiendo esta region quedan entre las cinco restantes416 dıas lluviosos y 3 · 416 = 1248 dıas soleados.

Page 613: COMPENDIUM OME - ToomatesEotvos de Hungría, que se iniciaron en 1894, precisamente durante la efervescencia de fin de siglo, consecuencia de la cual fue también el proceso iniciado

E OME 30. Problema 4. Solucion

A

B C

D

b

a

a

ab−a

Con los datos del enunciado tenemosen el triangulo ABC, BAC = 36; ABC = ACB = 72,

en el triangulo CBD, BCD = 36; CDB = BDC = 72

en el triangulo ADC, DAC = ACD = 72; ADC = 108.Por tanto, los triangulos BCD y ADC son isosceles y ademas BCD es semejante al ABC.Para los lados se tiene: DC = AD = a; BD = b − a.Expresando la proporcionalidad derivada de la semejanza anterior tenemos las equivalen-cias:

b − a

a=

a

b⇐⇒ a2 = b2 − ab ⇐⇒ a2 + ab − b2 = 0 ⇐⇒

(a

b

)2

+a

b− 1 = 0.

y resolviendo quedaa

b=

√5 − 12

o bien

a =

(√5 − 1

)b

2es decir que a es la seccion aurea de b.

Page 614: COMPENDIUM OME - ToomatesEotvos de Hungría, que se iniciaron en 1894, precisamente durante la efervescencia de fin de siglo, consecuencia de la cual fue también el proceso iniciado

E OME 30. Problema 5. Solucion

Dispondremos el tablero en posicion vertical, es decir, con 7 filas y 3 columnas. Asignare-mos el color blanco a la cifra 0 y el negro a la cifra 1. De este modo cada fila representaun numero escrito en base 2.En primer lugar es facil ver que si en una fila se colocan todas las fichas del mismo color,por ejemplo el negro, necesariamente habra un rectangulo ya que no podemos colocar enninguna fila dos fichas negras y solo podemos llenar un maximo de 5 filas en total sinformar rectangulo.Por otra parte si dos numeros son iguales sus filas forman rectangulo, luego todas las filashan de representar numeros distintos. Por la consideracion anterior hemos de excluir losnumeros 000 y 111. Con tres cifras en base dos existen 23 = 8 numeros distintos, quitandolos anteriores quedan 6 para 7 filas por lo que necesariamente hemos de repetir y formarrectangulo. El problema tendrıa solucion en un tablero de 3 x 6 tal como se muestra en lafigura.

Page 615: COMPENDIUM OME - ToomatesEotvos de Hungría, que se iniciaron en 1894, precisamente durante la efervescencia de fin de siglo, consecuencia de la cual fue también el proceso iniciado

E OME 30. Problema 6. Solucion

Como hay m triangulos, hay 3m lados; de ellos 3m − n son interiores, y como un lado

interior pertenece a dos triangulos, hay3m − n

2lados interiores distintos. En particular

3m − n es par, luego m y n tienen la misma paridad y m + n es par.Supongamos que el numero de vertices v solo depende de m y n. Razonemos por induccionsobre v.Si no hay ningun vertice interior (v = 0), uniendo un vertice del polıgono con los otros, sedivide en n − 2 = n + 2v − 2 triangulos.Supongamos que hay v vertices interiores y n+2v−2 triangulos. Al anadir un vertice haydos posibilidades:a) El vertice esta en el interior de un triangulo, y para que se cumplan las condiciones delenunciado, debe unirse a cada uno de los tres vertices del triangulo que se divide en tresy el numero de triangulos ahora es:

n + 2v − 2 + 2 = n + 2 (v + 1) − 2.

b) El vertice esta en un lado, y entonces hay que unirlo con el vertice opuesto de cada unode los dos triangulos que comparten ese lado, cada triangulo se descompone en dos y elnumero de triangulos es ahora:

n + 2v − 2 + 2 = n + 2 (v + 1) − 2

En conclusion,

m = n + 2v − 2, de donde v =m − n + 2

2.

Page 616: COMPENDIUM OME - ToomatesEotvos de Hungría, que se iniciaron en 1894, precisamente durante la efervescencia de fin de siglo, consecuencia de la cual fue también el proceso iniciado

E OME 31. Problema 1. Solucion

Sea A un conjunto que cumpla las condiciones del enunciado. Queremos calcular la sumade los perımetros de los triangulos, y lo haremos por dos procedimientos distintos.

Primer procedimiento

Sea k ∈ A y contemos el numero de triangulos que tienen lado k, y su contribucion a lasuma de perımetros.Hay un solo triangulo equilatero de lado k, y contribuye con el valor 3k.Hay 99 triangulos isosceles de lados k, r, r, (k = r) y el lado k contribuye con 99k.Hay 99 triangulos isosceles de lados k, k, r, (k = r) y el lado k contribuye con 2 · 99 k.Hay

(992

)= 99·49 triangulos escalenos de tipo k, r, s (con k, r, s distintos), y su contribucion

es 99 · 49k.En total, la contribucion de k ∈ A es de

k(3 + 99 + 2 · 99 + 49 · 99) = 5151 k.

Segundo procedimiento

Debemos escoger de A tres elementos con posibles repeticiones. El numero total de for-mas de hacerlo corresponde al numero de combinaciones con repeticion de 100 elementostomados 3 a tres,

CR3100 =

(100 + 3 − 1

3

)=

(1023

)=

102 · 101 · 1006

, triangulos,

que corresponden a102 · 101 · 100

63 lados,

y como que hay 100 valores posibles para los lados, cada uno de ellos aparece

102 · 1016

3 = 5151.

En ambos procedimientos, acabamos el problema observando que la suma de perımetrossera por tanto 5151 S(A), donde S(A) indica la suma de elementos de A, suma que debemoshacer mınima.Sean ahora a y c el mınimo y el maximo de A. El triangulo caa debe ser no oblicuangulo,luego c2 ≤ 2a2. Si para hacer mınimo S(A) exigimos que los elementos de A sean conse-cutivos, sera c = a + 99 y resulta a + 99 ≤

√2 a, o bien a ≥ 240. El conjunto es

A = 240, 241, 242, . . . , 338, 339

cuya suma es 28950, y la suma de perımetros mınima es 5151 · 28950 = 149 121 450.

Page 617: COMPENDIUM OME - ToomatesEotvos de Hungría, que se iniciaron en 1894, precisamente durante la efervescencia de fin de siglo, consecuencia de la cual fue también el proceso iniciado

E OME 31. Problema 2. Solucion

La frontera de las piezas recortadas (que no sean cırculos completos) esta formada porarcos concavos y convexos (vistos desde fuera) que se cortan en puntos que llamaremosvertices. En un vertice pueden concurrir dos arcos concavos o uno concavo y otro convexo,pero nunca dos convexos ya que estos unicamente provienen de la frontera de los cırculosiniciales. Ademas, los angulos que forman los arcos en cada vertice no son de 0 ni de 180

ya que excluimos las tangencias interiores.

P

Supongamos que tenemos un cırculo obtenido ensamblando piezas recortadas. Existe almenos un punto P de la frontera de dicho cırculo en el que concurren tres o mas arcos de lafrontera de las pieza ensambladas (P es vertice de dos o mas piezas). La tangente al cırculoen P deja a todos los arcos en un mismo semiplano. Elegido un sentido de rotacion en Pa partir de la tangente, y avanzando en este sentido, el primer arco que encontramos esconvexo y el ultimo concavo. Por lo tanto es necesario que existan dos arcos consecutivosuno convexo y el otro concavo los cuales forman parte de la frontera de una de las piezasensambladas. Como el arco que forman dichas piezas no puede ser ni 0o ni 180o, el puntoP es un vertice de la pieza. Esto es contradictorio pues en ningun vertice pueden concurrirdos arcos convexos vistos desde fuera.

Observacion. Hay que entender en el enunciado que quedan excluidas las tangencias inte-riores. De no ser ası pueden encontrarse contraejemplos como el “despiece” que se muestraen la figura:

Page 618: COMPENDIUM OME - ToomatesEotvos de Hungría, que se iniciaron en 1894, precisamente durante la efervescencia de fin de siglo, consecuencia de la cual fue también el proceso iniciado

E OME 31. Problema 3. Solucion

Dupliquemos el triangulo trazando AD paralela a BC y CD paralela a BA como muestrala figura, y tomemos la longitud del lado AB como unidad. Llamando M a la interseccionde CD con la recta PQ y x = PB, 1 − x = AP , tenemos

A

B

C

P

Q

M

G

D

Por la semejanza de los triangulos AQP y QMC saleQC

QA=

MC

AP=

MC

1 − x.

Por la semejanza de los triangulos GPB y GMD quedaPB

MD=

GB

GD=

12.

Luego MD = 2x y MC = 1 − 2x.Sustituyendo en el primer miembro de la relacion del enunciado queda la cadena de condi-ciones equivalentes

PB

PA· QC

QA≤ 1

4⇐⇒ x(1 − 2x)

(1 − x)2≤ 1

4⇐⇒ 9x2 − 6x + 1 ≥ 0 ⇐⇒ (3x − 1)2 ≥ 0,

relacion valida para cualquier x.La igualdad se alcanza para PB = x = 1

3 , es decir para MC = 13 o bien cuando PQ es

paralela al lado BC.

Page 619: COMPENDIUM OME - ToomatesEotvos de Hungría, que se iniciaron en 1894, precisamente durante la efervescencia de fin de siglo, consecuencia de la cual fue también el proceso iniciado

E OME 31. Problema 4. Solucion

Ya que p es primo, es distinto de 0 y de 1. De la ecuacion resulta que p divide a x o pdivide a y. Como la ecuacion es simetrica respecto de x e y, si el par (α, β) es solucion,tambien lo sera el par (β, α).Si p divide a x, debe ser x = p·a, (a ∈ Z) y la ecuacion se puede poner como p(p a+y) = pay,de donde pa + y = ay y

y =pa

a − 1, ya que a es entero.

Ademas, a y a − 1 son primos entre sı, luego a − 1 divide a p. Al ser p primo solo haycuatro posibilidades: a − 1 = ±1 y a − 1 = ±p.Examinemos todos los casos.

i) a − 1 = −1, entonces a = 0, x = 0, y = 0.

ii) a − 1 = 1, entonces a = 2, x = 2p, y =2p

2 − 1= 2p.

iii) a − 1 = p, entonces a = p + 1, x = p(p + 1), y =p (p + 1)p + 1 − 1

= p + 1 .

iiii) a− 1 = −p, entonces a = 1 − p, x = p(1 − p), y =p (1 − p)1 − p − 1

= p − 1 .

En resumen las soluciones son: (0, 0); (2p, 2p);(p(p + 1), p + 1

);(p(1 − p), p− 1

); y por la

simetrıa anadimos(p + 1, p(p + 1)

);(p − 1, p(1 − p)

).

Page 620: COMPENDIUM OME - ToomatesEotvos de Hungría, que se iniciaron en 1894, precisamente durante la efervescencia de fin de siglo, consecuencia de la cual fue también el proceso iniciado

E OME 31. Problema 5. Solucion

Sea α la raız comun de ambas ecuaciones. Entonces

α3 + mα = n

y sustituyendo en la segunda ecuacion se obtiene, tras hacer operaciones:

6mα4 + 8m2α2 + 2m3 = 0.

Si suponemos m = 0, entonces simplificando la relacion anterior queda:

3α4 + 4mα2 + m2 = 0.

Resolviendo respecto de m obtenemos

m = −α2 (i)

−3α2 (ii)

Analicemos cada caso.(i) Si m = −α2, sustituyendo en la primera ecuacion y despejando n queda

n = α3 − α3 = 0

en contra de lo supuesto. Por tanto (i) queda descartado.

(ii) Si m = −3α2, sustituyendo en la primera ecuacion y despejando n resulta n = α3 −3α3 = −2α3 y la primera ecuacion queda

x3 − 3α2x + 2α3 = (x − α)(x2 + αx − 2α2

)= (x − α)2 (x + 2α)

que, efectivamente, tiene la raız α doble.

De n = −2α3 obtenemos α = 3

√−n

2.

Entonces la segunda ecuacion es de la forma

−2α3(x3 + 9αx2 + 15α2x − 25α3

)= 0,

y, dividiendo por (x − α), resulta −2α3 (x − α) (x + 5α)2 = 0, cuyas raıces son α y −5αsiendo doble la ultima.

Observacion. Si m = 0 las dos ecuaciones son iguales y sus tres raıces son las mismas perola primera no tiene dos raıces iguales por lo que en el enunciado deberıa haberse anadidom = 0.

Page 621: COMPENDIUM OME - ToomatesEotvos de Hungría, que se iniciaron en 1894, precisamente durante la efervescencia de fin de siglo, consecuencia de la cual fue también el proceso iniciado

E OME 31. Problema 6. Solucion

Para resolver este problema utilizaremos muchas veces lapropiedad siguiente: si desde un punto P de la circunferenciacircunscrita a un triangulo equilatero trazamos segmentos quelo unan a los tres vertices, dichos segmentos forman angulos de60, 60 y 120; y este ultimo corresponde al angulo formadopor los segmentos que une P con los vertices mas proximos. Yrecıprocamente, si dos de los angulos que forman los segmentosque unen un punto P con los vertices de un triangulo equilateroson de 60 y 60, entonces dicho punto P pertenece a la circun-ferencia circunscrita al triangulo.

P

a) Tracemos las circunferencias circunscritas a los triangulos ACB′ y CBA′, y sea P elpunto de interseccion distinto del C. Desde P se ven AC y CB bajo angulo de 60, luego seve AB bajo angulo de 120, y por lo tanto la circunferencia circunscrita a ABC ′ tambienpasa por P . Ademas, desde P vemos CB bajo angulo de 60 y CB′ bajo angulo de 120, yesto implica que B′, P y B estan alineados. Un razonamiento analogo demuestra que A′,P y A estan alineados. En cuanto a la alineacion de P , C, C ′, basta observar que tantoC ′B como CB se ven desde P bajo angulo de 60.

A BC

PA′

B′

C′

b) Si observamos que el angulo APB es de 120 deducimos que el lugar geometrico de Pes el arco de circunferencia circunscrita a ABC ′ que esta por debajo del segmento AB.

Page 622: COMPENDIUM OME - ToomatesEotvos de Hungría, que se iniciaron en 1894, precisamente durante la efervescencia de fin de siglo, consecuencia de la cual fue también el proceso iniciado

c) Para demostrar esta parte utilizaremos que la cuerda comun a dos circunferencias se-cantes es perpendicular a la lınea de los centros. El segmento A′′B′′ es perpendicular aCP , el C ′′B′′ es perpendicular a AP y el C ′′A′′ es perpendicular a BP . Como que CP yAP forman un angulo de 60, tambien forman ese angulo las A′′B′′ y B′′C ′′. Repitiendoel razonamiento, vemos que los tres angulos de A′′B′′C ′′ son de 60, y el triangulo esequilatero.

A BC

PA′

B′

C′

B′′

C′′

A′′A BC

PA′

B′

C′

B′′

C′′

A′′

d) El cuadrilatero A′′PB′′C marcado en la figura tiene las dos diagonales que son la cuerdacomun de dos circunferencias y la lınea de los centros. Por lo tanto es simetrico respectode esta ultima y A′′CB′′ = A′′PB′′. Pero A′′CB′′ = 180 − 30 − 30 = 120 (los angulossenalados son de 30), de forma que A′′PB′′ = 120 y esto nos dice que desde P se ve ellado A′′B′′ del triangulo equilatero A′′B′′C ′′ bajo angulo de 120, es decir que P esta enla circunferencia circunscrita a este triangulo.

Page 623: COMPENDIUM OME - ToomatesEotvos de Hungría, que se iniciaron en 1894, precisamente durante la efervescencia de fin de siglo, consecuencia de la cual fue también el proceso iniciado

E OME 32. Problema 1. Solucion

Se tienea + 1

b+

b + 1a

=a2 + b2 + a + b

ab∈ Z.

Sea d = mcd (a, b). Como a b es divisible por d2, entonces a2 + b2 + a + b es divisible pord2 y como tambien lo es a2 + b2, debe serlo a + b. Al ser a y b naturales, se tiene

a + b ≥ d2

de donde √a + b ≥ d.

Page 624: COMPENDIUM OME - ToomatesEotvos de Hungría, que se iniciaron en 1894, precisamente durante la efervescencia de fin de siglo, consecuencia de la cual fue también el proceso iniciado

E OME 32. Problema 2. Solucion

Primera solucion

Teniendo en cuenta el teorema de la mediana, la relacion del enunciado se escribe

c − b =23

(√a2 + c2

2− b2

4−

√a2 + b2

2− c2

4

),

y multiplicando y dividiendo por la expresion conjugada queda:

c − b =23

34

(c2 − b2

)mc + mb

que es equivalente a (c − b)(

mc + mb −c + b

2

)= 0.

Probaremos que el segundo factor es positivo, de donde se deducira la conclusion.Llamando B′ y C ′ a los puntos medios de AC y AB respectivamente, en los triangulosCC ′A y BB′A tenemos por la desigualdad triangular

mb +b

2> c; mc +

c

2> b.

Sumando ambas desigualdades se obtiene el resultado.

Segunda solucion

B CA′

A

C ′ B′

G

M

Llamando A′, B′, C ′ a los puntos medios de los lados BC, AC y AB, respectivamente, ydividiendo por dos la condicion del enunciado, esta podemos escribirla como

C ′A + C ′G = B′A + B′G,

es decir, los puntos C ′ y B′ estan en una elipse de focos A y G.Llamando M al punto medio de C ′B′, M esta en la mediana AA′ y no es el centro de laelipse (punto medio del segmento AG), y por tanto C ′B′ ha de ser perpendicular a AA′,y entonces AA′ ademas de mediana es altura y el triangulo es isosceles.

Page 625: COMPENDIUM OME - ToomatesEotvos de Hungría, que se iniciaron en 1894, precisamente durante la efervescencia de fin de siglo, consecuencia de la cual fue también el proceso iniciado

E OME 32. Problema 3. Solucion

Podemos conseguir coeficientes A, B, C, tales que se tenga identicamente

f(x) = Ax(x + 1) + Bx(x − 1) + C(x2 − 1).

Particularizando para x = 1, -1 ,0 y resolviendo el sistema queda

f(x) =f(1)

2x(x + 1) +

f(−1)2

x(x − 1) + f(0)(1 − x2).

De aquı se deduce

|f(x)| ≤ 12|x(x + 1)| + 1

2|x(x − 1)| +

∣∣1 − x2∣∣ ;

como −1 ≤ x ≤ 1, 1 + x ≥ 0, 1 − x ≥ 0 y 1 − x2 ≥ 0, resulta

|f(x)| ≤ |x|2

(1 + x) +|x|2

(1 − x) + 1 − x2 = −x2 + |x| + 1 =54−

(|x| − 1

2

)2

≤ 54.

Por otra parte, para x = 0, es g(x) = x2f(

1x

). Entonces

g(x) =f(1)

2(1 + x) +

f(−1)2

(1 − x) + f(0)(x2 − 1

)valido para −1 ≤ x ≤ 1. Ası pues

|g(x)| ≤ 1 + x

2+

1 − x

2+ 1 − x2 = 2 − x2 ≤ 2.

Page 626: COMPENDIUM OME - ToomatesEotvos de Hungría, que se iniciaron en 1894, precisamente durante la efervescencia de fin de siglo, consecuencia de la cual fue también el proceso iniciado

E OME 32. Problema 4. Solucion

Si p < 0, entonces√

x2 − p > x; como 2√

x2 − 1 ≥ 0, no existe solucion. Por tanto p ≥ 0.Aislando un radical y elevando al cuadrado dos veces se llega a la ecuacion:

8 (2 − p)x2 = (4 − p)2 , de donde x =|4 − p|√8 (2 − p)

.

Como x ∈ R, debe ser p < 2, ası que

x =4 − p√8 (2 − p)

.

Sustituyendo en la ecuacion dada se obtiene

√(4 − 3p)2

8 (2 − p)+ 2

√p2

8 (2 − p)=

4 − p√8 (2 − p)

y como p > 0, es |p| = p; y finalmente

|4 − 3p| + 2p = 4 − p =⇒ |4 − 3p| = 4 − 3p =⇒ 4 − 3p ≥ 0 ⇐⇒ 0 ≤ p ≤ 43.

Page 627: COMPENDIUM OME - ToomatesEotvos de Hungría, que se iniciaron en 1894, precisamente durante la efervescencia de fin de siglo, consecuencia de la cual fue también el proceso iniciado

E OME 32. Problema 5. Solucion

Diremos que los agentes A y B son neutrales si A no vigila a B ni B vigila a A.Sean A1, A2, . . . , An los agentes. Sea ai el numero de agentes que vigilan a Ai, bi elnumero de agentes que son vigilados por Ai, y ci el numero de agentes que son neutralescon Ai.Es claro que

ai + bi + ci = 15, ai + ci ≤ 8, bi + ci ≤ 8 para i = 1, 2, ....16.

Notemos que si una cualquiera de las dos ultimas desigualdades no se cumpliese, entoncesno se podrıan numerar 10 espıas en la forma indicada.Combinando las relaciones anteriores obtenemos ci ≤ 1. Por tanto para cualquier espıa elnumero de sus colegas neutrales es 0 o 1.Razonemos por reduccion al absurdo.Supongamos que hubiera un grupo de 11 espıas que NO se pudiera numerar en la formadescrita. Sea B uno cualquiera de los espıas de este grupo.Numeramos los otros 10 espıas como C1, C2, . . . , C10 de modo que C1 vigila C2, . . . , C10

vigila a C1. Supongamos que ninguno de los Ci sea neutral respecto de B. Entonces siC1 vigila a B, B no puede vigilar a C2, pues en tal caso C1, B, C2, . . . , C10 formarıa ungrupo en las condiciones del problema; luego C2 vigila B , etc. De este modo llegamos ala contradiccion de que todos los espıas del grupo vigilan a B. Por tanto cada uno de los11 espıas debe tener uno y solo uno del grupo neutral con el, lo cual es imposible.

Page 628: COMPENDIUM OME - ToomatesEotvos de Hungría, que se iniciaron en 1894, precisamente durante la efervescencia de fin de siglo, consecuencia de la cual fue también el proceso iniciado

E OME 32. Problema 6. Solucion

La figura formada por el agua es un tronco de piramide pentagonal cuya base menor es elpentagono dado y cuya base mayor es otro pentagono regular que tiene por lado la diagonaldel anterior paralela a la arista de la base como se muestra en la siguiente figura.

Este piramide aparece en la figura en forma invertida para una mejor comprension deldibujo.

Si llamamos d a la diagonal del pentagono regular de lado uno , sabemos que

1d − 1

=d

1⇐⇒ d2 − d − 1 = 0 =⇒ d =

1 +√

52

= ϕ (1)

ϕ es el llamado numero aureo y representa la relacion entre la diagonal y el lado de unpentagono regular. En nuestro caso es la relacion de semejanza entre las bases del troncode piramide. Ademas, cos 36 = d

2= ϕ

2= 1+

√5

4, y para el radio r sale sen36 = 1

2r, de

donde

r =1

2 sen36=

1√4 − ϕ2

. (2)

Llamando V al volumen de la piramide grande, v al de la pequena, sabemos que V = ϕ3 v;y para el volumen del tronco de cono Vt queda

Vt = V − v = ϕ3v − v = v(ϕ3 − 1) =13ah(ϕ3 − 1);

Page 629: COMPENDIUM OME - ToomatesEotvos de Hungría, que se iniciaron en 1894, precisamente durante la efervescencia de fin de siglo, consecuencia de la cual fue también el proceso iniciado

siendo a el area del pentagono de lado 1. Solo nos queda calcular a, h, sustituir y operar:

Hh

r

RH−h 1

R−r

El area a la calculamos sumado 5 triangulos isosceles de lados iguales r, con angulo desigualde 72.

a =52r2 sen 72 =

52

r2 2 sen36 cos 36o =52

r cos 36 =54

r ϕ.

(Hemos usado 2r sen36 = 1).Para calcular h, por la semejanza de los triangulos de la figura central, tenemos:

H

R=

h

r=

H − h

R − r

implica que

h =r (H − h)

R − r=

r√

1 − (R − r)2

r(ϕ − 1)=

√1 − r2(ϕ − 1)2

ϕ − 1=

√1 − (ϕ−1)2

4−ϕ2

ϕ − 1.

Como ϕ cumple la ecuacion (1), es ϕ2 = ϕ +1; tenemos para la expresion de h

h =

√1 − (ϕ−1)2

4−ϕ2

ϕ − 1=

√4 − ϕ2 − ϕ2 + 2ϕ − 1(ϕ − 1)

√4 − ϕ2

=√

3 − 2ϕ − 2 + 2ϕ

(ϕ − 1)√

4 − ϕ2=

1(ϕ − 1)

√4 − ϕ2

.

Sustituyendo las expresiones de a y h y poniendo ϕ3-1= (ϕ-1)(ϕ2 + ϕ + 1) queda

Vt =13

54

ϕ√4 − ϕ2

(ϕ3 − 1)(ϕ − 1)

√4 − ϕ2

=512

ϕ(ϕ2 + ϕ + 1)4 − ϕ2

=56

ϕ(ϕ + 1)3 − ϕ

=56

2ϕ + 13 − ϕ

y sustituyendo el valor de ϕ de (1), queda finalmente:

Vt =53

2 +√

55 −

√5

=15 + 7

√5

12∼= 2,554m3.

Page 630: COMPENDIUM OME - ToomatesEotvos de Hungría, que se iniciaron en 1894, precisamente durante la efervescencia de fin de siglo, consecuencia de la cual fue también el proceso iniciado

E OME 33. Problema 1. Solucion

Sea la progresion, a, a + d, a + 2d, . . . , a + 99d; tenemos que hallar

S = a2 + (a + d)2 + . . . + (a + 99d)2 == 100a2 + 2ad (1 + 2 + . . . + 99) + d2

(12 + 22 + . . . + 992

).

Para calcular a y d resolvemos el sistema

(a + a + 99d) 50 = −1

(a + d + a + 99d) 25 = 1

que, despues de hacer operaciones, da a = −2, 98; d = 0, 06.El resto es facil de calcular. Los parentesis son progresiones de primer y segundo orden.

1 + 2 + ... + 99 = 4950; 12 + 22 + .... + 992 = 328350.

El resultado final es S = 299,98

Page 631: COMPENDIUM OME - ToomatesEotvos de Hungría, que se iniciaron en 1894, precisamente durante la efervescencia de fin de siglo, consecuencia de la cual fue también el proceso iniciado

E OME 33. Problema 2. Solucion

Numeremos los 16 puntos como indica la tabla siguiente.

13 14 15 169 10 11 125 6 7 81 2 3 4

Por simple tanteo se obtiene un conjunto de seis puntos que cumple la condicion delenunciado, por ejemplo 1, 2, 3, 8, 12, 16.Supongamos que hubiera un conjunto M de 7 puntos que tambien la cumpliese. Notemosque si cuatro puntos forman un cuadrado, a lo sumo figuraran dos de ellos en M . Lospuntos de los conjuntos

1, 4, 16, 13, 2, 8, 15, 9, 3, 12, 14, 5

forman cuadrados y su union forma el “contorno exterior” de A, luego a los sumo 6 delos puntos elegidos deben estar en M y por tanto al menos un punto de M debe ser delconjunto “interior” de A 6, 7, 10, 11. Por la simetrıa de la figura supongamos que es el 7.Como 7, 16, 9 y 1, 7, 14 forman triangulos rectangulos isosceles, a lo sumo 2 de lospuntos del conjunto 1, 9, 14, 16 deberan figurar en M . Ademas, 5, 7, 13, 15 formanun cuadrado; por tanto, a lo sumo podremos elegir dos numeros entre 5, 13, 15 , deello se deduce en M deben figurar al menos tres puntos de 2, 3, 4, 6, 8, 10, 11, 12. Sidescomponemos este conjunto en dos subconjuntos “cuadrados” y disjuntos

3, 6, 11, 8 y 2, 4, 10, 12

forzosamente de uno de ellos habremos de tomar dos puntos y uno de otro.Si tomamos dos puntos del primero, las unicas posibilidades son 3, 11 y 6, 8, ambasincompatibles con cualquier eleccion del punto restante en el segundo conjunto.Si los dos puntos se eligen del segundo, las unicas maneras son 2, 12 y 4, 10, de nuevoincompatibles con cualquier eleccion del punto que falta en el primer conjunto.En resumen el numero maximo de elementos es 6.

Page 632: COMPENDIUM OME - ToomatesEotvos de Hungría, que se iniciaron en 1894, precisamente durante la efervescencia de fin de siglo, consecuencia de la cual fue también el proceso iniciado

E OME 33. Problema 3. Solucion

Primera solucion

Sean α y β las raıces. Los tres puntos que definen la circunferencia son A(α, 0), B(β, 0) yC(0, q), cumpliendose α + β − p = 0 y αβ = q (*).La mediatriz de AB es la recta paralela al eje OY de ecuacion x = −p

2 .Hallando la mediatriz de AC, cortando con la anterior y teniendo en cuenta las relaciones(*), se obtiene para el centro de la circunferencia las coordenadas (−p

2, q+1

2) y para el radio

r =√

p2+(1−q)2

4. La ecuacion de la circunferencia es (x + p

2)2 + (y − q+1

2)2 = p2+(1−q)2

4,

que una vez operada queda:

x2 + y2 + px − (1 + q)y + q = 0,

que se cumple para el punto (0, 1), con independencia de p y q, como se comprueba porsimple sustitucion.Claramente el punto fijo se puede obtener a partir de tres circunferencias concretas.

Segunda solucion

Puesto que la parabola corta al eje de abscisas en dos puntos, se podra escribir en la forma:

y = (x − a) (x − b)

y los puntos de interseccion son

A(a, 0), B(b, 0), C(0, a b).

La inversion de polo el origen que transforma A en B, transforma C en U (0, 1) , ası quelos cuatro puntos A, B, C, U son concıclicos y todas las circunferencias pasan por el puntofijo U .

C

U

A B

Page 633: COMPENDIUM OME - ToomatesEotvos de Hungría, que se iniciaron en 1894, precisamente durante la efervescencia de fin de siglo, consecuencia de la cual fue también el proceso iniciado

E OME 33. Problema 4. Solucion

Si ponemos√

k2 − kp = n nos queda k2 − pk − n2 = 0, de donde se deduce

k =p ±

√p2 + 4n2

2. (*)

El radicando ha de ser cuadrado perfecto; llamemosle a. Se tiene

p2 + 4n2 = a2 o bien p2 = (a + 2n)(a − 2n).

Como p es primo y a+2n ≥ a−2n, solo hay dos posibilidades. Una de ellas es a+2n = p2

y a − 2n = 1, y la otra a + 2n = p y a − 2n = p.

En el primer caso es a = p2+12

y n = p2−14

, lo que exige p = 2, (n natural).En el segundo caso resulta a = p y n = 0.Sustituyendo los valores de a en (*) y operando queda:Si p = 2 , entonces k = 2 o k = 0.Si p = 2, entonces quedan los cuatro valores:

k1 =(

p + 12

)2

, k2 = −(

p − 12

)2

, k3 = p, k4 = 0.

Page 634: COMPENDIUM OME - ToomatesEotvos de Hungría, que se iniciaron en 1894, precisamente durante la efervescencia de fin de siglo, consecuencia de la cual fue también el proceso iniciado

E OME 33. Problema 5. Solucion

Primera solucion

Sea el cuadrilatero de lados a, b, c, d y diagonales p yq. Trazando la paralelas por cada vertice a la diagonalque no pasa por el se forma un paralelogramo de area 2 ylados p y q.Por el teorema isoperimetrico, de todos los paralelogramosde area 2, el cuadrado tiene perımetro mınimo que vale4√

2, luego

2 (p + q) ≥ 4√

2 ⇔ p + q ≥ 2√

2. (1)

En cuanto al los lados, por el mismo teorema para unacuadrado de area 1 el perımetro es 4 luego

a + b + c + d ≥ 4. (2)

Sumando(1) y (2) se obtiene el resultado.

a

bc

d

q

p

Segunda solucion

Esta solucion no usa la propiedad isoperimetrica sino que establece directamente las de-sigualdades (1) y (2).Si α es el angulo que forman las diagonales, tenemos:

1 =p q

2sen α ≤ p q

2de donde p q ≥ 2

pero (p + q)2 = (p − q)2 + 4p q ≥ 4p q ≥ 8, de donde

p + q ≥√

8 = 2√

2. (1)

Para los lados, si descomponemos el cuadrilatero en dos triangulos mediante la diagonalq, tenemos:

1 ≤ a b

2+

c d

2.

Descomponiendo ahora en dos triangulos mediante la diagonal p resulta

1 ≤ bc

2+

da

2y de ambas desigualdades se obtiene a b + b c + c d + d a ≥ 4.Pero (a + b + c + d)2 =

((a + c) − (b + d)

)2 + 4(a + c)(b + d) ≥ 4(a + c)(b + d) ≥ 16, dedonde

a + b + c + d ≥ 4. (2)

Basta sumar (1) y (2) para obtener lo pedido.

Page 635: COMPENDIUM OME - ToomatesEotvos de Hungría, que se iniciaron en 1894, precisamente durante la efervescencia de fin de siglo, consecuencia de la cual fue también el proceso iniciado

E OME 33. Problema 6. Solucion

Primera solucion

1

2

3

4 5

67

1

2

3

Sean c1, c2, . . . , cn las cantidades de combustible en cada uno de los n depositos y seand1, d2, . . . , dn las distancias a recorrer desde cada deposito hasta el siguiente.Hagamos el grafico del consumo comenzando en un punto de aprovisionamiento cualquiera.Notemos que los tramos inclinados tienen todos la misma pendiente. Los tramos bajo el eje

representan las situaciones imposibles. La pendiente de los tramos inclinados vale −∑

ci∑di

.

La hipotesis de que el total de combustible es la cantidad exacta para dar la vuelta setraduce en que la grafica comienza y termina en el eje OX .La funcion resultante (trazo continuo) tiene un mınimo, en la figura en el punto 3. Bastacomenzar en ese punto para asegurar que el recorrido es posible.En efecto, graficamente equivale a trasladar el eje OX en sentido vertical hasta el puntomas bajo con lo que aseguramos que ninguna zona queda bajo el eje. La nueva grafica puedetrazarse a partir del punto 3 siguiendo el mismo trazado hacia la derecha y trasladandola parte anterior (tramos 1-2 y 2-3) al punto final de la grafica anterior (de puntos en lafigura).

Segunda solucion

Se numeran los depositos de 1 a n comenzando por uno cualquiera en sentido antihorario.Llamamos a1, a2, . . . , an a la cantidad de gasolina de cada deposito; y b1, b2, . . . , bn a lacantidad de gasolina necesaria para ir del deposito ai al siguiente. Finalmente, ponemosd1 = a1 − b1, d2 = a2 − b2, . . . , dn = an − bn. Diremos que un deposito es positivo onegativo segun lo sea di.Si di = 0, la ubicacion del deposito i no influye en la ordenacion del recorrido. Por ellopodemos suponer sin perdida de generalidad que di = 0 para todo i.Por otra parte, si hay varios depositos consecutivos positivos o negativos, el tramo limitadopor ellos se puede considerar como un unico tramo positivo o negativo. Ası, el problemase reduce a tener un numero par de depositos alternativamente positivos o negativos.Agrupando los tramos por parejas, estas resultaran positivas o negativas y volvemos arepetir el proceso.Ası reducimos el caso a un numero de depositos n1 ≤ n

2 .Como n < 2k, a lo sumo en k − 1 etapas llegaremos a tener 2 depositos, uno con masgasolina que otro, en cuyo caso empezando por el que tenga mas combustible se puedecompletar el circuito.El caso de un solo deposito es trivial: se empieza y termina en ese unico deposito.

Page 636: COMPENDIUM OME - ToomatesEotvos de Hungría, que se iniciaron en 1894, precisamente durante la efervescencia de fin de siglo, consecuencia de la cual fue también el proceso iniciado

E OME 34. Problema 1. Solucion

Primera solucion

AB

C D

A′

B′

C′

D′

O

PM

Por la simetrıa bastara considerar 0 < α < 90, ya que la funcion es periodica con periodode un cuarto de vuelta. El area pedida S(α) sale restando del area del cuadrado cuatrotriangulos como el PA′M .Llamando x al cateto PA′ e y al cateto A′M , el area de cuatro triangulos vale 2xy. Comoel lado B′A′ vale 1, tenemos:

x + y = 1 −√

x2 + y2, (1)

relacion que elevada al cuadrado y simplificada queda

2xy = 1 − 2√

x2 + y2, (2)

perox =

√x2 + y2 cosα, y =

√x2 + y2 senα,

y sustituyendo en (1) resulta

√x2 + y2(1 + cosα + senα) = 1 de donde

√x2 + y2 =

11 + senα + cosα

.

Sustituyendo en (2) y operando obtenemos

2xy = 1 − 21 + senα + cosα

=senα + cosα − 1senα + cosα + 1

.

Finalmente, para el area pedida obtenemos

S(α) = 1 − senα + cosα − 1senα + cosα + 1

=2

senα + cosα + 1con 0 ≤ α ≤ 90.

Page 637: COMPENDIUM OME - ToomatesEotvos de Hungría, que se iniciaron en 1894, precisamente durante la efervescencia de fin de siglo, consecuencia de la cual fue también el proceso iniciado

Segunda solucion

AB

C D

A′

B′

C′

D′

O

PM

El area pedida consta de 8 triangulos como el sombreado en la figura OPM . Tomandocomo base b = MP , la altura es constante (de trazos en la figura) y vale 1/2. En eltriangulo PA′M se tiene MA′ = b cosα, PA′ = b senα; pero BM = MA′ y PA = PA′, yademas

BM + MP + PA = 1 o bien b cosα + b + b senα = 1,

de dondeb =

1senα + cosα + 1

y el area pedida es:

S(α) = 812

12

1senα + cosα + 1

=2

senα + cosα + 1con 0 ≤ α ≤ 90.

Page 638: COMPENDIUM OME - ToomatesEotvos de Hungría, que se iniciaron en 1894, precisamente durante la efervescencia de fin de siglo, consecuencia de la cual fue también el proceso iniciado

E OME 34. Problema 2. Solucion

Sea n un numero que cumpla las condiciones del enunciado, y s la suma de sus cifras.Como 1000 ≤ n ≤ 9999 y n = s3, resulta

11 ≤ s ≤ 21. (1)

Si n = xyzt, tenemos1000x + 100y + 10z + t = s3

x + y + z + t = s

(2)

y restando queda999x + 99y + 9z = s3 − s (3)

cuyo segundo miembro ha de ser multiplo de 9 (por serlo el primero) y, habida cuenta deque

s3 − s = (s − 1)s(s + 1)

y por (1), solo hay tres valores de s3 − s que son multiplos de 9

16 · 17 · 18; 17 · 18 · 19 y 18 · 19 · 20.

Sustituimos en (3) y analizamos cada caso.

Primer caso.

999x + 99y + 9z = 16 · 17 · 18 implica que 111x + 11y + z = 544

y resulta inmediatamente x = 4, y = 9, z = 1, valores que llevados a (2) con s = 17, dant = 3 y finalmente n = 4913.

Segundo caso.

999x + 99y + 9z = 17 · 18 · 19 implica que 111x + 11y + z = 646,

de donde x = 5, y = 8, z = 3, valores que llevados a (2) con s = 18 dan t = 2 y finalmenten = 5832.

Tercer caso.

999x + 99y + 9z = 18 · 19 · 20 implica que 111x + 11y + z = 760,

y resulta x = 6; y = 8; z = 6, valores que llevados a (2) con s = 19 dan una contradiccion.Resumiendo, las unicas soluciones son n = 4913 y n = 5832.

Page 639: COMPENDIUM OME - ToomatesEotvos de Hungría, que se iniciaron en 1894, precisamente durante la efervescencia de fin de siglo, consecuencia de la cual fue también el proceso iniciado

E OME 34. Problema 3. Solucion

A

B D E C

M

Los triangulos ABC y ADC son semejantes pues tienen los tres angulos iguales ya queADC = BCM = BAC. La primera igualdad sale por ser AD y CM paralelas y la segundapor ser BCM un angulo semiinscrito. El angulo ACD es comun.Estableciendo la proporcionalidad entre sus lados, resulta:

CD

AC=

AC

BCo bien CD · BC = AC2. (1)

De modo analogo, los triangulos ABC y ABE son semejantes pues AEB = EBM = BAC

y el angulo ABE es comun.Estableciendo la proporcionalidad entre sus lados, resulta:

BE

AB=

AB

BCo bien BE · BC = AB2. (2)

Dividiendo las igualdades (1) y (2) se obtiene el resultado.

Page 640: COMPENDIUM OME - ToomatesEotvos de Hungría, que se iniciaron en 1894, precisamente durante la efervescencia de fin de siglo, consecuencia de la cual fue también el proceso iniciado

E OME 34. Problema 4. Solucion

Sean α, β, γ los tres angulos y supongamos α ≤ β ≤ γ.Si fuera γ ≥ π

2, tendrıa que ser α < π

4y entonces tanα no serıa entero.

Si tanα > 1, entonces α ≥ arctan 2 > arctan√

3 = π3 , imposible ya que α + β + γ = π.

Por tanto arctanα = 1 y β + γ = 3π4 , con lo que

tan(β + γ) = −1 =tanβ + tan γ

1 − tanβ tan γ,

relacion que operada se convierte en

(tanβ − 1)(tan γ − 1) = 2

de donde, por ser enteros positivos, se sigue tanβ = 2 y tan γ = 3.

Existe una visualizacion “sin palabras” de la solucion: arctan 1 + arctan 2 + arctan 3 = π:

αβ

γ

Page 641: COMPENDIUM OME - ToomatesEotvos de Hungría, que se iniciaron en 1894, precisamente durante la efervescencia de fin de siglo, consecuencia de la cual fue también el proceso iniciado

E OME 34. Problema 5. Solucion

Supongamos f(1) = b. Entonces, f(1 + b) = 2b, y como f es estrictamente creciente, setiene:

b = f(1) < f(1 + 1) < · · · < f(1 + b) = 2b = b + b

y resulta que f(1), f(2), . . . , f(1+ b) son b+1 numeros naturales distintos, el primero valeb y el ultimo 2b, y por tanto han de ser consecutivos.Resulta entonces

f(1) = b, f(2) = 1 + b, f(3) = 2 + b, . . . , f(1 + b) = b + b.

En general, y haciendo un razonamiento parecido para n > 1, si f(n) = c, sera f(n + c) =2c = c + c y resulta que

c = f(n) < f(n + 1) < · · · < f(n + c) = c + c

y los numeros f(n), f(n + 1), . . . , f(n + c) son consecutivos.Ası pues,

f(n) = n − 1 + f(1).

Page 642: COMPENDIUM OME - ToomatesEotvos de Hungría, que se iniciaron en 1894, precisamente durante la efervescencia de fin de siglo, consecuencia de la cual fue también el proceso iniciado

E OME 34. Problema 6. Solucion

Si n es multiplo de 4, evidentemente existe solucion, como indica la figura siguiente.

Recıprocamente, supongamos que el cuadrado n × n se puede recubrir. Como cada piezatiene 4 cuadrados, debera ser n2 multiplo de 4, y n multiplo de 2.El caso n = 4k ya sabemos que tiene solucion. Queda solo considerar el caso n = 4k + 2.Veamos que en ese caso la repuesta es negativa.Supongamos que fuera posible.Si pintamos cada cuadradito del cuadrado dado alternativamente de blanco y negro comoen un tablero de ajedrez, hay dos posibilidades para cada pieza.Sea a el numero de piezas del tipo de las de la izquierda y b el numero de piezas del tipode las de la derecha.

B N B N B N

B N

Tipo a Tipo b

Tenemos

a + b =(4k + 2)2

4= (2k + 1)2 = 4k2 + 4k + 1

luego a + b ha de ser impar.Por otra parte, como hay tantas casillas blancas como negras, se tiene: 3a + b = 3b + a, dedonde a = b, y a + b = 2a ha de ser par en contradiccion con lo anterior.

Page 643: COMPENDIUM OME - ToomatesEotvos de Hungría, que se iniciaron en 1894, precisamente durante la efervescencia de fin de siglo, consecuencia de la cual fue también el proceso iniciado

E OME 35. Problema 1. Solucion

B

AM

C

t

t′

x

y

Sean A(a, a2), B(b, b2) los dos puntos de la parabola. Las ecuaciones de t y t′ son

t : y = 2ax − a2; t′ : y = 2bx − b2

y su interseccion C es C(a + b

2, a b).

La mediana CM esta en la recta x =a + b

2, paralela al eje OY . Las coordenadas de M

son (a + b

2,a2 + b2

2).

Tenemos que m = CM =(a − b)2

2y si h es la altura del triangulo BMC resulta

h =|b− a|

2=

√m

2.

Poniendo [XY Z] para denotar el area del triangulo de vertices X , Y , Z, queda finalmente:

[ABC] = 2[BMC] = 212

m

√m

2=

√m3

2.

Page 644: COMPENDIUM OME - ToomatesEotvos de Hungría, que se iniciaron en 1894, precisamente durante la efervescencia de fin de siglo, consecuencia de la cual fue también el proceso iniciado

E OME 35. Problema 2. Solucion

Lo haremos por induccion sobre n. Para n = 2 basta tomar a1 = 3, a2 = 4 con 32+42 = 52.Supongamos por hipotesis de induccion que

a21 + a2

2 + · · ·+ a2n = k2.

Veamos que podemos encontrar un entero positivo an+1 tal que k2 + a2n+1 = p2.

En efecto, k2 = p2 − a2n+1 = (p+ an+1) (p− an−1). Si ponemos a = p+ an+1, b = p− an+1

tenemos p =a + b

2, an+1 =

a − b

2y k2 =

a + b

2· a − b

2.

La ultima expresion exige que a y b sean de la misma paridad. Distinguiremos dos casos

Primer caso. Si a y b son pares, entonces k2 = 4m . Tomando a = 2m, b = 2 queda

p = m + 1 =k2

4+ 1, an+1 = m − 1 =

k2

4− 1.

Segundo caso. Si a y b son impares, entonces k2 = 2m + 1. Tomando a = 2m + 1, b = 1queda

p = m + 1 =k2 − 1

2+ 1, an+1 = m =

k2 − 12

.

En ambos casos hemos encontrado el entero an+1 que cumple las condiciones del enunciado.

Page 645: COMPENDIUM OME - ToomatesEotvos de Hungría, que se iniciaron en 1894, precisamente durante la efervescencia de fin de siglo, consecuencia de la cual fue también el proceso iniciado

E OME 35. Problema 3. Solucion

En el tablero, hay casillas de tres tipos: vertice, lado, o interiores. Cada una de ellas tiene,respectivamente, dos, cuatro o seis casillas vecinas.Si pudieramos retirar todas las fichas del tablero, habrıa un momento en que quedarıasobre el una unica ficha negra. Esa ficha era inicialmente blanca, luego ha tenido quecambiar de color un numero impar de veces. Pero esto es imposible, porque una ficha sevuelve cada vez que se retira una ficha vecina, y ninguna ficha tiene un numero impar decasillas vecinas.

Page 646: COMPENDIUM OME - ToomatesEotvos de Hungría, que se iniciaron en 1894, precisamente durante la efervescencia de fin de siglo, consecuencia de la cual fue también el proceso iniciado

E OME 35. Problema 4. Solucion

Hay 27 posibles resultados para la suma de dıgitos (de 1 a 27). Las sumas 1 y 27 solose puede obtener de un modo (100 y 999) En el caso mas desfavorable al sacar 52 (27 +25) tarjetas todas repetiran suma dos veces y en la siguiente (extraccion 53) una de ellasaparecera por tercera vez.Por tanto el numero pedido es 27 + 25 + 1 = 53.

Page 647: COMPENDIUM OME - ToomatesEotvos de Hungría, que se iniciaron en 1894, precisamente durante la efervescencia de fin de siglo, consecuencia de la cual fue también el proceso iniciado

E OME 35. Problema 5. Solucion

Primera solucion

i) Es sabido que uniendo G con cada vertice, se forman tres triangulos: BGC de base ay altura ga , AGC de base b y altura gb y AGB de base c y altura gc. Los tres tienen lamisma area que es un tercio del area total del triangulo.Por tanto, llamando S al area de ABC

a · ga = b · gb = c · gc =2S

3. (1)

Por otra parte, sabemos que r (a + b + c) = 2S (basta unir el incentro con los tres verticesy quedan tres triangulos de bases a, b, c y altura comun r).Sustituyendo 2S en (1), y despejando queda:

ga =r

3a + b + c

a, gb =

r

3a + b + c

b, gc =

r

3a + b + c

c(2)

y por la desigualdad triangular b + c ≥ a, resultaa + b + c

a= 1 +

b + c

a≥ 2, de donde

ga ≥ 2r

3y de modo analogo para gb y gc .

ii) De (1), haciendo los inversos y sumando resulta:

1ga

+1gb

+1gc

=3a

r(a + b + c)+

3b

r(a + b + c)+

3c

r(a + b + c)=

3r.

Finalmente, aplicando la desigualdad entre las medias aritmetica y armonica, salega + gb + gc

3≥ 3

1ga

+ 1gb

+ 1gc

=33r

= r que equivale aga + gb + gc

r≥ 3.

Observacion. Sumando las tres desigualdades de i) solo obtenemosga + gb + gc

r≥ 2.

A

B C

ga

gbgc

G

a

bc

Primera solucion

A

B C

ga

G

ha

HA GA MA

Segunda solucion

Page 648: COMPENDIUM OME - ToomatesEotvos de Hungría, que se iniciaron en 1894, precisamente durante la efervescencia de fin de siglo, consecuencia de la cual fue también el proceso iniciado

Segunda solucion

i) Consideremos los puntos MA, HA, GA, indicados en la figura.Denotaremos por hA la altura correspondiente a A, p el semiperımetro y S el area de ABC.Los triangulos AMAHA y GMAGA son semejantes, siendo 3 la razon de semejanza (pro-piedad del baricentro sobre cada mediana) y es

hA = 3gA. (3)

Por la desigualdad triangular tenemos la cadena de equivalencias

b + c ≥ a ⇐⇒ 2p ≥ 2a ⇐⇒ p ≥ a ⇐⇒ a

p≤ 1

y multiplicando por hA y teniendo en cuenta (3) queda:

gA ≥ ahA

3po bien gA ≥ 2S

3p

y finalmente, como S = p r, resulta gA ≥ 23r. Analogamente obtendrıamos las correspon-

dientes desigualdades para gB y gC .

ii) Usaremos la desigualdad x +1x

≥ 2, valida para x > 0, que se deduce de la obvia

(x − 1)2 ≥ 0. Tenemos entonces(

a

b+

b

a

)+

(b

c+

c

b

)+

( c

a+

a

c

)≥ 6.

Sumando, ordenando y operando resulta:

1 +a

b+

a

c+

b

a+ 1 +

b

c+

c

a+

c

b+ 1 ≥ 9

que equivale a

a

(1a

+1b

+1c

)+ b

(1a

+1b

+1c

)+ c

(1a

+1b

+1c

)≥ 9

y sacando factor comun, dividiendo por 3 y poniendo 2p = a + b + c, queda

2p

3a+

2p

3b+

2p

3c≥ 3. (4)

Por otra parte, como 3ga = hA, 3gb = hB, 3gc = hC , resulta 2S = 3gaa = 3gbb = 3gcc.Despejando 3a, 3b y 3c y sustituyendo en (4), queda:

(ga + gb + gc)p

S≥ 3.

Finalmente usando de nuevo S = p r, resultaga + gb + gc

r≥ 3.

Page 649: COMPENDIUM OME - ToomatesEotvos de Hungría, que se iniciaron en 1894, precisamente durante la efervescencia de fin de siglo, consecuencia de la cual fue también el proceso iniciado

E OME 35. Problema 6. Solucion

Supongamos que hay x rectas en la primera familia, y en la segunda y z en la tercera. Lasx rectas de la primera familia determinan x + 1 regiones. La primera recta de la segundafamilia determina en el plano 2(x + 1) regiones, las dos primeras 3(x + 1), etc., hasta lasy-esima, que determina (y + 1)(x + 1) regiones.La primera recta de la tercera familia es cortada por las x+y rectas existentes en x+y +1partes y cada una de estas partes divide en dos a cada region existente de modo que elnumero de regiones se incrementa en x + y + 1 regiones. Cada recta de la tercera familiaaumenta las regiones existentes en la misma cantidad; luego el numero total N de regionesvale

N = (x + 1)(y + 1) + z(x + y + 1) = x + y + z + xy + xz + yz + 1 = n + m + 1

con n = x + y + z y m = xy + xz + yz.Tenemos

m = x2 + y2 + z2 − 12((y − z)2 + (z − x)2 + (x − y)2

)≤ x2 + y2 + z2.

Entonces

n2 = x2 + y2 + z2 + 2m ≥ 3m que equivale a m ≤ n2

3y N = n + m + 1 ≤ n +

n2

3+ 1.

Para n = 76 es n2 + n2/3 + 1 > 2002. Ası, si n = 76 = x + y + z con x = 26, y = 25,z = 25, resulta m = 1925 y N = 2002.

Page 650: COMPENDIUM OME - ToomatesEotvos de Hungría, que se iniciaron en 1894, precisamente durante la efervescencia de fin de siglo, consecuencia de la cual fue también el proceso iniciado

E OME 36. Problema 1. Solucion

Las raıces comunes a ambos polinomios seran raıces de la diferencia

P (x) − Q(x) = (a − c)x3 + (c − a)x.

Resolvemos la ecuacion P (x) − Q(x) = 0, sacando primero x factor comun

x((a − c)x2 + (c − a)

)= 0.

Las tres raıces son 0, 1 y −1, y entre ellas tienen que estar las raıces comunes.Como 0 no es raız ni de P (x) ni de Q(x), las dos raıces comunes tienen que ser 1 y −1.Sustituyendo estos valores en P (x) y Q(x) obtenemos el sistema

2 + a + b + c = 02 − a + b− c = 0

que nos da las condicionesb = −2a = −c

y los polinomios quedan en la forma:

P (x) = x4 + ax3 − 2x2 − ax + 1Q(x) = x4 − ax3 − 2x2 + ax + 1.

Para resolver las ecuaciones P (x) = 0, Q(x) = 0, separamos por Ruffini las raıces conocida1 y −1 y quedan las ecuaciones en la forma

P (x) = (x + 1)(x − 1)(x2 + ax − 1) = 0Q(x) = (x + 1)(x − 1)(x2 − ax − 1) = 0.

Resolviendo las ecuaciones de segundo grado queda finalmente:Soluciones de P (x) = 0,

x = 1, x = −1, x =−a +

√a2 + 4

2, x =

−a −√

a2 + 42

.

Soluciones de Q(x) = 0,

x = 1, x = −1, x =a +

√a2 + 42

, x =a −

√a2 + 42

.

Page 651: COMPENDIUM OME - ToomatesEotvos de Hungría, que se iniciaron en 1894, precisamente durante la efervescencia de fin de siglo, consecuencia de la cual fue también el proceso iniciado

E OME 36. Problema 2. Solucion

Definamos un sistema de coordenadas con origen enA y unidad el lado de un cuadrado.Como P y Q recorren caminos de longitud mınima,P solo puede ir a la derecha o arriba y Q a laizquierda o abajo. Todos los caminos tienen lon-gitud 7, y P y Q solo se podran encontrar entre eltercero y el cuarto movimiento. En la figura se hanmarcado con puntos blancos todas las posibles posi-ciones de P tras el tercer movimiento y con puntosnegros las de Q.

A

B

Caso 1. P llega a (0, 3).

La probabilidad de que P llegue a (0, 3) es 12

12

12

= 18. Solo se puede cruzar con Q si este

esta en (1, 3), lo que sucede tambien con probabilidad 12

12

12

= 18.

P esta obligado a pasar a (1, 3) pero Q pasa a (0, 3) con probabilidad 12 . La probabilidad

de que se crucen entre (0, 3) y (1, 3) es 18

18

12

= 127 .

Caso 2. P llega a (1, 2).

La probabilidad de que P llegue a (1, 2) es 3(

12

)3 = 38

ya que hay tres modos de llegar(1, 2). Solo se puede cruzar con Q si este esta en (1, 3) o en (2, 2). Distingamos amboscasos:a) Q llega a (1, 3) con probabilidad 1

8, y entonces se cruzaran entre (1, 2) y (1, 3) si P se

mueve hacia (1, 3) y Q hacia (1, 2), ambos movimientos con probabilidad 12. La probabili-

dad de cruzarse es 38

18

12

12 = 3

28 .b) Q llega a (2, 2) con probabilidad 3

8 , y entonces se cruzaran entre (1, 2) y (2, 2) si P semueve hacia (2, 2) y Q hacia (1, 2), ambos movimientos con probabilidad 1

2. La probabili-

dad de cruzarse es 38

38

12

12

= 928 .

Caso 3. P llega a (2, 1).

Procediendo de modo analogo, la probabilidad de cruzarse entre los puntos (2, 1) y (2, 2)es 9

28 y la de cruzarse entre (2, 1) y (3, 1) es 928 .

Caso 4. P llega a (3, 0). La probabilidad de cruzarse entre (3, 0) y (3, 1) es 328 y la de

cruzarse entre (3, 0) y (4, 0) es 127 .

La probabilidad pedida es la suma de todos los casos y resulta ser

127

+328

+928

+928

+928

+328

+127

=37256

Page 652: COMPENDIUM OME - ToomatesEotvos de Hungría, que se iniciaron en 1894, precisamente durante la efervescencia de fin de siglo, consecuencia de la cual fue también el proceso iniciado

E OME 36. Problema 3. Solucion

Pr

B Qr

M2

M1

M

O

C2C1

εε γ

Sea O el punto medio del segmento M1M2. Vamos a demostrar que todas las mediatricesde los segmentos PrQr pasan por el simetrico de B respecto de O.Sean ε = PrBM1, γ = M1BM2. Entonces:

M2BQr = 180 − (γ + ε)

y como el triangulo M2BQr es isosceles,

BM2Qr = 180 − 2 M2BQr = −180 + 2 (γ + ε)

y por tanto,

MM2Qr = 180 − γ + BM2Qr = 180 − γ − 180 + 2 (γ + ε) = γ + 2 ε.

De modo analogo, por ser el triangulo PrM1B isosceles, se tiene

PrM1B = 180 − 2 ε

y

PrM1M = 360 − (PrM1B + 180 − γ) = 360 − 180 + 2 ε − 180 + γ = 2 ε + γ.

Resulta que para cualquier posicion de la recta variable, los triangulos MM1Pr y MM2Qr

son iguales y por tanto MPr = MQr y M esta en la mediatriz de PrQr.Como M no depende de la recta variable queda probada la propiedad del enunciado.

Page 653: COMPENDIUM OME - ToomatesEotvos de Hungría, que se iniciaron en 1894, precisamente durante la efervescencia de fin de siglo, consecuencia de la cual fue también el proceso iniciado

E OME 36. Problema 4. Solucion

De la condicion a) sale z = E

(N

3

)= 111 · k para todo k ∈ N, 1 ≤ k ≤ 9.

De la condicion b) sale z = E

(N

3

)= 1 + 2 + 3 + · · · + n, o bien, z =

n(n + 1)2

, que

equivale a n2 + n − 2z = 0, es decir, n =−1 +

√1 + 8z

2. (La otra raız es negativa).

Juntando las dos condiciones, queda:

n =−1 +

√1 + 8 · 111 · k

2

Como n es natural, el radicando ha ser cuadrado perfecto, lo que ocurre solo para k = 6,que sustituido en la expresion anterior resulta n = 36.Recuperando la condicion a) sale

z = E

(N

3

)= 111 · 6 = 666

lo que da

667 >N

3≥ 666 o bien 2001 > N ≥ 1998.

En consecuancia, el mayor N que cumple a) y b) es N = 2000.

Page 654: COMPENDIUM OME - ToomatesEotvos de Hungría, que se iniciaron en 1894, precisamente durante la efervescencia de fin de siglo, consecuencia de la cual fue también el proceso iniciado

E OME 36. Problema 5. Solucion

Vamos a demostrarlo por reduccion al absurdo. Supongamos que distribuimos 4 puntosen el cuadrado de manera que cada una de las seis distancias sea mayor que 1. Entonceshay dos posibilidades:1) Los cuatro puntos forman un cuadrilatero convexo.2) Los cuatro puntos forman un cuadrilatero no convexo.

P1

P2

P3

P4

α

β

γ

δ

Veamos ambos casos:Caso 1) Sean α, β, γ, δ los angulos del cuadrilatero convexo. Sabemos que α + β + γ + δ =360. Ademas cualquier pareja de puntos del interior (o frontera) del cuadrado estan auna distancia d ≤

√2 ya que el diametro de dicho cuadrado es

√2.

De la condicion α +β + γ + δ = 360, se deduce que necesariamente uno de los angulos hade ser mayor o igual que 90, digamos por ejemplo α ≥ 90.Tenemos (ver figura):

PiPj > 1, i = j

luego

P1P32

= P1P22

+ P2P32 − 2P1P2 · P2P3 cosα

y como el cuadrilatero es convexo, 90 ≤ α ≤ 180 y por tanto cosα ≤ 0 y en consecuencia:

P1P32 ≥ P1P2

2+ P2P3

2> 2 de donde sale P1P3 >

√2.

lo que es imposible.Caso 2) Si se forma un cuadrilatero no convexo podemos elegir tres de los cuatro puntosformando un triangulo de modo que el cuarto punto sea interior. Supongamos que el punto

Page 655: COMPENDIUM OME - ToomatesEotvos de Hungría, que se iniciaron en 1894, precisamente durante la efervescencia de fin de siglo, consecuencia de la cual fue también el proceso iniciado

interior es P4.

C

√2

Cada lado de dicho triangulo es menor o igual que√

2 (diametro del cuadrado) y por tanto

estara contenido en un triangulo equilatero de lado√

2, y circunradio√

2√

32

23 =

√23 < 1.

Si su centro es C, el punto P4 estara en el interior de uno de los tres triangulos que resultande unir C con cada vertice y la distancia de P4 a uno de los vertices sera menor o igual

que el circunradio, es decir menor que√

23 y por tanto menor que 1. Hemos encontrado

pues un par de puntos a distancia menor o igual que 1.Por ultimo si tres puntos estan alineados se reduce al caso b) y si los cuatro puntos estanalineados llamando x1, x2, x3 a las distancias entre puntos consecutivos, tenemos

x1 + x2 + x3 ≤√

2

y por el principio del palomar, uno de ellos, digamos x1, cumple x1 ≤√

23

< 1.

Page 656: COMPENDIUM OME - ToomatesEotvos de Hungría, que se iniciaron en 1894, precisamente durante la efervescencia de fin de siglo, consecuencia de la cual fue también el proceso iniciado

E OME 36. Problema 6. Solucion

Supongamos que exista f : N → N tal que f(f(n)

)= n + 1.

Se tiene que f(0) = a ∈ N . Por el enunciado

f(f(0)

)= 1; f

(f(0)

)= f(a) = 1,

y del mismo modo,

f(1) = a + 1, f(a + 1) = 2, f(2) = a + 2, . . .

Supongamos que f(n−1) = a+n−1; entonces f(a+n−1) = a+n. Luego hemos probadopor induccion que

f(f(n)

)= f(a + n) = 2a + n.

Entonces se tiene que cumplir,2a + n = n + 1

lo que implica

a =12

/∈ N.

Hemos llegado a una contradiccion y la condicion supuesta es falsa con lo que quedademostrado la inexistencia de la funcion f .

Page 657: COMPENDIUM OME - ToomatesEotvos de Hungría, que se iniciaron en 1894, precisamente durante la efervescencia de fin de siglo, consecuencia de la cual fue también el proceso iniciado

E OME 37. Problema 1. Solucion

Supongamos primero que exista el polinomio P que cumple las condiciones requeridas. Seax − a = h o x = a + h. Entonces

P (a − h) = b − h Q(h2)P (a + h) = b + h Q(h2)

yP (a − h) + P (a + h)

2= b, para todo h ∈ R.

Esto indica que la grafica de P es simetrica respecto del punto A(a, b).Sea x = a + h, P (x) = P (a + h) = R(h).

Como P (a−h) = R(−h), la condicionP (a − h) + P (a + h)

2= b es equivalente a R(−h)+

R(h) = 2h. Para R(h) = a0 +a1h+ · · ·+anhn, la condicion anterior se escribe de la forma

a0 + a1h + · · · + anhn + a0 − a1 + a2h2 − · · · + (−1)nanhn = 2b

es decir a0 + a2h2 + · · ·+ amhm = b, para cada h ∈ R, con m = n si n es par o m = n− 1

si n es impar.Deducimos que a2 = a4 = · · · = am = 0, a0 = b.Por tanto ahora se tiene que los terminos de R son el independiente y los que contienenpotencias impares de h

R(h) = b + a1h + a3h3 + · · ·

y ası existe un polinomio Q tal que R(h) = b + h Q(h2), para algun polinomio Q.Por ultimo, volvemos a la variable x: P (x) = R(h) = b + (x − a) Q

((x − a)2

).

Page 658: COMPENDIUM OME - ToomatesEotvos de Hungría, que se iniciaron en 1894, precisamente durante la efervescencia de fin de siglo, consecuencia de la cual fue también el proceso iniciado

E OME 37. Problema 2. Solucion

BP

Q

A

R

C

Los triangulos ABC y PBQ son semejantes pues tienen un angulo igual ABC = PBQ ylos lados que lo forman proporcionales

c

a=

BP

BQ.

De modo analogo, ABC es semejante a APR, y por tanto PBQ y APR son semejantes(y al ser PB = PA son iguales). En particular, ARP = ACB y BQP = ACB

Llamando α = BAP = ABP , resulta:

QPR = 360 − (180 − 2α) − (A + B) = 180 + 2α − (180 − ACB) = 2α + ACB,

QCR = ACB + 2α,

PRC = 180 − 2α − ARP = 180 − 2α − ACB,

PQC = 180 − 2α − BQP = 180 − 2α − ACB.

Las cuatro igualdades establecen que los dos pares de angulos opuestos del cuadrilateroPQCR son iguales y es un paralelogramo.La alineacion es un caso particular y se producira cuando ACB + 2α = 180, es decircuando

α =180 − ACB

2.

Page 659: COMPENDIUM OME - ToomatesEotvos de Hungría, que se iniciaron en 1894, precisamente durante la efervescencia de fin de siglo, consecuencia de la cual fue también el proceso iniciado

E OME 37. Problema 3. Solucion

Supongamos que 0 < a1 ≤ a2 ≤ a3 ≤ a4 ≤ a5. Si ningun triangulo es acutangulo,tendrıamos:

a21 + a2

2 ≤ a23 (1)

a22 + a2

3 ≤ a24 (2)

a23 + a2

4 ≤ a25 (3)

Pero por la desigualdad triangular,

a5 < a1 + a2, luego a25 < a2

1 + a22 + 2a1a2. (4)

Sumando las desigualdades (1),(2),(3) y (4) tenemos

a21 + 2a2

2 + 2a23 + a2

4 + a25 < a2

3 + a24 + a2

5 + a21 + a2

2 + 2a1a2,

es decir,a22 + a2

3 < 2a1a2.

Como a2 ≤ a3, resulta 2a22 ≤ a2

2 + a23 < 2a1a2, y por tanto a2 < a1, en contradiccion con

la ordenacion inicial.

Page 660: COMPENDIUM OME - ToomatesEotvos de Hungría, que se iniciaron en 1894, precisamente durante la efervescencia de fin de siglo, consecuencia de la cual fue también el proceso iniciado

E OME 37. Problema 4. Solucion

Consideremos la distribucion

a b cd e fg h i

Resulta

S = abc + def + ghi + adg + beh + cfi == 100(a + c + b + a + d + g) + 10(d + e + f + b + e + h) + (g + h + i + c + f + i) =

= 200a + 110b + 101c + 110d + 20e + 11f + 101g + 11h + 2i.

Calculando modulo 9 tenemos

S = 2(a + b + · · · + h + i) = 2 · 45 = 0.

Como 2001 no es multiplo de 9, no habra ninguna distribucion para la que la suma indicadatome el valor 2001.

Page 661: COMPENDIUM OME - ToomatesEotvos de Hungría, que se iniciaron en 1894, precisamente durante la efervescencia de fin de siglo, consecuencia de la cual fue también el proceso iniciado

E OME 37. Problema 5. Solucion

A O B

D

C

ab

α β

Sea O el centro de la semicircunferencia. Pongamos a = BC; b = AD; p = CD; 2α =BOC; 2β = AOD; 2γ = COD.La condicion necesaria y suficiente para que ABCD admita una circunferencia inscrita es

p + 2 = a + b. (1)

Como 2α + 2β + 2γ = 180, entonces β = 90 − (α + γ) y ademas

a = 2 sen α; p = 2 sen γ; b = 2 sen β = 2 cos(α + γ) = 2 cos α cos γ − 2 sen α sen γ.

Vamos a expresar la condicion (1) en funcion del angulo α y el dato p que determina porcompleto el cuadrilatero.

cos γ =

√1 − p2

4=

√4 − p2

2,

de dondeb =

√4 − p2 cos α − p sen α

sustituyendo en (1), queda

p + 2 = 2 sen α +√

4 − p2 cos α − p sen α

o lo que es lo mismo √4 − p2 cos α + (2 − p) sen α = p + 2. (2)

Por tanto, existira circunferencia inscrita para los valores de p que hagan compatible laecuacion (2) en la incognita α.Puede expresarse el seno en funcion del coseno y estudiar el discriminante de la ecuacionde segundo grado que se obtiene, pero es mas rapido interpretar la ecuacion (2) como el

Page 662: COMPENDIUM OME - ToomatesEotvos de Hungría, que se iniciaron en 1894, precisamente durante la efervescencia de fin de siglo, consecuencia de la cual fue también el proceso iniciado

producto escalar de los vectores u = (cos α, sen α) de modulo 1 y v = (√

4 − p2, 2− p). Lacondicion (2) queda:

|v| cos δ = p + 2 (3)

siendo δ el angulo formado por los vectores u y v.Para que (3) sea compatible debe cumplirse p + 2 ≤ |v| =

√4 − p2 + (2 − p)2, y elevando

al cuadrado y operando quedap2 + 8p − 4 ≤ 0.

Las raıces de la ecuacion son p = ±2√

5 − 4. Como p es positivo la condicion final es:

0 ≤ p ≤ 2√

5 − 4.

Page 663: COMPENDIUM OME - ToomatesEotvos de Hungría, que se iniciaron en 1894, precisamente durante la efervescencia de fin de siglo, consecuencia de la cual fue también el proceso iniciado

E OME 37. Problema 6. Solucion

Para cada numero natural n definimos f(n) como la suma de las cifras de la expresion de nescrito en base 2. Esta claro que esta funcion f cumple las condiciones a) y b). Ademas,es la unica funcion que las cumple, porque el valor de f(n) viene determinado por lascondiciones a) y b).Probamos esa afirmacion por induccion sobre n. Si n = 1 o n = 2s, entonces f(n) = 1.Supongamos n > 1, n = 2s y que es conocido f(m) para todo m < n; se puede escribirn = 2s + m con m < 2s tomando 2s la mayor potencia de 2 que es menor que n; entoncesf(n) = f(m) + 1.Ahora, es facil resolver las dos cuestiones que nos plantean:En el primer caso, se trata de ver cuantos unos puede tener como maximo un numeromenor o igual que 2001 escrito en base 2. Ese numero, escrito en base 2, es, obviamente,1111111111, que corresponde a n = 1023 = 210 − 1. Es f(n) = 10.En el segundo caso, razonando de manera analoga, se observa que la respuesta es

n = 22001 − 1.

Page 664: COMPENDIUM OME - ToomatesEotvos de Hungría, que se iniciaron en 1894, precisamente durante la efervescencia de fin de siglo, consecuencia de la cual fue también el proceso iniciado

E OME 38. Problema 1. Solucion

La ecuacion funcional dada

P (x2 − y2) = P (x + y)P (x − y) (*)

es equivalente a la ecuacion funcional

P (uv) = P (u)P (v) (**)

con el cambio de variables u = x + y y v = x − y, para todo u, v ∈ R.Poniendo u = v = 0 en (**) se obtiene P (0) = (P (0))2, de donde P (0) = 1 o P (0) = 0. SiP (0) = 1, haciendo v = 0 en (**) se deduce que P (0) = P (u)P (0) para todo u ∈ R, es decirP (u) ≡ 1. Sea ahora P (0) = 0. Entonces P (u) = uQ(u), siendo Q(u) un polinomio degrado una unidad inferior al grado de P (u). Facilmente se comprueba que Q(u) satisfacela ecuacion funcional (**). Por tanto P (u) = un con n ∈ N.Recıprocamente, se comprueba sin dificultad que P (x) ≡ 1 y P (x) = xn con n ∈ N

satisfacen la ecuacion funcional inicial (*).Ademas esta la solucion trivial P (x) ≡ 0.

Page 665: COMPENDIUM OME - ToomatesEotvos de Hungría, que se iniciaron en 1894, precisamente durante la efervescencia de fin de siglo, consecuencia de la cual fue también el proceso iniciado

E OME 38. Problema 2. Solucion

Para resolver la primera cuestion tenemos,

BA′ = c cosB, tanHBA′ = cotC =HA′

BA′ , AA′ = c senB.

Deducimosk =

AA′

HA′ =c senB

c cosB cotC, de donde tanB · tanC = k. (1)

Resolvamos la segunda cuestion. Poniendo a = BC, tomando unos ejes con origen en elpunto medio de BC y eje OX sobre el lado BC, resulta B(−a

2 , 0), C(a2 , 0), y llamando

A(x, y), la condicion (1) se escribe

ya2− x

· ya2

+ x= k que tambien es y2 = k

(a2

4− x2

)

que, una vez operada resulta:x2

a2

4

+y2

ka2

4

= 1, (2)

ecuacion de una elipse. Podremos distinguir dos casos.Si k < 1, elipse con eje mayor sobre OX , semidistancia focal c1 =

a

2√

1 − k y semieje

mayor a1 =a

2

BF ′ O A′ F

C

A

H

BO A′ C

A

H

F

F ′

Si k > 1, se trata de una elipse con eje mayor sobre OY , semidistancia focal c1 =a

2√

k − 1

y semieje mayor a1 =a

2.

Page 666: COMPENDIUM OME - ToomatesEotvos de Hungría, que se iniciaron en 1894, precisamente durante la efervescencia de fin de siglo, consecuencia de la cual fue también el proceso iniciado

E OME 38. Problema 3. Solucion

Dado cualquier natural n, consideramos su representacion binaria,

n = ak2k + ak−12k−1 + · · ·+ a12 + a0 = ak . . . a1a0 (2,

donde aj = 0 o 1.

Probaremos por induccion sobre k que g(n) =k∑

j=0

aj.

Para k = 0 es cierto: g(1 (2) = g(1) = 1.Supuesto cierto para k, hay dos casos para k + 1:

g(ak . . . a1a00(2

)= g

(2 · ak . . . a1a0 (2

)=

k∑j=0

aj ,

g(ak . . . a1a01 (2

)= g

(1 + 2 · ak . . . a1a0 (2

)= 1 +

k∑j=0

aj

donde se han aplicado las propiedades de g y la hipotesis inductiva.Entonces g(n) es el numero de unos de n escrito en base 2.Como 211 = 2048 > 2002 > 1024 = 210, resulta M = 10.Hay cinco valores de n que hacen g(n) = 10: 1023, 1535, 1791, 1919 y 1983.

Page 667: COMPENDIUM OME - ToomatesEotvos de Hungría, que se iniciaron en 1894, precisamente durante la efervescencia de fin de siglo, consecuencia de la cual fue también el proceso iniciado

E OME 38. Problema 4. Solucion

Tenemos las expresiones (en base 10)

n = abc = c + 10b + 100a

m = cba = 100c + 10b + a

que, sustituidas en 2m + S = n nos da

200c + 20b + 2a + (a + b + c) = 100a + 10b + c,

es decir200c + 11b − 97a = 0.

Por lo tanto, 200c − 97a es multiplo de 11.Calculando modulo 11 la expresion anterior sale 2(c + a) ≡ 0 , y como mcd (2, 11) = 1,resulta que a + c es congruente con 0 modulo 11.Calculando modulo 9: 2(c + a + b) ≡ 0, y c + a + b es congruente con 0.Por la primera congruencia, c + a = 0, o bien c + a = 11.Si c + a = 0, entonces a = c = 0 y no hay solucion por ser numeros de tres cifras. Si c + a= 11, entonces b = 7 y por lo tanto, 200c − 97a es multiplo de 7.Trabajando modulo 7 queda 4c + a es congruente con 0 modulo 7, es decir;

4c + a = 0, 7, 14, 21, 28, 35, 42.

Como a + c = 11, tenemos que 3c debe tomar uno de los valores −11, −4, 3, 10, 17, 24, o31 y ser multiplo de 3. Luego c = 1 o c = 8.Si c = 1, entonces a = 10, lo que es imposible.Si c = 8, debe ser a = 3. Pero n = 378 no es solucion y, en conclusion, no existen numeroscon las condiciones pedidas.

Page 668: COMPENDIUM OME - ToomatesEotvos de Hungría, que se iniciaron en 1894, precisamente durante la efervescencia de fin de siglo, consecuencia de la cual fue también el proceso iniciado

E OME 38. Problema 5. Solucion

Cada segmento determina dos vectores de igual modulo y sentido opuesto.Consideramos los 2·2002 = 4004 vectores ası obtenidos y los ordenamos por sus direccionesentre 0 y 2π respecto de un sistema de referencia ortonormal arbitrario.Construimos ahora un polıgono convexo de 4004 lados “uniendo” los vectores uno a con-tinuacion del otro, a partir de uno cualquiera dado.Claramente el perımetro de este polıgono es 2. Ademas es un polıgono centrado y simetrico,respecto de un punto O.Tomamos entonces uno de los lados mas proximos a O; sea d el segmento perpendiculara ese lado y a su opuesto que pasa por el centro O. La proyeccion del polıgono sobre larecta que contiene a este segmento es d y por tanto la suma de las proyecciones sobre la

recta anterior es tambien d. Por otra parte la circunferencia de centro O y radiod

2esta

totalmente contenida en el interior del polıgono y entonces su circunferencia es menor queel perımetro del polıgono.

Es decir: dπ < 2 y d <2π

<23.

Falta considerar el caso trivial de que todos los segmentos tengan la misma direccion encuyo caso no hay polıgono. Pero podemos tomar la recta perpendicular a la direccioncomun sale d = 0.

Page 669: COMPENDIUM OME - ToomatesEotvos de Hungría, que se iniciaron en 1894, precisamente durante la efervescencia de fin de siglo, consecuencia de la cual fue también el proceso iniciado

E OME 38. Problema 6. Solucion

Debido a que el numero de lados del polıgono H deja de resto 1 al dividirse entre seis, cadadiagonal y cada lado del polıgono pertenece solo (exactamente) a tres triangulos isoscelesdistintos.Denotamos por AA, AR y RR los numeros de segmentos que son lados y diagonales cuyosextremos respectivamente estan coloreados ambos de azul, de azul y de rojo o ambos derojo. Analogamente denotamos por AAA, AAR, ARR y RRR el numero de triangulosisosceles cuyos vertices son los tres azules, dos azules y uno rojo, uno azul y dos rojos olos tres rojos y ninguno azul, respectivamente.Entonces 3 × AA = 3 × AAA + AAR, porque cada diagonal o lado de H pertenece a trestriangulos isosceles y los triangulos isosceles con tres vertices azules tienen tres lados consus dos extremos azules. Los triangulos isosceles con dos vertices azules tienen solo un ladocon sus extremos de color azul y los triangulos isosceles con menos de dos vertices azulesno tiene ningun lado con los extremos del mismo color azul.Analogamente establecemos 3×RA = 2×AAR + 2×ARR y 3×RR = ARR + 3×RRR.Las tres relaciones obtenidas conducen a

AAA + RRR = RR + AA − 12× RA =

12× R × (R − 1) +

12× A × (A − 1) − 1

2× R × A,

donde A es el numero de vertices azules, A = 6n + 1 − R. Esto completa la prueba.Observemos que el resultado es tambien cierto si el polıgono H tiene 6n + 5 lados.

Page 670: COMPENDIUM OME - ToomatesEotvos de Hungría, que se iniciaron en 1894, precisamente durante la efervescencia de fin de siglo, consecuencia de la cual fue también el proceso iniciado

E OME 39. Problema 1. Solucion

Sea ai el numero compuesto por i nueves ai =

i︷ ︸︸ ︷99 · · ·9.

Supongamos que existe p tal que p ∣∣ai ∀i ∈ N, para probar por contradiccion el enunciado.

Considerense en dicho caso los numeros a1, a2, . . . , ap. En este conjunto sabemos porhipotesis que no hay ningun ai ≡ 0 (p). Por tanto al haber p numeros y solo p − 1 restosposibles modulo p, se sabe que existen m, n tales que am − an ≡ 0 (p). Suponemos sinperdida de generalidad que m > n y tenemos

p∣∣(am − an) =

m︷ ︸︸ ︷99 . . .9−

n︷ ︸︸ ︷99 . . .9 =

m−n︷ ︸︸ ︷99 . . .9

n︷ ︸︸ ︷00 . . .0 = am−n · 10n.

Como p = 2 y p = 5 resulta que p ∣∣ 10n = 2n · 5n, es decir que p

∣∣am−n y como am−n

pertenece al conjunto escogido por ser m − n < n y m − n ≥ 1, se ha llegado a unacontradiccion. Por consiguiente

∀p ∃ ai tal que p∣∣ai

y el enunciado queda probado.

Page 671: COMPENDIUM OME - ToomatesEotvos de Hungría, que se iniciaron en 1894, precisamente durante la efervescencia de fin de siglo, consecuencia de la cual fue también el proceso iniciado

E OME 39. Problema 2. Solucion

Como M es finito, necesariamente estara acotado.Pongamos M ⊂ [x, y], con x = min(M) e y = max(M).Supongamos en primer lugar que x ≤ 0. Tenemos x ≤ 0 ⇒ 2x ≤ x ⇒ 2x − k2 < x paracualquier numero k ∈ M . Esto contradice que x sea el mınimo de M . Por tanto x debeser mayor que 0 y se tiene 0 < x < y.En cualquier caso debe ser

x ≤ 2x − y2 ≤ y (1)

y ademasx ≤ 2y − y2 ≤ y. (2)

De (1) se desprende que : x ≤ 2x − y2 o bien que 0 ≤ x − y2, es decir y2 ≤ x < y; estosolo se cumple si y ∈ (0, 1).De (2) obtenemos que 2y − y2 ≤ y, es decir, y− y2 ≤ 0, o bien y ≤ y2; y esto solo es ciertosi y ∈ [1, +∞).Como (1) y (2) deben cumplirse a la vez, no existe ningun y ∈ R que pueda ser maximode M por lo que M no puede estar acotado y no puede ser finito.

Page 672: COMPENDIUM OME - ToomatesEotvos de Hungría, que se iniciaron en 1894, precisamente durante la efervescencia de fin de siglo, consecuencia de la cual fue también el proceso iniciado

E OME 39. Problema 3. Solucion

Primer caso: C < 90.

Llamaremos A′ al punto en que la altura de A corta al lado BC del triangulo ABC, y C′

al punto donde la altura de C corta al lado AB del triangulo ABC.El angulo CHA′ es igual al angulo AHC′. En el triangulo CA′H, el angulo CA′H esrecto, y por tanto el angulo HCA′ es 90 − α.En el triangulo AHC′, el angulo HC′A es recto, y por tanto el angulo HAC′ es 90 − α.El angulo HAC′ es igual al angulo A′AB del triangulo A′AB que es rectangulo, y portanto el angulo A′BA es α.De aquı concluimos que los triangulos CHA′ y A′AB son semejantes, y como CH = AB,son triangulos iguales; de donde obtenemos que AA′ = CA′, y por tanto el valor de latangente del angulo pedido es tanC = 1, y C = 45.

C A′ B

A

B′

C′

H

α

α90−α

A

A′

H

C′

B

C

B′

α

α

α

β

β=90−α

Segundo caso: C > 90.

Procediendo de modo analogo, se tiene A′CH =C′CB . En el triangulo C′CB, el angulo CA′H esrecto, y por tanto, A′HC = 90 − α, y en eltriangulo CC′B el angulo CC′B es recto y por tanto C′BC = 90 − α.El triangulo AA′B es rectangulo en A′ y por ello BAA′ = α.Entonces los triangulos AA′B y A′CH son semejantes y tienen la hipotenusa igual, luegoson iguales y deducimos AA′ = A′C, de donde la tangente de C vale tan C = −1 y C =135o.

Tercer caso: C = 90.

En este caso C coincide con H y CH = 0. Como AB = 0, este valor de C no es valido.

Page 673: COMPENDIUM OME - ToomatesEotvos de Hungría, que se iniciaron en 1894, precisamente durante la efervescencia de fin de siglo, consecuencia de la cual fue también el proceso iniciado

E OME 39. Problema 4. Solucion

Primero veamos que x no puede ser entero. Esto puede hacerse teniendo en cuenta que silo fuese, serıa un divisor de 20, y basta probar los 8 divisores para comprobar que ningunocumple la ecuacion.Otro modo de verlo es comprobar que f(x) = x3 + 2x2 +10x−20 es estrictamente creciente(su derivada es positiva para todo x) y ademas f(1) = 13 y f(2) = 36. Luego no hay raıcesenteras.Veamos que x no puede ser racional por reduccion al absurdo. Supongamos que x = p/qcon q ≥ 1 y p/q irreducible. Entonces

p3 = 20q3 − 10q2p − 2qp2 = q(20q2 − 10qp − 2p2).

Puesto que q es divisor del segundo miembro, tambien lo es del primero p3, lo que esabsurdo si p y q son primos entre si y q > 1. Luego la ecuacion no puede tener raıcesracionales y x es irracional.Para la irracionalidad de x2 basta ver que

x(x2 + 10

)= 20 − 2x2 =⇒ x =

20 − 2x2

x2 + 10,

y si x2 fuese racional, tambien lo serıa x en contra de lo probado.

Observacion.La demostracion de la irracionalidad de x se basa en dos propiedades conocidas de lospolinomios con coeficientes enteros:1) Las unicas raıces enteras posibles son divisores del termino independiente.2) Las unicas raıces racionales posibles deben tener el numerador divisor del termino in-dependiente y el denominador divisor del coeficiente del monomio de grado maximo.

Page 674: COMPENDIUM OME - ToomatesEotvos de Hungría, que se iniciaron en 1894, precisamente durante la efervescencia de fin de siglo, consecuencia de la cual fue también el proceso iniciado

E OME 39. Problema 5. Solucion

La idea es prolongar los lados para formar un triangulo equilatero.

a b c

a

f

e e

d

c

Tenemos a + b + c + d + e + f = 21 y = a + b + c = c + d + e = e + f + a de donde sale3 = 21 + a + c + e, y por tanto,

= 7 +a + c + e

3.

El valor mas pequeno de a + c + e es 6 y el mas grande 15, ası que

9 ≤ ≤ 12.

Si a + c + e = 6, entonces son:

(a, c, e) = (1, 2, 3) y (b, f, d) = (4, 5, 6).

Si a + c + e = 9, el unico caso posible es:

(a, c, e) = (1, 3, 5) y (b, f, d) = (2, 4, 6).

Si a + c + e = 12, el unico caso posible es (a, c, e) = (2, 4, 6).Si a + c + e = 15, el unico posible es (a, c, e) = (4, 5, 6).

Como el area del triangulo de lado l es l2√

34

y la del hexagono es√

34

(l2 − (a2 + c2 + e2)

),

las areas posibles son:

Si a + c + e = 6, entonces l = 9 y el area es67

√3

4.

Si a + c + e = 9, entonces l = 10 y el area es65

√3

4.

Si a + c + e = 4, entonces l = 11 y el area es65

√3

4.

Si a + c + e = 5, entonces l = 12 y el area es67

√3

4.

Page 675: COMPENDIUM OME - ToomatesEotvos de Hungría, que se iniciaron en 1894, precisamente durante la efervescencia de fin de siglo, consecuencia de la cual fue también el proceso iniciado

E OME 39. Problema 6. Solucion

Tenemos la cadena con el total de 4n bolas, 2n blancas y 2n negras. Cogemos un grupode un extremo con 2n bolas; este grupo tendra x bolas negras e y bolas blancas, de formaque la diferencia es x − y = 2k para k ∈ −n,−n + 1, . . . , 1, 0, 1, . . . , n − 1, n.Vamos moviendonos de una en una posicion hacia el extremo contrario y en cada movi-miento la diferencia varıa en 2 o no varıa, es decir k aumenta en 1, disminuye en 1 o nocambia.La diferencia varıa en 2 si la bola que se deja y que se coge son de distinto color y semantiene si son del mismo color.La posicion final, es decir en el otro extremo, tendra las bolas al reves, x bolas blancas ey bolas negras con lo que la diferencia (blancas - negras) sera ahora y − x = −2k, para elmismo k.Es decir que k pasa de una posicion a su opuesta con el mismo valor absoluto. Como ksolo puede variar de 1 en 1 tiene que pasar por el cero ya que no se lo puede saltar.En el momento en que k = 0, se tiene x = y = n.Por lo tanto siempre se podra cortar un segmento de longitud 2n con n bolas blancas y nbolas negras.

Page 676: COMPENDIUM OME - ToomatesEotvos de Hungría, que se iniciaron en 1894, precisamente durante la efervescencia de fin de siglo, consecuencia de la cual fue también el proceso iniciado

E OME 40. Problema 1. Solucion

Denotaremos por aji al elemento de la fila i-esima y columna j-esima del rectangulo.

Pongamos n para el numero de filas, m para el de columnas y S para la suma de los n×melementos.Con notacion matricial queda

M =

a11 a2

1 · · · am1

a12 a2

2 · · · am2

· · · · · · · · · · · ·a1

n a2n · · · am

n

.

Sumando por filas y llamando Sk a la suma de la fila k, resulta

S1 =a11 + am

1

2m

S2 =a12 + am

2

2m

.......................

.......................

Sn =a1

n + amn

2m

y sumando miembro a miembro queda:

S = S1 + S2 + · · ·+ Sn =

=m

2((a1

1 + a12 + · · ·+ a1

n) + (am1 + am

2 + · · ·+ amn )

)=

=n m

4(a11 + a1

n + am1 + am

n

).

De aquı sale

a11 + a1

n + am1 + am

n =4 S

n m=

4 · 110721221

= 2004.

Page 677: COMPENDIUM OME - ToomatesEotvos de Hungría, que se iniciaron en 1894, precisamente durante la efervescencia de fin de siglo, consecuencia de la cual fue también el proceso iniciado

E OME 40. Problema 2. Solucion

Primera solucion

Bastara probar que el area de cada cuadrilatero es la cuarta parte del area total.La quebrada APC divide al cuadrilatero en dos partes de igual area pues AP es la medianade ABD y PC lo es de CBD (Figura 1).La quebrada TPZ divide al cuadrilatero APCD (sombreado) en dos partes de igual areapues PT es mediana de APD y PZ es mediana de CPD (Figura 2).

B

A

D

C

Y

XT

Z

P

QO

Figura 1

B

A

D

C

Y

XT

Z

P

QO

Figura 2

B

A

D

C

Y

XT

Z

P

QO

Figura 3

Tenemos ya probado que el area del cuadrilatero TPZD es la cuarta parte del area delcuadrilatero inicial.Finalmente TZ es paralela a OP por serlo ambas a AC; luego los triangulos TPZ y TOZtienen la misma area y lo mismo les ocurre a los cuadrilateros TPZD y TOZD (Figura 3).Del mismo modo se probarıa para los otros tres cuadrilateros.

Page 678: COMPENDIUM OME - ToomatesEotvos de Hungría, que se iniciaron en 1894, precisamente durante la efervescencia de fin de siglo, consecuencia de la cual fue también el proceso iniciado

Segunda solucion

B

A

D

C

Y

XT

Z

P

Q O

Figura 4

Al ser OP paralela a AC, los triangulos OXY , PXY tienen la misma base e igual alturay por tanto la misma area.De ahı que los cuadrilateros OXBY , PXBY tambien tienen la misma area, pero el area dePXBY (en gris en la Figura 4) es la cuarta parte del cuadrilatero inicial al ser semejantescon razon 2 del grande al pequeno.

Page 679: COMPENDIUM OME - ToomatesEotvos de Hungría, que se iniciaron en 1894, precisamente durante la efervescencia de fin de siglo, consecuencia de la cual fue también el proceso iniciado

E OME 40. Problema 3. Solucion

Primeramente observemos que f(x + n f(y)

)= f(x) − n y.

Para n = 0 es obvio, y por induccion, suponemos que para el entero n ≥ 1 se cumple

f(x + (n − 1)f(y)) = f(x) − (n − 1) y.

Entonces

f(x + n f(y)

)= f

(x + (n − 1)f(y) + f(y)

)=

= f(x + (n − 1)f(y)

)− y =

= f(x) − (n − 1)y − y = f(x) − n y.

Analogamente se prueba la misma propiedad para cada entero n ≤ −1.Por tanto, haciendo y = 1 y n = f(1), sale

f(1 + f(1) f(1)

)= f(1) − n = 0.

Poniendo k = 1 + f(1) f(1) = 1 +(f(1)

)2> 0, se tiene f(x) = f

(x+ f(k)

)= f(x)− k que

es una contradiccion.Deducimos que no existen funciones que satisfagan la condicion requerida.

Page 680: COMPENDIUM OME - ToomatesEotvos de Hungría, que se iniciaron en 1894, precisamente durante la efervescencia de fin de siglo, consecuencia de la cual fue también el proceso iniciado

E OME 40. Problema 4. Solucion

Supongamos que exista tal potencia de 2, es decir, que haya dos potencias de 2 cuyasexpresiones decimales solo difieran en el orden de colocacion de los dıgitos. Claramenteninguna de las dos potencias es divisible por 3 y ambas dejan el mismo resto cuando sedividen por 9. Esto ultimo se debe a que el resto de un numero al dividirse por 9 escongruente, modulo 9, con la suma de sus dıgitos.Por otra parte la mayor de ambas potencias se obtiene de la menor multiplicando esta por2, 4 u 8 (de otra manera no tendrıan ambas el mismo numero de dıgitos). Sin embargo almultiplicar la menor potencia de las dos por 2, 4 u 8, cambia el resto obtenido al dividirpor 9. Los restos de las sucesivas potencias de 2 al dividirse por 9 forman una sucesionperiodica; los restos de

1, 2, 4, 8, 16, 32, 64, 128, 256, 512, 1024, 2048, 4096, . . .

al dividir por 9 son1, 2, 4, 8, 7, 5, 1, 2, 4, 8, 7, 5, 1, . . .

Esta sucesion tiene periodo 6 porque para todo n entero positivo

2n+6 − 2n = 2n(26 − 20) = 2n · 63,

y este numero es divisible por 9 por lo que ambas potencias dejan el mismo resto. (Laperiodicidad tambien se deduce directamente de la observacion de los restos, ya que cadauno de ellos depende solo del anterior y a partir del primero que se repite, necesariamenteresulta una sucesion periodica. Las congruencias de Fermat y Euler nos indican los posiblesvalores del periodo; en este caso hay que aplicar la de Euler).

No es posible por tanto, reordenar los dıgitos de una potencia de 2 para obtener otrapotencia distinta de 2.

Page 681: COMPENDIUM OME - ToomatesEotvos de Hungría, que se iniciaron en 1894, precisamente durante la efervescencia de fin de siglo, consecuencia de la cual fue también el proceso iniciado

E OME 40. Problema 5. Solucion

La condicion es necesaria.

Sea ABC un triangulo y K el punto medio de AC. Supongamos que la mediana BK cortaa la circunferencia inscrita en dos puntos, M y N , tales que

BM = MN = NK = x.

Sea T el punto de tangencia del cırculo inscrito con el lado BC.En cualquier triangulo se cumplen las igualdades siguientes

BT = p − b =a + c − b

2con 2p = a + b + c

BK2 =2a2 + 2c2 − b2

4

La primera se deduce sin mas de BT + CT = a, BT −CT = c− b y no es otra cosa que laformula que da las distancias de los vertices a los puntos de contacto del cırculo inscrito;la segunda — formula de Apolonio o de la mediana — se puede obtener completando eltriangulo ABC hasta obtener un paralelogramo; o tambien aplicando el teorema del cosenoa los triangulos BCK y ABK calculando a2 y c2.

B A

C

T

I

HM N

ab

c

K

Entonces resulta2a2 + 2c2 − b2 = 36x2. (1)

La potencia del vertice B respecto del cırculo inscrito se puede escribir de dos maneras

BT 2 = BM · BN,

con lo cual(a + c − b)2 = 8 x2. (2)

Como, evidentemente, en el triangulo del problema, los puntos B y K estan igualmentealejados del centro del cırculo inscrito, resulta BC = KC, de donde

b = 2a.

Page 682: COMPENDIUM OME - ToomatesEotvos de Hungría, que se iniciaron en 1894, precisamente durante la efervescencia de fin de siglo, consecuencia de la cual fue también el proceso iniciado

Sustituyendo esta ultima igualdad en (1) y (2), obtenemos

c2 − a2 = 18 x2, (c − a)2 = 8 x2

y ya que c − a = 0, x = 0, resulta

c + a

c − a=

94, de donde

c

a=

135

.

Por lo tanto,a

5=

b

10=

c

13.

La condicion es suficiente.

No hay perdida de la generalidad en suponer que a = 5, b = 10, c = 13.Sustituyendo los valores de los lados en las formulas utilizadas en la primera parte, resulta

BK = 6√

2, BT 2 = 16 = BM · BN

y para el radio de la circunferencia inscrita

r =S

p=

6√

1414

donde S se ha calcuado por medio de la formula de Heron.El triangulo BCK es isosceles, ası que la bisectriz del angulo C es tambien altura. SeaH = CI ∩ BK y consideremos el triangulo rectangulo BIT ; entonces BI2 = 42 + r2 =22 × 65

14.

Por otra parte, en BIH , se tiene HI2 =47, y finalmente en IHM , HM2 = r2 −HI2 = 2.

Como H es el punto medio de MN , resulta MN = 2√

2, luego la mediana BK queda, enefecto, dividida en tres partes iguales por el cırculo inscrito.

Page 683: COMPENDIUM OME - ToomatesEotvos de Hungría, que se iniciaron en 1894, precisamente durante la efervescencia de fin de siglo, consecuencia de la cual fue también el proceso iniciado

E OME 40. Problema 6. Solucion

Numeremos las fichas desde 1 hasta 2004: la 1 es negra y las restantes son blancas.Cada ficha inicialmente blanca debe ser tocada un numero par de veces, para que al finaldel proceso siga teniendo la cara blanca hacia arriba. Cada movimiento posible cambia elnumero de fichas negras en un numero impar:

BNB pasa a NBN: el numero de fichas negras aumenta en 1

NNB pasa a BBN: el numero de fichas negras disminuye en 1

BNN pasa a NBB: el numero de fichas negras disminuye en 1

NNN pasa a BBB: el numero de fichas negras disminuye en 3

Como inicialmente hay exactamente una ficha negra, el numero total de movimientos paratener las 2004 fichas con la cara blanca hacia arriba debe se impar.Designamos por xi el numero de movimientos realizados eligiendo la ficha i (que debe sernegra).La ficha que ocupa el lugar i cambia de color en los movimientos en que la elegimos a ella(xi), a la de su izquierda (xi−1) o a la de su derecha (xi+1). Por lo tanto, xi−1 +xi +xi+1

es el numero de veces que hemos dado la vuelta a la ficha que ocupa el lugar i. (2004 + 1se identifica con 1, y 2003 + 2 se identifica con 1).El numero total de movimientos sera:

N = (x1 + x2 + x3) + x4 + · · ·+ (x2002 + x2003 + x2004) .

Como 2004 es multiplo de 3, N es la suma del numero de veces que hemos dado la vueltaa las fichas en los lugares 2, 5 ... 3k + 2, . . . , 2003, todas ellas blancas al principio: asıque N , suma de numeros pares, deberıa ser par: contradiccion, pues N es impar. Por lotanto, no sera posible conseguir que las 2004 fichas tengan la cara blanca hacia arriba.Con 2003 fichas si es posible: iniciando el movimiento sobre la ficha 1, (unica negra al prin-cipio), y repitiendolo sobre las fichas que ocupan los lugares 2, . . . , 2001, 2002 llegarıamosa la configuracion

NNN NNN . . . NNN BB

Eligiendo ahora las fichas que ocupan los lugares 2, 5, . . . , 3k + 2, . . . , 2000, tendrıamos

BBB BBB . . . BBB BB

en la que todas las fichas tendrıan la cara blanca hacia arriba.

Page 684: COMPENDIUM OME - ToomatesEotvos de Hungría, que se iniciaron en 1894, precisamente durante la efervescencia de fin de siglo, consecuencia de la cual fue también el proceso iniciado

5/3/2020 Primera sesión

www.olimpiadamatematica.es/platea.pntic.mec.es/_csanchez/olimp2005.html 1/3

XLI Olimpiada Matemática Española

Fase nacional 2005 (Santiago de Compostela)Primera sesión (21 de marzo)

1.- Sean a y b enteros. Demostrar que la ecuación

admite a lo sumo una solución entera.

2.- ¿Es posible colorear los puntos del plano cartesiano Oxy de coordenadas enteras con tres colores, de tal modo que cadacolor aparezca infinitas veces en infinitas rectas paralelas al eje Ox y tres puntos cualesquiera, cada uno de distinto color, no esténalineados? Justificar la contestación.

3.- Diremos que un triángulo es multiplicativo si el producto de las longitudes de dos de sus lados

es igual a la longitud del tercer lado.Sea ABC...XYZ un polígono regular de n lados con todos sus lados de longitud 1. Las n – 3 diagonalesque salen del vértice A dividen al triángulo ZAB en n – 2 triángulos más pequeños. Probar que cada unode esos triángulos es multiplicativo.

Segunda sesión (22 de marzo)

4.- Probar que para todo entero positivo n, la expresión decimal de

es periódica mixta.

5.- Sean números reales cualesquiera. Probar que:

mín

6.- En un triángulo de lados a, b, c el lado a es la media aritmética de b y c. Probar:a) 0º ≤ A ≤ 60º.b) La altura relativa al lado a es tres veces el inradio r.c) La distancia del circuncentro al lado a es R – r , siendo R el circunradio.

No está permitido el uso de calculadoras. Cada problema se puntúa sobre 7 puntos. El tiempo de cada sesión es de 3,5 horas.

Page 685: COMPENDIUM OME - ToomatesEotvos de Hungría, que se iniciaron en 1894, precisamente durante la efervescencia de fin de siglo, consecuencia de la cual fue también el proceso iniciado

5/3/2020 Primera sesión

www.olimpiadamatematica.es/platea.pntic.mec.es/_csanchez/olimp2005.html 2/3

Soluciones en formato Microsoft Word 2000 (Comprimido .zip, 88 Kb)

Ganadores de la OlimpiadaMedallas de oro

Miguel Teixidó RománElisa Lorenzo García

Javier de la Nuez GonzálezAnas El Barkani

Pau Labarta BajoHugo Fernández Hervás

Medallas de plata

Marc Viñals PérezAlberto Camacho Martínez

Nuria Gómez GonzálezAbel Molina Prieto

María Calvo CerveraGabriel Poussif

Víctor Martínez MartínezIgnacio Somoza SotillosRicardo Martín Brualla

Zhuoli ChenLander Ramos Garrido

Marc Sabaté Vidales

Medallas de bronce

Diego Gimeno SanzJosé Carpio Pinedo

Ding Ru CaiGuillermo Vicente García

Kristina Krasimirova VladimirovaAlberto Soria Barroso

Laurent Dietrich LangkemperAlberto Alejandro Merino Madrid

Javier Fresán LealÁlvaro Morante Amat

Mikael Rodríguez ChalaMiguel Ángel Asensio Álvarez

Andrés Parrilla GómezAlba Hiniesta Lozano de las Heras

Jorge Ruiz NavarroAntonio José Berenguer Verdú

Jaime Bonnín RocaAdrián Coso Rodríguez

Breve estudio de los resultados

Los problemas se calificaron sobre 7 puntos, Las medias y desviaciones de cada problema fueron:

P1 P2 P3 P4 P5 P6Medias 3,03 0,64 1,25 0,80 0,99 0,69

Desviaciones 3,14 1,42 1,94 1,72 2,04 1,29

Page 686: COMPENDIUM OME - ToomatesEotvos de Hungría, que se iniciaron en 1894, precisamente durante la efervescencia de fin de siglo, consecuencia de la cual fue también el proceso iniciado

5/3/2020 Primera sesión

www.olimpiadamatematica.es/platea.pntic.mec.es/_csanchez/olimp2005.html 3/3

|Página principal| |Info Alumnos| |Info Profesores| |Otros sitios de interés| |Problemas| Copyright © C. Sánchez-Rubio

Actualizado 26 marzo 2005

Page 687: COMPENDIUM OME - ToomatesEotvos de Hungría, que se iniciaron en 1894, precisamente durante la efervescencia de fin de siglo, consecuencia de la cual fue también el proceso iniciado

-1-

SOLUCIONES 2005

Problema 1

Sean a y b enteros. Demostrar que la ecuación

3 1 0x a x b x

admite a lo sumo una solución entera.

Solución.

Sea el entero p una raíz, entonces: 3 1x a x b x se anula para x = p, es decir

3 1p a p b p

Distingamos varios casos

1.- 3 1 4p p

entonces para los otros factores tenemos dos posibilidades:

1 1 5

1 1 3

p a a p

p b b p

sustituyendo queda la ecuación

2 3 23 5 1 11 39 44 0x x x x x

y una vez separada la raíz 4 resulta la ecuación 2 7 11 0x x que no tiene raíces enteras.

1 1 3

1 1 5

p a a p

p b b p

idéntico al anterior.

2.- 3 1 2p p

entonces para los otros factores tenemos dos posibilidades:

1 1 1

1 1 1

p a a p

p b b p

sustituyendo queda la ecuación

2 3 21 3 1 5 7 2 0x x x x x

y después de separar la raíz 2 resulta 2 3 1 0x x que no tiene raíces enteras.

Finalmente,

1 1 3

1 1 3

p a a p

p b b p

sustituyendo queda la ecuación

3 3 23 1 9 27 26 0x x x x

y después de separar la raíz 2 resulta 2 7 13 0x x que no tiene raíces.

Problema 2

¿Es posible colorear los puntos del plano cartesiano Oxy de coordenadas enteras con tres colores,

de tal modo que cada color aparezca infinitas veces en infinitas rectas paralelas al eje Ox y tres

puntos cualesquiera, cada uno de distinto color, no estén alineados? Justificar la contestación.

Solución oficial.

Probemos que tal coloración es posible. Pintemos el punto ( , )x y de rojo si x y es par, de blanco

si x es impar e y es par y de azul si x es par e y es impar.

Claramente se satisface la condición de que cada color aparezca infinitas veces en infinitas rectas

paralelas al eje .OX

Page 688: COMPENDIUM OME - ToomatesEotvos de Hungría, que se iniciaron en 1894, precisamente durante la efervescencia de fin de siglo, consecuencia de la cual fue también el proceso iniciado

-2-

Supongamos ahora que 1 1( , )x y sea rojo, 2 2( , )x y sea blanco y 3 3( , )x y sea azul. Entonces 2 1x x e

2 1y y tienen paridad opuesta y 3 2x x e 3 2y y son ambos impares. Por tanto:

2 1 3 2 3 2 2 1( )( ) ( )( ),y y x x y y x x con lo cual : 3 2 2 1

3 2 2 1

( ) ( ),

( ) ( )

y y y y

x x x x

lo que significa que tres

puntos cualesquiera, cada uno de distinto color, no están alineados.

Solución de Miguel Teixidó Román

Consideremos una coloración de 2 d acuerdo con las siguientes reglas:

-Azul si sus dos coordenadas son pares.

-Verde si ambas coordenadas son impares.

-Naranja cuando tienen una coordenada de cada paridad.

Demostraremos que tal coloración cumple las condiciones requeridas.

Observamos que tres puntos A, B, C de 2 están alineados si y sólo si existe k tal que AB k AC .

Caso 1. Supongamos que A azul, B verde y C naranja con la primera coordenada par están

alineados, la condición AB k AC sobre la primera componente queda: x x x xb a k c a y

reduciéndola módulo 2 resulta: 1 0 0 0 1 0·k k contradicción que prueba que no existen

tales puntos.

Caso 2. Igual que el anterior con la primera coordenada de C impar y por tanto la segunda par.

Razonando del mismo modo sobre la segunda componente tenemos:

y y y yb a k c a y módulo 2 resulta 1 0 0 0 1 0·k k y la correspondiente

contradicción.

La segunda condición también se cumple:

El azul se repite infinitamente en las rectas de la forma y k para k par.

El verde se repite infinitamente en las rectas de la forma y k para k impar.

El naranja se repite infinitamente en las rectas de la forma y k para cualquier k .

Problema 3

Diremos que un triángulo es multiplicativo si el producto de las

longitudes de dos de sus lados es igual a la longitud del tercer lado.

Sea ABC...XYZ un polígono regular de n lados con todos sus lados de

longitud 1. Las n – 3 diagonales que salen del vértice A dividen al

B

A

Z

RQP

X

Y

Page 689: COMPENDIUM OME - ToomatesEotvos de Hungría, que se iniciaron en 1894, precisamente durante la efervescencia de fin de siglo, consecuencia de la cual fue también el proceso iniciado

-3-

triángulo ZAB en n – 2 triángulos más pequeños. Probar que cada uno de esos triángulos es

multiplicativo.

Solución oficial.

El ángulo formado pos dos diagonales consecutivas con un extremo en A es el mismo por inscrito

en el mismo arco .

Sean PQ y QR dos segmentos adyacentes sobre el segmento ZB, determinados por tres diagonales

consecutivas. Entonces

1 1· · ·

2 2

1 1· · ·

2 2

APQ

AQR

S h PQ AP AQ sen

S h QR AQ AR sen

donde h es la altura común a todos los triángulos. Por tanto

· ·AP AQ AQ AR

PQ QR ,

es decir, la razón r del producto de dos lados a la base es la misma para cualquier par de triángulos

adyacentes, y por tanto es la misma para todos ellos. Pero el primero y el último son claramente

multiplicativo al tener dos lados iguales y el tercero igual a 1. Se deduce que r = 1 y todos los

triángulos son multiplicativos.

Solución de Anas El Barkani que mereció una mención especial del jurado.

Vamos a probar el enunciado demostrando que si un triángulo

es multiplicativo, también lo es el colindante.

Obsérvese primero que el triángulo ABQ es isósceles puesto

que los ángulos A y B abarcan el mismo arco. Es obvio que el

triángulo ABQ es multiplicativo pues 1AB .

· ·1QB AQ AB AQ AQ (1)

Ahora bien, si aplicamos el teorema de la bisectriz al triángulo

APB tenemos que:

·PQ QB

AQ PQ AQ APAP AB

lo que prueba que APQ también es multiplicativo.

c.q.d.

Problema 4

Probar que para todo entero positivo n, la expresión decimal de

1 1 1

1 2n n n

es periódica mixta.

Solución:

Tenemos

21 1 1 3 6 2

1 2 1 2

n n

n n n n n n

Sabemos que para que una fracción origine un decimal periódico mixto, una vez reducida debe

tener en el denominador algún factor primo del conjunto 2,5 y alguno que no sea ni 2 ni 5.

Z

A B

C

PQ

Page 690: COMPENDIUM OME - ToomatesEotvos de Hungría, que se iniciaron en 1894, precisamente durante la efervescencia de fin de siglo, consecuencia de la cual fue también el proceso iniciado

-4-

Veamos primero que la fracción anterior tiene en el denominador al menos una factor 2 más que en

el numerador, en efecto

Si n es par tenemos 2n k que una vez sustituido resulta:

2 2 21 1 1 3 6 2 12 12 2 6 6 1

1 2 1 2 2 2 1 2 2 2 2 1 1

n n k k k k

n n n n n n k k k k k k

el numerador es impar y el denominados par.

Si n es impar tenemos 2 1n k que una vez sustituido resulta:

2 21 1 1 3 6 2 12 24 11

1 2 1 2 2 1 2 2 2 3

n n k k

n n n n n n k k k

el numerador es impar y el denominados par.

En ambos casos el denominador tiene al menos un factor 2 que no está en el numerador.

Además la expresión

21 1 1 3 6 2

1 2 1 2

n n

n n n n n n

muestra que el numerador no contiene el

factor primo 3 (da resto 2 al dividirlo entre tres) mientras el denominador al ser producto de tres

números consecutivos es múltiplo de tres.

Problema 5

Sean , , ,r s u v números reales cualesquiera. Probar que:

mín 2 2 2 2 1, , , .

4r s s u u v v r

Solución.

Supongamos que los cuatro números 2 2 2, ,r s s u u v y 2v r son mayores estrictamente que

1.

4 Entonces 2 2 2 2 1 1 1 1

,4 4 4 4

r s s u u v v r pero esta expresión es equivalente a

2 2 2 21 1 1 1

02 2 2 2

r s u v

que es una contradicción.

Problema 6

En un triángulo de lados a, b, c el lado a es la media aritmética de b y c. Probar:

a) 0º ≤ A ≤ 60º.

b) La altura relativa al lado a es tres veces el inradio r.

c) La distancia del circuncentro al lado a es R – r.

Solución.

a) Por la desigualdad triangular:

3 312

31 3

32 3

b c bb c b c

bc

b c b cc b c b

c

Por el teorema de coseno:

2 2 2

2 2 3 3 22 cos cos

4 8

b c b c bcb c bc A A

bc

r

da

R

a

cb

ha

BC

A

I

T

Page 691: COMPENDIUM OME - ToomatesEotvos de Hungría, que se iniciaron en 1894, precisamente durante la efervescencia de fin de siglo, consecuencia de la cual fue también el proceso iniciado

-5-

dividiendo numerador y denominador por c2

y llamando por comodidad de escritura b

xc

, queda:

23 2 3 3 1 3

cos8 8 4 8

x xA f x x

x x

con

13

3x .

Fácilmente se comprueba que 1

3 13

f f

y que la derivada se anula en x = 1 donde hay un

mínimo que vale 1

12

f .

También puede localizarse el mínimo sin recurrir a la derivada teniendo en cuenta la desigualdad de

las medias:

23 2 3 3 1 3 1 3 1 1 3

cos ·28 8 4 8 4 8 4 8

x xA f x x x

x x x

con igualdad para x = 1.

Resumiendo queda 1

cos 1 60º 0º2

A A .

b) Designando A, B y C a los vértices opuestos a los lados a, b y c respectivamente, I al incentro y

ha a la altura correspondiente al lado a como se indica en la figura, S al área y p al semiperímetro,

tenemos:

3

3 12 23

1 2 2

2

a a

a

a b c arS pr r

arah r h

S ah

c) Pongamos da a la distancia entre el circuncentro y el lado a.

De una parte 2

2 2

4a

ad R (1)

y de otra 2

2

A r rtg

p a a

.

Como 2a

RsenA

y

12

22

Atg

senAA

tg

, resulta:

2

2 222

41

2 24 4 4

r

a aaR a r Rr rr r

a

que sustituida en (1) queda:

2

22 2 2 224

a a

ad R R Rr r R r d R r

Solución de Elisa Lorenzo García.

Los apartados a) y b) son esencialmente iguales a los de la solución oficial.

Para el apartado c) se da la solución que sigue sin utilizar trigonometría.

Q es la intersección de la bisectriz de A con la mediatriz de a que está en el punto medio del arco

BC.

Llamando x PB , resulta

Page 692: COMPENDIUM OME - ToomatesEotvos de Hungría, que se iniciaron en 1894, precisamente durante la efervescencia de fin de siglo, consecuencia de la cual fue también el proceso iniciado

-6-

23

4

b ca x y

c bb y z x

c z x

Aplicando el teorema de la bisectriz:

c b

BQ CQ

y

2

2 2 2

b c c bcBQ CQ a BQ

b c

además '4

b cBA

por ser la mitad de a.

Calculemos

2 23 3' ' 2 0

4 2 2 4 4

b c c bc c b b b c c c bcQA PQ BA BQ BQ BP c c

b c b c

de donde 'QA PQ y como ' 90ºIPQ QA S y 'IQP A QS , resulta que los triángulos PIQ y

A’SQ son iguales y 'IP A S r de donde queda finalmente:

' 'OA OS A S R r

c

a

b

A

B CA'

OI

P

S

Q

Page 693: COMPENDIUM OME - ToomatesEotvos de Hungría, que se iniciaron en 1894, precisamente durante la efervescencia de fin de siglo, consecuencia de la cual fue también el proceso iniciado

5/3/2020 Primera sesión

www.olimpiadamatematica.es/platea.pntic.mec.es/_csanchez/olimp2006.html 1/4

XLII Olimpiada Matemática Española Fase nacional 2006 (Sevilla)

Primera sesión (24 de marzo)

1.- Sea un polinomio con coeficientes enteros. Demostrar que si existe un entero k tal que

ninguno de los enteros es divisible por k, entonces no tiene raíces enteras.

2.- Las dimensiones de un ortoedro de madera son enteras. Pintamos toda su superficie (las seis caras),lo cortamos mediante planos paralelos a las caras en cubos de una unidad de arista y observamos queexactamente la mitad de los cubos no tienen ninguna cara pintada. Probar que el número de ortoedros con talpropiedad es finito.

(Puede resultar útil tener en cuenta que )..

3.- ABC es un triángulo isósceles con AB = AC. Sea P un punto cualquiera de la circunferencia tangente

a los lados AB en B y a AC en C.Llamamos a, b y c a las distancias desde P a los lados BC, AC y AB respectivamente. Probar que:

Segunda sesión (25 de marzo)

4.- Hallar todas las funciones que satisfacen la ecuación

para todo par de números reales e positivos, siendo un número real positivo tal que

Page 694: COMPENDIUM OME - ToomatesEotvos de Hungría, que se iniciaron en 1894, precisamente durante la efervescencia de fin de siglo, consecuencia de la cual fue también el proceso iniciado

5/3/2020 Primera sesión

www.olimpiadamatematica.es/platea.pntic.mec.es/_csanchez/olimp2006.html 2/4

5.- Probar que el producto de cuatro naturales consecutivos no puede ser ni cuadrado ni cubo perfecto

6.- Las diagonales y de un cuadrilátero convexo se cortan en Denotamos por y a las áreas de los triángulos y del cuadrilátero respectivamente. Prueba que

¿Cuándo se alcanza la igualdad?

No está permitido el uso de calculadoras. Cada problema se puntúa sobre 7 puntos. El tiempo de cada sesión es de 3,5 horas.

Soluciones en formato Microsoft Word 2000 (Comprimido .zip, 72 Kb)

Ganadores de la OlimpiadaMedallas de oro

Hugo Fernández HervásJavier de la Nuez González

Diego Gimeno SanzÁngel David Martínez Martínez

Xavier Ros OtónMarc Viñals Pérez

Medallas de plata

Rafael Ballester RipollDiego Izquierdo Arseguet

María Calvo CerveraIván Fernández ColungaGabriel Adrián Poussif

Yassin EzbakheDaniel Alsina Leal

David Alfaya SánchezCarlos Ramírez Carrillo

Adrián Rodrigo EscuderoDiego González-Pinto González

Martín Citoler Saumell

Medallas de bronce

David Garcés Urzainqui

Page 695: COMPENDIUM OME - ToomatesEotvos de Hungría, que se iniciaron en 1894, precisamente durante la efervescencia de fin de siglo, consecuencia de la cual fue también el proceso iniciado

5/3/2020 Primera sesión

www.olimpiadamatematica.es/platea.pntic.mec.es/_csanchez/olimp2006.html 3/4

Carlos Corrales BenítezGerard Planas Conangla

Raúl Úbeda SalaMiguel Ángel Asensio Álvarez

Michael Ernesto López LehmannAlex Barceló Cuerda

Xiaolei Jin LinYago Bea Besada

José María Pérez RamosDaniel Illana González

Javier Goñi MolaLaura Carrillo Corbalán

Marta Guillén LlorMariana Ioana Ionascu

Juan Pedro Bolívar PuenteDongqing Qiu

Bernat Serra Montolí

Breve estudio de los resultadosLos problemas se calificaron sobre 7 puntos, Las medias y desviaciones de cada problema fueron:

P1 P2 P3 P4 P5 P6Medias 1,28 0,82 1,58 0,94 0,61 1,76

Desviaciones 2,37 0,87 1,73 1,42 1,40 2,52

Page 696: COMPENDIUM OME - ToomatesEotvos de Hungría, que se iniciaron en 1894, precisamente durante la efervescencia de fin de siglo, consecuencia de la cual fue también el proceso iniciado

5/3/2020 Primera sesión

www.olimpiadamatematica.es/platea.pntic.mec.es/_csanchez/olimp2006.html 4/4

|Página principal| |Info Alumnos| |Info Profesores| |Otros sitios de interés| |Problemas| Copyright © C. Sánchez-Rubio

Actualizado 28 marzo 2006

Page 697: COMPENDIUM OME - ToomatesEotvos de Hungría, que se iniciaron en 1894, precisamente durante la efervescencia de fin de siglo, consecuencia de la cual fue también el proceso iniciado

SOLUCIONES 2006

Problema 1

Sea P x un polinomio con coeficientes enteros, demostrar que si existe un entero k tal que

ninguno de los enteros 1 , 2 , ,P P P k es divisible por k, entonces P x no tiene raíces

enteras.

Solución.

Por reducción al absurdo. Si n fuese una raíz, por una parte tenemos

P x x n Q x

y por otra siempre existen enteros q y r tales que n kq r , con 1 r k (basta hacer la división

entera y en el caso de ser resto cero se rebaja el cociente en una unidad), entonces

P r r n Q r kqQ r

en contra de lo supuesto en el enunciado.

Problema 2

Las dimensiones de un paralelepípedo de madera son enteras. Pintamos toda su superficie (las seis

caras) y lo cortamos en cubos de una unidad de arista y observamos que exactamente la mitad de los

pequeños cubos no tienen ninguna cara pintada. Probar que el número de paralelepípedos con tal

propiedad es finito.

(Puede resultar útil tener en cuenta que 31

1,79... 1,82

).

Solución . Consiste en acotar el número de soluciones de la ecuación diofántica:

2 2 2 2abc a b c

y para ello suponiendo que a b c , se pone en la forma: 1 2 2 2

· ·2

a b c

a b c

, y como

3

32 1 2 1 2 1

2 2 2

a a a

a a a

fácilmente se obtiene que 4 < a < 10.

Fijado a se acotan de modo análogo los posibles valores de b:

De una parte

22 2 2 2

·2 2 2 2

a b c b a b

a b c b a b

y de ahí se sigue

2 5

12 2 2 2 6

a b a

a b a

con lo que para cada valor de a, b sólo puede tomar un conjunto finito de valores.

Finalmente fijados a y b a lo sumo hay un valor para c que cumpla la ecuación inicial lo que prueba

que el número de soluciones es finito.

Problema 3

ABC es un triángulo isósceles con AB = AC. Sea P un punto cualquiera de la circunferencia

tangente a los lados AB en B y a AC en C.

Page 698: COMPENDIUM OME - ToomatesEotvos de Hungría, que se iniciaron en 1894, precisamente durante la efervescencia de fin de siglo, consecuencia de la cual fue también el proceso iniciado

Pongamos a, b y c a las distancias desde P a los lados BC, AC y AB respectivamente. Probar que:

2 ·a b c

Solución:

Pongamos m = PB; n = PC, Q, R y S las proyecciones de P sobre

cada lado y sea P’ el punto diametralmente opuesto a P .

Por la semejanza de los triángulos PBP’ y PBS se tiene:

222

m rm cr

c m (1)

De modo análogo por la semejanza de PCP’ y PBC se cumple:

222

n rn br

b n (2)

Por el teorema de los senos en PBC:

( )2

nsen PBC

r

y en el triángulo rectángulo PQB:

( )a

sen PBCm

de donde 2 2

2

22 4

mn m na a

r r y por (1) y (2) queda finalmente:

2

2

2 ·2·

4

cr bra b c

r

Problema 4.

Hallar todas las funciones : (0, )f R que satisfacen la ecuación

( ) ( ) 2 ( )f x f y f f f xyx y

para todo par de números reales x e y positivos, siendo un número real positivo tal que

( ) 1.f

Solución:

Haciendo 1x y en la ecuación funcional dada se tiene que 2 2(1) ( ) 2 (1).f f f De este

modo 2

(1) 1 0f y así (1) 1.f Sustituyendo ahora 1y en la ecuación funcional resulta

( ) (1) ( ) 2 ( ),f x f f f f xx

que es equivalente a ( )f x f

x

para todo 0.x Tomemos a

continuación yx

y observemos que

( ) ( ) 2 ( ).f x f f f x fx x

Entonces ( ) 1,f x fx

por lo que 2 ( ) 1f x para todo 0.x

m na

bc

CB

A

P

Q

RS

P'

Page 699: COMPENDIUM OME - ToomatesEotvos de Hungría, que se iniciaron en 1894, precisamente durante la efervescencia de fin de siglo, consecuencia de la cual fue también el proceso iniciado

Sustituyendo x y t de nuevo en la ecuación funcional inicial se tiene que

2 2 2 ( )f t f f tt

y debido a ser el miembro de la izquierda positivo f es positivo y

( ) 1f x para todo 0.x Se comprueba fácilmente que esta función constante e idénticamente

igual a 1 es solución de la ecuación funcional del enunciado y además es la única.

Problema 5

Probar que el producto de cuatro naturales consecutivos no puede ser ni cuadrado ni cubo perfecto.

Solución. Si el producto 1 1 2N n n n n fuese un cuadrado, basta ponerlo en la forma

2

2 2 21 1 2 2 1 1N n n n n n n n n n n

de donde se sigue una contradicción (no hay dos cuadrados consecutivos).

Si N fuese cubo perfecto, podemos suponer que 2n (si n = 2 , N = 24).

Distinguimos ahora dos casos:

a) n impar, entonces n es primo con los otros tres factores y si N es cubo perfecto, también lo es

3 21 1 2 2 2M n n n n n n

pero si 33 3 22 2 2 1n n n n n n y ya tenemos la contradicción pues entre dos cubos

consecutivos no puede haber otro cubo.

b) si n es par, 1n es impar y por tanto 1n es primo con el producto

3 21 2 2M n n n n n n

que también debe ser cubo perfecto.

Finalmente, como 33 3 22 2 1x x x x x x se sigue la contradicción.

Problema 6

Las diagonales AC y BD de un cuadrilátero convexo ABCD se cortan en .E Denotamos por

1 2,S S y S a las áreas de los triángulos ,ABE CDE y del cuadrilátero ABCD respectivamente.

Prueba que 1 2 .S S S ¿Cuándo se alcanza la igualdad?

Solución:

Denotando, como en el enunciado, la áreas de los triángulos BCE y DAE por 3S y 4S

respectivamente, tenemos que probar que 1 2 1 2 3 4 .S S S S S S

Elevando al cuadrado la anterior desigualdad, se obtiene la desigualdad equivalente

1 2 3 42 S S S S (1).

Sean ahora K y L los pies de las perpendiculares en la diagonal AC trazadas desde D y

B respectivamente. Estos puntos K y L pueden estar dentro o fuera del segmento .AC Llamamos

, , , .b BL d DK m AE n CE Entonces

1 2 3 1

1 1 1 1, , , .

2 2 2 2S mb S nd S nb S md

Page 700: COMPENDIUM OME - ToomatesEotvos de Hungría, que se iniciaron en 1894, precisamente durante la efervescencia de fin de siglo, consecuencia de la cual fue también el proceso iniciado

Sustituyendo esta expresión en la desigualdad (1) se llega a 1

( ),2

mb nd nb md que es

precisamente la desigualdad entre la media aritmética y la media geométrica de los dos productos

nb y .md

Esta última desigualdad se alcanza si y sólo si b m

nb mdd n

(2).

Las rectas BL y DK son paralelas. Así ,b BL BE

d DK DE por la semejanza entre los triángulos BLE

y .DKE La relación (2) se convierte en BE AE

DE CE (3). Y recíprocamente por la semejanza de

triángulos (3) se verifica si y sólo si AB y CD son paralelos, es decir el cuadrilátero dado es un

trapecio con los lados paralelos AB y CD . Esta es la condición para que se alcance (1).

Page 701: COMPENDIUM OME - ToomatesEotvos de Hungría, que se iniciaron en 1894, precisamente durante la efervescencia de fin de siglo, consecuencia de la cual fue también el proceso iniciado

5/3/2020 Primera sesión

www.olimpiadamatematica.es/platea.pntic.mec.es/_csanchez/olimp2007.html 1/4

XLIII Olimpiada Matemática Española Fase nacional 2007 (Torrelodones)

Primera sesión (23 de marzo)

1.- Sean cinco números positivos en progresión aritmética de diferencia d. Probar que

2.- Determinar todos los posibles valores enteros no negativos que puede tomar la expresión

siendo y enteros no negativos tales que

3.- Sea O el circuncentro de un triángulo ABC. La bisectriz que parte de A corta al lado opuesto en P.

Probar que se cumple:

Segunda sesión (24 de marzo)

4.- ¿Cuáles son los números enteros positivos que se pueden obtener de exactamente 2007 manerasdistintas, como la suma de al menos dos números enteros positivos consecutivos? ¿Cuál es el menor de todosellos?Ejemplo: el número 9 se escribe exactamente de dos maneras distintas:

9 = 4 + 59 = 2 + 3 + 4

Page 702: COMPENDIUM OME - ToomatesEotvos de Hungría, que se iniciaron en 1894, precisamente durante la efervescencia de fin de siglo, consecuencia de la cual fue también el proceso iniciado

5/3/2020 Primera sesión

www.olimpiadamatematica.es/platea.pntic.mec.es/_csanchez/olimp2007.html 2/4

5.- Sea un número real positivo y un entero mayor que 1. Demostrar que

6.- Dada una semicircunferencia de diámetro AB = 2R, se considera una cuerda CD de longitud fija c.Sea E la intersección de AC con BD y F la intersección de AD con BC.Probar que el segmento EF tiene longitud constante y dirección constante al variar la cuerda CD sobre lasemicircunferencia.

No está permitido el uso de calculadoras. Cada problema se puntúa sobre 7 puntos. El tiempo de cada sesión es de 3,5 horas.

Soluciones en formato Microsoft Word 2000 (Comprimido .zip, 83 Kb)

Ganadores de la OlimpiadaMedallas de oro

Diego Izquierdo Arseguet (*)Adrián Rodrigo Escudero (*)

Daniel Remón RodríguezGabriel Fürstenheim Milerud

David Alfaya SánchezGlenier Lázaro Bello Burguet

(*) Las dos primeras medallas de oro tienen exactamente la misma puntuación.

Medallas de plata

Albert Santiago BoílBernat Serra Montolí

Alejandro Gimeno SanzArnau Messegué Buisan

José Ramón Sierra Gómez de LeónAndrés Rodríguez ReinaÁlvaro Mateos González

Josep Marc Mingot HidalgoCarlos Sánchez Ramírez

Daniel de la Concepción SáezDavid Garcés UrzainquiAdrián Moro Calderón

Medallas de bronce

Zhin Lin

Page 703: COMPENDIUM OME - ToomatesEotvos de Hungría, que se iniciaron en 1894, precisamente durante la efervescencia de fin de siglo, consecuencia de la cual fue también el proceso iniciado

5/3/2020 Primera sesión

www.olimpiadamatematica.es/platea.pntic.mec.es/_csanchez/olimp2007.html 3/4

Bartomeu Montserrat SánchezJavier Bolea Moll

Paloma Arenas GuerreroSantiago Ruiz - ValdepeñasAlberto Calderón González

Manuel López SheriffAntonio Pérez Hernández

Adrián Rey RodríguezTeresa Rodrigo Rey

Alberto Sánchez MoleroCarlos Corrales Benítez

José Luis Lado VillanuevaEloi Marín Amat

Víctor Rodríguez RodríguezFrancisco Salamanca González

Fernando Sánchez MadrigalDavid Tena Cucal

Breve estudio de los resultadosLos problemas se calificaron sobre 7 puntos, Las medias y desviaciones de cada problema fueron:

P1 P2 P3 P4 P5 P6Medias5.43 0.53 1.17 0.27 1.58 1.59

Desviaciones2.27 0.52 2.05 0.59 1.74 2.30

Page 704: COMPENDIUM OME - ToomatesEotvos de Hungría, que se iniciaron en 1894, precisamente durante la efervescencia de fin de siglo, consecuencia de la cual fue también el proceso iniciado

5/3/2020 Primera sesión

www.olimpiadamatematica.es/platea.pntic.mec.es/_csanchez/olimp2007.html 4/4

|Página principal| |Info Alumnos| |Info Profesores| |Otros sitios de interés| |Problemas| Copyright © C. Sánchez-Rubio

Actualizado 28 marzo 2007

Page 705: COMPENDIUM OME - ToomatesEotvos de Hungría, que se iniciaron en 1894, precisamente durante la efervescencia de fin de siglo, consecuencia de la cual fue también el proceso iniciado

1

SOLUCIONES 2007

Problema 1.

Sean 0 1 2 3 4, , , ,a a a a a cinco números positivos en progresión aritmética de razón d. Probar que

3 3 3 3 3

2 0 1 3 4

14 4 .

10a a a a a

Solución 1 (del autor de la propuesta).

La desigualdad dada puede escribirse como

3 3 3 3 3

2 0 1 3 410 4 4 .a a a a a

y sumando 3

26a a ambos miembros se convierte en

3 3 3 3 3 3

2 0 1 2 3 4

14 6 4 .

16a a a a a a

Por otro lado como 0 1 2 3 4, , , ,a a a a a están en progresión aritmética, entonces

0 1 2 3 4 0 4 1 3 2

4

2 2 2 2 2

4 4 4 4 4 4 4 4

0 1 2 3 4 0 1 2

4 4 4 4 4 4 4 42 2 2 .

0 1 2 0 1 2 3 4

a a a a a a a a a a

a a a a a

Aplicando la desigualdad de Jensen a la función 3f t t , convexa en 0, , con

4 1,0 4

2k k

p kk

resulta

4 4

0 0

k k k k

k k

f p a p f a

o equivalentemente,

3 3 3 3 3 3 3 3 3 3 3

2 0 1 2 3 4 0 1 2 3 4

4 4 4 4 41 14 6 4

0 1 2 3 42 16ka a a a a a a a a a a

.

Obsérvese que la igualdad tiene lugar cuando los cinco números son iguales y hemos terminado.

Solución 2.

Llamando a al término central y d a la diferencia, la progresión es 2 , , , , 2a d a d a a d a d y

tenemos:

33 3 2 2 3

0

33 3 2 2 3

4

33 3 2 2 3

1

33 3 2 2 3

3

2 6 12 8

2 6 12 8

4 4 4 12 12 4

4 4 4 12 12 4

a a d a a d ad d

a a d a a d ad d

a a d a a d ad d

a a d a a d ad d

sumando:

Page 706: COMPENDIUM OME - ToomatesEotvos de Hungría, que se iniciaron en 1894, precisamente durante la efervescencia de fin de siglo, consecuencia de la cual fue también el proceso iniciado

2

3 3 3 3 3 2

0 1 3 44 4 10 48a a a a a ad

dividiendo por 10 queda

3 3 3 3 3 2

0 1 3 4

14 4 4,8 0

10a a a a a ad

con independencia del valor de d.

Solución 3.

Como se trata de cinco términos en progresión aritmética, se tiene

0 4 2 1 32a a a a a

o también

0 4 2 22 2 .a a a d a d

Entonces

33 3 3

0 4 0 4 0 4 0 4 2 0 4 23 8 6 ,a a a a a a a a a a a a

y

33 3 3

1 3 1 3 1 3 1 3 2 1 0 24 3 4 8 6a a a a a a a a a a a a a

.

Entonces, lo que hay que probar es

3 3 3

2 2 0 1 2 2 1 3 2

18 6 32 24

10a a a a a a a a a

cuyo segundo miembro es

3

2 2 0 4 1 3

64 4 ;

10a a a a a a

trasponiendo términos, la desigualdad a probar se escribe como

2 2 2 2 3 3 2 3 2

2 2 2 2 2 2

3 24 244 4 4 3 3 3 0

5 5 5a a d a d a a d a d

la última desigualdad es cierta y hemos terminado.

Problema 2.

Determinar todos los posibles valores enteros no negativos que puede tomar la expresión

,1

22

mn

nmnm siendo m y n enteros no negativos tales que .1mn

Solución.

Sea Nkkmn

nmnm

,

1

22

(naturales con el ).0

En el caso ,nm el número 1

33

2

mk es un entero positivo si 0m ó ;2m de donde

0k ó 4k respectivamente.

Page 707: COMPENDIUM OME - ToomatesEotvos de Hungría, que se iniciaron en 1894, precisamente durante la efervescencia de fin de siglo, consecuencia de la cual fue también el proceso iniciado

3

El caso 0n lleva a que 2mk y por tanto .0 km

Consideremos ahora las soluciones ),( nm tales que .0 nm Como la relación dada es

equivalente a ,0)1( 22 knmnkm observamos que si ),( nm es una solución y

,0)1( mnkn entonces ))1(,( mnkn es también una solución. La desigualdad

0)1( mnk es cierta en todos los casos porque se convierte sucesivamente en una desigualdad

obvia:

,n

nmk

,

1

22

n

nm

mn

nmnm

.3 nmn

Del mismo modo la desigualdad mnkn )1( es sucesivamente equivalente a:

,2

n

nmk

,

2

1

22

n

nm

mn

nmnm

1

32

n

nnm para .1n

Si ,13 2 nn esto es si ,4n la desigualdad anterior es cierta y por tanto para cada solución

),( nm con 4 nm encontramos una solución ),( pn con .0 pn De este modo, cada

solución es tal que .3n

Para ,1n obtenemos ,1

32

mmk 4m ó .7,2 km

Para ,2n obtenemos ,12

21524

mmk 4m ó 4,11 km ó .7k

Para ,3n obtenemos ,13

911039

mmk que no conduce a ninguna solución.

Entonces los posibles valores k enteros no negativos de la expresión del enunciado son 4,0 y .7

Problema 3.

Sea O el circuncentro de un triángulo ABC. La bisectriz que parte de A corta al lado opuesto en P.

Probar que se cumple:

2 2 2AP OA OP bc

Solución:

Prolongamos AP hasta que corte en M al circuncírculo.

Los triángulos ABM y APC son semejantes al tener dos

ángulos iguales.(ACB = AMB por inscritos en el

mismo arco y BAN = CAN por bisectriz).

Entonces:

·c AP

bc AM APAM b

como AM = AP + PM, queda:

2 ·bc AP AP PM AP AP PM

c

b

A

B C

O

P

M

Page 708: COMPENDIUM OME - ToomatesEotvos de Hungría, que se iniciaron en 1894, precisamente durante la efervescencia de fin de siglo, consecuencia de la cual fue también el proceso iniciado

4

AP·PM es la potencia de P respecto de la circunferencia circunscrita y su valor es OA2 – OP

2 sólo

queda sustituir y resulta:

2 2 2bc AP OA OP

Problema 4.

¿Cuáles son los números enteros positivos que se pueden obtener de exactamente 2007 maneras

distintas, como la suma de al menos dos números enteros positivos consecutivos? ¿Cuál es el menor

de todos ellos?

Ejemplo: el número 9 se escribe exactamente de dos maneras distintas:

9 = 4 + 5

9 = 2 + 3 + 4

Solución

1 2

1 2 1 22

n a nN a a a n N n a n

Si n es par 1n es impar y 2a n es par.

Si n es impar 1n es par y 2a n es impar.

Siempre que 2N en su descomposición en factores primos tenga un factor impar distinto de uno,

existe una descomposición de N en suma de números consecutivos.

En efecto, sea 2 1N q Q con q, Q naturales y 1q , 1Q , entonces:

a) )2)(1().12(2 nanQq , y hacemos la siguiente descomposición:

112 nq , es decir qn 2 y 1n

qaQ 222 , de donde qQa Si 0a ya hemos terminado, en otro caso

0Q q y entonces,

b) 12 nQ , de donde 12 Qn , es decir 1n y 12122 qQa de donde Qqa 1 , es

decir 1a y ya hemos terminado.

Entonces necesitamos el menor número que en su descomposición tenga 2007 factores impares sin

contar el 1.

Si sólo tiene un factor primo es:

2.32 2007N

Si tiene más de uno .. rqp , el número de divisores que tiene es )......1)(1)(1( , y este

producto debe ser 2008 para que excepto l uno tenga 2007 divisores impares distintos.

)......1)(1)(1(251.22008 3 , entonces:

7250 5.3N , o bien 7.5.3 3250N , o bien 11.7.5.3250N y este último es el menor.

Problema 5.

Sea 1a un número real positivo y n un entero positivo. Demostrar que 2

1

2.

2

n na an

a a

Solución.

Page 709: COMPENDIUM OME - ToomatesEotvos de Hungría, que se iniciaron en 1894, precisamente durante la efervescencia de fin de siglo, consecuencia de la cual fue también el proceso iniciado

5

La desigualdad dada 2

1

2

2

n na an

a a

es equivalente a

2

2 2

2

21 1

2 2

,

n n

a a

n

a a

que a su vez equivale a

que 1

,n n

n

siendo .a Entonces, usando la desigualdad aritmético-geométrica, se tiene

la desigualdad pedida:

21 1 2 4 2 2 1 2 4 ... (2 2) 1 1

1 2

1(1 ... ) .

1

n n nnn n n n n n nn n n

Problema 6.

Dada una semicircunferencia de diámetro AB = 2R, se considera una cuerda CD de longitud fija c.

Sea E la intersección de AC con BD y F la intersección de AD con BC.

Probar que el segmento EF tiene longitud constante y dirección constante al variar la cuerda CD

sobre la semicircunferencia.

Solución.

Como los triángulos EFC y EDF son rectángulos, el

cuadrilátero EDFC es inscriptible y EF es el diámetro.

Llamemos r al radio del circuncírculo de ECD, por el

teorema de los senos en ECD:

2 1sen

cEF r

E

Pongamos BOD y COD. Entonces

18090

2 2E

expresión que prueba que el ángulo E es constante al serlo y además el punto E se mueve en el

arco capaz de 2

90

sobre AB. Sustituyendo en (1) queda:

2 2

cos2

cEF r

, por otra parte 3

R2

c

2sen

; eliminando entre (2) y (3) y despejando

EF resulta: 2 2

2

4

cREF

R c

expresión que muestra que EF es constante al serlo c y R.

Además F es el ortocentro del triángulo ABE como intersección de las alturas AD y BC, por ello EF

que está sobre la tercera altura es siempre perpendicular a AB.

También admite solución analítica aunque mucho más larga.

F

E

D

OA B

C

Page 710: COMPENDIUM OME - ToomatesEotvos de Hungría, que se iniciaron en 1894, precisamente durante la efervescencia de fin de siglo, consecuencia de la cual fue también el proceso iniciado

XLIV Olimpiada Matemática Española Fase nacional 2008 (Valencia)

PRIMERA SESIÓN (28 de marzo)

• 1.- Halla dos enteros positivos a y b conociendo su suma y su mínimo común múltiplo. Aplícalo en el caso de que la suma sea 3972 y el mínimo común múltiplo .985928

• 2.- Prueba que para cualesquiera números reales ba, tales que ,1,0 << ba se cumple la desigualdad siguiente:

.2)1()1()1)(1( 2222 <−−+−−++ bababaab

• 3.- Sea 3≥p un número primo. Se divide cada lado de un triángulo en p partes iguales y se une cada uno de los puntos de división con el vértice opuesto. Calcula el número máximo de regiones, disjuntas dos a dos, en que queda dividido el triángulo.

No está permitido el uso de calculadoras. Cada problema se valora hasta 7 puntos.

El tiempo de cada sesión es de tres horas y media.

Page 711: COMPENDIUM OME - ToomatesEotvos de Hungría, que se iniciaron en 1894, precisamente durante la efervescencia de fin de siglo, consecuencia de la cual fue también el proceso iniciado

XLIV Olimpiada Matemática Española Fase nacional 2008 (Valencia)

SEGUNDA SESIÓN (29 de marzo)

• 4.- Sean p y q dos números primos positivos diferentes. Prueba que existen enteros positivos a y ,b tales que la media aritmética de todos los divisores positivos del número baqpn = es un número entero.

• 5.- Dada una circunferencia y en ella dos puntos fijos A, B, otro variable P y una recta ;r se trazan las rectas PA y PB que cortan a r en C y D respectivamente. Determina dos puntos fijos de ,r M y N, tales que el producto ·CM DN sea constante al variar P.

• 6.- A cada punto del plano se le asigna un solo color entre siete colores distintos. ¿Existirá un trapecio inscriptible en una circunferencia cuyos vértices tengan todos el mismo color?

No está permitido el uso de calculadoras. Cada problema se valora hasta 7 puntos.

El tiempo de cada sesión es de tres horas y media.

Page 712: COMPENDIUM OME - ToomatesEotvos de Hungría, que se iniciaron en 1894, precisamente durante la efervescencia de fin de siglo, consecuencia de la cual fue también el proceso iniciado

1

XLIV Olimpiada Matemática Española Fase nacional 2008 (Valencia)

PRIMERA SESIÓN (28 de marzo) 1.- Halla dos enteros positivos a y b conociendo su suma y su mínimo común múltiplo. Aplícalo en el caso de que la suma sea 3972 y el mínimo común múltiplo .985928 SOLUCIÓN: Sea p un número primo que divide a la suma ba + y a su mínimo común múltiplo [ ].,ba Como [ ]bap , al menos divide a uno de los dos enteros a ó .b Si ,ap al

dividir p a la suma ,ba + también .bp (Obviamente el mismo razonamiento vale si

hubiéramos supuesto que ).bp Por tanto podemos dividir los dos números a y b por p y también su mínimo común múltiplo [ ],,ba para obtener dos enteros 1a y 1b

tales que pbb

paa == 11 , y [ ] [ ].,, 11 p

baba = Sea el máximo común divisor de a y ,b

( ).,bad = Repitiendo el proceso anterior llegaremos a obtener dos enteros A y B tales que dBbdAa == , y ( ) .1, =BA Entonces [ ] ., ABBA = Ahora es fácil

determinar A y B a partir del sistema de ecuaciones [ ] .,

⎪⎪⎩

⎪⎪⎨

=

+=+

dbaAB

dbaBA

Es decir A y

B son las raíces de la ecuación de segundo grado [ ] .0,)(2 =++− batbatd Observamos que el discriminante de esta ecuación es no negativo. En efecto: [ ] .0)(4)(,4)( 222 ≥−=−+=−+=∆ baabbabadba Si a y b son distintos, la ecuación anterior tiene por soluciones los dos enteros

positivos daA = y .

dbB =

En particular cuando 3972=+ ba y [ ] ,985928, =ba tenemos que .4)985928,3972( ==d Por tanto Aa 4= y Bb 4= siendo A y B las raíces de la ecuación .098592839724 2 =+− tt Es decir 491=A y 502=B y los números buscados son 1964=a (año de la primera OME) y 2008=b (año de la actual edición de la OME).

Page 713: COMPENDIUM OME - ToomatesEotvos de Hungría, que se iniciaron en 1894, precisamente durante la efervescencia de fin de siglo, consecuencia de la cual fue también el proceso iniciado

2

XLIV Olimpiada Matemática Española Fase nacional 2008 (Valencia)

PRIMERA SESIÓN (28 de marzo) 2.- Prueba que para cualesquiera números reales ba, tales que ,1,0 << ba se cumple la desigualdad siguiente:

.2)1()1()1)(1( 2222 <−−+−−++ bababaab

SOLUCIÓN:

Se verifica que 3 xx < para todo ).1,0(∈x Teniendo en cuenta que ,12

0 <+

<ba

utilizando la desigualdad anterior y aplicando la desigualdad entre las medias aritmética y geométrica, se tiene:

232

223

bababa

baabbaab +=

⎟⎠⎞

⎜⎝⎛ +

++≤⎟

⎠⎞

⎜⎝⎛ +

<⎟⎠⎞

⎜⎝⎛ +

y

.2

13

2111

21)1()1(

21)1()1( 3

bababa

babababa

+−=

+−+−+−

≤⎟⎠⎞

⎜⎝⎛ +−−−<⎟

⎠⎞

⎜⎝⎛ +−−−

Sumando las expresiones anteriores resulta

,12

1)1()1(2

<⎟⎠⎞

⎜⎝⎛ +−−−+⎟

⎠⎞

⎜⎝⎛ + bababaab

o equivalentemente

( ) ,1)1()1()1)(1(2

1 2222 <−−+−−++ bababaab

de donde se obtiene inmediatamente la desigualdad del enunciado.

Page 714: COMPENDIUM OME - ToomatesEotvos de Hungría, que se iniciaron en 1894, precisamente durante la efervescencia de fin de siglo, consecuencia de la cual fue también el proceso iniciado

3

XLIV Olimpiada Matemática Española Fase nacional 2008 (Valencia)

PRIMERA SESIÓN (28 de marzo) 3.- Sea 3≥p un número primo. Se divide cada lado de un triángulo en p partes iguales y se une cada uno de los puntos de división con el vértice opuesto. Calcula el número máximo de regiones, disjuntas dos a dos, en que queda dividido el triángulo. SOLUCIÓN: En primer lugar veremos que tres de estos segmentos (cevianas) no pueden ser concurrentes. Sea el triángulo ABC y ZYX ,, puntos de las divisiones interiores de los lados ABACBC ,, respectivamente. Si BYAX , y CZ fueran concurrentes aplicando el teorema de Ceva tendríamos

.1=⋅⋅YACY

XCBX

ZBAZ

Por otro lado, por la forma en que hemos construido los puntos de división, existen enteros positivos ,1,...,2,1,, −∈ pmlk tales que

.,,mp

mYACY

lpl

XCBX

kpk

ZBAZ

−=

−=

−=

Sustituyendo en la expresión anterior

),()()( mplpkpmlk −−−= o equivalentemente,

).()()(2 lpkpplkpmpmlk −−=−−+ De aquí resulta que p divide al producto ,klm que es imposible y nuestra afirmación inicial queda probada. Dibujando las cevianas desde el vértice A el triángulo ABC queda dividido en p triángulos. Las cevianas trazadas desde B dividen cada uno de los p triángulos

anteriores en p partes disjuntas, teniendo en total 2p regiones. Cada ceviana trazada desde el vértice C aumenta el número de regiones en un número exactamente igual a su número de intersecciones con las rectas que encuentra (incluido el lado AB ). Es decir,

.121)1(2 −=+− pp Por tanto, el número máximo de regiones disjuntas dos a dos, en que queda dividido el triángulo ABC es .133)12)(1( 22 +−=−−+ ppppp

Page 715: COMPENDIUM OME - ToomatesEotvos de Hungría, que se iniciaron en 1894, precisamente durante la efervescencia de fin de siglo, consecuencia de la cual fue también el proceso iniciado

1

XLIV Olimpiada Matemática Española Fase nacional 2008 (Valencia)

SEGUNDA SESIÓN (29 de marzo)

4.- Sean p y q dos números primos positivos diferentes. Prueba que existen enteros positivos a y ,b tales que la media aritmética de todos los divisores positivos del número baqpn = es un número entero. SOLUCIÓN: La suma de todos los divisores de n viene dada por la fórmula

),...1()...1( 22 ba qqqppp ++++++++ como se puede comprobar desarrollando los paréntesis. El número n tiene

)1)(1( ++ ba divisores positivos y la media aritmética de todos ellos es

.)1()1(

)...1()...1( 22

++++++++++

=ba

qqqpppmba

Si p y q son ambos impares, tomando pa = y ,qb = es fácil ver que m es un entero. Efectivamente: cada factor ppppp +++++ ...1 32 y qqqqq +++++ ...1 32 tiene un número par de sumandos y por ejemplo, el primero se puede escribir como sigue

)....1()1()1(...)1()1(...1

12

1232

++++

=++++++=+++++p

pp

pppppppppppp

Análogamente el segundo factor )....1()1(...1 1232 −++++=+++++ qq qqqqqqq Entonces ),...1()...1( 142142 −− ++++++++= qp qqqpppm que es un entero positivo. Si 2=p y q es impar, se eligen qb = y ....11 142 −++++=+ qqqqa Entonces

aq

qqqq

qqqqqqqqm

q

2...221)1()...1(

)...1()2...221( 2142

32...2 142

++++=+++++

+++++++++= −

+++ −

, que es

entero. Para 2=q y p impar, análogamente al caso anterior se eligen pa = y

142 ... −+++= ppppb y m es entero. Alternativamente y de una manera casi directa, se obtiene una solución completa observando que si p y q son primos impares se toman 1== ba y si 2=p y q primo

impar, entonces se consideran 2

1−=

qa y .1=b

Page 716: COMPENDIUM OME - ToomatesEotvos de Hungría, que se iniciaron en 1894, precisamente durante la efervescencia de fin de siglo, consecuencia de la cual fue también el proceso iniciado

2

XLIV Olimpiada Matemática Española Fase nacional 2008 (Valencia)

SEGUNDA SESIÓN (29 de marzo) 5.- Dada una circunferencia y en ella dos puntos fijos A, B, otro variable P y una recta ;r se trazan las rectas PA y PB que cortan a r en C y D respectivamente. Determina dos puntos fijos de ,r M y N, tales que el producto ·CM DN sea constante al variar P. SOLUCIÓN: Trazamos las paralelas a r por A y B que cortan a la circunferencia en A y ´B respectivamente de modo que ´´BBAA es un trapecio isósceles. Las intersecciones de ´AB y ´BA con r determinan los puntos M y N buscados. En efecto, los triángulos AMC y DNB (sombreados en la figura) son semejantes ya que tienen dos ángulos iguales:

,´ NDBBPBMAC ∠=∠=∠ donde la primera igualdad es cierta por ser ángulos inscritos en el mismo arco y la segunda por ser ´BB paralela a .r

,´ DNBBABAMC ∠=∠=∠ con argumentos análogos a los anteriores. Estableciendo la proporcionalidad de los lados resulta

,BNAMNDMCBNND

MCAM

⋅=⋅⇔=

cantidad que no depende de P.

r NM

B'

A'

C

DA

B

P

Page 717: COMPENDIUM OME - ToomatesEotvos de Hungría, que se iniciaron en 1894, precisamente durante la efervescencia de fin de siglo, consecuencia de la cual fue también el proceso iniciado

3

Se observa que si la recta r pasa por el punto ,A ,CAM == no se forma el triángulo

.AMC En este caso 0=CM y el producto ,0=⋅DNCM es constante. Análogamente este producto es cero si la recta r pasa por B o por los puntos A y B en cuyo caso

.0== DNCM

XLIV Olimpiada Matemática Española Fase nacional 2008 (Valencia)

SEGUNDA SESIÓN (29 de marzo) 6.- A cada punto del plano se le asigna un solo color entre siete colores distintos. ¿Existirá un trapecio inscriptible en una circunferencia cuyos vértices tengan todos el mismo color? SOLUCIÓN: La idea inicial es considerar una circunferencia C de radio r y sobre ella bloques de 8 puntos 821 ,...,, AAA igualmente espaciados; es decir que los arcos 7,...,11 =+ iAA ii tengan igual longitud 0>λ (que se elegirá convenientemente) para cada uno de los bloques. Se elige un sentido dado (por ejemplo, el antihorario). Se disponen entonces, en este sentido antihorario, 50177 =+× bloques de 817 =+ puntos cada uno en la semicircunferencia superior de ,C tales que dos bloques distintos no se intersequen o solapen, para lo cual se toma λ suficientemente pequeño, por

ejemplo .400

0 rπλ <<

Se observa que al menos hay dos puntos del mismo color en cada bloque. Se eligen dos de esos puntos y su color se le asocia al bloque. Y la distancia entre estos dos puntos que es uno de los siete números ,7,6,5,4,3,2,1, ∈= nndn λ se le asigna también al bloque. De este modo a cada uno de los 50 bloques se le hace corresponder el par (color, distancia), indicado anteriormente. Como el número total de posibles pares es

,49 por el principio el palomar, existirán dos bloques R y Q a los que se les asocia el mismo par (color, distancia). Por tanto los cuatro puntos determinados por estos dos bloques tienen el mismo color. Y como los dos puntos del bloque R distan igual que los dos puntos del bloque ,Q estando los cuatro puntos sobre la circunferencia

,C necesariamente, estos cuatro puntos son los vértices de un trapecio inscriptible. NOTA: Este mismo razonamiento se podría hacer considerando un arco de circunferencia de radio r de

longitud .20, rll π≤< En este caso se tomaría .400

0, l<< λλ

Y también se podría generalizar a un número de colores )2( ≥cc cualesquiera, considerando 1+c

puntos en vez de 17 + y 12 +c bloques disjuntos de 1+c puntos cada uno, en vez de 172 + bloques

de 17 + puntos cada uno. Y ahora λ debe cumplir que .)1)(1(

0 2 ++<<

cclλ

Page 718: COMPENDIUM OME - ToomatesEotvos de Hungría, que se iniciaron en 1894, precisamente durante la efervescencia de fin de siglo, consecuencia de la cual fue también el proceso iniciado

Fase Nacional de la XLV Olimpiada Matemática Española Sant Feliu de Guixols (Girona), 27 de marzo de 2009

PRIMERA SESIÓN

1.- Halla todas las sucesiones finitas de números naturales consecutivos con tales que

n,,...,, 21 naaa ,3≥n .2009...21 =+++ naaa

2.- Sean un triángulo acutángulo, ABC I el centro del círculo inscrito en el triángulo

,ABC r su radio y R el radio del círculo circunscrito al triángulo Se traza la altura con perteneciente al lado Demuestra que

.ABC,ahAD = D .BC

).2()2(2 rhhRDI aa −−=

3.- Se pintan de rojo algunas de las aristas de un poliedro regular. Se dice que una coloración de este tipo es buena, si para cada vértice del poliedro, existe una arista que concurre en dicho vértice y no está pintada de rojo. Por otra parte, se dice que una coloración donde se pintan de rojo algunas de las aristas de un poliedro regular es completamente buena, si, además de ser buena, ninguna cara del poliedro tiene todas sus aristas pintadas de rojo. ¿Para qué poliedros regulares es igual el número máximo de aristas que se pueden pintar en una coloración buena y en una completamente buena? Justifica la respuesta.

No está permitido el uso de calculadoras. Cada problema vale siete puntos.

El tiempo de cada sesión es de tres horas y media.

Page 719: COMPENDIUM OME - ToomatesEotvos de Hungría, que se iniciaron en 1894, precisamente durante la efervescencia de fin de siglo, consecuencia de la cual fue también el proceso iniciado

Fase Nacional de la XLV Olimpiada Matemática Española Sant Feliu de Guixols (Girona), 28 de marzo de 2009

SEGUNDA SESIÓN

4.- Determina justificadamente todos los pares de números enteros que verifican la ecuación

),( yx.200942 =− yx

5.- Sean números reales positivos tales que cba ,, .1=abc Prueba la desigualdad

siguiente

+⎟⎠⎞

⎜⎝⎛+

2

1 aba

+⎟⎠⎞

⎜⎝⎛

+

2

1 bcb

43

1

2

≥⎟⎠⎞

⎜⎝⎛+ cac

6.- En el interior de una circunferencia de centro y radio O ,r se toman dos puntos A y simétricos respecto de Se considera ,B .O P un punto variable sobre esta circunferencia y se

traza la cuerda perpendicular a Sea el punto simétrico de ´,PP .AP C B respecto de Halla el lugar geométrico del punto intersección de con al variar sobre la circunferencia.

´.PP,Q ´PP ,AC P

No está permitido el uso de calculadoras. Cada problema vale siete puntos.

El tiempo de cada sesión es de tres horas y media.

Page 720: COMPENDIUM OME - ToomatesEotvos de Hungría, que se iniciaron en 1894, precisamente durante la efervescencia de fin de siglo, consecuencia de la cual fue también el proceso iniciado

Fase Nacional de la XLV Olimpiada Matemática Española Sant Feliu de Guixols (Girona), 27 de marzo de 2009

PRIMERA SESIÓN

SOLUCIONES

PROBLEMA 1.- Halla todas las sucesiones finitas de números naturales consecutivos con tales que

n,,...,, 21 naaa ,3≥n .2009...21 =+++ naaa

Primera solución:

Supongamos que es la suma de números naturales consecutivos empezando por Entonces

N n.1+k

=++++++= )(...)2()1( nkkkN [ ] [ ] =+++−++++++++ knkkk ...21)(...)1(...21

.2

)12(2

)1(2

)1)(( ++=

+−

+++ nknkknknk

Teniendo en cuenta que 41492877200912009 ×=×=×= se tienen los

siguientes casos:

(1) Si y 7=n ( ) 2872

12=

++ nk resulta 283=k con lo que

.2902892882872862852842009 ++++++=

(2) Si 72=

n y ,28712 =++ nk resulta 136=k con lo que

.150...1381372009 +++=

(3) Si y 41=n ( ) 492

12=

++ nk resulta 28=k con lo que

.69...3130292009 ++++=

(4) Si y 49=n ( ) 412

12=

++ nk resulta 16=k con lo que

.65...1918172009 ++++=

(5) Los otros casos dan valores de k que no verifican el enunciado.

Page 721: COMPENDIUM OME - ToomatesEotvos de Hungría, que se iniciaron en 1894, precisamente durante la efervescencia de fin de siglo, consecuencia de la cual fue también el proceso iniciado

Segunda solución:

Claramente, es ,11 −+= naan de donde .2

)1(2009 1−

+=nnan Entonces

,2

)1(2

)1(20092−

>−

≥nnn luego ,40181<−n y al ser

resulta que ,6339694018409664 22 =>>= ,644018 < con lo que .65<n Supongamos en primer lugar que n es impar. Entonces, obviamente divide a

y puede tomar los valores pues cualquier otro divisor impar de es mayor o igual que

n4172009 2 ×= n 41,7 ó ,49

2009 .287417 =× Se obtiene entonces

,2

120091

−−=

nn

a con valores respectivos y es decir, se obtienen

las tres sucesiones

29,284 ,17

;290,...,285,284 ;69,...,30,29 .65,...,18,17

No hay otras sucesiones con un número impar de términos.

Supongamos finalmente que n es par. Entonces 2n divide a con lo que

ya que cualquier otro valor de ha de ser mayor o igual que lo que no es posible. Entonces

,2009

,14=n n,6582412 >=× ,1371 =a y la única sucesión

con número par de términos es .150,...,138,137

Page 722: COMPENDIUM OME - ToomatesEotvos de Hungría, que se iniciaron en 1894, precisamente durante la efervescencia de fin de siglo, consecuencia de la cual fue también el proceso iniciado

Fase Nacional de la XLV Olimpiada Matemática Española Sant Feliu de Guixols (Girona), 27 de marzo de 2009

PRIMERA SESIÓN

SOLUCIONES PROBLEMA 2 .- Sean un triángulo acutángulo, ABC I el centro del círculo inscrito en el triángulo ,ABC r su radio y R el radio del círculo circunscrito al triángulo Se traza la altura con perteneciente al lado Demuestra que

.ABC,ahAD = D .BC

).2()2(2 rhhRDI aa −−= Primera solución: Sean E y M las proyecciones ortogonales de I sobre y respectivamente.

BC ,AD

Se tiene: ;

2Asen

rAI = pSr = ⇒

2Asenp

SAI⋅

= (1) donde, evidentemente,

es el área del triángulo y S ABC p es su semiperímetro.

Por otra parte, ,2

cos22

AAsenbcsenAbcS ⋅⋅=⋅

= así que (1) se puede escribir

como ,2cos

p

AbcAI

⋅= y ya que ,)(

2cos2

bcappA −

= obtenemos

papbcAI )(2 −

= (2).

Teniendo en cuenta que ,2 ahRbc ⋅= ,rSp = ,2

ahSa = la expresión (2) se

escribe como )2(2212

2

122 rhRhrhR

rShS

hRAI aa

aa

a −=⎟⎟⎠

⎞⎜⎜⎝

⎛−⋅=

⎟⎟⎟⎟

⎜⎜⎜⎜

−⋅=

(3).Como el cuadrilátero IEDM es un rectángulo, .rIEMD == Aplicando el teorema de Pitágoras generalizado a ADI tenemos ⇔⋅−+= AMhAIhDI aa 2222 ),(2222 MDhhAIhDI aaa −−+=y teniendo en cuenta los resultados anteriormente obtenidos resulta, finalmente,

c.q.d. ),2()2()(2)2(222 rhhRrhhrhRhDI aaaaaa −−=−−−+=

Page 723: COMPENDIUM OME - ToomatesEotvos de Hungría, que se iniciaron en 1894, precisamente durante la efervescencia de fin de siglo, consecuencia de la cual fue también el proceso iniciado

Segunda solución: Sean las longitudes de los lados y cba ,, CABC , AB respectivamente, y sea T el punto donde la circunferencia inscrita es tangente al lado Por el teorema de Pitágoras, al ser

.BC,// DTADIT ⊥ se tiene que

222222 ADATrDTrDI −+=+= y la igualdad a demostrar es equivalente a .)(24 22

ahrRrRrAT +=++Ahora bien, llamando al área de es conocido que S ,ABC

.2

)()()()(41

2)(

4ahacbabacacbcbacbar

RabcS =−+−+−+++=

++==

De aquí se deduce que ,24cba

abcRr++

= ,)()()(2

cbacbabacacbr

++−+−+−+

=

bchR a =2 y .2

)()()(2a

cbabacacbhR a−+−+−+

=

Obsérvese que .4

2224222

2 cbacabcabrRr −−−++=+ Ahora bien, se

sabe que ,2

bacBT −+= .

2cbaCT −+

=

Por el teorema de Stewart, tenemos que

=⋅−⋅+⋅

= CTBTBC

ABCTACBTAT22

2

acbcbbcacb

2)()(

4233 2222 +−

−−−+

con lo que

,2

)()(2

42222

22

acbcbcaabacbrRrAT +−

=++−+

=++

e identificando términos se comprueba que esto coincide con el valor de ahrR )(2 +

y de este modo la igualdad requerida queda demostrada. Tercera solución: Nótese en primer lugar que

,2

22

2

222)(222

2

2

2

RIA

AsenR

rAsen

CsenBsenr

aracb

aarSrha ===

−+=

−=−

donde es el área de y se ha utilizado que S ABC .222

4 CsenBsenAsenRr =

Sea ahora el punto P en el que la paralela a por BC I corta a la altura Claramente,

.AD

).2()2()2()( 22222 rhhRrhhIArhIArDI aaaaa −−=−−=−−+=

Page 724: COMPENDIUM OME - ToomatesEotvos de Hungría, que se iniciaron en 1894, precisamente durante la efervescencia de fin de siglo, consecuencia de la cual fue también el proceso iniciado

Fase Nacional de la XLV Olimpiada Matemática Española Sant Feliu de Guixols (Girona), 27 de marzo de 2009

PRIMERA SESIÓN

SOLUCIONES

PROBLEMA 3.- Se pintan de rojo algunas de las aristas de un poliedro regular. Se dice que una coloración de este tipo es buena, si para cada vértice del poliedro, existe una arista que concurre en dicho vértice y no está pintada de rojo. Por otra parte, se dice que una coloración donde se pintan de rojo algunas de las aristas de un poliedro regular es completamente buena, si además de ser buena, ninguna cara del poliedro tiene todas sus aristas pintadas de rojo. ¿Para qué poliedros regulares es igual el número máximo de aristas que se pueden pintar en una coloración buena y en una completamente buena? Justifica la respuesta.

Solución: Claramente, las coloraciones completamente buenas son un subconjunto de las coloraciones buenas, con lo que si el máximo número de aristas que se pueden pintar de rojo para obtener una coloración buena se puede alcanzar con una coloración completamente buena, la pregunta del enunciado tiene respuesta afirmativa. NOTA: En cada caso véase el recuadro de las figuras al final de la solución. En el caso de un tetraedro, existen 6 aristas, tales que en cada vértice confluyen

de ellas. El número máximo de aristas pintadas de rojo en una coloración

buena sería por lo tanto

3

,4632

=× pues en caso contrario existiría algún vértice

donde más de 32 de las aristas estuvieran pintadas de rojo, es decir, todas las

aristas estarían pintadas de rojo. La figura muestra una coloración completamente buena de un tetraedro con aristas rojas (el tetraedro ha sido deformado para poder ser dibujado en el plano).

4

De igual forma, en el cubo existen 12 aristas, tales que en cada vértice confluyen de ellas. El número máximo de aristas pintadas de rojo en una

coloración buena sería, por lo tanto,

3

.81232

=× La figura muestra una

coloración completamente buena con 8 aristas rojas. Finalmente, en el dodecaedro existen 30 aristas, tales que en cada vértice confluyen de ellas. El número máximo de aristas pintadas de rojo en una 3

Page 725: COMPENDIUM OME - ToomatesEotvos de Hungría, que se iniciaron en 1894, precisamente durante la efervescencia de fin de siglo, consecuencia de la cual fue también el proceso iniciado

coloración buena sería, por lo tanto, .203032

=× La figura muestra una

coloración completamente buena de un dodecaedro con 12 aristas rojas. De lo anterior se deduce que para el tetraedro, el cubo y el dodecaedro, el número máximo de aristas rojas en una coloración buena se alcanza con una coloración completamente buena. En el octaedro existen 12 aristas, tales que en cada vértice confluyen 4 de ellas. El número máximo de aristas pintadas de rojo en una coloración buena es

por lo tanto .91243

=× La figura muestra una coloración buena de un octaedro

con aristas rojas. Ahora bien, como cada arista pertenece a dos caras, sumando para las 8 caras el número de aristas pintadas de rojo en dicha cara, obtenemos con lo que hay, al menos, dos caras con 3 aristas pintadas de rojo y una coloración buena con el número máximo de 9 aristas pintadas de rojo nunca puede ser completamente buena.

9

,282×18 +=

Finalmente, en el icosaedro existen 30 aristas, tales que en cada vértice confluyen de ellas. El número máximo de aristas pintadas de rojo en una

coloración buena es por lo tanto

5

.243054

=× La figura muestra una

coloración buena de un icosaedro con 24 aristas rojas. Ahora bien, eso quiere decir, que sumando el número de aristas rojas de cada cara para todas las caras, obtenemos es decir, existen, al menos, caras con todas sus aristas rojas, y una coloración buena con el número máximo de aristas pintadas de rojo nunca puede ser completamente buena.

,820248 +×= 824

Por lo tanto, los poliedros regulares que tienen la propiedad descrita en el enunciado son el tetraedro, el cubo y el dodecaedro (es decir, los poliedros tales que en cada vértice confluyen exactamente 3 aristas) y los que no la tienen son el octaedro e icosaedro (en cuyos vértices confluyen más de 3 aristas).

Page 726: COMPENDIUM OME - ToomatesEotvos de Hungría, que se iniciaron en 1894, precisamente durante la efervescencia de fin de siglo, consecuencia de la cual fue también el proceso iniciado

FIGURAS FIGURAS

Page 727: COMPENDIUM OME - ToomatesEotvos de Hungría, que se iniciaron en 1894, precisamente durante la efervescencia de fin de siglo, consecuencia de la cual fue también el proceso iniciado

Fase Nacional de la XLV Olimpiada Matemática Española Sant Feliu de Guixols (Girona), 28 de marzo de 2009

SEGUNDA SESIÓN

SOLUCIONES

PROBLEMA 4. - Determina justificadamente todos los pares de números enteros

que verifican la ecuación ),( yx .200942 =− yx

Solución: Dada una solución cualquiera, es claro que también son soluciones

y con lo que se puede asumir sin pérdida de generalidad que Supongamos entonces que es así. Es claro que

),( yx),(),,( yxyx −− ),,( yx −−

.0, ≥yx.417)()( 222 ⋅=+− yxyx

Si y no son primos entre sí, su máximo común divisor al cuadrado divide a luego es 7 y divide a

y a con lo que existen enteros no negativos y v tales que

2yx − 2yx +,4172009 2 ⋅=

xyxyx 2)()( 22 =−++ ,2)()( 222 yyxyx =−−+u ,7ux = y

Como ambos factores han de ser enteros, se tiene que

y con lo que

vy 7=.41)7)(7( 22 =−+ vuvu

417 2 =+ vu ,17 2 =− vu 21=u y .7

102 =v No existen pues

soluciones enteras en este caso. Si y son primos entre sí, un posible caso es que y

con lo que absurdo pues Resta entonces tan sólo el caso en que y que produce

con lo que la única solución con enteros no negativos es e y las únicas soluciones en enteros son

2yx − 2yx + 12 =− yx,20092 =+ yx ,10042 =y .32100496131 22 <<=

412 =− yx ,492 =+ yx,45=x ,42 =y

45=x ,2=y ).2,45(),( ±±=yx

Page 728: COMPENDIUM OME - ToomatesEotvos de Hungría, que se iniciaron en 1894, precisamente durante la efervescencia de fin de siglo, consecuencia de la cual fue también el proceso iniciado

Fase Nacional de la XLV Olimpiada Matemática Española Sant Feliu de Guixols (Girona), 28 de marzo de 2009

SEGUNDA SESIÓN

SOLUCIONES

PROBLEMA 5. - Sean números reales positivos tales que Prueba la desigualdad siguiente

cba ,, .1=abc

+⎟⎠⎞

⎜⎝⎛+

2

1 aba

+⎟⎠⎞

⎜⎝⎛+

2

1 bcb

43

1

2

≥⎟⎠⎞

⎜⎝⎛+ cac

Solución:

Como ,1=abc entonces .11

222

⎟⎠⎞

⎜⎝⎛+

=⎟⎠⎞

⎜⎝⎛

+=⎟

⎠⎞

⎜⎝⎛+ c

cacabc

caab

a Análogamente se

obtienen 22

11⎟⎠⎞

⎜⎝⎛+

=⎟⎠⎞

⎜⎝⎛+ a

abbc

b y .11

22

⎟⎠⎞

⎜⎝⎛+

=⎟⎠⎞

⎜⎝⎛+ b

bcca

c Por tanto la

desigualdad requerida se convierte en

+⎟⎠⎞

⎜⎝⎛+

2

1 aab

+⎟⎠⎞

⎜⎝⎛+

2

1 bbc ,

43

1

2

≥⎟⎠⎞

⎜⎝⎛+ cca equivalente a

.21

11131 222

≥⎥⎥⎦

⎢⎢⎣

⎡⎟⎠⎞

⎜⎝⎛+

+⎟⎠⎞

⎜⎝⎛+

+⎟⎠⎞

⎜⎝⎛+ c

cab

bca

ab

Usando ahora la desigualdad entre las medias aritmética y cuadrática, se obtiene

.1113

11113

1 222

⎥⎦

⎤⎢⎣

⎡⎟⎠⎞

⎜⎝⎛+

+⎟⎠⎞

⎜⎝⎛+

+⎟⎠⎞

⎜⎝⎛+

≥⎥⎥⎦

⎢⎢⎣

⎡⎟⎠⎞

⎜⎝⎛+

+⎟⎠⎞

⎜⎝⎛+

+⎟⎠⎞

⎜⎝⎛+ c

cab

bca

abc

cab

bca

ab

Así es suficiente demostrar que 23

111≥

++

++

+ cca

bbc

aab o equivalentemente

,23

)1()1()1(≥

++

++

+ cbabc

baabc

acabc que a su vez equivale a que

.23

)1(1

)1(1

)1(1

≥+

++

++ cbbaac

Page 729: COMPENDIUM OME - ToomatesEotvos de Hungría, que se iniciaron en 1894, precisamente durante la efervescencia de fin de siglo, consecuencia de la cual fue también el proceso iniciado

Poniendo zyb

yxa == , y

xzc = en la última desigualdad resulta

+⎟⎟⎠

⎞⎜⎜⎝

⎛+

−1

zx

yx

+⎟⎠⎞

⎜⎝⎛ +

−1

xy

zy .

23

1

≥⎟⎟⎠

⎞⎜⎜⎝

⎛+

yz

xz Sustituyendo ahora

yx1,1

== βα y

,1z

=γ se llega a la desigualdad de Nessbit .23

≥+

++

++ βα

γαγ

βγβ

α

La igualdad se alcanza si y sólo si .1=== cba

Page 730: COMPENDIUM OME - ToomatesEotvos de Hungría, que se iniciaron en 1894, precisamente durante la efervescencia de fin de siglo, consecuencia de la cual fue también el proceso iniciado

Fase Nacional de la XLV Olimpiada Matemática Española Sant Feliu de Guixols (Girona), 28 de marzo de 2009

SEGUNDA SESIÓN

SOLUCIONES

PROBLEMA 6. - En el interior de una circunferencia de centro O y radio ,r se toman dos puntos y simétricos respecto de Se considera un punto variable A ,B .O P sobre esta circunferencia y se traza la cuerda perpendicular a Sea el punto simétrico de

´,PP .AP CB respecto de Halla el lugar geométrico del punto intersección de ´.PP ,Q

´PP con al variar ,AC P sobre la circunferencia.

Primera solución: Establezcamos primero que es constante. ACMétodo 1. Se obtiene C a partir de A aplicando un giro de con centro en O

seguido de la simetría de eje PP º180´.

1 A

O AB

PC

Q

A'

P'

e1

e2

MDescomponiendo el giro en producto de dos simetrías de ejes perpendiculares e paralelo a P y

perpendicular a resulta que el triángulo es rectángulo en

2e ,APCAA A y además:

2´AA = ,MP2´CA = ;OM de donde es decir ;4444 2222 rOPMPOM ==+2AC =

2AC ,r= con independencia de la posición de P .

Método 2

O AB

P

P '

C

P ''

Prolongamos hasta que corte de nuevo a la circunferencia en P Se tiene CP

PA´´. ´´ P == ´´.APB

Además BP´ ´´;PPCA ´

es paralelo a luego el segmento es la imagen del segmento ´´PP mediante la

traslación de vector AP´´ P y como ´´´PPP∠ es recto y ´´´PP es un diámetro, resulta

.2´´´P rPAC == Finalmente, al ser PP la mediatriz de ´ ,BC

Page 731: COMPENDIUM OME - ToomatesEotvos de Hungría, que se iniciaron en 1894, precisamente durante la efervescencia de fin de siglo, consecuencia de la cual fue también el proceso iniciado

;QBQC = se deduce entonces que rACQAQCQAQB 2==+=+ y Q describe la elipse de focos y A B y constante La recta .2 r ´PP es la tangente en a la elipse. Q

Segunda solución:

Tomamos 1=r y unos ejes de coordenadas en los que la ecuación de la

circunferencia es y las coordenadas de con

,122 =+ yx ),0,(),0,( aBaA −.10 << a

En vez de empezar por sea con la condición Por las condiciones del problema, es el punto medio de llamando

a las coordenadas de se tiene Entonces la ecuación de la

recta CA es

,P ),(´ 00 yxP .120

20 =+ yx

´P ;BC ,( 11 yx

,C .22

01

01

⎩⎨⎧

=+=

yyaxx

),(0

0 axxy

y −= es decir .0000 =+− ayxyyx

)

Las pendientes de BP´ y de PP´ son respectivamente ax

y+0

0 y .0

0

yax +

− Por

tanto la ecuación de PP´ es .01)( 000 =+−++ axaxxyy Las coordenadas del punto intersección de y son: ,Q AC PP´

.1

)1(,

1 0

20

0

0⎟⎟⎠

⎞⎜⎜⎝

⎛+−

++

axay

axax

Q Denotando por yx, a las coordenadas de Q y

despejando los valores de e se obtiene 0x 0y .1

,1 00 −

−ax

y=

−−

= yax

xax

Imponiendo ahora la condición se llega a ,120

20 =+ yx

1)1()1(

)(2

2

2

2

=−

+−−

axy

axxa

y mediante operaciones se transforma en la ecuación ,11 2

22 =

−+

ayx que es la

ecuación de una elipse. Tercera solución: Demostraremos en primer lugar que, dados dos puntos del plano, el conjunto de los puntos (del mismo plano) tales que

BA,P 22 PBPA + es constante

y mayor que es una circunferencia de centro el punto medio de y que tiene a los puntos

,2AB ABA y B en su interior.

En efecto, supongamos )0,(),0,( dBdA −== y sea cualquier punto. Se tiene entonces

),( yxP =

.222 22222 dyxPBPA ++=+

Así que si se tiene que ,4 222 dkPBPA ≥=+ .2

2222 ddkyx ≥−=+

Page 732: COMPENDIUM OME - ToomatesEotvos de Hungría, que se iniciaron en 1894, precisamente durante la efervescencia de fin de siglo, consecuencia de la cual fue también el proceso iniciado

Sea entonces ahora R el punto donde corta a BC ´PP (que es perpendicular a ). Este punto BC R satisface

,22222 APARBRBPPR −=−= luego R está en la circunferencia y es distinto de , con lo que P ´.PR =Ahora bien, se tiene

,22

´)(´2

´)(´´222222 ABkBPAPPPkBPAPBPAPBPAP −

=−−−

=−−+

=⋅

donde .´´ 2222 BPAPBPAPk +=+=Además, la potencia de respecto de la circunferencia es A

,2

22

2222 ABkdkdr −=−=−

con lo que el segundo punto en el que S AP corta a la circunferencia es tal que ´.´ CPBPAS ==Como ,´´ APPPCP ⊥⊥ se tiene que es paralelo a y es un paralelogramo.

AS ´CP CASP´

Finalmente, ,442´´2´´ 222222222 rdkABkBPAPAPASASAPPSPP =−=−++=⋅⋅++=+

es decir, Como 'P S AC r= = 2 . ,2 rACBQAQ ==+ el lugar de es la elipse interiormente tangente a la circunferencia dada, con

QA y B como

focos.

Page 733: COMPENDIUM OME - ToomatesEotvos de Hungría, que se iniciaron en 1894, precisamente durante la efervescencia de fin de siglo, consecuencia de la cual fue también el proceso iniciado

OlimpiadaMatemáticaEspañola RSME

XLVI Olimpiada Matematica Espanola

Fase nacional 2010 (Valladolid)

Primera sesion (26 de marzo)

• Problema 1

Una sucesion pucelana es una sucesion creciente de dieciseis numeros impares positivosconsecutivos, cuya suma es un cubo perfecto. ¿Cuantas sucesiones pucelanas tienensolamente numeros de tres cifras?

• Problema 2

Sean N0 y Z el conjunto de todos los enteros no negativos y el conjunto de todos losenteros, respectivamente. Sea f : N0 → Z la funcion que a cada elemento n de N0 leasocia como imagen el entero f(n) definido por

f(n) = −f(⌊n

3

⌋)− 3

n

3

donde x es la parte entera del numero real x y x = x − x su parte decimal.Determina el menor entero n tal que f(n) = 2010.

NOTA: La parte entera de un numero real x, denotada por x es el mayor entero queno supera a x. Ası 1,98 = 1, −2,001 = −3, 7π − 8,03 = 13.

• Problema 3

Sea ABCD un cuadrilatero convexo. Sea P la interseccion de AC y BD. El angulo APD = 60. Sean E, F , G y H los puntos medios de los lados AB, BC, CD y DArespectivamente. Halla el mayor numero real positivo k tal que

EG + 3HF ≥ kd + (1 − k)s

siendo s el semiperımetro del cuadrilatero ABCD y d la suma de las longitudes de susdiagonales. ¿Cuando se alcanza la igualdad?

No esta permitido el uso de calculadoras.Cada problema se puntua sobre siete puntos.

El tiempo de cada sesion es de tres horas y media.

Page 734: COMPENDIUM OME - ToomatesEotvos de Hungría, que se iniciaron en 1894, precisamente durante la efervescencia de fin de siglo, consecuencia de la cual fue también el proceso iniciado

OlimpiadaMatemáticaEspañola RSME

XLVI Olimpiada Matematica Espanola

Fase nacional 2010 (Valladolid)

Segunda sesion (27 de marzo)

• Problema 4

Sean a, b, c tres numeros reales positivos. Demuestra que

a + b + 3c

3a + 3b + 2c+

a + 3b + c

3a + 2b + 3c+

3a + b + c

2a + 3b + 3c≥ 15

8

• Problema 5

Sea P un punto cualquiera de la bisectriz del angulo A en el triangulo ABC, y sean A′,B′, C′ puntos respectivos de las rectas BC, CA, AB, tales que PA′ es perpendicular aBC, PB′ es perpendicular a CA y PC′ es perpendicular a AB. Demuestra que PA′ yB′C′ se cortan sobre la mediana AM , siendo M el punto medio de BC.

• Problema 6

Sea p un numero primo y A un subconjunto infinito de los numeros naturales. Sea fA(n) elnumero de soluciones distintas de la ecuacion x1+x2+· · ·+xp = n, con x1, x2, . . . , xp ∈ A.¿Existe algun numero natural N tal que fA(n) sea constante para todo n > N ?

No esta permitido el uso de calculadoras.Cada problema se puntua sobre siete puntos.

El tiempo de cada sesion es de tres horas y media.

Page 735: COMPENDIUM OME - ToomatesEotvos de Hungría, que se iniciaron en 1894, precisamente durante la efervescencia de fin de siglo, consecuencia de la cual fue también el proceso iniciado

OlimpiadaMatemáticaEspañola RSME

XLVI Olimpiada Matematica Espanola

Fase nacional 2010 (Valladolid, 26 y 27 de Marzo)Soluciones oficiales

• Problema 1

Una sucesion pucelana es una sucesion creciente de dieciseis numeros impares positivos con-secutivos, cuya suma es un cubo perfecto. ¿Cuantas sucesiones pucelanas tienen solamentenumeros de tres cifras?

Solucion:Sea la sucesion n, n + 2, . . . , n + 30. Entonces la suma es 1

2 16(2n + 30) = 8(2n + 30). Portanto, es necesario que 2n + 30 sea un cubo perfecto. Ahora hay que contar el numero detales n que son impares y verifican 101 ≤ n ≤ 969. Los cubos pares entre 232 y 1968 son512, 1000 y 1728, que corresponden a valores de n de 241, 485 y 849. Por lo tanto hayexactamente tres sucesiones pucelanas.

• Problema 2

Sean N0 y Z el conjunto de todos los enteros no negativos y el conjunto de todos los enteros,respectivamente. Sea f : N0 → Z la funcion que a cada elemento n de N0 le asocia comoimagen el entero f(n) definido por

f(n) = −f(⌊n

3

⌋)− 3

n

3

donde x es la parte entera del numero real x y x = x−x su parte decimal. Determinael menor entero n tal que f(n) = 2010.

NOTA: La parte entera de un numero real x, denotada por x es el mayor entero que nosupera a x. Ası 1,98 = 1, −2,001 = −3, 7π − 8,03 = 13.

Solucion:Se prueba facilmente por induccion que, si n = (akak−1 . . . a0)3, entonces

f(n) =k∑

j=0j impar

aj −k∑

j=0j par

aj

En efecto, f(0) = 0, f(1) = −1, f(2) = −2.

Supongamos que, para todo n menor que 3t, f(n) =∑k

j=0j impar

aj −∑k

j=0j par

aj . Entonces, si

t =(bk . . . b0

)3, 3t =

(bk . . . b00

)3, 3t+1 =

(bk . . . b01

)3, 3t+2 =

(bk . . . b02

)3. Por lo tanto,

Page 736: COMPENDIUM OME - ToomatesEotvos de Hungría, que se iniciaron en 1894, precisamente durante la efervescencia de fin de siglo, consecuencia de la cual fue también el proceso iniciado

como f(3t) = −f(t), f(3t+1) = −f(t)−1, f(3t+2) = −f(t)−2, la propiedad sigue siendocierta para todo entero n menor que 3t + 2.Luego, para todo n = (akak−1 . . . a0)3, f(n) =

∑kj=0

j imparaj −

∑kj=0

j paraj.

De esta forma, se obtiene el menor n =(2020 . . .20

)3, tal que f(n) = 2010. Este numero

contiene 1005 doses; su valor en base decimal es:

3 · 32010 − 14

• Problema 3

Sea ABCD un cuadrilatero convexo. Sea P la interseccion de AC y BD. El angulo APD = 60. Sean E, F , G y H los puntos medios de los lados AB, BC, CD y DArespectivamente. Halla el mayor numero real positivo k tal que

EG + 3HF ≥ kd + (1 − k)s

siendo s el semiperımetro del cuadrilatero ABCD y d la suma de las longitudes de susdiagonales. ¿Cuando se alcanza la igualdad?

Solucion:Probaremos que k = 1 +

√3 y que la igualdad se da si, y solo si, ABCD es un rectangulo.

X

Z

YW

H

G

F

E

P

A

B C

D

Sean W , X , Y y Z cuatro puntos exteriores a ABCD de modo que los triangulos ABW yDCY sean equilateros, el triangulo BCX sea isosceles en X , el triangulo AZD sea isoscelesen Z y BXC = AZD = 120.Los cuadrilateros WAPB, XBPC, Y CPD y ZDPA son cıclicos. Luego, por el teoremade Ptolomeo, se obtiene que:

WP = PA + PB , XP√

3 = PB + PC , Y P = PC + PD , ZP√

3 = PD + PA

Por otro lado,

WPY = WPB + 60 + CPY = WAB + 60 + CDY = 180

Page 737: COMPENDIUM OME - ToomatesEotvos de Hungría, que se iniciaron en 1894, precisamente durante la efervescencia de fin de siglo, consecuencia de la cual fue también el proceso iniciado

Luego W , P , Y estan alineados y, de forma analoga, Z, P , X estan alineados. Luego:

WY = WP + PY = PA + PB + PC + PD = AC + BD

XZ = XP + PZ =1√3(PB + PC + PD + PA) =

1√3(AC + BD)

Por la desigualdad, triangular:

WY ≤ WE + EG + GY , XZ ≤ XF + FH + HZ

Luego:

AC + BD ≤ AB

√3

2+ EG + DC

√3

2,

1√3(AC + BD) ≤ BC

2√

3+ FH +

AD

2√

3

Por lo tanto, sumando,

AC + BD ≤ AB√

32 + EG + DC

√3

2√3(AC + BD) ≤ BC

√3

2 + 3FH + AD√

32(

1 +√

3)(AC + BD) ≤ EG + 3FH + s

√3

o sea,EG + 3FH ≥

(1 +

√3)

d − s√

3

Luego, si k = 1 +√

3, entonces EG + 3FH ≥ kd + (1 − k)s.La igualdad se dara si, y solo si, por un lado, W , E, G, Y estan alineados y, por otro lado, X ,F , H, Z tambien estan alineados. Como WE es perpendicular a AB y GY es perpendiculara DC, AB y DC deben ser paralelas y, de forma analoga, BC y AD tambien deben serparalelas, luego ABCD debe ser un paralelogramo. Ademas, la recta EG es perpendiculara DC, lo que implica que ABCD es un rectangulo y se comprueba facilmente que si ABCDes un rectangulo, entonces se da la igualdad. Luego, la igualdad se da si, y solo si, ABCDes un rectangulo.Ahora, sea un numero real positivo l tal que EG+3HF ≥ ld+(1− l)s. Entonces, si ABCDes un rectangulo,

kd + (1 − k)s ≥ ld + (1 − l)s

o seak(d − s) ≥ l(d − s)

Pero la desigualdad triangular implica que d > s, lo que implica que k ≥ l. Luego el numeroreal buscado es k = 1 +

√3 y la igualdad se da si, y solo si, ABCD es un rectangulo.

• Problema 4

Sean a, b, c tres numeros reales positivos. Demuestra que

a + b + 3c

3a + 3b + 2c+

a + 3b + c

3a + 2b + 3c+

3a + b + c

2a + 3b + 3c≥ 15

8

Page 738: COMPENDIUM OME - ToomatesEotvos de Hungría, que se iniciaron en 1894, precisamente durante la efervescencia de fin de siglo, consecuencia de la cual fue también el proceso iniciado

Solucion:Haciendo a = x1, b = x2, c = x3 y llamando s = x1 + x2 + x3, resulta que el lado izquierdode la desigualdad se escribe como

S =s + 2x1

3s − x1+

s + 2x2

3s − x2+

s + 2x3

3s − x3

Por otro lado,

S + 6 =3∑

k=1

(s + 2xk

3s − xk+ 2

)=

3∑k=1

7s

3s − xk

con lo que

S + 6 =7s

3s − x1+

7s

3s − x2+

7s

3s − x3= 7s

3∑k=1

13s − xk

Dado que∑3

k=1 (3s − xk) = 8s, entonces s = 18

∑3k=1 (3s − xk), y

S + 6 =78

3∑k=1

(3s − xk)3∑

k=1

13s − xk

≥ 638

de donde resulta que

S =s + 2x1

3s − x1+

s + 2x2

3s − x2+

s + 2x3

3s − x3≥ 63

8− 6 =

158

La igualdad tiene lugar cuando x1 = x2 = x3, es decir, cuando a = b = c.

• Problema 5

Sea P un punto cualquiera de la bisectriz del angulo A en el triangulo ABC, y sean A′, B′,C′ puntos respectivos de las rectas BC, CA, AB, tales que PA′ es perpendicular a BC,PB′ es perpendicular a CA y PC′ es perpendicular a AB. Demuestra que PA′ y B′C′ secortan sobre la mediana AM , siendo M el punto medio de BC.

Solucion:Sea E el punto de interseccion de PA′ y B′C′. Si P se mueve sobre la bisectriz AI (I es elincentro), la figura PB′C′E es homotetica de sı misma con respecto al punto A. Luego Edescribe una recta que pasa por A. La bisectriz AI corta a la circunferencia circunscrita aABC en F , que se proyecta en el punto medio Am de BC; si P = F , la recta B′C′ es larecta de Simson de F , luego el lugar geometrico de E es la mediana AAm.

• Problema 6

Sea p un numero primo y A un subconjunto infinito de los numeros naturales. Sea fA(n) elnumero de soluciones distintas de la ecuacion x1 +x2 + · · ·+xp = n, con x1, x2, . . . , xp ∈ A.¿Existe algun numero natural N tal que fA(n) sea constante para todo n > N ?

Solucion:

Page 739: COMPENDIUM OME - ToomatesEotvos de Hungría, que se iniciaron en 1894, precisamente durante la efervescencia de fin de siglo, consecuencia de la cual fue también el proceso iniciado

Para demostrar el enunciado procederemos por contradiccion. Supongamos que existe unnumero N para el que se cumpla la propiedad anterior. Como el conjunto A es infinito,tomemos a ∈ A mayor que N . Vamos a estudiar el valor de fA(pa) y fA(pa + 1). Porhipotesis, se cumple que fA(pa) = fA(pa + 1).Sea S = (s1, s2, . . . , sp) solucion de la ecuacion

x1 + x2 + · · ·+ xp = n , s1, s2, . . . , sp ∈ A

Entonces, cualquier permutacion de los ındices da lugar a una nueva solucion de la ecuacion(posiblemente repetida si se permutan valores iguales). Diremos que una solucion S =(s1, s2, . . . , sp) es asociada a una solucion S′ =

(s′1, s

′2, . . . , s

′p

)si la primera se obtiene a

partir de la segunda mediante permutacion de ındices. Sea S = (s1, s2, . . . , sp) una soluciondel problema y sea Q = (q1, q2, . . . , qp) una solucion asociada a S con la propiedad queq1 = q2 = · · · = qr1 = qr1+1 = qr1+2 = · · · qr1+r2 , y ası de manera sucesiva hasta llegar aqr1+r2+···+rk

= qp. En otras palabras, Q se obtiene a partir de S agrupando los valores si

que son iguales. En particular, r1 + r2 + · · · + rk = p. Con esta notacion, el numero de

soluciones asociadas a S (contando tambien S) es igual ap!

r1!r2! · · · rk!.

Observese que si todos los ri son estrictamente menores que p, entonces dicha expresion escongruente con 0 modulo p, puesto que el cociente de factoriales es un numero natural y enel denominador no hay ningun termino multiplo de p.Ya tenemos todas las herramientas que necesitabamos. Volviendo al problema original,observar que (a, a, . . . , a)︸ ︷︷ ︸

p

es solucion de x1 + x2 + · · · + xp = pa.

Por lo tanto fA(pa) ≡ 1 (mod p): la solucion (a, a, . . . , a)︸ ︷︷ ︸p

no se asocia a ninguna otra,

mientras que cualquier otra solucion de la ecuacion x1 +x2 + · · ·+xp = pa tiene un numeromultiplo de p de asociadas. Por otro lado no existen soluciones de x1 +x2+ · · ·+xp = pa+1

con todas las xi iguales (su valor tendrıa que ser a +1p), con lo que segun lo anterior

fA(pa + 1) ≡ 0 (mod p) y llegamos a una contradiccion.

Page 740: COMPENDIUM OME - ToomatesEotvos de Hungría, que se iniciaron en 1894, precisamente durante la efervescencia de fin de siglo, consecuencia de la cual fue también el proceso iniciado

XLVII Olimpiada Matemática Española

Fase nacional 2011 (Pamplona) Primera sesión (25 de marzo)

Problema 1 En un polígono regular de 67 lados trazamos todos los segmentos que unen dos vértices, incluidos los lados del polígono. Elegimos n de estos segmentos y asignamos a cada uno de ellos un color entre 10 colores posibles. Halla el valor mínimo de n que garantiza, que independientemente de cuáles sean los n segmentos elegidos y de cómo se haga la asignación de colores, siempre habrá un vértice del polígono que pertenece a 7 segmentos del mismo color.

Problema 2 Sean cba ,, números reales positivos. Demuestra que

.25

222 ≥++++

++

++

++ cba

cabcabba

cac

bcb

a

¿Cuándo se alcanza la igualdad?

Problema 3 Sean DCBA ,,, cuatro puntos en el espacio, tales que no hay ningún plano que pasa por los cuatro a la vez. Los segmentos DACDBCAB ,,, son tangentes a una misma esfera. Demuestra que los cuatro puntos de tangencia están en un mismo plano.

No está permitido el uso de calculadoras. Cada problema se puntúa sobre siete puntos.

El tiempo de cada sesión es de tres horas y media.

Page 741: COMPENDIUM OME - ToomatesEotvos de Hungría, que se iniciaron en 1894, precisamente durante la efervescencia de fin de siglo, consecuencia de la cual fue también el proceso iniciado

XLVII Olimpiada Matemática Española

Fase nacional 2011 (Pamplona) Segunda sesión (26 de marzo)

Problema 4 Sea ABC un triángulo con CB ∠=∠ 2 y o90>∠A . Sean D el punto de la recta AB tal que CD es perpendicular a ,AC y M el punto medio de BC . Demuestra que

DMCAMB ∠=∠ .

Problema 5 Cada número racional se pinta de un color, usando sólo dos colores, blanco y rojo. Se dice que una tal coloración es sanferminera cuando para cada dos números racionales ,, yx con

,yx ≠ si se cumple una de las tres condiciones siguientes: a) ,1=xy b) ,0=+ yx c) ,1=+ yx entonces x e y están pintados de distinto color. ¿Cuántas coloraciones sanfermineras hay?

Problema 6 Sea ),( nS con ,0≥n la sucesión definida por:

(i) 1=nS para .20110 ≤≤ n (ii) ,20112012 nnn SSS += ++ para .0≥n

Prueba que, para todo entero no negativo ,a se cumple que aa SS −2011 , es múltiplo de 2011.

No está permitido el uso de calculadoras. Cada problema se puntúa sobre siete puntos.

El tiempo de cada sesión es de tres horas y media.

Page 742: COMPENDIUM OME - ToomatesEotvos de Hungría, que se iniciaron en 1894, precisamente durante la efervescencia de fin de siglo, consecuencia de la cual fue también el proceso iniciado

1

XLVII Olimpiada Matemática Española Fase nacional (Pamplona)

25 y 26 de marzo de 2011

ENUNCIADOS Y SOLUCIONES OFICIALES 1. En un polígono regular de 67 lados trazamos todos los segmentos que unen dos vértices, incluidos los lados del polígono. Elegimos n de estos segmentos y asignamos a cada uno de ellos un color entre 10 colores posibles. Halla el valor mínimo de n que garantiza, que independientemente de cuáles sean los n segmentos elegidos y de cómo se haga la asignación de colores, siempre habrá un vértice del polígono que pertenece a 7 segmentos del mismo color. SOLUCIÓN: Veamos en primer lugar que con 2010=n no es suficiente. Diremos que un segmento es de tamaño r si une dos vértices entre los que, por el camino más corto siguiendo los lados del polígono, hay otros 1−r vértices. Elegimos los 2010 segmentos de tamaño mayor que .3 Para cada ,10,...,2,1∈r asignamos el color r a los segmentos de tamaño 23,13 ++ rr y .33 +r Es obvio que cada vértice pertenece a 6 segmentos de cada color. Ahora vamos a probar que si ,2011=n hay algún vértice que está en 7 segmentos del mismo color. En efecto, en los 2011 segmentos intervienen, contando repeticiones, 4022 vértices, luego, por el principio del palomar, como ,67604022 ×> algún vértice interviene en, al menos, 61 segmentos, de los cuales, de nuevo por el principio del palomar, al menos, 7 serán del mismo color. 2. Sean cba ,, números reales positivos. Demuestra que

.25

222 ≥++++

++

++

++ cba

cabcabba

cac

bcb

a

¿Cuándo se alcanza la igualdad?

SOLUCIÓN: Nótese en primer lugar que, en virtud de la desigualdad entre medias aritmética y geométrica, se tiene

23

23

21

3

2

222

222

222

222

=⎟⎟⎠

⎞⎜⎜⎝

++++

++++

≥++++

+++++

cbacabcab

cabcabcba

cbacabcab

cabcabcba ,

dándose la igualdad si y sólo si a2+ b2+ c2= ab + bc + ca, o equivalentemente, si y sólo si (a− b)2 + (b− c)2 + (c− a)2 = 0, es decir, si y sólo si a = b = c. Nos bastaría entonces, para concluir el problema, con demostrar que

cabcabcba

bac

acb

cba

++++

+≥+

++

++

222

211 .

Esto puede conseguirse por fuerza bruta, viéndose en primer lugar que el miembro de la derecha puede escribirse como

( )( )( ) 1333

++++

+++accbba

abccba ,

Page 743: COMPENDIUM OME - ToomatesEotvos de Hungría, que se iniciaron en 1894, precisamente durante la efervescencia de fin de siglo, consecuencia de la cual fue también el proceso iniciado

2

y a partir de aquí comparando las fracciones restantes, llegándose a la conclusión tras algo de cálculo. Sin embargo, se puede obtener el resultado deseado de una forma más elegante, ya que podemos escribir

222222

⎟⎠

⎞⎜⎝

⎛+

+⎟⎠

⎞⎜⎝

⎛+

+⎟⎠

⎞⎜⎝

⎛+

=+

++

++

=+

++

++ bcca

cabbc

bcaab

abcca

cabbc

bcaab

aba

cac

bcb

a

, ( ) ( ) ( ) ( )222

2 bccaabbccaabcabcab +++++=++ , y por la desigualdad del producto escalar, se tendría

( )

,

2

cbabccabcca

cabbcabbc

bcaabcaab

aba

cac

bcb

acabcab

++=++

+++

+++

≥+

++

++

++

es decir, ( )

( ) ( ) 122

2222

+++

++=

++++

≥+

++

++ cabcab

cbacabcab

cbaba

cac

bcb

a ,

con lo que se concluye el problema. Nótese que la igualdad se da en la última desigualdad si y sólo si

bccac

bccaabbcb

abbccaaba

caab+

+=+

+=+

+ ,

es decir, si y sólo si ,cba == que es por lo tanto también la condición necesaria y suficiente para que se dé la igualdad en la desigualdad propuesta. 3. Sean DCBA ,,, cuatro puntos en el espacio, tales que no hay ningún plano que pasa por los cuatro a la vez. Los segmentos DACDBCAB ,,, son tangentes a una misma esfera. Demuestra que los cuatro puntos de tangencia están en un mismo plano. SOLUCIÓN: Sean R, S, T, U los puntos de tangencia respectivos de la esfera con los segmentos AB, BC, CD, DA. Siendo O el centro de la esfera, claramente OU, OR son respectivamente perpendiculares a DA, AB, con lo que los triángulos OAU y OAR son rectángulos respectivamente en U, R, y al ser OU = OR el radio de la esfera, ambos triángulos son iguales, luego AU = AR = a, y de forma análoga,

BR = BS = b, CS = CT = c, y DT = DU = d.

Page 744: COMPENDIUM OME - ToomatesEotvos de Hungría, que se iniciaron en 1894, precisamente durante la efervescencia de fin de siglo, consecuencia de la cual fue también el proceso iniciado

3

Las rectas UR y BD están en el plano del triángulo ABD, así o bien se cortan o son paralelas. Si UR y BD son paralelas, del teorema de Thales resulta b = d, y del recíproco de Thales resulta que también son paralelas TS y DB. Entonces, UR y TS son paralelas y los cuatro puntos están en un plano. Si UR y BD no son paralelas, sea E su punto de intersección. Tampoco pueden entonces ser paralelas TS y DB, porque si lo fueran, lo serían UR y BD, en contra de lo supuesto. Sea entonces F el punto de intersección de TS y DB. Aplicando el teorema de Menelao en ABD cortado por UR, y en BCD cortado por TS, obtenemos

bd

EDBE

RBAR

UADU

EDBE

==1 , bd

FDBF

SBCS

TCDT

FDBF

==1 , FDBF

EDBE

= .

Entonces, E y F dividen a DB en la misma razón. Como ninguno de los dos puntos (E y F) pueden estar entre D y B – pues BCD tendría tres puntos de intersección con ST – entonces E y F tienen que coincidir. Las rectas UR y ST se cortan entonces en E = F, luego están en un mismo plano y en él, en particular, están los puntos U, R, S y T. 4. Sea ABC un triángulo con CB ∠=∠ 2 y o90>∠A . Sean D el punto de la recta AB tal que CD es perpendicular a ,AC y M el punto medio de BC . Demuestra que DMCAMB ∠=∠ . SOLUCIÓN: La recta que pasa por A y es paralela a BC corta a DM y a DC en los puntos N y F respectivamente. Se sigue que MCNFDMDNBMAN ::: == . Pues MCBM = , resulta NFAN = . Como que o90=∠ACF , tenemos NCAN = , de manera que ACBNACNCA ∠=∠=∠ . Así pues, ABCACBNCB ∠=∠=∠ 2 . Como BCAN , el cuadrilátero ABCN es un trapecio isósceles con NCAB = . Por consiguiente, NCMABM ∆≡∆ y DMCNMCAMB ∠=∠=∠

.

α

α

α2α90

B C

A

M

D

FN

Page 745: COMPENDIUM OME - ToomatesEotvos de Hungría, que se iniciaron en 1894, precisamente durante la efervescencia de fin de siglo, consecuencia de la cual fue también el proceso iniciado

4

5. Cada número racional se pinta de un color, usando sólo dos colores, blanco y rojo. Se dice que una tal coloración es sanferminera cuando para cada dos números racionales ,, yx con ,yx ≠ si se cumple una de las tres condiciones siguientes: a) ,1=xy

b) ,0=+ yx c) ,1=+ yx

entonces x e y están pintados de distinto color. ¿Cuántas coloraciones sanfermineras hay? SOLUCIÓN: Si una coloración es sanferminera, podemos hallar otra coloración sanferminera intercambiando simultáneamente el color de cada racional, de rojo a blanco y de blanco a rojo; si en la coloración inicial dos racionales tienen distinto color, también lo tendrán en la resultante. Hallemos entonces el número de coloraciones sanfermineras tales que, sin pérdida de generalidad, el racional 1 está pintado de rojo, y el número total de coloraciones sanfermineras será exactamente el doble. Supongamos entonces que tenemos una coloración sanferminera con el 1 pintado de rojo. Dado el color del racional x>0, el color del racional −x < 0 tiene que ser distinto, pues ;0)( =−+ xx al mismo tiempo, como 1 es rojo y ,101 =+ necesariamente 0 es blanco. Luego fijado el color de cada racional positivo en una coloración sanferminera, quedaría fijado el color de todos los racionales. Al mismo tiempo, para cada racional positivo x, como ,1)()1( =−++ xx entonces 1+x y x− tienen color distinto, luego x y 1+x tienen el mismo color. Por inducción sobre n, se comprueba entonces fácilmente que para cada racional positivo x y para cada entero positivo n, los racionales x y n + x tienen el mismo color. En particular, todos los enteros positivos serán necesariamente de color rojo. Coloreados entonces todos los enteros positivos de color rojo, procedemos ahora de la siguiente manera para los racionales positivos q que no tienen un color asociado

todavía: expresamos vumq += , donde m es la parte entera de q, y u < v son enteros

positivos primos entre sí; esta expresión es claramente única para cada racional positivo q, y para dos racionales positivos que difieran en un entero, los valores de u, v son

claramente los mismos. Si uv tiene ya un color asociado, entonces le asociamos a q el

color opuesto; si uv no tiene todavía color asociado, entonces procedemos de la misma

forma con uv , hasta llegar a un racional que sí tenga color asociado, pudiendo entonces

asociarle un color a uv , y asociándole a q el color opuesto; nótese que en cualquier caso

x y n + x tendrán el mismo color asociado. El proceso termina necesariamente, ya que

expresando ''

vum

uvq +== , se tiene que ''' uuvumvv += , luego al ser primos entre sí u,

v, entonces u divide a v', y al ser primos entre sí u', v', entonces v' divide a u, luego u = v' > u', y tras a lo sumo u pasos, llegaríamos a u' = 0, es decir, a un entero, que sí tiene

Page 746: COMPENDIUM OME - ToomatesEotvos de Hungría, que se iniciaron en 1894, precisamente durante la efervescencia de fin de siglo, consecuencia de la cual fue también el proceso iniciado

5

asociado un color. Nótese además que el proceso es único ya que, en cada paso, los valores de m, u, v quedan unívocamente determinados por el valor del racional positivo q, con lo que al ser todos los enteros positivos rojos, cada racional positivo podrá tener uno y sólo un color asociado. Esta coloración única para los racionales positivos se extiende también de forma única a todos los racionales, como ya hemos visto. Existe entonces a lo sumo una coloración sanferminera pintando el racional 1 de rojo. Comprobemos que en efecto esta coloración única que hemos construido satisface las condiciones del enunciado:

1) Si x + y = 0 con x ≠ y, sin pérdida de generalidad x > 0 > y = −x, luego y tiene, por la forma de extender la coloración a todos los racionales, color distinto al de x.

2) Si xy = 1 con x ≠ y, o ambos son positivos, o ambos son negativos, teniendo en el segundo caso x, y colores respectivamente opuestos a los de −x, −y, que serían positivos, reduciéndose la comprobación al primer caso. Si x, y son ambos

positivos, sin pérdida de generalidad x>1> y, con vuy += 0 para enteros

positivos u < v, y por construcción a y se le asigna color opuesto al de uvx = .

3) Si x + y = 1 con x ≠ y, entonces sin pérdida de generalidad x > y, y bien x = 1 es rojo, y = 0 es blanco, bien x > 1> 0 > y, bien 1 > x > y > 0. En el segundo caso, el racional positivo x−1 tiene por construcción el mismo color que x, luego por 1), x tiene color opuesto a 1−x = y. En el tercer caso, existen enteros positivos u < v

primos entre sí tales que vu

vx+

= ,vu

uy+

= . Por construcción y por 2), x tiene

color opuesto al de vu

vvu

+=+ 1 , luego x tiene el mismo color que

uv . Pero

también por construcción y por 2), y tiene color opuesto al de uv

uvuy +=

+= 1 ,

luego opuesto al de uv , y opuesto al de x.

Luego la coloración construida es sanferminera, y es la única que puede ser sanferminera con el racional 1 pintado de rojo. Restaurando la generalidad, hay exactamente dos coloraciones sanfermineras. 6. Sea ),( nS con ,0≥n la sucesión definida por:

(i) 1=nS para .20110 ≤≤ n (ii) ,20112012 nnn SSS += ++ para .0≥n

Demuestra que, para todo entero no negativo ,a se cumple que aa SS −2011 , es múltiplo de 2011. SOLUCIÓN: Sea 2011p = . Observemos que p es primo. Sea nA el número de formas de cubrir un tablero de [ ] ( )filas 1 columnasn p× +⎡ ⎤⎣ ⎦ con

fichas de dimensiones ( )1 1p× + (que podemos colocar horizontal o verticalmente). Probemos primero que .nn SA =

Page 747: COMPENDIUM OME - ToomatesEotvos de Hungría, que se iniciaron en 1894, precisamente durante la efervescencia de fin de siglo, consecuencia de la cual fue también el proceso iniciado

6

Tenemos que: • para 1 n p≤ ≤ , .1 nn SA == • .2 11 ++ == pp SA • para 0n ≥ , 1n p n p nA A A+ + += + ; en efecto, al cubrir un tablero de

( ) ( )1 filas 1 columnasn p p+ + × +⎡ ⎤ ⎡ ⎤⎣ ⎦ ⎣ ⎦ con fichas de ( )1 1p× + , se da uno de los dos casos siguientes: *o bien la última fila, de ( )1p + casillas, está cubierta por una ficha colocada horizontalmente: en este caso, hay n pA + formas de cubrir el tablero.

*o bien las ( )1p + últimas filas, cada una de ( )1p + casillas, están cubiertas por

( )1p + fichas en posición vertical: en este caso, hay nA formas de cubrir el tablero.

Deducimos que, para 0n > , .nn SA = Hallemos ahora una expresión de nA en función de n. Escribamos la división euclidiana de n por ( )1p + : ( )1n q p r= + + . Observemos que, al cubrir el tablero, se pueden juntar las fichas colocadas en posición vertical para formar cuadrados de ( ) ( )1 1p p+ × + : p+1 p + 1 n Llamemos bloques a estos cuadrados. Sea k

nA el número de formas de cubrir el tablero con k bloques, para 0 k q≤ ≤ . Dada una de estas formas de cubrir el tablero, llamemos 1,..., kB B los bloques ordenados de arriba hacia abajo, y sean 0a el número de filas situadas encima de 1B , 1a el número de filas entre 1B y 2B , 2a el número de filas entre 2B y 3B ,…, ka el número de filas debajo de kB . Por ejemplo, en el dibujo anterior, 0 1 22, 5, 3, 0k a a a= = = = . Entonces se tiene que ( )0 ... 1ka a n p k+ + = − + .

Recíprocamente, a una ( )1 -uplak + de enteros no negativos ( )0 ,..., ka a tal que

( )0 ... 1ka a n p k+ + = − + , le podemos asociar una única forma de cubrir el tablero con k bloques, colocando de forma alternada y da arriba hacia abajo 0a fichas horizontales, un

Page 748: COMPENDIUM OME - ToomatesEotvos de Hungría, que se iniciaron en 1894, precisamente durante la efervescencia de fin de siglo, consecuencia de la cual fue también el proceso iniciado

7

bloque, 1a fichas horizontales, un bloque, …, 1ka − fichas horizontales, un bloque, ka fichas horizontales. Por tanto, k

nA es el número de ( )1 -uplask + de enteros no negativos ( )0 ,..., ka a tales

que ( )0 ... 1ka a n p k+ + = − + , luego kn

n pkA

k−⎛ ⎞

= ⎜ ⎟⎝ ⎠

.

Deducimos que .0

∑=

⎟⎟⎠

⎞⎜⎜⎝

⎛ −==

q

knn k

pknAS

Estudiemos finalmente ∑⎥⎦

⎥⎢⎣

⎢+

=⎟⎟⎠

⎞⎜⎜⎝

⎛ −=

1

0

)(ppc

kpc k

kcpS )(mod p .

Para ello, utilizamos el lema siguiente: Lema: Para todo primo p y enteros a y b, se tiene

(i) ).(mod pba

pbpa

⎟⎟⎠

⎞⎜⎜⎝

⎛≡⎟⎟

⎞⎜⎜⎝

(ii) Si p no divide a b, entonces |pa

pb

⎛ ⎞⎜ ⎟⎝ ⎠

.

Si admitimos el lema,

c

pc

k

ppc

p

k

ppc

kpc S

kpkc

kk

pkck

kcpS ≡⎟⎟

⎞⎜⎜⎝

⎛ −=≡⎟⎟

⎞⎜⎜⎝

⎛ −≡⎟⎟

⎞⎜⎜⎝

⎛ −= ∑∑∑

⎥⎦

⎥⎢⎣

⎢+

=

⎥⎥⎦

⎢⎢⎣

⎢⎥⎦

⎥⎢⎣

⎢+

=

⎥⎦

⎥⎢⎣

⎢+

=

1

0

11

0

1

0

)(),(mod p

que es lo que queríamos probar. Demostración del lema: (i)

( ) ( )

( ) ( ) ( ) ( )

( )

( ) ( )

( )( )( )( ) ( )( )

( )

1

0 11 1

0 0 1 1

1 ... 1

1 ... 1 1 ... 1

1 !mod

1 ! 1 !

a a

k kb a b b a b

k k k ka

b a b

pk pk p pkpapb pk pk p pk pk p pk pk

p a ap

b bp p

= =− − − −

= = = =

+ + −⎛ ⎞

= ⋅⎜ ⎟⎝ ⎠ + + − + + −

− ⎛ ⎞ ⎛ ⎞≡ ⋅ ≡⎜ ⎟ ⎜ ⎟

− − ⎝ ⎠ ⎝ ⎠

∏ ∏

∏ ∏ ∏ ∏

(ii) Si n es el cociente de b dividido por p,

( ) ( ) ( ) ( ) ( )1 1

1 1 2 10

a a n a a n

p p p p p pk n k k n k

pav v pk v pk v pk v pk v pa

b

− − − −

= + = = + =

⎛ ⎞⎛ ⎞= − = − + >⎜ ⎟⎜ ⎟

⎝ ⎠⎝ ⎠∑ ∑ ∑ ∑ ,

puesto que ( ) ( )1

2 10

a a n

p pk n k

v pk v pk− −

= + =

− ≥∑ ∑ .

Page 749: COMPENDIUM OME - ToomatesEotvos de Hungría, que se iniciaron en 1894, precisamente durante la efervescencia de fin de siglo, consecuencia de la cual fue también el proceso iniciado

OlimpiadaMatemáticaEspañola RSME

XLVIII Olimpiada Matematica Espanola

Fase nacional 2012 (Santander)

Primera sesion (24 de marzo)

• Problema 1

Determinar razonadamente si el numero λn =√

3n2 + 2n + 2 es irracional para todoentero no negativo n.

• Problema 2

Hallar todas las funciones f : R −→ R de variable real con valores reales, tales que

(x − 2)f(y) + f(y + 2f(x)) = f(x + yf(x))

para todo x, y ∈ R.

• Problema 3

Sean x y n enteros tales que 1 ≤ x < n. Disponemos de x+1 cajas distintas y n−x bolasidenticas. Llamamos f(n, x) al numero de maneras que hay de distribuir las n − x bolasen las x+1 cajas. Sea p un numero primo. Encontrar los enteros n mayores que 1 para losque se verifica que el numero primo p es divisor de f(n, x) para todo x ∈ 1, 2, . . . , n − 1.

No esta permitido el uso de calculadoras.Cada problema se puntua sobre siete puntos.

El tiempo de cada sesion es de tres horas y media.

Page 750: COMPENDIUM OME - ToomatesEotvos de Hungría, que se iniciaron en 1894, precisamente durante la efervescencia de fin de siglo, consecuencia de la cual fue también el proceso iniciado

OlimpiadaMatemáticaEspañola RSME

XLVIII Olimpiada Matematica Espanola

Fase nacional 2012 (Santander)

Segunda sesion (25 de marzo)

• Problema 4

Hallar todos los numeros enteros positivos n y k tales que (n + 1)n = 2nk + 3n + 1.

• Problema 5

Una sucesion (an)n≥1 se define mediante la recurrencia

a1 = 1 , a2 = 5 , an =a2

n−1 + 4an−2

, para n ≥ 3

Demostrar que todos los terminos de la sucesion son numeros enteros y encontrar unaformula explıcita para an.

• Problema 6

Sea ABC un triangulo acutangulo, ω su circunferencia inscrita de centro I, Ω su cir-cunferencia circunscrita de centro O, y M el punto medio de la altura AH, donde Hpertenece al lado BC. La circunferencia ω es tangente a este lado BC en el punto D. Larecta MD corta a ω en un segundo punto P , y la perpendicular desde I a MD corta a BCen N . Las rectas NR y NS son tangentes a la circunferencia Ω en R y S respectivamente.Probar que los puntos R, P , D y S estan en una misma circunferencia.

No esta permitido el uso de calculadoras.Cada problema se puntua sobre siete puntos.

El tiempo de cada sesion es de tres horas y media.

Page 751: COMPENDIUM OME - ToomatesEotvos de Hungría, que se iniciaron en 1894, precisamente durante la efervescencia de fin de siglo, consecuencia de la cual fue también el proceso iniciado

1

SOLUCIONES DE LA 48ª OME

1. Determinar razonadamente si el número 223 2 ++= nnnλ es irracional para todo entero no negativo .n SOLUCIÓN. Supongamos que n es par. Entonces, nn 23 2 + es múltiplo de 4 y

223 2 ++ nn es múltiplo de 2 pero no de ,4 con lo que no puede ser un cuadrado perfecto. Supongamos que n es impar. Cualquier cuadrado perfecto impar da resto 1 al dividir

entre ;8 este resultado se demuestra trivialmente, escribiendo el cuadrado de cualquier

entero impar 12 +m en la forma 1)1(4 ++mm y observando que, bien ,m bien ,1+m

ha de ser par. Se tiene entonces que si n es impar y 223 2 ++ nn fuera un cuadrado

perfecto, entonces 223 2 ++ nn daría resto 1 al dividir entre ,8 o equivalentemente,

n2 daría resto 4321 −=−− al dividir entre ,8 con lo que n2 sería múltiplo de 4 y n

par, contradicción. Luego para cualquier entero, positivo o negativo, 223 2 ++ nn es

un entero que no es un cuadrado perfecto, por tanto nλ es siempre irracional para

cualquier entero ,n positivo o negativo.

Nótese también que nλ es siempre real, incluso cuando n es un entero negativo, pues

.0)1(223 22 ≥+>++ nnn

2. Hallar todas las funciones RRf →: de variable real con valores reales, tales que

))(())(2()()2( xfyxfxfyfyfx +=++− (1) para todo ., Ryx ∈ SOLUCIÓN. Supongamos primeramente que .0)0( =f Haciendo 0=x en (1),

0)( =yf para todo .Ry ∈ Esta función satisface la ecuación funcional dada (1). Sea .0)0( ≠f Haciendo 0=y en (1), se obtiene )())(2()0()2( xfxfffx =+− para todo .Rx ∈ Claramente esto implica que f es inyectiva porque si ),()( 21 xfxf = entonces ==+− )())(2()0()2( 111 xfxfffx )())(2()0()2( 222 xfxfffx =+− y por tanto 0)0()( 21 =− fxx y .21 xx = Poniendo ahora 2=x en (1), ))2(2())2(2( yfffyf +=+ para todo .Ry ∈ Al ser f inyectiva )2(2)2(2 yffy +=+ para todo .Ry ∈ En esta igualdad si ,0=y

.1)2( =f Al ser inyectiva ;1)3( ≠f por tanto con 3=x e )3(1

3f

y−

= se llega a

Page 752: COMPENDIUM OME - ToomatesEotvos de Hungría, que se iniciaron en 1894, precisamente durante la efervescencia de fin de siglo, consecuencia de la cual fue también el proceso iniciado

2

.0)3(2)3(1

3=⎟⎟

⎞⎜⎜⎝

⎛+

−f

ff Hemos de demostrar que f tiene un cero. Sea ,a tal que

.0)( =af Poniendo ahora ay = en (1) resulta ))(())(2( xafxfxfaf +=+ para todo .Rx ∈ Así por la inyectividad de ,f )()(2 xafxxfa +=+ para todo .Rx ∈ Como

,2≠a .2

)(aaxxf

−−

= Sustituyendo esta función en (1) resulta que ,1=a lo que

proporciona dos únicas soluciones de la ecuación funcional inicial 0)( =xf y .1)( −= xxf

3. Sean x y n enteros tales que .1 nx <≤ Disponemos de 1+x cajas distintas y

xn − bolas idénticas. Llamamos ),( xnf al número de maneras que hay de distribuir las xn − bolas en las 1+x cajas. Sea p número primo, encontrar los enteros n mayores que 1 para los que se verifica que el número primo p es divisor de ),( xnf para todo .1...,,2,1 −∈ nx SOLUCIÓN. Claramente ),( xnf es el número de combinaciones con repetición de

1+x elementos tomados de xn − en .xn − Es decir,

.1)()1(

),1(),( ⎟⎟⎠

⎞⎜⎜⎝

⎛=⎟⎟

⎞⎜⎜⎝

⎛−

−−++=−+=

xn

xnxnx

xnxCRxnf

Vamos a probar que los n buscados son todos los de la forma ap con a entero positivo. Sea pm la p -parte del entero positivo ,m es decir si qpm a= (con 1≥q entero),

,ap pm = siendo 1≥a entero. Ahora probaremos el siguiente resultado previo:

Si ,ap pm = entonces pp iim =− )( para cada .1...,,2,1 −∈ api

En efecto, si kp pi = entonces ak < y es obvio que ),( impk − luego .)( pp imi −≤

Recíprocamente, si ,)( kp pim =− ha de ser ak < porque si no sería .ipa Ahora,

ip k porque mp k y ).( imp k − Es decir .)( pp iim ≤− A continuación probaremos que si p es primo y n un entero mayor que 1. Entonces p

divide a ⎟⎟⎠

⎞⎜⎜⎝

⎛xn

para todo 1...,,2,1 −∈ nx si y sólo si apn = con a entero.

Si ⎟⎟⎠

⎞⎜⎜⎝

⎛xn

p para todo ,1...,,2,1 −∈ nx .1

nn

p =⎟⎟⎠

⎞⎜⎜⎝

⎛ Poniendo ,a

p pn = se tiene:

( ) 12...1)1...()1(

⋅−+−−

=⎟⎟⎠

⎞⎜⎜⎝

⎛aa

a

a pppnnn

pn

y por el resultado previo concluimos que la −p parte de ⎟⎟⎠

⎞⎜⎜⎝

⎛ap

n es ,1 luego .apn =

Recíprocamente, si ,apn = para cada 1...,,2,1 −∈ apx ,

( ) 12...1)1...()1(

⋅−+−−

=⎟⎟⎠

⎞⎜⎜⎝

⎛xx

xpppxn aaa

Page 753: COMPENDIUM OME - ToomatesEotvos de Hungría, que se iniciaron en 1894, precisamente durante la efervescencia de fin de siglo, consecuencia de la cual fue también el proceso iniciado

3

y de nuevo por el resultado previo, la −p parte de ⎟⎟⎠

⎞⎜⎜⎝

⎛xn

es ,p

a

xp que es múltiplo de p

por ser .apx < 4. Hallar todos los números enteros positivos n y ,k tales que

.132)1( ++=+ nnn kn SOLUCIÓN. Para 1=n , la ecuación se escribe 2 = 6, claramente falsa. Luego n ≥ 2. Por la fórmula del binomio de Newton,

( ) ...32

11 322 +⎟⎟⎠

⎞⎜⎜⎝

⎛+⎟⎟

⎞⎜⎜⎝

⎛+=−+ n

nn

nnn n

es múltiplo de n2. Tenemos entonces dos casos a analizar:

• k = 1. Entonces, n2 divide a 2n1+3n = 5n, es decir, n divide a 5, con lo que n = 5,

y la ecuación se convierte en 65=26, claramente falsa.

• k ≥ 2. Entonces, n2 divide a 2nk + 3n, pero como divide a 2nk, también ha de

dividir a 3n, es decir, n divide a 3, con lo que n=3. Se comprueba fácilmente

que para n = 3, la ecuación se convierte en 43 = 2×3k + 10, luego en 3k = 27 = 33,

que es cierta si y sólo si k = 3.

5. Una sucesión ( ) 1≥nna se define mediante la recurrencia

,4

,5,12

21

21−

− +===

n

nn a

aaaa para .3≥n

Demostrar que todos los términos de la sucesión son números enteros y encontrar una fórmula explícita para .na SOLUCIÓN. Observamos a partir de la definición que 42

12 += −− kkk aaa y 42

11 +=−+ kkk aaa . Restando a la segunda ecuación de la primera, resulta 2

12

211 −−−+ −=− kkkkkk aaaaaa ⇔ =+ +−− 112

1 kkk aaa ,22

++ kkk aaa que es equivalente a su vez a ).()( 2111 −+−− +=+ kkkkkk aaaaaa Haciendo que ,3 nk ≤≤ se obtienen

),()( 133422 aaaaaa +=+ ),()( 244533 aaaaaa +=+ ),()( 355644 aaaaaa +=+

…………………………. ),()( 31122 −−−−− +=+ nnnnnn aaaaaa

).()( 2111 −+−− +=+ nnnnnn aaaaaa Multiplicando las igualdades anteriores y simplificando términos, resulta

Page 754: COMPENDIUM OME - ToomatesEotvos de Hungría, que se iniciaron en 1894, precisamente durante la efervescencia de fin de siglo, consecuencia de la cual fue también el proceso iniciado

4

)()( 31112 aaaaaa nnn +=+ +− y teniendo en cuenta que 5,1 21 == aa y ,203 =a resulta .6 11 −+ −= nnn aaa De este modo es inmediato que todos los términos de la sucesión son

enteros. Para encontrar una formula explícita de na ensayamos con n

n ta = con lo que obtenemos a partir de la última expresión que

06 11 =−− −+ nnn ttt ⇔ .0)16( 21 =+−− ttt n Se tienen tres soluciones. La primera ,0=t que no cumple con el enunciado. Las otras dos 223 ±=t combinadas linealmente, sí lo harán. Es decir la solución que buscamos

es de la forma ( ) +−=n

na 223λ ( ) ,223n

+µ ., R∈µλ Para determinar las constantes λ y µ utilizamos que 11 =a y 52 =a y tenemos

⎪⎩

⎪⎨⎧

=++−

=++−

5)223()223(

1)223()223(22 µλ

µλ.

Resolviendo este sistema 4

22 +=λ y ,

422 −

=µ con lo que

( ) +−+

=n

na 2234

22 ( ) .2234

22 n+

6. Sea ABC un triángulo acutángulo, ω su circunferencia inscrita de centro ,I Ω su circunferencia circunscrita de centro ,O y M el punto medio de la altura ,AH donde H pertenece al lado .BC La circunferencia ω es tangente a este lado BC en el punto .D La recta MD corta a ω en un segundo punto ,P y la perpendicular desde I a MD corta a BC en .N Las rectas NR y NS son tangentes a la circunferencia Ω en R y S respectivamente. Probar que los puntos

DPR ,, y S están en una misma circunferencia. SOLUCIÓN. Supongamos que .cb = Entonces, el pie de la altura H coincide con el punto de tangencia ,D luego DM es perpendicular a BC y N no está definido. Asumiremos entonces sin pérdida de generalidad que .cb > Sea U el punto de la recta BC cuya potencia es la misma respecto de ω y .Ω Claramente, hay exactamente dos tangentes a cada una de ambas circunferencias que pasan por ,U siendo D el punto de tangencia de una de ellas con ;ω llamemos E al punto de tangencia con ω de la segunda recta que pasa por .U La distancia de U a los cuatro puntos de tangencia es la misma, luego existe una circunferencia de centro U que pasa por los cuatro puntos, es decir, si demostramos que ,NU = el problema quedaría resuelto. Ahora bien, el eje radical de la circunferencia descrita con centro U y ,ω es claramente la recta DE y la perpendicular a esta recta por I es la mediatriz de la cuerda ,DE luego pasa por .U Basta entonces con demostrar que el punto W de la altura AH cuya potencia es la misma respecto a la circunferencia de centro U por D y por ,E y respecto a ,ω es el punto medio de ,AH con lo que sería EP = y .UN = Ahora bien, dicha potencia es

2222 IWIDUWUD −=−

Page 755: COMPENDIUM OME - ToomatesEotvos de Hungría, que se iniciaron en 1894, precisamente durante la efervescencia de fin de siglo, consecuencia de la cual fue también el proceso iniciado

5

Pero ,222 WHUHUW += ,)( 222 HDIDWHIW +−= con lo que la anterior condición

es equivalente a

( ),22 222 HDUDUDHDUHIDWHUD −=−=⋅− ,ID

UDHDWH ⋅=

y el problema se reduce a demostrar que esta última expresión es la mitad de la altura.

Llamando s al semiperímetro de ,ABC tenemos que ,bsBD −= ,csCD −=

,cos BcBH = y al estar U definido como el punto sobre BC tal que su potencia es la

misma respecto de ω y ,Ω y llamando Σ al área de ABC y usando la fórmula de

Herón para la misma, tenemos

( )( )CDUDBDUDUD +−=2 , ( )( )( )( ) .

2

ascbscbcsbs

BDCDCDBDUD

−−Σ

=−

−−=

−⋅

=

Luego

( )( )( )

( )( )( )( ) =

−+−+−−−++

=−−

−−=

acbcbbcaabcbaah

ascbBcbsahWH

22222

cos2

( )( )( ) ,

22

22 hacbcbbcabach

=−+−+−−

=

como queríamos demostrar.

Page 756: COMPENDIUM OME - ToomatesEotvos de Hungría, que se iniciaron en 1894, precisamente durante la efervescencia de fin de siglo, consecuencia de la cual fue también el proceso iniciado

XLIX Olimpiada Matemática Española Fase nacional 2013 (Bilbao) Primera sesión (5 de abril)

Problema 1 Sean ba, y n enteros positivos tales que ba y .1 2nab Prueba que

.34 nba Indica justificadamente cuando se alcanza la igualdad.

Problema 2 Determina todos los números enteros positivos ,n para los cuales

nnnn zyxS

es constante, cualesquiera que sean zyx ,, reales tales que, 1xyz y .0 zyx

Problema 3 Sean k y n enteros, con .3 kn Se consideran 1n puntos en el plano, no alineados entre sí tres a tres. A cada segmento que une entre sí dos de esos puntos se le asigna un color de entre k colores dados. Se dice que un ángulo es bicolor si tiene por vértice uno de los n+1 puntos, y por lados, dos de los segmentos anteriores que sean de distinto color. Demuestra que existe una coloración tal que el número de ángulos bicolores es estrictamente mayor que

.2

2

k

knn

OBSERVACIÓN: Se denota por t la parte entera del número real ;t es decir el mayor entero .tn

No está permitido el uso de calculadoras. Cada problema se puntúa sobre siete puntos.

El tiempo de cada sesión es de tres horas y media.

Page 757: COMPENDIUM OME - ToomatesEotvos de Hungría, que se iniciaron en 1894, precisamente durante la efervescencia de fin de siglo, consecuencia de la cual fue también el proceso iniciado

XLIX Olimpiada Matemática Española

Fase nacional 2013 (Bilbao) Segunda sesión (6 de abril)

Problema 4 ¿Existen infinitos enteros positivos que no pueden representarse de la forma

,119753 edcba donde edcba ,,,, son enteros positivos? Razónese la respuesta.

Problema 5 Estudia si existe una sucesión estrictamente creciente de enteros ...0 210 aaa , que cumple las dos condiciones siguientes: i) Todo número natural puede ser escrito como suma de dos términos, no necesariamente distintos, de la sucesión.

ii) Para cada entero positivo ,n se cumple que .16

2nan

Problema 6 Sea ABCD un cuadrilátero convexo tal que:

ACCDAB 2 y .2 BDDABC ¿Qué forma tiene el cuadrilátero ?ABCD

No está permitido el uso de calculadoras. Cada problema se puntúa sobre siete puntos.

El tiempo de cada sesión es de tres horas y media.

Page 758: COMPENDIUM OME - ToomatesEotvos de Hungría, que se iniciaron en 1894, precisamente durante la efervescencia de fin de siglo, consecuencia de la cual fue también el proceso iniciado

SOLUCIONES DE LOS PROBLEMAS DE LA OME 49ª 1. Sean ba, y n enteros positivos tales que ba y .1 2nab Prueba que

.34 nba Indica justificadamente cuándo se alcanza la igualdad. SOLUCIÓN: Supongamos que el resultado a demostrar fuera falso. Entonces .)12()1(4344)()( 2222 nnnabbaba Pero como ba es entero y ,12 nba entonces nba 2 y por la desigualdad entre las medias

aritmética y geométrica ,2

1 22

2 nbaabn

que es una contradicción. Luego el

resultado a demostrar es cierto. El caso de la igualdad requiere que ,)12()1(434)( 222 nnnba debido a la identidad .4)()( 22 abbaba Por tanto .12 nba La igualdad 34 nba se alcanza cuando el radicando sea necesariamente un cuadrado perfecto impar; es decir

2)12(34 un para algún entero no negativo ,u con lo que 12 uun y .12 uba

Además, ,12 uunb luego .2212 2 uuuba Se comprueba fácilmente que en efecto .11)1(2232 222234 nuuuuuuab Luego hay igualdad si y sólo si

,222 uua 12 ub y ,12 uun con ,12 uba para todo entero no negativo .u 2. Determina todos los números enteros positivos ,n para los cuales

nnnn zyxS

es constante, cualesquiera que sean zyx ,, reales tales que, 1xyz y .0 zyx SOLUCIÓN: Supongamos z conocido. Entonces yx, son dos números reales con suma z y

producto ;1z

es decir son las raíces de la ecuación ,012 z

z o bien:

.2

4

,

2

zzz

yx

Así una posible solución es por ejemplo 3 4z y .2

13

yx Sin embargo,

podemos hacer z arbitrariamente grande en valor absoluto, siendo entonces uno de los dos números x ó y también arbitrariamente grande en valor absoluto y de signo contrario a .z Y el otro arbitrariamente pequeño en valor absoluto y de signo negativo. Se deduce inmediatamente que n no puede ser par cuando nS es constante, ya que por

una parte toma un valor finito y bien determinado cuando 3 4z y por otra puede

Page 759: COMPENDIUM OME - ToomatesEotvos de Hungría, que se iniciaron en 1894, precisamente durante la efervescencia de fin de siglo, consecuencia de la cual fue también el proceso iniciado

tomar valores arbitrariamente grandes y siempre positivos cuando z crece arbitrariamente. Para finalizar el problema utilizaremos el siguiente resultado. Lema: Para todo 3n impar, fijado ,z se puede escribir nn yx como un polinomio en z de grado n (más algún término posiblemente de exponente negativo) y con coeficientes iguales a ,1 para los términos de grado ,n iguales a ,0 para los términos de grado 2,1 nn y n para los términos de grado .3n Demostración: Para 3n tenemos que .3)(3)( 3333 zyxxyyxyx

Para 5n tenemos .55)(10)(5)( 252233555

zzzyxyxyxxyyxyx

Si el resultado es cierto para un entero impar ,n entonces para 2n tenemos

ununuu

u

nnn yxyxu

nyxyx

n

2222

1

2222

1

2)(

.2

)(2 2222

2

22

1

u

unun

u

nnn

zyx

un

yxz

nzn

Por hipótesis de inducción la suma para 2u no contiene ningún término de grado superior a ,4n y también por hipótesis de inducción, el término de mayor grado del

segundo sumando del miembro de la derecha es .1)2()(2

nn znzz

n

De este modo queda probado el lema por inducción completa.

Del razonamiento inicial se deduce que si nS es constante, no puede ser n par y por el lema cuando z para algún 5n impar dado, se tiene que

,3 nnnn nzzyx con signo positivo además por ser 3n par en este caso. Luego sólo podrían ser constantes

1S y :3S 01 S es constante por hipótesis y 33 S como consecuencia de un resultado hallado durante la demostración del lema. Por tanto, se concluye que nS es constante con las condiciones del enunciado, si y sólo si n =1 ó n =3.

3. Sean k y n enteros, con .3 kn Se consideran 1n puntos en el plano, noalineados entre sí tres a tres. A cada segmento que une entre sí dos de esos puntos se le asigna un color de entre k colores dados. Se dice que un ángulo es bicolor si tiene por vértice uno de los n+1 puntos, y por lados, dos de los segmentos anteriores que sean de distinto color. Demuestra que existe una coloración tal que el número de ángulos bicolores es estrictamente mayor que

.2

2

k

knn

OBSERVACIÓN: Se denota por t la parte entera del número real ;t es decir el mayor entero .tn

SOLUCIÓN:

Utilizaremos en la solución el siguiente resultado o lema:

Para todo entero k3, se pueden colorear, usando k colores, todos los lados y diagonales de un polígono regular de k lados, de forma que los k1 lados y diagonales

Page 760: COMPENDIUM OME - ToomatesEotvos de Hungría, que se iniciaron en 1894, precisamente durante la efervescencia de fin de siglo, consecuencia de la cual fue también el proceso iniciado

que confluyen en cada vértice estén coloreados de distinto color. Se puede además elegir la coloración de forma que, para cada dos vértices consecutivos del polígono, el color ausente para cada uno de ambos (es decir, aquel color del que no está pintado ningún lado o diagonal que concurre en cada uno de dichos vértices) sea distinto.

Prueba: Cada lado o diagonal del polígono regular de k lados es una cuerda de su circunferencia circunscrita, y su mediatriz es eje de simetría del polígono. Como es conocido que el polígono tiene k ejes de simetría, podemos, a cada eje de simetría, asociarle un color, y pintar de dicho color todos los lados y diagonales que son perpendiculares al mismo (aquellos para los que el eje de simetría del polígono es la mediatriz).Luego dos lados o diagonales tienen el mismo color si y sólo si son paralelos, y como los k1 lados o diagonales que confluyen en un vértice uniéndolo a los k1 restantes no pueden ser paralelos dos a dos, estos k1 lados o diagonales tienen distinto color, como queríamos demostrar. Además, dados cuatro vértices consecutivos que numeramos ordenadamente P1, P2, P3, P4, el color del que está pintada la diagonal P1P3 corresponde a todos los lados y diagonales perpendiculares al eje de simetría que pasa por P2, luego está ausente del vértice P2, y de forma análoga el color del que está pintada la diagonal P2P4 está ausente del vértice P3, y estos dos vértices consecutivos tienen colores ausentes distintos. Para 3k se cumple trivialmente, siendo suficiente pintar cada lado del triángulo equilátero, de uno de los 3 colores disponibles.

Dados n y k cumpliendo las restricciones del enunciado, construimos una coloración en la que el número de ángulos bicolores sea mayor que la cota propuesta. Para ello, elegimos primero un punto O, al que llamamos “pivote”, y distribuimos los n puntos

restantes en k subconjuntos A1, A2,..., Ak, colocando primero

kn puntos en cada

subconjunto, y distribuyendo los demás a voluntad. Numeramos ahora P1, P2,..., Pk los vértices de un polígono regular de k lados, y asociamos a cada subconjunto Ai, el vértice Pi. Coloreamos además los lados y diagonales del polígono regular de k lados de forma que en cada vértice concurran k1 lados y diagonales de k1 colores distintos, de acuerdo al Lema.

Para cada punto Q de los n1 dados y distinto del pivote, pintamos todos los segmentos que lo unen a cada punto R de los n restantes, de la siguiente forma:

Si Q y R están en distintos subconjuntos Ai y Aj, pintamos el segmento que losune del mismo color que el lado o diagonal PiPj en el polígono regular.

Si Q y R están en el mismo subconjunto Ai, o si RO es el pivote , pintamos elsegmento que los une del color que no está presente en ninguno de los lados ydiagonales que confluyen en el vértice Pi del polígono regular.

Claramente, con esta coloración cada punto Q de un subconjunto Ai, está unido a al menos

kn

puntos por segmentos pintados con cada uno de los k colores. En efecto, para cada uno de los k1 colores de los que están pintados los lados y diagonales que confluyen en el vértice Pi del

polígono regular, el punto Q está unido a los al menos

kn

puntos del subconjunto Aj asociado

al vértice Pj que está unido al vértice Pi por dicho color; y para el color restante, el punto Q está

unido a los al menos 1

kn

puntos restantes del conjunto Ai, más al pivote O.

De lo anterior, deducimos que, en la coloración propuesta y para cada uno de los n puntos

distintos del pivote, hay al menos

kn

segmentos de cada uno de los k colores que confluyen

Page 761: COMPENDIUM OME - ToomatesEotvos de Hungría, que se iniciaron en 1894, precisamente durante la efervescencia de fin de siglo, consecuencia de la cual fue también el proceso iniciado

en dicho vértice. Para cada par de colores distintos, hay por lo tanto 2

kn

posibles pares de

segmentos que forman un ángulo bicolor cuyos lados son estos dos colores, y como hay

2k

posibles formas de elegir dos colores de entre los k existentes, para cada uno de los n puntos

distintos del pivote hay al menos

2

2 kkn ángulos bicolores que lo tienen por vértice, para un

total de al menos

2

2 kknn

ángulos bicolores.

Ahora bien, por la segunda parte del Lema, los segmentos que unen al pivote con los puntos de dos subconjuntos Ai y Aj, asociados a vértices consecutivos del polígono regular de k lados, están pintados de los colores ausentes en cada uno de los dos vértices, que son distintos, y hay al menos un ángulo bicolor con vértice en el pivote O, y por lo tanto, para la coloración construida, el número de ángulos bicolores es mayor que

2

2 kknn .

4. ¿Existen infinitos enteros positivos que no pueden representarse de la forma,119753 edcba donde edcba ,,,, son enteros positivos?

Razona la respuesta.

SOLUCIÓN:

Como ,346511975 veamos cuántos enteros podemos obtener que sean menores o iguales que .3465N Al ser 119753 ,,,, edcba positivos, cada uno de ellos es menor que

,3465N y tenemos ,1155Na ,693Nb ,495Nc 385Nd y .315Ne

Luego al ser ,30433153854956931155 hay menos de 3043N tales números que se pueden poner en la forma indicada, y hay más de 3464N números entre los 3465N primeros que no se pueden representar en la forma propuesta. Al crecer N arbitrariamente, también aumenta el número de enteros positivos que no pueden representarse en la forma indicada y por lo tanto sí existen infinitos enteros que no se pueden representar de la manera propuesta.

5. Estudiar si existe una sucesión estrictamente creciente de enteros ...0 210 aaa , que cumple las dos condiciones siguientes: i) Todo número natural puede ser escrito como suma de dos términos, nonecesariamente distintos, de la sucesión.

ii) Para cada entero positivo ,n se verifica que .16

2nan

Page 762: COMPENDIUM OME - ToomatesEotvos de Hungría, que se iniciaron en 1894, precisamente durante la efervescencia de fin de siglo, consecuencia de la cual fue también el proceso iniciado

SOLUCIÓN:

Sea na la sucesión de todos los números naturales ,k que en el sistema de numeración binario sólo tiene unos en las posiciones pares o sólo en los lugares impares. De esta forma se satisface la condición primera. Probemos a continuación que la estimación de la condición segunda es también válida. Consideremos todos los enteros no negativos que son menores que ,22r es decir los números que no tienen más de r2 dígitos (cifras) en su representación binaria. Contamos el número de elementos de la sucesión que hay entre esos números: hay r2 elementos que tienen ceros en todos los lugares pares y r2 elementos que tienen ceros en todos los lugares impares y 0 es el único número que se cuenta dos veces. Por tanto hay 12 1 r elementos de la sucesión que son menores que r22 y así .22

12 1r

ra

Para cada número natural, se puede encontrar un entero ,r tal que .1212 21 rr n

Por tanto 4

2 nr y esto implica que ..16

22

212 1

na rr

6. Sea ABCD un cuadrilátero convexo tal que:ACCDAB 2 y .2 BDDABC

¿Qué forma tiene el cuadrilátero ?ABCD

SOLUCIÓN :

Sean M es el punto medio de AB, N el de BC, P el de CD y Q el de DA. El punto medio de la diagonal AC es E y el de la diagonal BD es F. Las longitudes respectivas de los lados AB, BC, CD y DA son a, b, c y d. Y e y f las de las diagonales AC y BD, respectivamente. Las relaciones de Euler en los cuadriláteros convexos las formulamos como

2 2 2 2 2 2 2

2 2 2 2 2 2 2

2 2 2 2 2 2 2

) 4) 4) 4

i a c MP b d e fii b d NQ a c e fiii e f EF a b c d

Recordemos brevemente la demostración de una de ellas, por ejemplo i).

MP es una mediana en el triángulo APB, así que el teorema de la mediana en él nos da 2 2 2 24 2MP PA PB AB ;

ahora bien, PA es mediana en ACD y PB lo es en BCD, así que volviendo a aplicar en esos dos triángulos el teorema de la mediana y sustituyendo en la expresión anterior se obtiene i).

Page 763: COMPENDIUM OME - ToomatesEotvos de Hungría, que se iniciaron en 1894, precisamente durante la efervescencia de fin de siglo, consecuencia de la cual fue también el proceso iniciado

Vamos a demostrar ahora que se verifica la desigualdad

12

QN AB CD ,

y que el signo igual se verifica si y sólo si AB es paralelo a CD. En efecto, sobre la siguiente figura,

en el triángulo EQN se verifica obviamente QN QE EN , valiendo el signo igual cuando E está sobre QN; puesto que QE es paralela media en ACD (paralela a CD e igual a su mitad) y EN también es paralela media en ABC (paralela a AB e igual a su mitad), se tiene, inmediatamente,

12

QN AB CD . (1)

(El signo igual vale, entonces, cuando ABCD es un trapecio de bases AB y CD). A continuación vamos a demostrar que Si ABCD es un cuadrilátero convexo, entonces

2 2 2 2 2e f d b ac , (2) y el signo igual vale si y sólo si AB es paralelo a CD. En efecto, multipliquemos (1) por 2 y elevemos al cuadrado:

224QN AB CD , y sustituyamos aquí la expresión de QN2 dada por (ii):

2 2 2 2 2 2 24b d NQ a c e f ; obtendremos

2 2 2 2 2AC BD AD BC AB CD , que es (2), y el signo igual es válido si y sólo si AB es paralelo a CD. De una forma completamente similar obtendríamos

2 2 2 2 2e f a c bd , (3) donde ahora el signo = es válido si y sólo si AD es paralelo a BC. Sumando (2) y (3) llegamos entonces a que

2 22 22 e f a c b d , (4)Donde el signo igual vale si AB es paralelo a CD y BC lo es a AD, es decir, si ABCD es un paralelogramo. Puesto que elevando al cuadrado las dos relaciones del enunciado y sumando se obtiene (4) y hemos terminado.

CONSTRUCCIÓN GRÁFICA:

Utilizaremos las notaciones siguientes. Lados: AB = a, BC = b, CD = c, DA = d. Diagonales del cuadrilátero: AC = d1, y BD = d2. Y para fijar ideas supongamos que

.21 dd Además, M es el punto medio de AB, N el de BC, P el de CD y Q el de DA. El punto medio de la diagonal AC es E y el de la diagonal BD es F.

Page 764: COMPENDIUM OME - ToomatesEotvos de Hungría, que se iniciaron en 1894, precisamente durante la efervescencia de fin de siglo, consecuencia de la cual fue también el proceso iniciado

Según el enunciado, se cumplen las relaciones

1 22 y 2a c d b d d . Dados d1 y d2, se construyen sendos triángulos rectángulos isósceles cuyos catetos son d1 y d2; entonces sus hipotenusas son a + c y b + d respectivamente.

El punto M (que coincidirá con A una vez terminada la construcción) será centro de una circunferencia de radio d1; el punto N será centro de una circunferencia de radio d2. A uno de los puntos de intersección de ambas circunferencias se le llama O. Sea P el simétrico de M respecto de O. En el triángulo NOP, el ángulo NOP es el suplementario de .MON Apliquemos el teorema del coseno en los triángulos MON y NOP:

En :NOP cos2 2122

21

2 ddddNP y

en :MNO cos22 2122

21

2

1 ddddd

Sumando miembro a miembro y simplificando, obtenemos 2 2

2 22 2NP d NP d b d . Trazando por P la paralela a MN y por M la paralela a NP obtenemos el punto Q. El paralelogramo MNPQ es semejante al que buscamos; éste se obtiene trazando las paralelas a los lados por el punto O. Para que la construcción sea posible, debe ser

1 2 1 2 1 1 2 12 2 1 2 1d d d d d d d d .

Si 1 2 cos 0d d y se obtendría un cuadrado.

Page 765: COMPENDIUM OME - ToomatesEotvos de Hungría, que se iniciaron en 1894, precisamente durante la efervescencia de fin de siglo, consecuencia de la cual fue también el proceso iniciado

L Olimpiada Matemática Española Fase nacional 2014, Requena

Viernes, 28 de marzo PRIMERA SESIÓN

Problema 1 ¿Es posible disponer sobre una circunferencia los números 0,1, 2,…,9 de tal manera que la suma de tres números sucesivos cualesquiera sea, como mucho a) 13, b) 14, c) 15?

Problema 2

Dados los números racionales positivos qr, y n tales que ,111qrrnqqnr

demostrar que 13

nn es un número racional.

Problema 3 Sean B y C dos puntos fijos de una circunferencia de centro O , que no sean diametralmente opuestos. Sea A un punto variable sobre la circunferencia, distinto de B y C , y que no pertenece a la mediatriz de BC . Sean ,H el ortocentro del triángulo ABC ; y M y N los puntos medios de los segmentos BC y ,AH respectivamente. La recta AM corta de nuevo a la circunferencia en ,D y, finalmente, NM y OD se cortan en un punto .P Determinar el lugar geométrico del punto P cuando A recorre la circunferencia.

No está permitido el uso de calculadoras. Cada problema se puntúa sobre siete puntos.

El tiempo de cada sesión es de tres horas y media.

Page 766: COMPENDIUM OME - ToomatesEotvos de Hungría, que se iniciaron en 1894, precisamente durante la efervescencia de fin de siglo, consecuencia de la cual fue también el proceso iniciado

L Olimpiada Matemática Española

Fase nacional 2014, Requena Sábado, 29 de marzo SEGUNDA SESIÓN

Problema 4 Sea nx la sucesión de enteros positivos definida por 21 x y nnn xxx

31 2 para todo

.1n Determinar la mayor potencia de 5 que divide al número .122014 x

Problema 5 El conjunto M está formado por números enteros de la forma 22 13ba , con a y b enteros distintos de cero.

i) Demostrar que el producto de dos elementos cualesquiera de M es un elemento de M . ii) Determinar, razonadamente, si existen infinitos pares de enteros ),( yx tales que yx no

pertenece a M, pero 1313 yx sí pertenece a M .

Problema 6 Se tienen 60 puntos en el interior de un disco unidad (es decir, un círculo de radio 1, y su circunferencia frontera). Demostrar que existe un punto V de la frontera del disco, tal que la suma de las distancias de V a los 60 puntos es menor o igual que .80

No está permitido el uso de calculadoras. Cada problema se puntúa sobre siete puntos.

El tiempo de cada sesión es de tres horas y media.

Page 767: COMPENDIUM OME - ToomatesEotvos de Hungría, que se iniciaron en 1894, precisamente durante la efervescencia de fin de siglo, consecuencia de la cual fue también el proceso iniciado

L Olimpiada matematica Espanola (Concurso Final)

Enunciados y Soluciones

1. Es posible disponer sobre una circunferencia los numeros 0, 1, 2, . . . , 9 de talmanera que la suma de tres numeros sucesivos cualesquiera sea, como mucho a) 13,b) 14, c) 15?

Solucion. Cortamos la circunferencia a la izquierda del 0. Descartado el 0, nosquedan 9 numeros, que podemos agrupar en tres trıos cuya suma total es

1 + 2 + . . . + 9 = 45

Por lo tanto, no puede suceder que las tres sumen menos de 15. Luego la respuestaa los apartados a) y b) es NO, mientras que al apartado c) es SI, colocando porejemplo los numeros en el siguiente orden:

0, 9, 5, 1︸ ︷︷ ︸, 8, 4, 3︸ ︷︷ ︸, 2, 7, 6︸ ︷︷ ︸

2. Dados los numeros racionales r, q y n, tales que1

r + qn+

1

q + rn=

1

r + q, probar

que

√n− 3

n + 1es un numero racional.

Solucion. Racionalizando, tenemos√n− 3

n + 1=

√(n− 3)(n + 1)

(n + 1)2=

√(n− 1)2 − 4

n + 1

Por tanto, el enunciado quedara probado si vemos que (n−1)2−4 es el cuadrado deun numero racional. Para ello, utilizamos la condicion y la escribimos en la formaequivalente

(r + qn)(r + q) + (q + rn)(r + q) = (r + qn)(q + rn)⇔ (r + q)2 = rq(n− 1)2

de donde resulta (n− 1)2 =(r + q)2

rq. Entonces, se tiene que

(n− 1)2 − 4 =(r + q)2

rq− 4 =

(r − q)2

rq=

(r − q)2(n− 1)2

(r + q)2=

((r − q)(n− 1)

(r + q)

)2

(En la penultima igualdad se ha utilizado que rq =(r + q)2

(n− 1)2). Por tanto, (n−1)2−4

es el cuadrado de un numero racional y hemos terminado.

1

Page 768: COMPENDIUM OME - ToomatesEotvos de Hungría, que se iniciaron en 1894, precisamente durante la efervescencia de fin de siglo, consecuencia de la cual fue también el proceso iniciado

3. Sean B y C dos puntos fijos de una circunferencia de centro O, que no sean dia-metralmente opuestos. Sea A un punto variable sobre la circunferencia, distinto deB y C, y que no pertenece a la mediatriz de BC. Sean H, el ortocentro del trianguloABC; y M y N los puntos medios de los segmentos BC y AH, respectivamente.La recta AM corta de nuevo a la circunferencia en D, y, finalmente, NM y ODse cortan en un punto P . Determinar el lugar geometrico del punto P cuando Arecorre la circunferencia.

Solucion. Empezaremos considerando el caso en que 4ABC es acutangulo. Enprimer lugar, denotaremos por A′ el punto diametralmente opuesto a A con lo que lostriangulos ACA′ y ABA′ son rectangulos. Los segmentos HB y CA′ son paralelospor ser perpendiculares a AC. Igualmente, HC y BA′ tambien son paralelos por serperpendiculares a AB.

7

Figura-1: Caso en que ABC sea acutángulo.

A

BC

A'

M

N

H O

MC

N

H

A

O

D

P

B

Figura-1a: Figura-1b

Análogamente se razonaría en el:

Figura-2: Caso en que ABC sea obtusángulo.

O

A'

C

H

AN

M B

O

H

NA

CM

PD

B

Figura-2a: Figura-2b

7

Figura-1: Caso en que ABC sea acutángulo.

A

BC

A'

M

N

H O

MC

N

H

A

O

D

P

B

Figura-1a: Figura-1b

Análogamente se razonaría en el:

Figura-2: Caso en que ABC sea obtusángulo.

O

A'

C

H

AN

M B

O

H

NA

CM

PD

B

Figura-2a: Figura-2b

Triangulo ABC acutangulo

Entonces, CHBA′ es un paralelogramo y, por tanto, M es el unto medio de HA′.Los triangulos AA′H y OA′M son semejantes con razon de semejanza conocida. Esdecir, tenemos que

OM

AH=

MA′

HA′=

1

2⇒ OM =

AH

2= AN = NH

Luego OMHN es otro paralelogramo (figura izquierda).

Sea D la interseccion de AM con la circunferencia y sea P el punto de corte deAD con NM . Puesto que 4AOD es isosceles, entonces ∠AOD = ∠ODA. ComoOM y AN son paralelos, pues ambos son perpendiculares al lado BC, ademas deiguales, entonces AOMN es tambien un paralelogramo. Y, de aquı, tenemos que∠OAM = ∠AMN = ∠PMD por ser opuesto por el vertice. Sintetizando, tenemosque ∠PMD = ∠OAM = ∠OAD = ∠ODA = ∠PDM con lo que 4PDM esisosceles y, por tanto, PM = PD.

Finalmente, tenemos que

OP + PM = OP + PD = OD = r = constante

Es decir, con A variable, el punto P se mueve sobre una elipse incompleta confocos en O y M , y eje mayor el radio de la circunferencia. En esta elipse hay quedescartar los cuatro vertices. En efecto, si el punto P estuviese sobre el eje mayorde la elipse, tambien estarıa D y por tanto A, lo cual esta excluido del enunciado yaque en este caso AD y NM son coincidentes. Si el punto P fuese uno de los verticesdel eje menor de la elipse, tendrıamos OP = PM = r/2. Como OD = r, entonces

2

Page 769: COMPENDIUM OME - ToomatesEotvos de Hungría, que se iniciaron en 1894, precisamente durante la efervescencia de fin de siglo, consecuencia de la cual fue también el proceso iniciado

PD = r/2. Supongamos que P esta del lado de B, entonces la paralela a BM por elpunto medio de OB es el eje menor de la elipse, con lo que el punto medio de OB esprecisamente P y D coincide con B, lo que implicara que A coincide con C, lo cualesta escluido del enunciado ya que ABC serıa degenerado. El resto de puntos de laelipse se pueden obtener cuando A es distinto de B y C y no esta en la mediatriz deBC.

A continuacion aparece la figura para el caso en que 4ABC sea obtusangulo.

7

Figura-1: Caso en que ABC sea acutángulo.

A

BC

A'

M

N

H O

MC

N

H

A

O

D

P

B

Figura-1a: Figura-1b

Análogamente se razonaría en el:

Figura-2: Caso en que ABC sea obtusángulo.

O

A'

C

H

AN

M B

O

H

NA

CM

PD

B

Figura-2a: Figura-2b

7

Figura-1: Caso en que ABC sea acutángulo.

A

BC

A'

M

N

H O

MC

N

H

A

O

D

P

B

Figura-1a: Figura-1b

Análogamente se razonaría en el:

Figura-2: Caso en que ABC sea obtusángulo.

O

A'

C

H

AN

M B

O

H

NA

CM

PD

B

Figura-2a: Figura-2b Triangulo ABC obtusangulo

4. Sea xnn≥1 la sucesion de enteros positivos definida por x1 = 2 y xn+1 =2x3n + xn para todo n ≥ 1. Determinar la mayor potencia de 5 que divide al numerox22014 + 1.

Solucion. Para n = 1 se tiene que x1 = 2 y x21 + 1 = 22 + 1 = 5 que es multiplo de5 pero no de 52. Para n = 2 es x2 = 2 · 23 + 2 = 18 y x22 + 1 = 182 + 1 = 325 quees multiplo de 52 pero no de 53. Esto nos sugiere conjeturar que x2n + 1 es multiplode 5n pero no de 5n+1. La demostraremos por induccion. Los casos n = 1 y n = 2ya estan probados. Supongamos que n ≥ 3 y que x2n + 1 = k 5n con (k, 5) = 1.Entonces,

x2n+1 + 1 = (2x3n + xn)2 + 1 = x2n(2x2n + 1)2 + 1 = (k5n − 1)(2k5n − 1)2 + 1

= (k5n − 1)(4k252n − 4k5n + 1) + 1 = 4k353n − 8k252n + k5n+1

= k5n+1(

4k252n−1 − 8k5n−1︸ ︷︷ ︸5

+1)

que es multiplo de 5n+1 pero no es multiplo de 5n+2 ya que para n ≥ 3 es 2n−1 ≥ 1y n− 1 ≥ 1 respectivamente y el termino entre parentesis no es multiplo de 5. Estoprueba la conjetura y, por tanto, la mayor potencia de 5 que divide a x22014 + 1 es52014.

5. El conjunto M esta formado por numeros enteros de la forma a2 + 13b2, con ay b enteros distinos de cero.

• Demostrar que el producto de dos elementos cualesquiera de M es un elementode M .

• Determinar, razonadamente, si existen infinitos pares de enteros (x, y) talesque x + y no pertenece a M , pero x13 + y13 sı pertenece a M .

3

Page 770: COMPENDIUM OME - ToomatesEotvos de Hungría, que se iniciaron en 1894, precisamente durante la efervescencia de fin de siglo, consecuencia de la cual fue también el proceso iniciado

Solucion. (1) Sean a2 +13b2 y c2 +13d2 dos elementos de M . Poniendo m = a, n =√13 b, p = c, q =

√13 d en la identitdad algebraica

(m2 + n2) (p2 + q2) = (mq − np)2 + (mp + nq)2 (Euler)

se obtiene

(a2+13b2) (c2+13d2) = (√

13ad−√

13bc)2+(ac+13bd)2 = (ac+13bd)2+13(ad−bc)2 ∈M

(2) Puesto que los cuadrados de los enteros son congruentes con 0 o con 1 modulo4, entonces los elementos de M no son congruentes con 3 modulo 4. Para que lasuma x + y de dos elementos de M no pertenezca a M, es suficiente que x + y seacongruente con 3 modulo 4. Ahora consideramos las sucesiones xkk≥1 e ykk≥1definidas, respectivamente por

xk = (213 + 1)(

(4k)2 + 13(4k + 1)2), yk = 2xk

Entonces,

xk + yk = 3xk = 3 (213 + 1)(

(4k)2 + 13(4k + 1)2)≡ 3 (mod 4)

y por tanto xk + yk /∈M para todo entero positivo k.

Calculamos ahora

x13k + y13k = x13k + 213x13k = (213 + 1)x13k = (213 + 1)14(

(4k)2 + 13(4k + 1)2)13∈M

al ser el producto de un cuadrado por un elemento de M.

6. Se tienen 60 puntos en el interior del cırculo unidad. Demostrar que existe unpunto V de la frontera del cırculo, tal que la suma de las distancias de V a los 60puntos es menor o igual que 80.

Solucion. Consideremos un triangulo equilatero PQR inscrito en la circunferenciafrontera del cırculo unidad dado. Observamos que si para cualquier punto X delcırculo se cumple que

|XP |+ |XQ|+ |XR| ≤ 4,

entonces sumando las correspondientes desigualdades para los 60 puntos Xk, (1 ≤k ≤ 60) se obtiene

60∑k=1

|XkP |+60∑k=1

|XkQ|+60∑k=1

|XkR| =60∑k=1

(|XkP |+ |XkQ|+ |XkR|) ≤ 60× 4 = 240

En consecuencia, una de las sumas del miembro izquierdo no excede de 240/3 = 80y, por tanto, alguno de los puntos P,Q o R verifica el enunciado. Ahora veremosque

|XP |+ |XQ|+ |XR| ≤ 4

4

Page 771: COMPENDIUM OME - ToomatesEotvos de Hungría, que se iniciaron en 1894, precisamente durante la efervescencia de fin de siglo, consecuencia de la cual fue también el proceso iniciado

En efecto, debido a la simetrıa delrecinto, sera suficiente probar la de-siguladad cuando el punto X este enel sector POQ, siendo O el centrodel cırculo. De hecho, el maximovalor de |XP | + |XQ| + |XR| se al-canza cuando el punto X esta so-bre el arco PQ. En este caso, apli-cando el Teorema de Ptolomeo alcuadrilatero cıclico RQXP se tieneque ax = ay + az de donde resultaque x = y + z y como x ≤ 2 entonces|XP | + |XQ| + |XR| ≤ 2 + 2 = 4 yhemos terminado.

z

y

x

a

a a

X

O

RQ

P

5

Page 772: COMPENDIUM OME - ToomatesEotvos de Hungría, que se iniciaron en 1894, precisamente durante la efervescencia de fin de siglo, consecuencia de la cual fue también el proceso iniciado

LI Olimpiada Matemática Española Fase nacional 2015, Badajoz

Viernes, 20 de marzo PRIMERA SESIÓN

Problema 1 Sobre la gráfica de una función polinómica con coeficientes enteros, se eligen dos puntos con coordenadas enteras. Probar que si la distancia entre ellos es un número entero, entonces el segmento que los une es paralelo al eje de abscisas. Problema 2 En el triángulo ,ABC sea A el punto simétrico de A respecto del circuncentro O de .ABC Probar que: a) La suma de los cuadrados de los segmentos de tangentes trazadas desde A y A ´a la circunferencia inscrita en ABC es igual a ,244 22 rRrR siendo R y r los radios de las circunferencias circunscrita e inscrita de ABC respectivamente. b) La circunferencia de centro A y radio IA corta a la circunferencia circunscrita de

ABC en un punto ,L tal que .ACABAL

Problema 3 En la pizarra está escrito un entero .2N Dos jugadores A y B juegan alternadamente, empezando por A. Cada jugador en su turno reemplaza el número existente por el que resulte de realizar una de estas dos operaciones: restar 1 o dividir entre 2, siempre que se obtenga un entero positivo. El jugador que llegue al número 1 gana. Determinar razonadamente el menor número par N que le exige a A jugar al menos 2015 veces para ganar (no se contabilizan los turnos de B).

No está permitido el uso de calculadoras. Cada problema se puntúa sobre siete puntos.

El tiempo de cada sesión es de tres horas y media.

Page 773: COMPENDIUM OME - ToomatesEotvos de Hungría, que se iniciaron en 1894, precisamente durante la efervescencia de fin de siglo, consecuencia de la cual fue también el proceso iniciado

LI Olimpiada Matemática Española

Fase nacional 2015, Badajoz Sábado, 21 de marzo SEGUNDA SESIÓN

Problema 4 Todas las caras de un poliedro son triángulos. A cada uno de los vértices de este poliedro se

le asigna de forma independiente uno de entre tres colores: verde, blanco o negro. Decimos

que una cara es extremeña si sus tres vértices son de distintos colores, uno verde, uno blanco

y uno negro. ¿Es cierto que, independientemente de cómo coloreemos los vértices, el número

de caras extremeñas de este poliedro es siempre par?

Problema 5 Sean p y n enteros positivos, tales que p es primo, n≥p, y 1+np es un cuadrado perfecto. Probar que 1n es suma de p cuadrados perfectos no nulos. Problema 6 Sean M y N puntos del lado BC del triángulo ABC tales que BM = CN, estando M en el

interior del segmento BN. Sean P, Q puntos que están respectivamente en los segmentos AN,

AM tales que PMC = MAB y QNB = NAC. ¿Es cierto que QBC = PCB?

No está permitido el uso de calculadoras. Cada problema se puntúa sobre siete puntos.

El tiempo de cada sesión es de tres horas y media.

Page 774: COMPENDIUM OME - ToomatesEotvos de Hungría, que se iniciaron en 1894, precisamente durante la efervescencia de fin de siglo, consecuencia de la cual fue también el proceso iniciado

LI Olimpiada matematica Espanola (Concurso Final)

Enunciados y Soluciones

1. Sobre la grafica de una funcion polinomica con coeficientes enteros, se eligen dospuntos con coordenadas enteras. Probar que si la distancia entre ellos es un numeroentero, entonces el segmento que los une es paralelo al eje de abscisas.

Solucion. Sea el polinomio f(x) =n∑

i=0

aixi, ai ∈ Z y sean A(c, f(c)) y B(d, f(d))

dos puntos con coordenadas enteras. Entonces

f(c)− f(d) =n∑

i=1

ai(ci − di)

Todos los sumandos de esta suma son divisibles por c− d, ası que

f(c)− f(d) =

n∑i=1

ai(ci − di) = k(c− d),

donde k es un entero. Como la distancia entre los puntos A y B, que denotamospor d(A,B) es un entero, entonces d2(A,B) es un cuadrado perfecto. Pero

d2(A,B) = (c− d)2 + k2(c− d)2 = (c− d)2(1 + k2)

luego la unica posibilidad para que esta expresion sea un cuadrado perfecto es quek = 0, en cuyo caso

f(c)− f(d) = 0

y efectivamente el segmento AB es paralelo al eje de abcisas.

2. En el triangulo ABC, sea A′ el punto simetrico de A respecto del circuncentroO de ABC.

a) Probar que la suma de los cuadrados de los segmentos de tangentes trazadasdesde A y A′ a la circunferencia inscrita en ABC es igual a 4R2− 4Rr− 2r2.

b) Sea I el incentro del 4ABC. Probar que la circunferencia de centro A′ y radioA′I corta a la circunferencia circunscrita de ABC en un punto L tal queAL =

√AB.AC.

1

Page 775: COMPENDIUM OME - ToomatesEotvos de Hungría, que se iniciaron en 1894, precisamente durante la efervescencia de fin de siglo, consecuencia de la cual fue también el proceso iniciado

Solucion. a) Si t y t′ son las longitudes de los segmentos de tangentes, se tiene

t2 = AI2 − r2 y t′2 = A′I2 − r2;

por lo tanto,

L

M'

M

t'

t

I O

A'

CB

A

t2 + t′2 = AI2 +A′I2 − 2r2

Aplicando el teorema de Apolonio al triangulo AIA′ con la mediana IO resulta

AI2 +A′I2 = 2IO2 + 2R2 = 2(R2 − 2Rr) + 2R2 = 4R2 − 4Rr

Sustituyendo en la expresion anterior resulta la relacion pedida.

b) Teniendo en cuenta a) y que AL2 = A′A2−A′L2 = 4R2−A′I2, ya que el trianguloAA′L es claramente rectangulo por ser AA′ un diametro, resulta

AL2 = 4R2 − (4R2 − 4Rr −AI2) = 4Rr +AI2 =abc

p+bc(p− a)

p= bc,

donde las dos ultimas expresiones se obtienen a partir de las relaciones entre angulosy lados en el triangulo, teniendo en cuenta que t = p− a siendo p el semiperımetro.

3. En la pizarra esta escrito un numero entero. Dos jugadores A y B jueganalternadamente, empezando por A. Cada jugador en su turno reemplaza el numeroexistente por el que resulte de realizar una de estas dos operaciones: restar 1 o dividirentre 2, siempre que se obtenga un entero positivo. El jugador que llegue al numero1 gana. Determinar razonadamente el menor numero par que le exige a A jugar almenos 2015 veces para ganar (no se contabilizan los turnos de B).

2

Page 776: COMPENDIUM OME - ToomatesEotvos de Hungría, que se iniciaron en 1894, precisamente durante la efervescencia de fin de siglo, consecuencia de la cual fue también el proceso iniciado

Solucion. Si el valor de N inicial es par, veamos que gana A: ya sea restando 1o bien dividiendo entre 2 (en el caso N = 4k + 2, N

2 = 2k + 1 impar), A siemprele dejara a B un impar, obligandolo a restar 1 por no poder dividir entre 2, con locual al jugar A volvera a encontrarse con un par, menor que el anterior. Ası, A seencontrara finalmente con un 2, y ganara.Hay que destacar que cuando A tiene dos opciones validas para dejarle a B un impar,siempre preferira dividir entre 2, para acercarse mas rapidamente al objetivo.

A continuacion usaremos el siguiente resultado:

Sea y un numero par con el cual se encuentra A en su turno. Entonces, dos turnosantes A se encontraba con un numero mayor o igual que 2y + 4.

En efecto, distinguiremos dos casos: (1) Caso y = 4k + 2. Proviene de una jugadaobligada de B desde 4k + 3. Antes, A podıa estar en 8k + 6, o bien 4k + 4 (estoes viable, ya que desde 4k + 4 A no puede dividir porque 2k + 2 es par). Es decir,B estaba antes en 8k + 7 o en 4k + 5. Si B estaba en 8k + 7, A pudo estar antesen 16k + 14 o en 8k + 8, mientras que si B estaba en 4k + 5 A solo pudo estar en8k + 10, no en 4k + 6, porque en ese caso habrıa preferido dividir en vez de restar.Resumiendo, si y = 4k+2, hace dos turnos A podıa encontrarse en 16k+14, 8k+10o 8k + 8, siendo la menor opcion justamente 2y + 4.

(2) Caso y = 4k. Razonando de forma similar, se deduce que hace dos turnos A seencontraba en 8k + 4 (2y + 4) o

La solucion al problema se obtendra aplicando repetidamente el resultado anterior.

Definimos la sucesion a0 = 2, an+1 = 2an + 4, cuya formula explıcita es

an = 6 · 2n − 4

Aplicando reiteradamente el resultado, vemos que para todo n se tiene que cualquierN desde el cual se llegue al 2 en 2n turnos, debe cumplir N ≥ an. Es decir, contandocon que A pasa del 2 al 1 en un turno, hemos probado que para todo n, los numerosN que le exigen a A al menos 2n+ 1 jugadas cumplen N ≥ an.Dado que para n = 1007 se tiene 2n+ 1 = 2015, se cumple que N es mayor o igualque el termino a1007 de la sucesion considerada, es decir, el numero 6 · 21007 − 4.Queda por probar que se alcanza la igualdad. Para ello, basta ver que para todon ≥ 1, si A se encuentra con an, tras dos turnos estara en an−1. En efecto,

6 · 2n − 4︸ ︷︷ ︸an

A→ 6 · 2n − 5B→ 6 · 2n − 6

A→ 3 · 2n − 3B→ 3 · 2n − 4 = 6 · 2n−1 − 4︸ ︷︷ ︸

an−1

4. Todas las caras de un poliedro son triangulos. A cada uno de los vertices de estepoliedro se le asigna de forma independiente uno de entre tres colores: verde, blanco

3

Page 777: COMPENDIUM OME - ToomatesEotvos de Hungría, que se iniciaron en 1894, precisamente durante la efervescencia de fin de siglo, consecuencia de la cual fue también el proceso iniciado

o negro. Decimos que una cara es extremena si sus tres vertices son de distintoscolores, uno verde, uno blanco y uno negro. Es cierto que, independientemente decomo coloreemos los vertices, el numero de caras extremenas de este poliedro essiempre par?

Solucion. Sea C el numero de caras del poliedro. Cada cara tiene 3 lados, cada unode los cuales pertenece exactamente a dos caras. Luego el numero total de aristasdel poliedro es 3C/2, que ha de ser entero. Luego el numero C de caras del poliedroes par.A una arista cuyos vertices extremos son del mismo color la llamaremos monocroma.Si sumamos las aristas monocromas de todas las caras, como cada una de ellas estaexactamente en dos caras, tendremos un numero par. A este numero no contribuyenlas caras extremenas, pues no contienen aristas monocromas, y las no extremenaslo hacen con un numero impar: 3, si los tres vertices son del mismo color, o 1 enotro caso. Por tanto, el numero de caras no extremenas tiene que ser par. Como elnumero total de caras es par, tambien sera par el numero de caras extremenas.

5. Sean p y n enteros positivos, tales que p es primo, n ≥ p, y 1+np es un cuadradoperfecto. Probar que n+ 1 es suma de p cuadrados perfectos no nulos.

Solucion. Sea 1 + np = k2, con k entero positivo. Entonces np = k2 − 1 =(k − 1)(k + 1). Ahora consideramos dos casos:

1. Si el primo p divide a k − 1, entonces k − 1 = p` y k = p` + 1, con ` enteropositivo. Por tanto

1 + np = k2 = (p`+ 1)2 = p2`2 + 2p`+ 1⇔ np = p2`2 + 2p`⇔ n = p`2 + 2`

Entonces, n+ 1 = p`2 + 2`+ 1 = (p− 1)`2 + (`+ 1)2 como querıamos probar.

2. Si el primo p divide a k+ 1, entonces k+ 1 = p` y k = p`− 1, con ` > 1 entero(` = 1 se corresponde con el caso n = p− 2, que no es posible). Por tanto

1 + np = k2 = (p`− 1)2 = p2`2 − 2p`+ 1⇔ np = p2`2 − 2p`⇔ n = p`2 − 2`

Entonces, n+ 1 = p`2 − 2`+ 1 = (p− 1)`2 + (`− 1)2 es suma de p cuadradosperfectos.

6. Sean M y N puntos del lado BC del triangulo ABC tales que BM = CN ,estando M en el interior del segmento BN . Sean P,Q puntos que estan respectiva-mente en los segmentos AN,AM tales que ∠PMC = ∠MAB y ∠QNB = ∠NAC.¿Es cierto que ∠QBC = ∠PCB?

Solucion. La idea clave de la solucion es considerar las circunferencias circunscritasde los triangulos BNQ (en verde en la figura) y PMC (en rojo). Si AM corta a

4

Page 778: COMPENDIUM OME - ToomatesEotvos de Hungría, que se iniciaron en 1894, precisamente durante la efervescencia de fin de siglo, consecuencia de la cual fue también el proceso iniciado

la circunferencia (BNQ) en X, y AN corta a la circunferencia (PMC) en Y , esevidente que los cuadrilateros BQNX y MPCY son cılicos. Puesto que ∠QBC =

β

β

α

α

NM

QP

Y

X

C

B

A

∠QBN y ∠PCB = ∠PCM, los angulos del enunciado del problema seran igualessi lo son ∠QBC y ∠PCM. Pero ∠QBN = ∠QXN = ∠MXN y por otra parte∠PCM = ∠PYM = ∠NYM. Entonces el problema estara resuelto en sentidoafirmativo si demostramos que ∠MXN = ∠NYM lo cual es tanto como decir quelos cuatro puntos M,N, Y,X estan en una circunferencia, para lo cual podemosintentar demostrar que

AM ·AX = AN ·AY ⇔ AM

AN=AY

AX

Para ello razonaremos de la siguiente manera:

Los triangulos ABM y ACN tienen la misma area, ya que sus bases son iguales porhipotesis y la altura desde A es la misma; entonces

AM ·AB · sinα = AN ·AC · sinβ, (1)

donde α = ∠MAB y β = ∠NAC. Por otra parte, dos de los angulos del trianguloABX son α y ∠BXQ = ∠QNB = β (en la circunferencia (BNQ)).

Analogamente, dos de los angulos del triangulo ACY son β y α. Por lo tanto esos dostriangulos son semejantes, y escribiendo la proporcionalidad entre lados homologos,se tiene

AY

AX=CY

AB(2)

5

Page 779: COMPENDIUM OME - ToomatesEotvos de Hungría, que se iniciaron en 1894, precisamente durante la efervescencia de fin de siglo, consecuencia de la cual fue también el proceso iniciado

Por ultimo, aplicando el teorema del seno en el triangulo ACY , tenemos

AC

sinα=

CY

sinβ⇔ sinβ

sinα=CY

AC

Utilizando (1) y teniendo en cuenta (2), resulta

AM

AN=AC · sinβAB · sinα

=AC

AB· CYAC

=AY

AX

y esto demuestra la igualdad de angulos indicada en el enunciado

6

Page 780: COMPENDIUM OME - ToomatesEotvos de Hungría, que se iniciaron en 1894, precisamente durante la efervescencia de fin de siglo, consecuencia de la cual fue también el proceso iniciado

LII Olimpiada Matemática Española Fase nacional 2016, Barcelona

Viernes, 1 de abril PRIMERA SESIÓN

Problema 1

Se tienen dos progresiones de números reales, una aritmética y otra geométrica no constante. Se verifica que , y . Estudiar si para cada

entero positivo existe un entero positivo tal que .

Problema 2

Sea un número primo positivo dado. Demostrar que existe un entero tal que es divisible por si y sólo si existe un entero tal que es divisible por

Problema 3

Sea A1 el punto diametralmente opuesto al vértice A del triángulo ABC en la circunferencia circunscrita, y sea A’ el punto en el que la recta AA1 corta al lado BC. La perpendicular a AA’ trazada por A’ corta a los lados AB y AC (o a sus prolongaciones) en M y N, respectivamente. Demostrar que los puntos A, M, A1 y N están en una circunferencia cuyo centro se encuentra en la altura desde A en el triángulo ABC.

No está permitido el uso de calculadoras. Cada problema se puntúa sobre siete puntos.

El tiempo de cada sesión es de tres horas y media.

Page 781: COMPENDIUM OME - ToomatesEotvos de Hungría, que se iniciaron en 1894, precisamente durante la efervescencia de fin de siglo, consecuencia de la cual fue también el proceso iniciado

LII Olimpiada Matemática Española Fase nacional 2016, Barcelona

Sábado, 2 de abril SEGUNDA SESIÓN

Problema 4

Sean un entero positivo, y enteros positivos distintos tales que Hallar todos los enteros tales que divida al producto y

Problema 5

Se consideran todas las permutaciones del conjunto ,entero) tales que es divisible por , para cada Calcular el número total de estas permutaciones .

Problema 6

Sea 2n ≥ entero. Determinar el menor número real positivo de modo que para cualesquiera números reales positivos y cualesquiera números reales , con

, verificando se tiene que

No está permitido el uso de calculadoras. Cada problema se puntúa sobre siete puntos.

El tiempo de cada sesión es de tres horas y media.

Page 782: COMPENDIUM OME - ToomatesEotvos de Hungría, que se iniciaron en 1894, precisamente durante la efervescencia de fin de siglo, consecuencia de la cual fue también el proceso iniciado

Problema 1.

Se tienen dos progresiones de numeros reales, una aritmetica (an)n∈N y otra geometrica (gn)n∈N noconstante. Se cumple que a1 = g1 = 0, a2 = g2 y a10 = g3. Decidir, razonadamente, si para cada enteropositivo p, existe un entero positivo m, tal que gp = am.

Solucion.

Sean d y r = 1 la diferencia y la razon, respectivamente, de las progresiones aritmetica (an)n∈N ygeometrica (gn)n∈N. En primer lugar tenemos g1r = g2 = a2 = a1 + d = g1 + d de donde d = g1(r − 1).En segundo lugar g1r

2 = g3 = a10 = a1 + 9d = g1 + 9g1(r − 1). De aquı sale r2 − 9r + 8 = 0 puestoque g1 = 0. Las soluciones son r = 1 (que debemos descartar ya que la progresion geometrica no esconstante) y r = 8 que es la razon buscada. De aquı tambien resulta d = 7g1.Sea p un entero positivo cualquiera. Debemos encontrar un m tal que gp = am, es decir gp = g18

p−1 =am = a1 + (m− 1)d = g1 + (m− 1)7g1 que es equivalente a 8p−1 + 6 = 7m. Puesto que las potencias de8 modulo 7 siempre son 1, resulta que 8p−1 + 6 es siempre multiplo de 7 y siempre podremos encontrar

m =8p−1 + 6

7.

Problema 2.

Sea p un numero primo positivo dado. Demostrar que existe un entero α tal que α(α− 1)+3 es divisiblepor p si y solo si existe un entero β tal que β(β − 1) + 25 es divisible por p.

Solucion.

Sean f(x) = x(x− 1) + 3 = x2 − x+ 3, g(x) = x(x − 1) + 25 = x2 − x+ 25.

Caso p = 2. No podemos encontrar ni un tal α ni un tal β porque para cualesquiera α y β enteros, f(α)y g(β) son impares, es decir, no multiplos de p simultaneamente, y por lo tanto el enunciado se cumple.

Caso p = 3. Ahora f(1) = 3, g(2) = 27 y el enunciado tambien se cumple.

Caso p ≥ 5. Decir que p divide a f(α) es lo mismo que decir que f(α) ≡ 0 mod p. En adelanteseguiremos con esta notacion de congruencias sobreentendiendo el modulo p. El enunciado es equivalentea ver que las congruencias f(x) = x2 − x+ 3 ≡ 0 y g(x) = x2 − x+ 25 ≡ 0 tengan o no tengan solucionsimultaneamente.Puesto que 2 no es congruente con p, se puede dividir por 2 modulo p. Tenemos

x2 − x+ 3 ≡(x− 1

2

)2

+11

4≡ 0 ⇐⇒ x ≡ 1±

√−11

2.

Analogamente

x2 − x+ 25 ≡(x− 1

2

)2

+99

4≡ 0 ⇐⇒ x ≡ 1± 3

√−11

2.

En consecuencia las congruencias f(x) ≡ 0 y g(x) ≡ 0 tienen o no solucion (a la vez) segun que −11 seacuadrado perfecto modulo p o no lo sea.

Observacion. Recordemos que esto se cumplira segun que

(−11)p−12 ≡ 1 o (−11)

p−12 ≡ −1.

Problema 3.

En la circunferencia circunscrita al triangulo ABC, sea A1 el punto diametralmente opuesto al verticeA. Sea A′ el punto en el que la recta AA1 corta al lado BC. La perpendicular a la recta AA′ trazadapor A′ corta a los lados AB y AC (o a sus prolongaciones) en M y N , respectivamente. Demostrar quelos puntos A, M , A1 y N estan en una circunferencia cuyo centro se encuentra en la altura desde A enel triangulo ABC.

Primera solucion.

Page 783: COMPENDIUM OME - ToomatesEotvos de Hungría, que se iniciaron en 1894, precisamente durante la efervescencia de fin de siglo, consecuencia de la cual fue también el proceso iniciado

Sea O el circuncentro de ABC, y D el pie de la altura desde A. Es conocido que AO y la altura desdeA son rectas isogonales en cualquier triangulo. En nuestro caso, los son en los dos triangulos ABC yAMN , por la manera como se construye el triangulo AMN .En ABC, AA1 es diametro de la circunferencia circunscrita Γ y la recta AD es altura. En AMN , AOes altura, ası que el centro de la circunferencia circunscrita Γ′ estara en la recta AD (isogonal de AO enAMN).Para terminar el problema hay que probar que A1 pertenece a Γ′. En primer lugar, es facil demostrar que

los triangulos ABC y AMN son semejantes (AMN = 90 − α = C). Escribiendo la proporcionalidadentre sus lados, obtenemos,

AB

AC=

AN

AM,

y esto quiere decir que las rectas MC y BN son antiparalelas y el cuadrilatero BMCN es cıclico. SeaΓ′′ la circunferencia que pasa por los vertices de dicho cuadrilatero.El eje radical de Γ y Γ′′ es BC; el de Γ′ y Γ′′ es MN . La interseccion de las rectas BC y MN , es decir,A′, es el centro radical de las tres circunferencias. El eje radical de Γ y Γ′ debe, necesariamente, pasarpor A y A′, luego no puede ser otro que AA′ y por lo tanto A1 estara en la circunferencia Γ′.

Segunda solucion.

Sean Γ y Γ′ las circunferencias circunscritas a ABC y AMN , respectivamente y AA1 un diametro de Γ(por construccion). El triangulo ABA1 es rectangulo en B y su angulo en A1 es C por ver, desde A1 lacuerda AB de Γ. En consecuencia, los triangulosABA1 y ADC son semejantesy los angulos marcados en A son iguales, α = 90 − C.El segmento MA1 se ve desde A′ y desde B bajo angulo recto (el primero por construccion y el segundopor inscrito que ve un diametro). Esto nos dice que los cuatro puntos B,M,A′, A1 son concıclicos. Yesto nos lleva a que γ = δ. De forma similar, el segmento A1N se ve desde A′ y C bajo angulo recto ylos puntos A1, A

′, C,N son concıclicos. De ahı que γ′ = δ = γ.

Se tiene que BA1C = 180− A por ser inscrito en Γ y ver la cuerda BC desde el arco contrario al vertice

A. Tenemos que MA1N = BA1C + γ′ − γ = 180 − A. Esto nos indica que A1 tiene que estar sobre lacircunferencia Γ′ en el arco opuesto al del vertice A.

Como que el triangulo AA′M es rectangulo por construccion, resulta que AMA′ = C. Los puntos M y

P de Γ′ ven sobre esta circunferencia la cuerda AN . Luego APN = C. En el triangulo APN los angulos

en A y en P suman 90, luego ANP = 90 y AP es un diametro de Γ′.

Problema 4.

Sean m ≥ 1 un entero positivo, a y b enteros positivos distintos mayores estrictamente que m2 y menoresestrictamente que m2 +m. Hallar todos los enteros d, que dividen al producto ab y cumplen m2 < d <m2 +m.

Solucion.

Sea d un entero positivo que divida a ab y tal que d ∈ (m2,m2+m). Entonces d divide a (a−d)(b−d) =ab− da− db+ d2. Como que |a− d| < m y |b− d| < m, deducimos que

∣∣(a− d)(b− d)∣∣ < m2 < d lo que

implica que (a− d)(b − d) = 0. Ası d = a o d = b.

Page 784: COMPENDIUM OME - ToomatesEotvos de Hungría, que se iniciaron en 1894, precisamente durante la efervescencia de fin de siglo, consecuencia de la cual fue también el proceso iniciado

Problema 5.

De entre todas las permutaciones (a1, a2, . . . , an) del conjunto 1, 2, ., n, (n ≥ 1 entero), se consideranlas que cumplen que 2(a1 + a2 + · · · + am) es divisible por m, para cada m = 1, 2, . . . , n. Calcular elnumero total de estas permutaciones.

Solucion.

Sea Pn el conjunto de permutaciones de 1, 2, . . . , n que cumplen las condiciones del enunciado. Elproblema consiste en calcular |Pn|. Observemos que, para cualquier n, las condiciones se cumplensiempre para m = 1, para m = 2 y para m = n, de manera que P1, P2 y P3 son, en cada caso, elconjunto de todas las permutaciones y |P1| = 1, |P2| = 2 y |P3| = 6.Supongamos que (a1, . . . , an) ∈ Pn. Tomando m = n − 1, debe cumplirse que(n − 1)|2(a1 + · · · + an−1) = 2(a1 + · · · + an) − 2an = n(n + 1) − 2an. Mirando esta relacion enforma de congruencias, tenemos 2− 2an ≡ 0 mod (n− 1), o bien que 2(an− 1) es multiplo de n− 1, quees equivalente a que an − 1 sea multiplo de n−1

2 . Dada la acotacion obvia an − 1 ≤ n− 1, resulta que losunicos valores que puede tomar an − 1 son 0, n−1

2 o n− 1. Entonces an solamente puede ser 1, n+12 o n.

Si fuese an = n+12 , entonces n deberıa ser impar. La propiedad de Pn para m = n − 2 nos dice,

con un calculo parecido al hecho antes, que (n − 2)|2(a1 + · · · + an−2) = n(n + 1) − 2an−1 − 2an =(n−1)(n+1)−2an−1. Mirando esta relacion en forma de congruencias modulo (n−2) queda 3−2an−1 ≡ 0mod (n − 2), de manera que 2an−1 − 3 tiene que ser multiplo de n − 2 y esto solo sucede si es n − 1.Pero esto conduce a que an−1 = n+1

2 = an, que es absurdo.En conclusion, an solamente puede tomar los valores 1 y n.Estudiemos estos dos casos.

Caso an = n. Entonces (a1, . . . , an−1) es una permutacion de 1, 2, . . . , n−1. Se comprueba facilmenteque es de Pn−1. Entonces habra tantas permutaciones de Pn con an = n como permutaciones en Pn−1.

Caso an = 1. Ahora a1, a2, . . . , an−1 > 1 y (a1 − 1, a2 − 1, . . . , an−1 − 1) es una permutacion de Pn−1.La correspondencia (a1, a2, . . . , an−1, 1) (a1 − 1, a2 − 1, . . . , an−1 − 1) es biyectiva. Habra tantaspermutaciones de Pn con an = 1 como permutaciones en Pn−1.

En definitiva, |Pn| = 2|Pn−1| si n > 3, de donde, |Pn| = 3 · 2n−2.

Observacion: La demostracion anterior nos da el algoritmo recurrente para obtener todas las permuta-ciones que cumplen la condicion. Por ejemplo, conocemos que las permutaciones de P3 son todas.Anadiendo un 4 al final de cada una obtenemos la mitad de las de P4. La otra mitad sale de sumar 1 acada elemento de cada permutacion y anadir un 1 al final.

Page 785: COMPENDIUM OME - ToomatesEotvos de Hungría, que se iniciaron en 1894, precisamente durante la efervescencia de fin de siglo, consecuencia de la cual fue también el proceso iniciado

Problema 6.

Sea n ≥ 2 un numero entero. Determinar el menor numero real positivo γ de modo que para cua-lesquiera numeros reales positivos x1, x2, . . . , xn y cualesquiera numeros reales y1, y2, . . . , yn con 0 ≤y1, y2, . . . , yn ≤ 1

2 que cumplan x1 + x2 + . . .+ xn = y1 + y2 + . . .+ yn = 1, se tiene que

x1x2 . . . xn ≤ γ (x1y1 + x2y2 + . . .+ xnyn),

Solucion.

Sean M = x1x2 . . . xn y Xi =M

xipara 1 ≤ i ≤ n. Consideremos la funcion

ϕ : (0,+∞) → R definida por ϕ(t) =M

tque es convexa como se prueba facilmente. Como los numeros

no negativos yi, (1 ≤ i ≤ n), son tales que y1 + y2 + . . .+ yn = 1, entonces aplicando la desigualdad deJensen a la funcion ϕ se tiene

ϕ

(n∑

i=1

yixi

)≤

n∑i=1

yiϕ(xi),

es decir,

M

(n∑

i=1

yixi

)−1

≤n∑

i=1

yiM

xi=

n∑i=1

yiXi. (1)

Ahora se trata de encontrar la menor cota superior del termino de la derecha de (1). Sin perdida degeneralidad, podemos suponer que x1 ≤ x2,≤ . . . ≤ xn e y1 ≥ y2 ≥ . . . ≥ yn. Entonces se tiene queX1 ≥ X2 ≥ . . . ≥ Xn como se comprueba inmediatamente. Aplicando la desigualdad del reordenamiento,

sabemos que entre todas las sumas de la forma

n∑i=1

yiXi la que alcanza el valor maximo es la que se obtiene

cuando y1 ≥ y2 ≥ . . . ≥ yn y X1 ≥ X2 ≥ . . . ≥ Xn.

Ahora observamos que

n∑i=1

yiXi = y1X1 + (y2X2 + . . .+ ynXn) ≤ y1X1 + (y2 + . . .+ yn)X2 = y1X1 + (1− y1)X2.

Al ser 0 ≤ y1 ≤ 1/2, se tiene que

n∑i=1

yiXi ≤1

2(X1 +X2) =

1

2

((x1 + x2)x3 . . . xn

)≤

≤ 1

2

((x1 + x2) + x3 + . . .+ xn

n− 1

)n−1

=1

2

(1

n− 1

)n−1

donde se ha utilizado la desigualdad entre las medias aritmetica y geometrica y la condicion x1 + x2 +. . .+ xn = 1. De lo anterior y (1), resulta

M ≤(

n∑i=1

yixi

)(n∑

i=1

yiXi

)≤ 1

2

(1

n− 1

)n−1(

n∑i=1

yixi

)

y

γ ≤ 1

2

(1

n− 1

)n−1

.

Si tomamos x1 = x2 =1

2(n− 1), x3 = x4 = . . . xn =

1

n− 1e y1 = y2 =

1

2, y3 = y4 = . . . = yn = 0,

entonces

M = x1x2 . . . xn =1

4

(1

n− 1

)n

=

=1

2

(1

n− 1

)n−1

(y1x1 + y2x2) =1

2

(1

n− 1

)n−1 n∑i=1

yixi

Page 786: COMPENDIUM OME - ToomatesEotvos de Hungría, que se iniciaron en 1894, precisamente durante la efervescencia de fin de siglo, consecuencia de la cual fue también el proceso iniciado

M = x1x2 . . . xn =1

4

(1

n− 1

)n

=1

2

(1

n− 1

)n−1

(y1x1 + y2x2) =1

2

(1

n− 1

)n−1 n∑i=1

yixi

y se concluye que

γ =1

2

(1

n− 1

)n−1

.

Page 787: COMPENDIUM OME - ToomatesEotvos de Hungría, que se iniciaron en 1894, precisamente durante la efervescencia de fin de siglo, consecuencia de la cual fue también el proceso iniciado

LIII Olimpiada Matemática Española

Concurso Final Nacional

PRIMERA SESIÓN

Alcalá de Henares, viernes 24 de marzo de 2017

Problema 1

Determina el número de valores distintos de la expresión

𝑛2 − 2

𝑛2 − 𝑛 + 2,

donde 𝑛 ∈ 1, 2, 3, … , 100.

Problema 2

Un trazador de puntos medios es un instrumento que dibuja el punto medio exacto de

dos puntos previamente señalados. Partiendo de dos puntos a distancia 1 y utilizando

sólo el trazador de puntos medios, debes obtener dos puntos a una distancia

estrictamente comprendida entre 1

2017 y

1

2016, trazando el menor número posible de

puntos. ¿Cuál es el mínimo número de veces que necesitas utilizar el trazador de puntos

medios, y qué estrategia seguirías para lograr tu objetivo?

Problema 3

Sea 𝑝 un primo impar y 𝑆𝑞 =1

2∙3∙4+

1

5∙6∙7+ ⋯ +

1

𝑞(𝑞+1)(𝑞+2), donde 𝑞 =

3𝑝−5

2.

Escribimos 1

𝑝− 2𝑆𝑞 en la forma

𝑚

𝑛, donde 𝑚 y 𝑛 son enteros. Demuestra que 𝑚 ≡

𝑛 (𝑚ó𝑑 𝑝); es decir, 𝑚 y 𝑛 dan el mismo resto al ser divididos por 𝑝.

No está permitido el uso de calculadoras,

ni dispositivos electrónicos o digitales de ningún tipo.

Cada problema se puntúa sobre siete puntos.

El tiempo de cada sesión es de TRES HORAS Y MEDIA.

Page 788: COMPENDIUM OME - ToomatesEotvos de Hungría, que se iniciaron en 1894, precisamente durante la efervescencia de fin de siglo, consecuencia de la cual fue también el proceso iniciado

LIII Olimpiada Matemática Española

Concurso Final Nacional

SEGUNDA SESIÓN

Alcalá de Henares, sábado 25 de marzo de 2017

Problema 4

Se dispone de una fila de 2018 casillas, numeradas consecutivamente de 0 a 2017.

Inicialmente, hay una ficha colocada en la casilla 0. Dos jugadores A y B juegan

alternativamente, empezando A, de la siguiente manera: En su turno, cada jugador

puede, o bien hacer avanzar la ficha 53 casillas, o bien hacer retroceder la ficha 2

casillas, sin que en ningún caso se sobrepasen las casillas 0 ó 2017. Gana el jugador

que coloque la ficha en la casilla 2017. ¿Cuál de ellos dispone de una estrategia

ganadora, y cómo tendría que jugar para asegurarse ganar?

Problema 5

Determina el máximo valor posible de la expresión

27𝑎𝑏𝑐 + 𝑎√𝑎2 + 2𝑏𝑐 + 𝑏√𝑏2 + 2𝑐𝑎 + 𝑐√𝑐2 + 2𝑎𝑏,

siendo 𝑎, 𝑏, 𝑐, números reales positivos tales que 𝑎 + 𝑏 + 𝑐 =1

√3.

Problema 6

En el triángulo 𝐴𝐵𝐶, los puntos medios respectivos de los lados 𝐵𝐶, 𝐴𝐵 y 𝐴𝐶 son 𝐷, 𝐸

y 𝐹. Sean: 𝑀 el punto donde la bisectriz interior del ángulo 𝐴𝐷𝐵 corta al lado 𝐴𝐵, y 𝑁

el punto donde la bisectriz interior del ángulo 𝐴𝐷𝐶 corta al lado 𝐴𝐶. Sean además 𝑂

el punto de intersección de las rectas 𝐴𝐷 y 𝑀𝑁, 𝑃 el punto de intersección de 𝐴𝐵 y 𝐹𝑂,

y 𝑅 el punto de intersección de 𝐴𝐶 y 𝐸𝑂. Demuestra que 𝑃𝑅 = 𝐴𝐷.

No está permitido el uso de calculadoras,

ni dispositivos electrónicos o digitales de ningún tipo.

Cada problema se puntúa sobre siete puntos.

El tiempo de cada sesión es de TRES HORAS Y MEDIA

Page 789: COMPENDIUM OME - ToomatesEotvos de Hungría, que se iniciaron en 1894, precisamente durante la efervescencia de fin de siglo, consecuencia de la cual fue también el proceso iniciado

LIII Olimpiada matematica Espanola (Concurso Final)

Enunciados y Soluciones

1. Determina el numero de valores distintos de la expresion

n2 − 2

n2 − n + 2,

donde n ∈ 1, 2, . . . , 100.

Solucion. Sumando y restando 2− n al numerador se obtiene

an =n2 − 2

n2 − n + 2=

n2 − 2− n + 2 + n− 2

n2 − n + 2= 1 +

n− 4

n2 − n + 2

Ahora vamos a ver si hay dos terminos iguales, es decir, cuando es ap = aqpara p 6= q. Esto es equivalente a encontrar los enteros p 6= q para los que

p− 4

p2 − p + 2=

q − 4

q2 − q + 2⇔ (p− q)(pq − 4p− 4q + 2) = 0

pq − 4p− 4q + 2 + 14 = 14⇔ (p− 4)(q − 4) = 14

De lo anterior se deduce que (p−4) | 14 y p−4 ∈ ±1,±2,±7,±14. Los valoresnegativos no son posibles porque ambos p, q ≥ 1 con lo que p−4 ∈ 1, 2, 7, 14.Como (p − 4)(q − 4) = 14 entonces q − 4 ∈ 14, 7, 2, 1 de donde resultanlos pares (p, q) = (5, 18) y (p, q) = (6, 11) para los que a5 = 22

23= a18 y

a6 = 1716

= a11. Finalmente, dado que todos los an son numeros racionals,entonces entre los 100 primeros terminos de la sucesion hay 98 que son distintos.

2. Un trazador de puntos medios es un instrumento que dibuja el punto medioexacto de dos puntos previamente senalados. Partiendo de dos puntos a distan-cia 1 y utilizando solo el trazador de puntos medios, debes obtener dos puntos

a una distancia estrictamente comprendida entre1

2017y

1

2016, trazando el

menor numero posible de puntos. ¿Cual es el mınimo numero de veces que

1

Page 790: COMPENDIUM OME - ToomatesEotvos de Hungría, que se iniciaron en 1894, precisamente durante la efervescencia de fin de siglo, consecuencia de la cual fue también el proceso iniciado

necesitas utilizar el trazador de puntos medios, y que estrategia seguirıas paralograr tu objetivo?

Solucion. Sin perdida de generalidad podemos trabajar con la recta real yconsiderar que uno de los puntos iniciales es el 0 y el otro el 1. Es facil com-probar que despues de k aplicaciones del trazador, todos los puntos hallados

son de la forman

2kcon 0 ≤ n ≤ 2k siendo la fraccion no necesariamente

irreducible. Esto lo probaremos por induccion. En efecto, para k = 1 se tienenlos puntos

0 =0

21,

1

2=

1

21, 1 =

2

21

Supongamos que en el paso k los puntos son

0 =0

2k,

1

2k, · · · , r

2k, · · · , s

2k, · · · , 1 =

2k

2k

Entonces, aplicando el trazador de nuevo, en el paso k+1 los puntos obtenidosson

0 =0

2k+1,

1

2k+1, · · · , 1

2

( r

2k+

s

2k

)=

r + s

2k+1, · · · , 1 =

2k+1

2k+1

Ahora nos centramos en las distancias entre puntos. Utilizando el trazadoruna vez, la distancia entre dos cualesquiera de los puntos obtenidos

0 =0

21,

1

2=

1

21, 1 =

2

21

es un multiplo de1

21y aplicando el trazador k veces resulta que la distancia

entre dos cualesquiera puntos es un multiplo de1

2k. Es inmediato comprobar

que entre64

217y

66

217se cumplen las desigualdades

64

217<

1

2017<

65

217<

1

2016<

66

217

Luego cualquier distancia estrictamente incluida entre 12017

y 12016

ha de tenernecesariamente en el denominador un exponente k ≥ 17.

Ahora vamos a ver que el numero mınimo de veces que hay que utilizar eltrazador de puntos medios es k = 17 y describiremos una estrategia paraconseguirlo. En efecto, usando el trazador una vez se obtiene el punto 1

2= 1

21,

luego lo utilizamos

2

Page 791: COMPENDIUM OME - ToomatesEotvos de Hungría, que se iniciaron en 1894, precisamente durante la efervescencia de fin de siglo, consecuencia de la cual fue también el proceso iniciado

de nuevo entre este punto y 1 para obtener el punto 34

= 322, y a partir de aquı

calculamos el punto medio entre el ultimo punto hallado hasta ese momento,

y el punto1

2, obteniendo sucesivamente los puntos

5

8=

5

23,

9

16=

9

24,

17

32=

17

25,

33

64=

33

26,

y tras un total de 7 aplicaciones del trazador, se obtiene el punto65

128=

65

27.

Aplicamos ahora 10 veces el trazador, para obtener el punto medio entre elultimo punto hallado hasta ese momento y el punto 0, obteniendose sucesiva-mente los puntos

65

28,

65

29,

65

210, · · · , 65

217

Este ultimo punto dista65

217tanto de 0 como del anterior punto hallado

65

216.

Luego en 17 pasos hemos conseguido nuestro objetivo, y hemos terminado.

3. Sean p un primo impar y Sq =1

2 · 3 · 4+

1

5 · 6 · 7+ . . . +

1

q(q + 1)(q + 2),

donde q =3p− 5

2. Escribimos

1

p− 2Sq en la forma

m

n, donde m y n son

enteros. Demuestra que m ≡ n (mod p); es decir, que m y n dan el mismoresto al ser divididos por p.

Solucion. Se tiene que

2

k(k + 1)(k + 2)=

(k + 2)− k

k(k + 1)(k + 2)=

1

k(k + 1)− 1

(k + 1)(k + 2)

=

(1

k− 1

k + 1

)−(

1

k + 1− 1

k + 2

)=

1

k+

1

k + 1+

1

k + 2− 3

k + 1

con lo que

2Sq =

(1

2+

1

3+

1

4+ · · ·+ 1

q+

1

q + 1+

1

q + 2

)− 3

(1

3+

1

6+ · · ·+ 1

q + 1

)Antes de considerar el caso general veamos que ocurre con los primeros casosparticulares. Para p = 3 es q = 2 y se cumple el enunciado trivialmente. Enel caso en que p = q = 5, se tiene

2S5 =2

2 · 3 · 4+

2

5 · 6 · 7=

(1

2+

1

3+

1

4+

1

5+

1

6+

1

7

)− 3

(1

3+

1

6

)3

Page 792: COMPENDIUM OME - ToomatesEotvos de Hungría, que se iniciaron en 1894, precisamente durante la efervescencia de fin de siglo, consecuencia de la cual fue también el proceso iniciado

y

n−m

n= 1− m

n= 1 + 2S5 −

1

5=

1

3+

1

4+

1

6+

1

7=

(1

3+

1

7

)+

(1

4+

1

6

)

=2 · 54 · 6

+2 · 53 · 7

Como en los denominadores no aparece el primo 5 entonces n−mn

es una fraccioncuyo numerador es multiplo de 5 mientras que su denominador no lo es, lo quepermite concluir que n−m es multiplo de 5 o n ≡ m (mod 5).

Notese que se han agrupado las p − 1 = 4 fracciones en p−12

= 2 parentesiscuyos denominadores suman 2p = 10.

Generalizando lo anterior, resulta

n−m

n= 1− m

n= 1 + 2Sq −

1

p

= 1+

(1

2+

1

3+

1

4+ · · ·+ 1

q+

1

q + 1+

1

q + 2

)−3

(1

3+

1

6+ · · ·+ 1

q + 1

)− 1

p

= 1 +

(1

2+

1

3+ · · ·+ 2

3p− 1

)−(

1 +1

2+ · · ·+ 2

p− 1

)− 1

p

=2

p + 1+ · · ·+ 1

p− 1+

1

p + 1+ · · ·+ 2

3p− 1︸ ︷︷ ︸p−1 sumandos

=

(1

p+12

+1

3p−12

)+ · · ·+

(1

p− 1+

1

p + 1

)︸ ︷︷ ︸

p−12

parentesis cuyos denominadores suman 2p

=2p

(p+12

)(3p−12

)+ · · ·+ 2p

(p− 1)(p + 1)︸ ︷︷ ︸p−12

terminos

Al igual que en el caso particular, todos los factores que aparecen en los deno-minadores son primos con p mientras que el numerador no lo es, lo que permiteconcluir que n−m es multiplo de p. Es dcecir, n ≡ m (mod p).

4. Se dispone de una fila de 2018 casillas, numeradas consecutivamente de 0a 2017. Inicialmente, hay una ficha colocada en la casilla 0. Dos jugadores

4

Page 793: COMPENDIUM OME - ToomatesEotvos de Hungría, que se iniciaron en 1894, precisamente durante la efervescencia de fin de siglo, consecuencia de la cual fue también el proceso iniciado

A y B juegan alternativamente, empezando A, de la siguiente manera: En suturno, cada jugador puede, o bien hacer avanzar la ficha 53 casillas, o bienhacer retroceder la ficha 2 casillas, sin que en ningun caso se sobrepasen lascasillas 0 o 2017. Gana el jugador que coloque la ficha en la casilla 2017.¿Cual de ellos dispone de una estrategia ganadora, y como tendrıa que jugarpara asegurarse ganar?

Solucion. Vamos a probar que el jugador A tiene estrategia ganadora. Co-mienza de la unica forma posible: llevando la ficha hasta la casilla 53. A partirde ahı, durante 38 turnos dobles BA, el jugador A hara lo contrario de B: si Bavanza 53, A retrocede 2, y viceversa. De este modo, la ficha queda en la casilla53 + 38×51 = 1991 y es turno de B. Los siguientes movimientos son forzados:7 turnos dobles BA de restar. La ficha queda en la casilla 1991−14×2 = 1963y es turno de B. Ahora:

1. Si B avanza 53, dejara la ficha en la casilla 2016 y tras 13 turnos doblesAB, forzados, la ficha queda en la casilla 2016− 26× 2 = 1964 y A ganasumando 53.

2. Si B resta 2, dejara la ficha en la casilla 1961. Entonces, A avanza 53para dejarla en 2014. Tras 12 turnos dobles forzados BA, la ficha quedaen 2014− 24× 2 = 1966. Despues, B esta obligado a restar 2 hasta 1964y, en su turno, A gana sumando 53.

5. Determina el maximo valor posible de la expresion

27abc + a√a2 + 2bc + b

√b2 + 2ca + c

√c2 + 2ab,

siendo a, b, c, numeros reales positivos tales que a + b + c =1√3.

Solucion. En primer lugar se observa que cuando a = b = c = 13√3

el valor

que toma la expresion es2

3√

3, lo cual sugiere conjeturar que

27abc + a√a2 + 2bc + b

√b2 + 2ca + c

√c2 + 2ab ≤ 2

3√

3

Para probar la conjetura, se puede aplicar la desigualdad de Cauchy, (~u ·~v)2 ≤||~u||2 ||~v||2 a los vectores ~u = (

√a,√b,√c) y ~v = (

√bc,√ca,√ab), obteniendose

9abc =(√

abc +√abc +

√abc)2≤ (a + b + c) (bc + ca + ab)

5

Page 794: COMPENDIUM OME - ToomatesEotvos de Hungría, que se iniciaron en 1894, precisamente durante la efervescencia de fin de siglo, consecuencia de la cual fue también el proceso iniciado

Multiplicando por 3 ambos miembros de la desigualdad anterior y teniendo encuenta la restriccion, resulta

27 abc ≤ 3 (a + b + c) (bc + ca + ab) =√

3 (bc + ca + ab)

Por otro lado, dado que a√a2 + 2bc =

1√3

√3a2(a2 + 2bc), aplicando la desi-

gualdad entre las medias aritmetica y geometrica a los numeros 3a2 y a2 + 2bcse obtiene

a√a2 + 2bc =

1√3

√3a2(a2 + 2bc) ≤ 1√

3

3a2 + (a2 + 2bc)

2=

1√3

(2a2 + bc)

Analogamnete, se tiene que

b√b2 + 2ca ≤ 1√

3(2b2 + ca)

y

c√c2 + 2ab ≤ 1√

3(2c2 + ab)

Combinando las desigualdades anteriores, y teniendo en cuenta otra vez larestriccion, se obtiene

27abc + a√a2 + 2bc + b

√b2 + 2ca + c

√c2 + 2ab

≤√

3 (bc + ca + ab) +1√3

(2a2 + bc + 2b2 + ca + 2c2 + ab)

=2√3

(a2 + b2 + c2 + 2ab + 2bc + 2ca) =2√3

(a + b + c)2 =2

3√

3

Esto prueba la conjetura y el maximo de la expresion es2

3√

3.

6. En el triangulo ABC los puntos medios respectivos de los lados BC,CAy AB son D,F,E. Sean: M el punto donde la bisectriz interior del angulo∠ADB corta al lado AB, y N el punto donde la bisectriz interior del angulo∠ADC corta al lado AC. Sean ademas O el punto de interseccion de lasrectas AD y MN , P el punto de interseccion de AB y FO, y R el punto deinterseccion de AC y EO. Demuestra que PR = AD.

Solucion. Aplicando el teorema la bisectriz a los triangulos ADB y ADC seobtiene

MB

AM=

BD

ADy

NC

AN=

DC

AD

6

Page 795: COMPENDIUM OME - ToomatesEotvos de Hungría, que se iniciaron en 1894, precisamente durante la efervescencia de fin de siglo, consecuencia de la cual fue también el proceso iniciado

RPO

NMFE

D CB

A

Como BD = DC,MB

AM=

NC

ANy entonces MN es paralelo a BC, de donde

AB

AM=

BC

MN. De aquı, junto con

MB

AM=

BD

AD, resulta

BD + AD

AD= 1 +

BD

AD= 1 +

MB

AM=

MB + AM

AM=

AB

AM=

BC

MN(1)

Como EF es la paralela media de ABC, EF y BC son paralelas, BD = EFy BC = 2 · EF . Sustituyendo esto en (1) obtenemos

BD + AD

AD=

BC

MN⇔ BD + AD

AD=

2 · EF

MN⇔ 2

MN=

BD + AD

AD · EF

que lo podemos escribir como

2

MN=

1

AD+

1

EF(2)

Como MN y EF son paralelas a BC, son paraleleas entre sı, y entonces

AE

EM=

AF

AN(3)

Aplicando dos veces el Teorema de Menelao al triangulo AMN , una con latransversal ER y otra con la transversal PF , se obtiene

AR

NR=

AE

EM· MO

ONy

AP

MP=

AF

FN· MO

ON,

7

Page 796: COMPENDIUM OME - ToomatesEotvos de Hungría, que se iniciaron en 1894, precisamente durante la efervescencia de fin de siglo, consecuencia de la cual fue también el proceso iniciado

ası que teniendo en cuenta (3) obtenemosAP

MP=

AR

NRlo cual es equivalente a

que PR es paralelo a MN . Pero MN es paralelo a EF y el cuadrilatero EPRFes un trapecio. Ya que MN es paralela a las bases por el punto de interseccionde las diagonales, MN es la media armonica de las bases del trapecio, es decir

1

PR+

1

EF=

2

MNy por (2),

1

PR=

1

AD

de donde se concluye que PR = AD.

8

Page 797: COMPENDIUM OME - ToomatesEotvos de Hungría, que se iniciaron en 1894, precisamente durante la efervescencia de fin de siglo, consecuencia de la cual fue también el proceso iniciado

LIV Olimpiada Matemática Española

Concurso Final Nacional

PRIMERA SESIÓN

Jaén, viernes 16 de marzo de 2018

Problema 1

Determina todos los enteros positivos 𝑥, tales que 2𝑥 + 1 sea un cuadrado perfecto,

pero entre los números 2𝑥 + 2, 2𝑥 + 3, … ,3𝑥 + 2, no haya ningún cuadrado perfecto.

Problema 2

Se colocan 2n+1 fichas, blancas y negras, en una fila ( 1)n . Se dice que una ficha está

equilibrada si el número de fichas blancas a su izquierda, más el número de fichas

negras a su derecha es n. Determina, razonadamente, si el número de fichas que están

equilibradas es par o impar.

Problema 3

Sea 𝑂 el circuncentro del triángulo acutángulo 𝐴𝐵𝐶 y sea 𝑀 un punto arbitrario del

lado 𝐴𝐵. La circunferencia circunscrita del triángulo 𝐴𝑀𝑂 interseca por segunda vez a

la recta 𝐴𝐶 en el punto 𝐾 y la circunferencia circunscrita del triángulo 𝐵𝑂𝑀 interseca

por segunda vez a la recta 𝐵𝐶 en el punto 𝑁.

Prueba que Á𝑟𝑒𝑎 (𝑀𝑁𝐾) ≥1

4Á𝑟𝑒𝑎 (𝐴𝐵𝐶) y determina el caso en que se alcanza la

igualdad.

No está permitido el uso de calculadoras,

ni dispositivos electrónicos o digitales de ningún tipo.

Cada problema se puntúa de cero a siete puntos.

El tiempo de cada sesión es de TRES HORAS Y MEDIA.

Page 798: COMPENDIUM OME - ToomatesEotvos de Hungría, que se iniciaron en 1894, precisamente durante la efervescencia de fin de siglo, consecuencia de la cual fue también el proceso iniciado

LIV Olimpiada Matemática Española

Concurso Final Nacional

SEGUNDA SESIÓN

Jaén, sábado 17 de marzo de 2018

Problema 4

Los puntos de una superficie esférica de radio 4, se pintan con cuatro colores distintos.

Prueba que existen dos puntos sobre la superficie que tienen el mismo color y que están

a distancia 4√3 o bien a distancia 2√6. NOTA: La distancia entre dos puntos es la distancia euclídea; es decir la longitud del segmento

rectilíneo que los une.

Problema 5

Sean 𝑎 y 𝑏 dos números positivos primos entre sí. Se dice que un entero positivo 𝑛 es

débil si no puede ser escrito en la forma 𝑛 = 𝑎𝑥 + 𝑏𝑦, para algunos enteros 𝑥 e 𝑦 no

negativos. Prueba que si 𝑛 es débil y 𝑛 <𝑎𝑏

6, existe un entero 𝑘 ≥ 2, tal que 𝑘𝑛 es

débil.

Problema 6

Sea 𝑅+el conjunto de los números reales positivos. Halla todas las funciones

𝑓: 𝑅+ → 𝑅+, tales que 𝑓(𝑥 + 𝑓(𝑦)) = 𝑦𝑓(𝑥𝑦 + 1), para todo 𝑥, 𝑦 > 0.

No está permitido el uso de calculadoras,

ni dispositivos electrónicos o digitales de ningún tipo.

Cada problema se puntúa de cero a siete puntos.

El tiempo de cada sesión es de TRES HORAS Y MEDIA

Page 799: COMPENDIUM OME - ToomatesEotvos de Hungría, que se iniciaron en 1894, precisamente durante la efervescencia de fin de siglo, consecuencia de la cual fue también el proceso iniciado

LIV Olimpiada matematica Espanola (Concurso Final)

Enunciados y Soluciones

1. Determina todos los enteros positivos x, tales que 2x + 1 sea un cuadradoperfecto, pero entre los numeros 2x + 2, 2x + 3, · · · , 3x + 2, no haya ninguncuadrado perfecto.

Solucion. Sea n un numero entero tal que 2x+1 = n2 y n2 ≤ 3x+2 < (n+1)2.De la primera ecuacion se obtiene x = (n2 − 1)/2 y sustituyendo este valor enla doble desigualdad, resulta

n2 ≤ 3n2 + 1

2< n2 + 2n+ 1⇔ 2n2 ≤ 3n2 + 1 < 2n2 + 4n+ 2

En la ultima expresion la primera desigualdad se cumple para todo n entero,la segunda puede escribirse como

n2 − 4n− 1 < 0⇔ 2−√

5 < n < 2 +√

5,

y se verifica para todos los enteros n ∈ 0, 1, 2, 3, 4. De entre estos valores,solo para n = 3, se obtiene que x = (32 − 1)/2 = 4 es entero positivo, y estaes la unica solucion del problema, ya que en este caso 2x + 1 = 9 y entre losnumeros 10, 11, 12, 13 y 14 no hay ningun cuadrado perfecto.

2. Se colocan 2n+ 1 fichas, blancas y negras, en una fila (n ≥ 1). Se dice queuna ficha esta equilibrada si el numero de fichas blancas a su izquierda, mas elnumero de fichas negras a su derecha es n. Determina, razonadamente, si elnumero de fichas que estan equilibradas es par o impar.

Solucion. Numeramos las posiciones en la fila desde 1 hasta 2n + 1, deizquierda a derecha. Definimos la valoracion de una cierta posicion k =1, 2, · · · , 2n+1 como el numero de fichas blancas a su izquierda mas el numerode fichas negras a su derecha, con lo que una ficha esta equilibrada si y solo sisu valoracion es igual a n.

1

Page 800: COMPENDIUM OME - ToomatesEotvos de Hungría, que se iniciaron en 1894, precisamente durante la efervescencia de fin de siglo, consecuencia de la cual fue también el proceso iniciado

Supongamos que las fichas en posiciones k y k + 1 tienen distinto color. Elnumero de fichas blancas a su izquierda en las posiciones 1, 2, · · · , k − 1 y elnumero de fichas negras a su derecha en las posiciones k+ 2, k+ 3, · · · , 2n+ 1son los mismos para ambas. La suma de estas dos cantidades es la valoracioncomun de ambas si la ficha en posicion k es negra y la ficha en posicion k + 1es blanca; pero en caso contrario, la ficha negra en k + 1 tiene una fichablanca adicional a su izquierda, y la ficha blanca en posicion k tiene una fichanegra adicional a su derecha, con lo que la valoracion de estas dos fichas es encualquiera de los dos casos la misma.

Tenemos entonces que, sean cuales sean sus colores, si intercambiamos lasfichas en posiciones k y k+ 1, la paridad del numero de fichas equilibradas novarıa. En efecto, la valoracion de las fichas en las posiciones 1, 2, · · · , k − 1y en las posiciones k + 2, k + 3, · · · , 2n + 1 no varıan con el cambio, si lasfichas en posiciones k y k+ 1 tienen el mismo color simplemente intercambiansus valoraciones, y si tienen distinto color, entonces ambas tienen la mismavaloracion antes del cambio, y ambas tienen la misma valoracion despues delcambio, luego el numero de fichas equilibradas permanece constante, puedeaumentar en 2, o reducirse en 2.

Como toda permutacion se puede descomponer en intercambios sucesivos deelementos contiguos, la paridad del numero de fichas equilibradas no cambiacuando situamos todas las fichas negras en las primeras posiciones de la fila,y todas las blancas en las ultimas posiciones de la fila.

Sea a el numero de fichas negras y b el de fichas blancas, con la condicion deque a+ b = 2n+ 1. Representamos la fila de fichas blancas y negras medianteun camino en el interior de un rectangulo a × b, que empieza por la esquinainferior izquierda del rectangulo. La fila se recorre de izquierda a derecha. Sila ficha es negra se marca un paso unidad hacia arriba, y si es blanca, un pasounidad hacia la derecha. Se considera el segmento L que une dos lados parale-los del rectangulo, pasa por su centro O, y forma angulos de 45 con los ladosdel mismo. El rectangulo queda ası dividido por L en dos polıgonos simetricosrespecto del punto O. Y las fichas equilibradas corresponden precisamente alos pasos del camino que cruzan L. Si consideramos la fila con todas las fichasnegras al principio de la fila a la izquierda de las fichas blancas (lo cual noaltera la paridad del numero de fichas equilibradas), entonces el correspondi-ente camino es formado por el lado vertical del rectangulo y el lado superiordel mismo. Ası L corta una vez al camino y por tanto el numero de fichasequilibradas es impar.

2

Page 801: COMPENDIUM OME - ToomatesEotvos de Hungría, que se iniciaron en 1894, precisamente durante la efervescencia de fin de siglo, consecuencia de la cual fue también el proceso iniciado

3. Sea O el circuncentro del triangulo acutangulo ABC y sea M un punto arbi-trario del lado AB. La circunferencia circunscrita al triangulo AMO intersecapor segunda vez a la recta AC en el punto K y la circunferencia circunscritaal triangulo BOM interseca por segunda vez a la recta BC en el punto N .Prueba que

Area (MNK) ≥ 1

4Area (ABC)

y determina el caso en que se alcanza la igualdad.

Solucion. Utilizaremos las notaciones habituales en Geometrıa del triangulo(R, radio de la circunferencia circunscrita al 4ABC, O su incentro; a longitud

del lado BC del triangulo ABC, etc.). SeaMA

MB= k > 0, entonces

k + 1 =MA+MB

MB=

c

MB⇒MB =

c

k + 1

O

NK

M

C

BA

En lo que sigue utilizaremos el parametro k para fijar la posicion del puntoM en el segmento AB. Si α = ∠MNB = ∠MOB y β = ∠MOA = ∠MKAentonces α+β = 2C. Sean r1 y r2 los radios de las circunferencias circunscritasa los triangulos OMA y OMB, respectivamente. Aplicando el teorema de lossenos generalizado en los dos triangulos OMB y OMA, habida cuenta que∠OMA+ ∠OMB = 180, resulta

OA

sin∠OMA= 2 r1 y

OB

sin∠OMB= 2 r2

De lo anterior se deudce que r1 = r2 ya que OA = OB = R y los angulos∠OMA y ∠OMB son suplementarios.

3

Page 802: COMPENDIUM OME - ToomatesEotvos de Hungría, que se iniciaron en 1894, precisamente durante la efervescencia de fin de siglo, consecuencia de la cual fue también el proceso iniciado

Aplicando el teorema de los senos a los triangulos MAK y MNB, se obtiene

MK

sinA= 2 r1 y

MN

sinB= 2 r2

De lo anterior, resulta

MK

sinA=MN

sinB⇒ MK

a=MN

b(1)

Por otra parte, se verifica que ∠KMA = 180 − (β + A) y ∠NMB = 180 −(α +B). Entonces,

∠KMN = 180 − (∠NMB + ∠KMA) = C (2)

De (1) y (2) se deduce que los triangulos MNK y ABC son semejantes y queMN es el lado homologo de b en la semejanza. Entonces, para el cociente entrelas areas, se tiene

[MNK]

[ABC]=MN2

b2

En los triangulos MOB y MNB se tiene

MB

sinα=

OM

sin(90 − C)=MN

sinB⇒MN =

OM · sinBcosC

Ademas, aplicando el Teorema del Coseno al triangulo MOB, resulta

OM2 = MB2 +R2 − 2MB ·R · cos(90 −C) = MB2 +R2 − 2MB ·R · sinC

Teniendo en cuenta que c = 2R sinC, entonces

MN2 =sin2B

cos2C

(c2

(k + 1)2+R2 − c2

k + 1

)=

sin2B

cos2C

(R2 − c2k

(k + 1)2

)=

sin2B

cos2C

(R2(k + 1)2 − 4R2 sin2C

(k + 1)2

)Llevando esto al cociente entre las areas de los triangulos MNK y ABC yteniendo en cuenta que b = 2R sinB, se obtiene

[MNK]

[ABC]=MN2

b2=

1

b2· sin2B

cos2C·R2 · (k + 1)2 − 4k sin2C

(k + 1)2

=(k − 1)2 + 4k cos2C

4(k + 1)2 cos2C

4

Page 803: COMPENDIUM OME - ToomatesEotvos de Hungría, que se iniciaron en 1894, precisamente durante la efervescencia de fin de siglo, consecuencia de la cual fue también el proceso iniciado

Dado que cos2C < 1, entonces se verifica que

[MNK]

[ABC]>

cos2C [(k − 1)2 + 4k]

4(k + 1)2 cos2C=

1

4

El cociente entre las areas es igual a 1/4 para k = 1, es decir, cuando M es elpunto medio del segmento AB.

4. Los puntos de una superficie esferica de radio 4, se pintan con cuatrocolores distintos. Prueba que existen dos puntos sobre la superficie que tienenel mismo color y que estan a distancia 4

√3 o bien a distancia 2

√6.

NOTA: La distancia entre dos puntos es la distancia euclıdea; es decir la lon-gitud del segmento rectilıneo que los une.

Solucion. Dado que se pintan los puntos de la esfera con cuatro coloresdistintos, para resolver el problema sera suficiente con encontrar cinco puntosdistintos que cumplan las condiciones del enuciado y aplicar el Principio delPalomar.

Esto puede hacerse, por ejemplo, observando que si inscribimos un trianguloequilatero en una circunferencia de diametro maximo de la esfera, entonces ellado de dicho triangulo mide 4

√3, y que tomando dos de tales triangulos,

contenidos en planos perpendiculares y que tengan un vertice comun, losvertices no comunes distan entre sı 2

√6. En efecto, en un sistema de co-

ordenadas cartesianas con origen en el centro de la esfera, la superficie de estaviene dada por los puntos cuyas coordenadas (x, y, z) satisfacen la ecuacionx2 + y2 + z2 = 16. Sobre esta superficie, consideremos los siguientes puntosA(4, 0, 0), B(−2, 2

√3, 0), C(B(−2,−2

√3, 0) que estan sobre la circunferencia

Γ1 = (x, y, z) |x2 + y2 = 16, z = 0

y son los vertices de un triangulo equilatero de lado 4√

3. Igualmente, los pun-tosA(4, 0, 0), D(−2, 0, 2

√3), E(B(−2, 0,−2

√3) son los vertices de un triangulo

equilatero inscrito en la circunferencia

Γ2 = (x, y, z) |x2 + z2 = 16, y = 0

Notese que los planos de las circuneferencias Γ1 y Γ2 son perpendiculares, quelos triangulos equilateros tienen el vertice comun A, y que

AB = AC = AD = AE = BC = DE = 4√

3 y BD = BE = CD = CE = 2√

6

Luego cada uno de estos cinco puntos A,B,C,D,E esta a distancia 4√

3 oa distancia 2

√6 de cada uno de los cuatro restantes, y por el principio del

palomar, hay dos del mismo color, como querıamos demostrar.

5

Page 804: COMPENDIUM OME - ToomatesEotvos de Hungría, que se iniciaron en 1894, precisamente durante la efervescencia de fin de siglo, consecuencia de la cual fue también el proceso iniciado

5. Sean a y b dos numeros positivos primos entre sı. Se dice que un enteropositivo n es debil si no puede ser escrito en la forma n = ax+by, para algunosenteros x e y no negativos. Prueba que si n es debil y n < ab

6, entonces existe

un entero k ≥ 2, tal que kn es debil.

Solucion. Trivialmente se observa que la suma de enteros positivos no debileses no debil. Esto motiva considerar para cada entero positivo n los enteros 2ny 3n. Si ambos no son debiles, entonces kn no es debil para cada k ≥ 2, ya quekn puede ser escrito en la forma 2nr + 3ns para algunos enteros no negativosr y s.

Sea n un entero positivo debil y supongamos que 2n y 3n son no debiles, con2n = ax1 + by1 y 3n = ax2 + by2. Entonces

0 = 2(3n)− 3(2n) = a(2x2 − 3x1) + b(2y2 − 3y1) (3)

yn = 3n− 2n = a(x2 − x1) + b(y2 − y1) (4)

De (4) por ser n debil, resulta que x2 < x1 o y2 < y1. Sin perdida de gen-eralidad, podemos suponer que x2 < x1. Entonces 2x2 − 3x1 < 0, y por (3),2y2 − 3y1 > 0. Como a y b son primos entre sı, (3) tambien implica quea | (2y2 − 3y1), y ası a ≤ 2y2 − 3y1 ≤ 2y2. Concluimos que

3n = ax2 + by2 ≥ by2 ≥ab

2⇒ n ≥ ab

6

Esto prueba que si n es debil y n < ab6

, entonces existe un entero k ≥ 2, talque kn es debil. (La desigualdad en el enunciado debe de ser estricta, comomuestra el ejemplo n = 1, a = 2, b = 3).

6. Sea R+ el conjunto de los numeros reales positivos. Halla todas las funcionesf : R+ → R+, tales que f(x+ f(y)) = yf(xy + 1), para todo x, y > 0.

Solucion. Supongamos que exista un numero real y tal que y − 1 y f(y)− 1son ambos no nulos y del mismo signo. Entonces podemos tomar el numeroreal positivo

x =f(y)− 1

y − 1

de donde se obtienef(y) = x(y − 1) + 1 y x + f(y) = xy + 1. Por tanto,f(x+ f(y)) = f(xy + 1) y utilizando el enunciado, resulta

y =f(x+ f(y))

f(xy + 1)= 1

6

Page 805: COMPENDIUM OME - ToomatesEotvos de Hungría, que se iniciaron en 1894, precisamente durante la efervescencia de fin de siglo, consecuencia de la cual fue también el proceso iniciado

Esto es una contradiccion, y por tanto, y−1 y f(y)−1 no puden ser ambos nonulos ni tener el mismo signo. Es decir, f(y) ≥ 1 para todo y < 1, y f(y) ≤ 1para todo y > 1.

Para todo y > 1, tomemos el numero real x = (y − 1)/y > 0. Entonces, setiene que

f

(y − 1

y+ f(y)

)= y f(y)

Ahora distinguimos dos casos:

• Si yf(y) > 1, entoncesy − 1

y+ f(y) ≤ 1, luego yf(y) ≤ 1, contradiccion.

• Si yf(y) < 1, entoncesy − 1

y+ f(y) ≥ 1, luego yf(y) ≥ 1, contradiccion.

Por tanto, yf(y) = 1 para todo y > 1 y f(y) =1

y.

Sea ahora y ≤ 1, y tomemos x = 1. Se tiene entonces que tanto x + f(y) =1 + f(y) como xy + 1 = y + 1 son numeros reales mayores que 1, con lo que

1

x+ f(y)=

y

xy + 1⇔ xy + 1 = xy + yf(y)

y concluımos que f(y) =1

ytambien para todo y ≤ 1. Luego la unica funcion

que puede satisfacer las condiciones del enunciado es f(y) =1

y. Vemos ademas

que

f(x+ f(y)) = f

(xy + 1

y

)=

y

xy + 1= yf(xy + 1),

luego en efecto f(y) =1

yes solucion, y no puede haber otras.

7

Page 806: COMPENDIUM OME - ToomatesEotvos de Hungría, que se iniciaron en 1894, precisamente durante la efervescencia de fin de siglo, consecuencia de la cual fue también el proceso iniciado

LV Olimpiada Matemática Española

Concurso Final Nacional

PRIMERA SESIÓN

Ourense, viernes 22 de marzo de 2019

Problema 1

Un conjunto de números enteros 𝑇 es orensano si existen enteros a < b < c tales que 𝑎

y 𝑐 pertenecen a 𝑇 y 𝑏 no pertenece a 𝑇. Hallar el número de subconjuntos 𝑇 de

1,2,...,2019 que son orensanos.

Problema 2

Determinar si existe un conjunto finito 𝑆 formado por números primos positivos de

manera que para cada entero 𝑛 ≥2, el número 22 + 32 + ⋯ + 𝑛2 sea múltiplo de algún

elemento de 𝑆.

Problema 3

Los números reales 𝑎, 𝑏 y 𝑐, verifican que el polinomio 𝑝(𝑥) = 𝑥4 + 𝑎𝑥3 + 𝑏𝑥2 +𝑎𝑥 + 𝑐 tiene exactamente tres raíces reales distintas; estas raíces son iguales a 𝑡𝑎𝑛𝑦,tan (2𝑦) y tan (3𝑦), para algún número real 𝑦. Hallar todos los posibles valores de 𝑦, 0 ≤ 𝑦 < 𝜋.

No está permitido el uso de calculadoras,

ni dispositivos electrónicos o digitales de ningún tipo.

Cada problema se puntúa de cero a siete puntos.

El tiempo de cada sesión es de TRES HORAS Y MEDIA.

Page 807: COMPENDIUM OME - ToomatesEotvos de Hungría, que se iniciaron en 1894, precisamente durante la efervescencia de fin de siglo, consecuencia de la cual fue también el proceso iniciado

LV Olimpiada Matemática Española

Concurso Final Nacional

SEGUNDA SESIÓN

Ourense, sábado 23 de marzo de 2019

Problema 4

Calcular todos los pares de enteros (𝑥, 𝑦) tales que 3423(𝑥2 + 𝑦2) = 𝑥3𝑦3.

Problema 5

Se consideran todos los pares (𝑥, 𝑦) de números reales tales que 0 ≤ 𝑥 ≤ 𝑦 ≤ 1. Sea

𝑀(𝑥, 𝑦) el máximo valor del conjunto

A= 𝑥𝑦, 1 − 𝑥 − 𝑦 + 𝑥𝑦, 𝑥 + 𝑦 − 2𝑥𝑦 .

Hallar el mínimo valor que puede tomar 𝑀(𝑥, 𝑦) para todos estos pares (𝑥, 𝑦).

Problema 6

En el triángulo escaleno 𝐴𝐵𝐶, la bisectriz del ángulo 𝐴 corta al lado 𝐵𝐶 en el punto 𝐷. Las rectas que pasan por 𝐷 y son tangentes a las circunferencias circunscritas de los

triángulos 𝐴𝐵𝐷 y 𝐴𝐶𝐷 𝑐ortan a las rectas 𝐴𝐶 y 𝐴𝐵 en los puntos 𝐸 y

𝐹 respectivamente. Si 𝐵𝐸 y 𝐶𝐹 se cortan en 𝐺, demostrar que los ángulos < 𝐸𝐷𝐺 y <𝐴𝐷𝐹 son iguales.

No está permitido el uso de calculadoras,

ni dispositivos electrónicos o digitales de ningún tipo.

Cada problema se puntúa de cero a siete puntos.

El tiempo de cada sesión es de TRES HORAS Y MEDIA

Page 808: COMPENDIUM OME - ToomatesEotvos de Hungría, que se iniciaron en 1894, precisamente durante la efervescencia de fin de siglo, consecuencia de la cual fue también el proceso iniciado

LV Olimpiada matematica Espanola (Concurso Final)

Enunciados y Soluciones

1. Un conjunto de numeros enteros T es orensano si existen enteros a < b < ctales que a y c pertenecen a T y b no pertenece a T . Hallar el numero desubconjuntos T de 1, 2, . . . , 2019 que son orensanos.

Solucion. El numero de subconjuntos de 1, 2, · · · , 2019 es 22019 como esbien conocido. Contemos ahora el numero de estos conjuntos que NO tienenla propiedad pedida. Claramente, el conjunto vacıo, y los subconjuntos de1, 2, . . . , 2019 con un unico elemento no tienen la propiedad pedida, ya quees imposible elegir a, c distintos en ellos. Hay en total(

2019

0

)+

(2019

1

)= 1 + 2019 = 2020

de estos conjuntos. Cualquier otro subconjunto que no tenga la propiedadpedida ha de estar formado por elementos consecutivos de 1, 2, . . . , 2019.En efecto, si m,M son respectivamente el mınimo y el maximo de T , para queT NO tenga la propiedad enunciada, todos los elementos m,m+1,m+2, . . . ,Mhan de estar en T , pues si alguno k no lo estuviera, podemos tomar a = m,b = k y c = M , cumpliendose la propiedad. El numero de tales subconjuntoses(20192

). En efecto, dado un tal subconjunto, su mınimo m y su maximo

M lo determinan biunıvocamente, y hay(20192

)formas distintas de elegir dos

numeros del conjunto 1, 2, . . . , 2019, de forma que el menor sera m, y elmayor M .

Finalmente, se tiene que el numero total de subconjuntos T de 1, 2, · · · , 2019con la propiedad pedida es

22019 − 2020−(

2019

2

)= 22019 − 2039191.

2. Determinar si existe un conjunto finito S formado por numeros primospositivos de manera que para cada entero n ≥ 2, el numero 22 + 32 + · · ·+ n2

sea multiplo de algun elemento de S.

1

Page 809: COMPENDIUM OME - ToomatesEotvos de Hungría, que se iniciaron en 1894, precisamente durante la efervescencia de fin de siglo, consecuencia de la cual fue también el proceso iniciado

Solucion. Es bien conocido que

12 + 22 + · · ·+ n2 =n(n+ 1)(2n+ 1)

6=

2n3 + 3n2 + n

6,

con lo que restando 1 a ambos lados, resulta

22 + 32 + · · ·+ n2 =2n3 + 3n2 + n− 6

6=

(n− 1) (2n2 + 5n+ 6)

6.

Esta expresion sugiere tomar p = (n− 1)/6 o n = 6p+ 1 con lo que se obtiene

22 + 32 + · · ·+ n2 = p(72p2 + 54p+ 13

).

Ahora elegimos n = 6p+ 1, donde p da resto 1 al dividir entre cualquier primode S distinto de 139, y da resto −1 al dividir entre 139. Notese que el resto aldividir entre cualquier primo de S distinto de 139 es 72 + 54 + 13 = 139, quees primo y por lo tanto no divisible por ningun primo de S distinto de 139,y el resto al dividir entre 139 es −72 + 54 − 13 = −31, claramente tampocodivisible por 139. Luego esta expresion es coprima con todos los primos de S,y la respuesta es que no existe tal conjunto S.

3. Los numeros reales a, b y c verifican que el polinomio p(x) = x4+ax3+bx2+ax + c tiene exactamente tres raıces reales distintas; estas rıces son iguales atan (y), tan (2y) y tan (3y), para algun numero real y. Hallar todos los posiblesvalores de y, 0 ≤ y < π.

Solucion. Sean r, r, s, t los tres ceros reales y distintos del polynomio p(x).Mediante las formula de Cardano-Viete, identificando los coeficientes cubico ylineal, se obtiene

2r + s+ t = r2s+ r2t+ 2rst⇔ 2r(1− st) = (r2 − 1)(s+ t)

De la ultima igualdad se deduce que si r2−1 = 0 entonces 1−st = 0 y viceversa.A continuacion, bajo la hipoteis r2 = st = 1 y considerando 0 ≤ y < π,analizaremos los siguientes casos:

• Si r = tan y entonces y ∈π

4,3π

4

pero para estos valores s = tan 2y

no esta definido.

• Si r = tan 2y entonces y ∈π

8,3π

8,5π

8,7π

8

. Poniendo s = tan y y

s = tan 3y se tiene que para todos los valores anteriores es rs = tan y ·tan 3y = 1, lo cual es cierto si y + 3y = 4y es un multiplo impar de π/2,y esto ocurre para todos los valores anteriores.

2

Page 810: COMPENDIUM OME - ToomatesEotvos de Hungría, que se iniciaron en 1894, precisamente durante la efervescencia de fin de siglo, consecuencia de la cual fue también el proceso iniciado

• Si r = tan 3y entonces se debe cumplir rs = tan y · tan 2y = 1 y la tan 3yno esta definida.

Por tanto solo queda por analizar los casos en que r2 − 1 6= 0 y 1 − st 6=0. Dividiendo los dos lados de la igualdad 2r(1 − st) = (r2 − 1)(s + t) por(r2 − 1)(1− st) resulta

2r

1− r2+

s+ t

1− st= 0.

Estas expresiones por separado serıan las correspondients a las formulas de latangente del angulo doble y de la tangente de la suma. Ahora, utilizando laultima expresion, distinguiremos los siguientes casos:

• Si r = tan y, s = tan 2y, t = tan 3y entonces tan 2y + tan 5y = 0 queadmite como soluciones los multiplos de π

2inferiores a 7π

2con lo que

y ∈π

7,2π

7,3π

7,4π

7,5π

7,6π

7

.

• Si r = tan 2y, s = tan y, t = tan 3y entonces tan 4y + tan 4y = 0 queadmite como soluciones los multiplos de π

2inferiores a 4π, con lo que

y ∈π

8,3π

8,5π

8,7π

8

,

y hemos rechazado y = π2

ya que entonces tan y no serıa un numero real,y tambien π

4y 3π

4ya que entonces tan(2y) tampoco serıa un numero real.

• Si r = tan 3y, s = tan y, t = tan 2y entonces tan 6y + tan 3y = 0 queadmite como soluciones los multiplos de π

2inferiores a 9π

2, con lo que

y ∈π

9,2π

9,π

3,4π

9,5π

9,2π

3,7π

9,8π

9

.

Estos son todos los valores, y hemos terminado.

4. Calcular todos los pares de enteros (x, y) tales que

3423(x2 + y2

)= x3y3.

Solucion. Notese en primer lugar que si xy = 0, entonces x2 + y2 = 0, dedonde resulta la solucion x = y = 0. Notese tambien que si xy < 0, entonces

3

Page 811: COMPENDIUM OME - ToomatesEotvos de Hungría, que se iniciaron en 1894, precisamente durante la efervescencia de fin de siglo, consecuencia de la cual fue también el proceso iniciado

x2 + y2 < 0, absurdo, luego x, y tienen ambos el mismo signo. Como cambiarsimultaneamente de signo a x y a y no altera la ecuacion, podemos asumir apartir de este momento y sin perdida de generalidad que x, y son ambos enterospositivos. Sean x = 3ma e y = 3nb con m,n enteros no negativos y a, b enteroscoprimos y no divisibles por 3. Supongamos que m ≥ n. Entonces la ecuaciondad se transforma en

8((3m−na)2 + b2) = 33m+n−4a3b3.

Como todo cuadrado perfecto da resto 0 o 1 al dividir entre 3, como es bienconocido, el miembro de la izquierda no es divisible por 3 y para que se cumplala ecuacion debe ser el exponente de 3 en el termino de la derecha igual a cero.Es decir, 3m+ n− 4 = 0 y como m ≥ n ≥ 0 esto implica que m = n = 1. Laecuacion ahora se simplifica y queda

8(a2 + b2) = a3b3.

Por la simetrıa de la expresion podemos suponer que a ≥ b. Entonces

16a2 ≥ a3b3 ⇔ 16 ≥ ab3

y tenemos dos posibilidades (1) b = 2 de donde resulta a = 2, (2) b = 1 peroen este caso los unicos valores posibles de a son 1, 2, 4, 8 y no satisfacen laecuacion a3 − 8a2 − 8 = 0. Se deduce que (a, b) = (2, 2) y (x, y) = (6, 6).Finalmente, se tiene que las unicas soluciones posibles son:

(x, y) = (−6,−6), (x, y) = (0, 0), (x, y) = (6, 6).

5. Se consideran todos los pares (x, y) de numeros reales tales que 0 ≤ x ≤y ≤ 1. Sea M(x, y) el maximo valor del conjunto

A = xy, xy − x− y + 1, x+ y − 2xy .

Hallar el mınimo valor que puede tomar M(x, y) para todos estos pares (x, y).

Solucion 1. Haciendo el cambio de variable xy = p, y x+ y = s y escribiendolos tres elementos del conjunto A en terminos de s y p, tenemos

a = xy = p, b = xy − x− y + 1 = (1− x)(1− y) = s− 1 + p, c = s− 2p

verificandose que a+ b+ c = 1. Observemos que s2 − 4p = (x− y)2 ≥ 0.

Ahora consideremos los siguientes casos:

4

Page 812: COMPENDIUM OME - ToomatesEotvos de Hungría, que se iniciaron en 1894, precisamente durante la efervescencia de fin de siglo, consecuencia de la cual fue también el proceso iniciado

• Si 0 ≤ x ≤ y ≤ 1

3⇒ p = xy ≤ 1

9⇔ b = (1− x)(1− y) ≥ 4

9.

• Si c = s−2p ≤ 4

9⇒ 0 ≤ s2−4p =

(2p+

4

9

)2

−4p = 4

(p− 1

9

) (p− 4

9

),

de donde se deduce que p ≤ 1

9o p ≥ 4

9. Si a = p ≤ 1

9⇒ b ≥ 4

9, c =

s − 2p ≤ 4

9y si a = p ≥ 4

9⇒ b ≤ 1

9, c ≤ 4

9. Por lo tanto, en cualquier

caso

maxA ≥ 4

9

Para hallar el mınimo4

9, las desigualdades deberan ser igualdades y se tiene:

• p =1

9, s− 2p =

4

9⇒ s =

2

3⇒ x = y =

1

3,

• p =4

9, s− 2p =

4

9⇒ s =

4

3⇒ x = y =

2

3.

Solucion 2. En primer lugar, se observa que xy + (xy − x − y + 1) + (x +y − 2xy) = 1. Ası, si uno de los tres elementos de A vale 1

9o menos, entonces

los otros dos suman 89

o mas, luego si el menor valor de A vale 19

o menos, elmayor valor de A vale 4

9o mas.

Supongamos que xy ≥ x+ y− 2xy. Entonces, por la desigualdad entre mediasaritmetica y geometrica se tiene que

3xy ≥ x+ y ≥ 2√xy,

√xy ≥ 2

3,

y el mayor valor de A es al menos xy ≥ 49

en este caso.

Supongamos que xy−x−y+1 ≥ x+y−2xy. Nuevamente por la desigualdadentre medias aritmetica y geometrica, se tiene que

3xy + 1 ≥ 2(x+ y) ≥ 4√xy ⇔ (3

√xy − 1) (

√xy − 1) ≥ 0.

La ultima desigualdad se verifica cuando√xy ≥ 1, con lo que el mayor valor

de A serıa al menos xy ≥ 1, o bien√xy ≤ 1

3, para xy ≤ 1

9, y por la observacion

inicial el mayor valor de A serıa 49

o mas.

En cualquier otro caso, x+ y− 2xy es el mayor valor de A, y supongamos quees inferior a 4

9. Es decir, x + y − 2xy < 4

9. Aplicando la desigualdad entre las

medias aritmetica y geometrica, se obtiene

2xy +4

9> x+ y ≥ 2

√xy ⇔

(√xy − 1

2

)2

>1

36=

(1

6

)2

5

Page 813: COMPENDIUM OME - ToomatesEotvos de Hungría, que se iniciaron en 1894, precisamente durante la efervescencia de fin de siglo, consecuencia de la cual fue también el proceso iniciado

Si√xy− 1

2> 1

6, entonces

√xy > 2

3y el mayor valor de A es al menos xy > 4

9,

contradiccion. Si 12−√xy > 1

6, entonces

√xy < 1

3, con lo que xy < 1

9, y por

la observacion inicial el mayor valor de A es mayor que 49, contradiccion.

Luego el mayor valor de A nunca puede ser menor que 49, y ese valor es en efecto

el mınimo que toma el maximo de A, pues se puede obtener A =

49, 19, 49

tomando x = y = 2

3, o A =

19, 49, 49

tomando x = y = 1

3. Viendo las

condiciones de igualdad en los dos primeros casos analizados, se tiene ademasque estos son todos los posibles pares de valores (x, y) para los que el mayorvalor de A toma dicho valor mınimo.

6. En el triangulo escaleno ABC, la bisectriz del angulo A corta al lado BC enel punto D. Las rectas que pasan por D y son tangentes a las circunferenciascircunscritas de los triangulos ABD y ACD cortan a las rectas AC y AB enlos puntos E y F , respectivamente. Si BE y CF se cortan en G, demostrarque los angulos ∠EDG y ∠ADF son iguales.

Solucion. Se verifican las igualdades de angulos ∠ADE = ∠B y ∠ADF =∠C, por ser angulos semiinscritos en las circunferencias (ABD) y (ADC),respectivamente. Por tanto el cuadrilatero AFDE es inscriptible y de eso se

GF E

D CB

A

deduce que

∠AFE = ∠ADE = ∠B y ∠AEF = ∠ADF = ∠C,

y por tanto, EF es paralela a BC. Ademas DE = DF por ser cuerdascorrespondientes al angulo ∠A/2, ası que el triangulo FDE es isosceles.

6

Page 814: COMPENDIUM OME - ToomatesEotvos de Hungría, que se iniciaron en 1894, precisamente durante la efervescencia de fin de siglo, consecuencia de la cual fue también el proceso iniciado

Sea M el punto medio de EF , L = AD ∩EF , y H = DG∩EF . Aplicando elteorema de Thales, se obtiene que

LE

LF=DC

BD

Por otro lado, de 4FGH ∼ 4CGD resulta

HF

DC=GH

GD,

de 4EGH ∼ 4BGD, se obtiene

HE

BD=GH

GD

y de aquı queHF

HE=DC

DB,

luegoH y L son simetricos respecto deM , como vermos mas tarde, y ∠LDM =∠HDM = α siendo

α = 90 − ∠ADC =∠C − ∠B

2si C > B,

y

α = 90 − ∠ADB =∠B − ∠C

2si B > C.

Finalmente, se tiene que

∠GDE = ∠ADE − α = ∠ADF si B > C,

y∠GDE = ∠ADE + α = ∠ADF si B < C.

Ahora falta probar que L y H son simetricos respecto a M . En efecto, sidenotamos LF + LE = HF +HE = s, entonces

LE

LF=HF

HE,

de dondes

LF=

s

HE⇒ LF = HE y LE = HF

con lo que HM = ML como se querıa demostrar.

7

Page 815: COMPENDIUM OME - ToomatesEotvos de Hungría, que se iniciaron en 1894, precisamente durante la efervescencia de fin de siglo, consecuencia de la cual fue también el proceso iniciado

Agradecimientos

Agradecemos a los proponentes de los problemas sus contribuciones para for-mar la Lista Corta de la que se han seleccionado los seis Problemas de lacompeticion y al profesor Juan Manuel Conde y sus colaboradores por la se-leccion de los mismos.

8

Page 816: COMPENDIUM OME - ToomatesEotvos de Hungría, que se iniciaron en 1894, precisamente durante la efervescencia de fin de siglo, consecuencia de la cual fue también el proceso iniciado

LVI Olimpiada Matematica Espanola

Concurso Final Nacional

Primera sesion

14 de julio de 2020

Olimpiada

Matemática

Española RSME

1. Decimos que un polinomio p(x), con coeficientes reales, es almeriense si tienela forma

p(x) = x3 + ax2 + bx+ a

y sus tres raıces son numeros reales positivos en progresion aritmetica. Hallatodos los polinomios almerienses tales que p(7/4) = 0.

2. Consideramos la sucesion de numeros enteros f(n)∞n=1

definida por:

• f(1) = 1.

• Si n es par, f(n) = f(n/2).

• Si n > 1 es impar y f(n− 1) es impar, entonces f(n) = f(n− 1)− 1.

• Si n > 1 es impar y f(n− 1) es par, entonces f(n) = f(n− 1) + 1

a) Calcula f(22020 − 1).

b) Demuestra que f(n)∞n=1

no es periodica, es decir, no existen enterospositivos t y n0 tales que f(n+ t) = f(n) para cualquier n ≥ n0.

3. A cada punto del conjunto A = (x, y, z) ∈ Z3, formado por los puntos del

espacio tridimensional cuyas coordenadas son enteras, le asignamos un colorde entre p colores posibles.

Demuestra que forzosamente existe algun paralelepıpedo recto (poliedro deseis caras en el que cada cara es un rectangulo) cuyos vertices pertenecen aA y son todos del mismo color.

No esta permitido el uso de calculadoras.

Cada problema se puntua sobre 7 puntos.

El tiempo de cada sesion es de 3 horas y media.

Page 817: COMPENDIUM OME - ToomatesEotvos de Hungría, que se iniciaron en 1894, precisamente durante la efervescencia de fin de siglo, consecuencia de la cual fue también el proceso iniciado

LVI Olimpiada Matematica Espanola

Concurso Final Nacional

Segunda sesion

15 de julio de 2020

Olimpiada

Matemática

Española RSME

4. Ana y Benito juegan a un juego que consta de 2020 rondas. Inicialmente,en la mesa hay 2020 cartas, numeradas de 1 a 2020, y Ana tiene una cartaadicional con el numero 0. En la ronda k-esima, el jugador que no tiene lacarta k − 1 decide si toma la carta k o si se la entrega al otro jugador. Elnumero de cada carta indica su valor en puntos. Al terminar el juego, ganaquien tiene mas puntos. Determina que jugador tiene estrategia ganadora, osi ambos jugadores pueden forzar el empate, y describe la estrategia a seguir.

5. En un triangulo acutangulo ABC, sea M el punto medio del lado AB y P elpie de la altura sobre el lado BC. Prueba que si AC+BC =

√2AB, entonces

la circunferencia circunscrita del triangulo BMP es tangente al lado AC.

6. Sea S un subconjunto finito de los numeros enteros. Definimos d2(S) y d3(S)de la siguiente manera:

• d2(S) es el numero de elementos a ∈ S para los que existen x, y ∈ Z

tales que x2 − y2 = a.

• d3(S) es el numero de elementos a ∈ S para los que existen x, y ∈ Z

tales que x3 − y3 = a.

a) Sea m un numero entero y sea S = m,m+1, . . . , m+2019. Prueba que

d2(S) >13

7· d3(S)

b) Sea n un entero positivo y sea Sn = 1, 2, . . . , n. Prueba que existe unnumero N de manera que si n > N ,

d2(Sn) > 4 · d3(Sn)

No esta permitido el uso de calculadoras.

Cada problema se puntua sobre 7 puntos.

El tiempo de cada sesion es de 3 horas y media.

Page 818: COMPENDIUM OME - ToomatesEotvos de Hungría, que se iniciaron en 1894, precisamente durante la efervescencia de fin de siglo, consecuencia de la cual fue también el proceso iniciado

OLIMPIADA MATEMÁTICA

PROBLEMAS Y SOLUCIONES 2020

REAL SOCIEDAD MATEMÁTICA ESPAÑOLA

Page 819: COMPENDIUM OME - ToomatesEotvos de Hungría, que se iniciaron en 1894, precisamente durante la efervescencia de fin de siglo, consecuencia de la cual fue también el proceso iniciado
Page 820: COMPENDIUM OME - ToomatesEotvos de Hungría, que se iniciaron en 1894, precisamente durante la efervescencia de fin de siglo, consecuencia de la cual fue también el proceso iniciado

LVI Olimpiada matematica Espanola (Concurso Final)

Enunciados y Soluciones

1. Decimos que un polinomio p(x), con coeficientes reales, es almeriense sitiene la forma

p(x) = x3 + ax2 + bx+ a

y sus tres raıces son numeros reales positivos en progresion aritmetica. Hallatodos los polinomios almerienses tales que p(7/4) = 0.

Solucion. Llamemos α ≤ β ≤ γ a las raıces del polinomio. De la condicion deestar en progresion aritmetica tenemos que existe un numero real no negativoδ de manera que α = β − δ y γ = β + δ. Por su parte, utilizando las formulasde Cardano–Viete, resulta que

αβγ = −a = α + β + γ,

o lo que es lo mismoβ(β2 − δ2) = 3β.

Como las raıces son distintas de cero, se llega finalmente a que

(β − δ)(β + δ) = β2 − δ2 = 3.

Tenemos que analizar los tres casos posibles:

(a) Si β− δ = 7/4, entonces β+ δ = 12/7. De aquı se tiene que β = 97/56 yδ = −1/56, lo cual contradice la hipotesis de que δ ≥ 0 (alternativamente,no es posible que β + δ < β − δ).

(b) Si β = 7/4, entonces δ2 = 1/4. Por lo tanto, las raıces del polinomio son(3/2, 7/4, 2) y se obtiene que

p(x) = (x− 3/2)(x− 7/4)(x− 2) = x3 − 21

4x2 +

73

8x− 21

4.

1

Page 821: COMPENDIUM OME - ToomatesEotvos de Hungría, que se iniciaron en 1894, precisamente durante la efervescencia de fin de siglo, consecuencia de la cual fue también el proceso iniciado

(c) Si β + δ = 7/4, entonces β − δ = 12/7. De aquı se tiene que β = 97/56y δ = 1/56. Por lo tanto, las raıces del polinomio son (12/7, 97/56, 7/4)y se obtiene que

p(x) = (x− 12/7)(x− 97/56)(x− 7/4) = x3 − 291

56x2 +

14113

1568x− 291

56.

2. Consideramos la sucesion de numeros enteros f(n)∞n=1 definida por:

f(1) = 1.

Si n es par, f(n) = f(n/2).

Si n > 1 es impar y f(n− 1) es impar, entonces f(n) = f(n− 1)− 1.

Si n > 1 es impar y f(n− 1) es par, entonces f(n) = f(n− 1) + 1

a) Calcula f(22020 − 1).

b) Demuestra que f(n)∞n=1 no es periodica, es decir, no existen enteros pos-itivos t y n0 tales que f(n+ t) = f(n) para cualquier n ≥ n0 .

Solucion. En primer lugar, hacemos notar que la sucesion esta bien definida:para cada n ∈ N, n > 1, el valor de f(n) esta perfectamente determinado apartir de los valores de f(r) con r < n.

Consideramos la sucesion g(n) dada por g(n) = 0 si la expresion binaria den tiene un numero par de unos; y g(n) = 1 si la expresion binaria de n tieneun numero impar de unos. Es obvio que g cumple todas las condiciones quedefinen a f , luego f(n) = g(n) para todo n ∈ N.De esta forma, puesto que 22020 − 1 se escribe en binario con 2020 unos,

f(22020 − 1) = 0

Finalmente, veamos que la sucesion no es periodica. Supongamos que lo fueraa partir de un valor n0 y con periodo t. Tomamos el entero, r, tal que 2r ≤ t <2r+1. Observamos que la expresion binaria de 2r + t se obtiene sustituyendo elprimer 1 (por la izquierda) de la expresion de t por dos dıgitos 10, con lo quef(2r + t) = f(t). Tomamos un entero k > r + 1 tal que 2k > n0. Ahora, setiene:Si f(t) = 1, entonces f(2k + t) = 0 y por tanto, f(2k + t) 6= f(2k) = 1.

2

Page 822: COMPENDIUM OME - ToomatesEotvos de Hungría, que se iniciaron en 1894, precisamente durante la efervescencia de fin de siglo, consecuencia de la cual fue también el proceso iniciado

Si f(t) = 0, entonces f(2k + 2r + t) = 1 + f(2r + t) = 1 + f(t) = 1 y por tanto,f(2k + 2r + t) 6= f(2k + 2r) = 0.

3. A cada punto del conjunto A = (x, y, z) ∈ Z3, formado por los puntosdel espacio tridimensional cuyas coordenadas son enteras, le asignamos uncolor de entre p colores posibles. Demuestra que forzosamente existe algun pa-ralelepıpedo recto (poliedro de seis caras en el que cada cara es un rectangulo)cuyos vertices pertenecen a A y son todos del mismo color.

Solucion. Ponemos n = p

(p+ 1

2

). Para cada j ∈ 0, 1, . . . , n, pensamos

en el conjunto A0j = (i, j, 0) ∈ A; 0 ≤ i ≤ p. Por el Principio del Palomar,

podemos asegurar que, para cada j hay dos puntos de A0j del mismo color.

Puede que haya mas de dos o que ocurra para mas de un color; no importa,de entre los p+ 1 marcamos dos que comparten color. Teniendo en cuenta que

hay p

(p+ 1

2

)+ 1 posibles valores de j, de nuevo por el Palomar, existiran al

menos r =

(p+ 1

2

)+ 1 de esos A0

j (digamos A0j1, . . . , A0

jr) en los que el color

repetido en todos ellos es el mismo (color c). Como el numero de maneras deelegir los dos lugares (valores de i) de cada uno de esos A0

jt , con t = 1, . . . , r,

es

(p+ 1

2

), usando de nuevo el Principio del Palomar, podemos asegurar que

existen Ajα y Ajβ , con α, β ∈ 1, . . . , r, en los que el color c aparece dos vecesen cada uno y en los mismos lugares. O sea, queda probado que en el planoz = 0, y con valores de la coordenada x ∈ 0, 1, . . . , p y de la coordenaday ∈ 0, 1, . . . , n, existen cuatros puntos

(i1, jα, 0), (i2, jα, 0) ∈ Ajα (i1, jβ, 0), (i2, jβ, 0) ∈ Ajβque son vertices de un rectangulo (siempre de Lados Paralelos a los Bordes dela Cuadrıcula: LPBC) y los cuatro de un mismo color.

El numero, m, de formas de elegir, en una cuadrıcula de (p + 1) × (n + 1)puntos, cuatro puntos que sean vertıces de un rectangulo LPBC es lo mismoque el numero de maneras de elegir dos valores de la abscisa y dos valores de

la ordenada, o sea m =

(p+ 1

2

)(n+ 1

2

).

Para cada k ∈ 0, 1, . . . , p ×m consideramos la cuadrıcula Ak = (i, j, k) ∈A; 0 ≤ i ≤ p, 0 ≤ j ≤ n. Tenemos pm + 1 cuadrıculas y en cada una deellas hay cuatros puntos que comparten color y son vertices de un rectangulo

3

Page 823: COMPENDIUM OME - ToomatesEotvos de Hungría, que se iniciaron en 1894, precisamente durante la efervescencia de fin de siglo, consecuencia de la cual fue también el proceso iniciado

LBPC. Por el Principio del Palomar, hay al menos p + 1 de esas cuadrıculasen las que los cuatro puntos de cada una de ellas ocupan los mismos lugaresen lo referido a sus dos primeras coordenadas. Finalmente, de nuevo por elPalomar, hay (al menos) dos de esas p+1 cuadrıculas tales que el color comunde los cuatro vertices de una es el mismo que el color comun de los cuatrovertices de la otra. Ası tenemos ocho puntos de A que comparten color y sonvertices de un paralelepıpedo recto.

4. Ana y Benito juegan a un juego que consta de 2020 rondas. Inicialmente,en la mesa hay 2020 cartas, numeradas de 1 a 2020, y Ana tiene una cartaadicional con el nœmero 0. En la ronda k-esima, el jugador que no tiene lacarta k − 1 decide si toma la carta k o si se la entrega al otro jugador. Elnumero de cada carta indica su valor en puntos. Al terminar el juego, ganaquien tiene mas puntos. Determina que jugador tiene estrategia ganadora, osi ambos jugadores pueden forzar el empate, y describe la estrategia a seguir.

Solucion. Ambos jugadores pueden forzar el empate. Dividimos el juego en505 etapas, cada una con cuatro rondas consecutivas de la forma

k, k + 1, k + 2, k + 3.

Vamos a demostrar que cada jugador puede conseguir al menos la mitad delos puntos de cada etapa, independientemente de que jugador tenga la cartak − 1. No importa que ocurra en la k-esima ronda. A partir de ahı:

El jugador que recibe la carta k (sin perdida de generalidad, podemossuponer que es Ana) puede asegurase al menos el empate en la etapa de4 turnos. En efecto, si Benito le entrega tambien la carta k+1 y la cartak + 2, entonces Ana ya ha recibido 3k + 3 puntos y gana la etapa de 4turnos. Si Benito entrega a Ana la carta k + 1 y se queda la k + 2, Anatoma la carta k + 3 y gana la etapa de 4 turnos. Si Benito se queda lacarta k + 1, Ana entrega la carta k + 2 a Benito y se queda con la cartak+ 3, con lo que empata la etapa de 4 turnos. En resumen, quien recibela carta k siempre puede conseguir al menos el empate.

El jugador que no recibe la carta k (sin perdida de generalidad, Benito)se queda con la carta k + 1. Si Ana le entrega la carta k + 2, Benito yatiene 2k + 3 puntos y se garantiza el empate en la etapa de 4 turnos. SiAna se queda la carta k + 2, Benito se queda con la carta k + 3, con loque acaba con 2k + 4 puntos y gana la etapa de 4 turnos. En resumen,quien no recibe la carta k siempre puede conseguir al menos el empate.

4

Page 824: COMPENDIUM OME - ToomatesEotvos de Hungría, que se iniciaron en 1894, precisamente durante la efervescencia de fin de siglo, consecuencia de la cual fue también el proceso iniciado

5. En un triangulo acutangulo ABC, sea M el punto medio del lado AB yP el pie de la altura sobre el lado BC. Prueba que si AC + BC =

√2AB,

entonces la circunferencia circunscrita del triangulo BMP es tangente al ladoAC.

Solucion. Sea S el punto de AC, al mismo lado de A que C, tal que AS =√2AB/2. Este punto cumple que AS2 = AB2

2= AB · AM , que es la potencia

de A con respecto a la circunferencia circunscrita de BMP ; luego si estacircunferencia circunscrita pasa por S entonces es tangente a AB. Vamos ademostrar que esto es cierto, probando que ∠MSB = ∠MPB. Sea N el

N S

P

M

C

B

A

punto medio de AC. Como AS =√2AB2

= AC+BC2

= AN + MN , tenemosNS = NM . De ahı obtenemos ∠NMS = ∠NSM .Observese que los triangulos ASB y AMS son semejantes, porque AB

AS=√

2 =ASAM

. De aquı se deduce que ∠AMS = ∠ASB. Entonces ∠ABC = ∠AMN =∠AMS − ∠NMS = ∠ASB − ∠NSM = ∠MSB.Por otro lado, por ser M el punto medio de la hipotenusa del triangulorectangulo APB, tenemos MP = MB. Esto quiere decir que ∠MPB =∠MBP = ∠ABC = ∠MSB, como querıamos demostrar.

6. Sea S un subconjunto finito de los numeros enteros. Definimos d2(S) yd3(S) de la siguiente manera:

d2(S) es el numero de elementos a ∈ S para los que existen x, y ∈ Z talesque x2 − y2 = a.

d3(S) es el numero de elementos a ∈ S para los que existen x, y ∈ Z talesque x3 − y3 = a.

5

Page 825: COMPENDIUM OME - ToomatesEotvos de Hungría, que se iniciaron en 1894, precisamente durante la efervescencia de fin de siglo, consecuencia de la cual fue también el proceso iniciado

(a) Sea m un numero entero y sea S = m,m + 1, . . . ,m + 2019. Pruebaque

d2(S) >13

7· d3(S).

(b) Sea n un entero positivo y sea Sn = 1, 2, . . . , n. Prueba que existe unnumero N de manera que si n > N ,

d2(Sn) > 4 · d3(Sn).

Solucion. En primer lugar, observamos que un numero se puede escribir comodiferencia de cuadrados si y solo si no es de la forma 4k + 2. Para ver esto,escribimos x2−y2 = α ·β, donde α y β tienen la misma paridad y simplementeponemos x = α+β

2, y = β−α

2. Por ejemplo, si n = 4k sirve α = 2, β = 2k; si

n = 4k + 1 o n = 4k + 3, sirve α = 1, β = n. Recıprocamente, una diferenciade cuadrados nunca podra dar resto 2 al dividir entre 4 (simplemente notandoque un cuadrado es 0 o 1 modulo 4).Analicemos ahora como pueden ser las diferencias de dos cubos. Observamosque la congruencia modulo 7 del cubo de un entero solo puede ser 0, 1 o 6; enconsecuencia, la diferencia de dos cubos, modulo 7, solo puede ser 0, 1, 2, 5 o6. Haciendo lo mismo modulo 9, vemos que la congruencia de un cubo modulo9 solo puede ser 0, 1 u 8; en consecuencia, la diferencia de dos cubos modulo9, solo puede ser 0, 1, 2, 7 u 8. Por tanto, hay a lo sumo 5 opciones modulo 7y 5 opciones modulo 9, y, por el teorema chino de los restos, tenemos tan solo25 opciones modulo 63.En un intervalo de longitud 2020 (multiplo de 4), exactamente 3/4 partes delos numeros seran no congruentes con 2 modulo 4 (esto es, d2(S) = 1515). Porsu parte, tenemos que 2020 = 32 · 63 + 4, de manera que

d3(S) ≤ 32 · 25 + 4 = 804.

Por tanto, tenemos que d2(S)/d3(S) ≥ 1515/804 > 13/7, como querıamosdemostrar.

Pasamos ahora a resolver la segunda parte. El mismo razonamiento usadoanteriormente muestra que

3n

4− 1

2≤ d2(Sn) ≤ 3n

4+

1

4.

Sera suficiente entonces con ver que asintoticamente d3(n) < 3n16

, esto es, quepara valores de n suficientemente grandes se cumple esa ultima desigualdad.

6

Page 826: COMPENDIUM OME - ToomatesEotvos de Hungría, que se iniciaron en 1894, precisamente durante la efervescencia de fin de siglo, consecuencia de la cual fue también el proceso iniciado

Supongamos que x > y > 0. Una primera observacion es que si x3 − y3 ≤ n,entonces

n ≥ x3 − y3 = (x− y)(x2 + xy + y2) ≥ x2 + xy + y2 > 3y2,

con lo que y <√n/3. Por su parte,

x3 ≤ n+ y3 ≤ n+ (n/3)3/2.

Esto es, x ≤ 3√n+ (n/3)3/2. Como el cociente n/(n/3)3/2 tiende a 0 cuando

n tiende a infinito, tenemos que para cualquier δ > 0 existira un numero Nsuficientemente grande de manera que cuando n > N ,

x ≤ 3

√n+ (n/3)3/2 < (1 + δ)

√n/3.

Como sabemos ademas que y < x, necesariamente el numero de parejas esta

acotado por (1+δ)2n/32

. Por tanto, podemos obtener a lo sumo (1 + δ)2n/6numeros. Observamos que no es necesario considerar el caso en el que x, y < 0,dado que los numeros obtenidos seran los mismos.En el caso en que y < 0 < x, tenemos que x3+(−y)3 ≤ n, y han de ser x, −y <n1/3, de manera que podemos obtener a lo sumo n2/3 numeros. Seleccionandoun δ suficientemente pequeno (por ejemplo δ = 0.01), concluimos que

d3(Sn) ≤ (1 + 0.01)2n/6 + n2/3 <3n

16

si n es suficientemente grande.

7

Page 827: COMPENDIUM OME - ToomatesEotvos de Hungría, que se iniciaron en 1894, precisamente durante la efervescencia de fin de siglo, consecuencia de la cual fue también el proceso iniciado
Page 828: COMPENDIUM OME - ToomatesEotvos de Hungría, que se iniciaron en 1894, precisamente durante la efervescencia de fin de siglo, consecuencia de la cual fue también el proceso iniciado
Page 829: COMPENDIUM OME - ToomatesEotvos de Hungría, que se iniciaron en 1894, precisamente durante la efervescencia de fin de siglo, consecuencia de la cual fue también el proceso iniciado
Page 830: COMPENDIUM OME - ToomatesEotvos de Hungría, que se iniciaron en 1894, precisamente durante la efervescencia de fin de siglo, consecuencia de la cual fue también el proceso iniciado

OLIMPIADA MATEMÁTICA

ACTIVIDADES 2019

REAL SOCIEDAD MATEMÁTICA ESPAÑOLA

Page 831: COMPENDIUM OME - ToomatesEotvos de Hungría, que se iniciaron en 1894, precisamente durante la efervescencia de fin de siglo, consecuencia de la cual fue también el proceso iniciado
Page 832: COMPENDIUM OME - ToomatesEotvos de Hungría, que se iniciaron en 1894, precisamente durante la efervescencia de fin de siglo, consecuencia de la cual fue también el proceso iniciado

Continuamos este año la publicación de este pequeño folleto que recoge los problemas propuestos en las principales actividades relacionadas con la Olimpiada Matemática Española: desde los utilizados en las fases local y nacional de su edición número 55, hasta los de las olimpiadas y concursos internacionales en las que los equipos españoles de matemáticas, allí seleccionados, participaron en el año 2019. Con ello pretendemos poner al alcance tanto de profesores como de estudiantes el valioso material de trabajo que suponen los buenos problemas. Esperamos y deseamos que pueda ser utilizado en diferentes actividades de formación, imprescindibles para poder enfrentarse con éxito a cualquier competición matemática; actividades que difícilmente pueden llegar a buen puerto sin la desinteresada y nunca reconocida labor de los profesores de secundaria.

En las páginas que siguen encontraréis sin duda problemas interesantes, atractivos y formativos. Un buen problema puede abordarse desde distintos enfoques, que os invitamos a explorar. Pero un buen problema es sobre todo un reto. Os animamos a aceptarlo, confiando en que disfrutaréis con ello.

La Olimpiada Internacional de Matemáticas (IMO), ha servido de modelo para el resto de competiciones, destinadas a estudiantes de secundaria, que cada año se realizan a lo largo y ancho de los cinco continentes. A diferencia de otras olimpiadas científicas no tiene establecido ningún temario, aunque por tradición los problemas que en ella se proponen, siempre abordables con técnicas elementales, se agrupan alrededor de cuatro grandes bloques: álgebra, geometría, teoría de números y combinatoria. En la web hay excelentes páginas en inglés con abundantes recursos: sirvan como ejemplo las de Art of Problem solving (www.artofproblemsolving.com), y la de Mathlinks (www.mathlinks.ro). Es de destacar asimismo The IMO Compendium (www.imomath.com), que recoge los problemas de las llamadas ”listas cortas”, entre los que cada año se eligen los seis que constituyen finalmente la prueba de la Olimpiada Internacional. No son muchas las páginas en español sobre el tema, aunque en la de la Olimpiada Matemática Argentina (www.oma.org.ar) puede encontrarse también variado material.

Deseamos agradecer de todo corazón el trabajo altruista de cuantos hacen posible, año tras año, la Olimpiada Matemática Española. Estamos especialmente agradecidos a los estudiantes, que cada año participan en ella con ilusión, y a sus profesores, que los alientan y ayudan. La RSME, a través de su Comisión de Olimpiadas, espera ser capaz de ofrecerles el apoyo y soporte matemático que necesiten.

María Gaspar Alonso-Vega

Page 833: COMPENDIUM OME - ToomatesEotvos de Hungría, que se iniciaron en 1894, precisamente durante la efervescencia de fin de siglo, consecuencia de la cual fue también el proceso iniciado
Page 834: COMPENDIUM OME - ToomatesEotvos de Hungría, que se iniciaron en 1894, precisamente durante la efervescencia de fin de siglo, consecuencia de la cual fue también el proceso iniciado

Introducción

Como cada año desde 1964, la Real Sociedad Matemática Española organizó el Concurso Final de la 55 Olimpiada Matemática Española (OME). Lo hizo a través de los Delegados de Distrito o Comunidad, y bajo la coordinación de la Comisión de Olimpiadas de la RSME con la colaboración del Ministerio de Educación y Ciencia. La OME se desarrolla en dos fases: la primera o Fase Local, se celebró a primeros de año y en ella se escogieron los estudiantes que representaron a cada distrito en la Segunda Fase. Este año el Concurso Final, que es de ámbito nacional, se celebró a finales de marzo en Orense y en ella se seleccionaron los miembros del equipo español que han representado a nuestro país en la Olimpiada Internacional (IMO), celebrada en Bath (UK) en julio pasado, y la Olimpiada Iberoamericana de Matemáticas celebrada en Guanajuato (México) en septiembre. Como hacen otros países de nuestro entorno es fundamental que nuestros estudiantes se preparen lo mejor posible, dentro de nuestras posibilidades, para competir en estos concursos. Cada año, el equipo español que acude a la IMO es invitado a participar en Olimpiadas internacionales de ámbito regional previas a la IMO. Como parte de la preparación del equipo español, este año los estudiantes seleccionados en el Concurso Final compartieron unas sesiones de trabajo con ex-olímpicos previas al inicio de la preparación que realizaron los estudiantes del equipo español en Barcelona antes de partir para la IMO. En este folleto se recogen los enunciados y soluciones de la Fase Local, los enunciados y soluciones del Concurso Final de la OME 55, Mathcontest y los de las Olimpiadas Mediterránea y el Concurso celebrado durante la preparación de Barcelona. También se han incluido los enunciados de la IMO y las soluciones de nuestros participantes. Finalmente, se han incluido los enunciados y soluciones de la Olimpiada Iberoamericana. José Luis Díaz Barrero

Page 835: COMPENDIUM OME - ToomatesEotvos de Hungría, que se iniciaron en 1894, precisamente durante la efervescencia de fin de siglo, consecuencia de la cual fue también el proceso iniciado
Page 836: COMPENDIUM OME - ToomatesEotvos de Hungría, que se iniciaron en 1894, precisamente durante la efervescencia de fin de siglo, consecuencia de la cual fue también el proceso iniciado

Índice

LIV Olimpiada Matemática Española (Primera Fase) ……………………..1

LV Olimpiada Matemática Española (Concurso Final)…..………………..9

Mathcontest………………………………………………………………………..16

Mediterranean Mathematical Olympiad …………………………………….20

Barcelona Contest ……………………………………………………………… 24

LX Olimpiada Internacional de Matemáticas ………………………………29

XXXIV Olimpiada Iberoamericana de Matemáticas ……………………….38

Page 837: COMPENDIUM OME - ToomatesEotvos de Hungría, que se iniciaron en 1894, precisamente durante la efervescencia de fin de siglo, consecuencia de la cual fue también el proceso iniciado
Page 838: COMPENDIUM OME - ToomatesEotvos de Hungría, que se iniciaron en 1894, precisamente durante la efervescencia de fin de siglo, consecuencia de la cual fue también el proceso iniciado

LV Olimpiada Matematica Espanola (Primera Fase)

Enunciados

1. Para cada numero de cuatro cifras distintas abcd, denotamos por S al numeroS = abcd− dcba. Demuestra que S es multiplo de 37 si y solo si las dos cifras centralesdel numero abcd son iguales.

2. Demuestra que para todo n ≥ 2 podemos encontrar n numeros reales x1, x2, · · · , xndistintos de 1 de manera que los productos

x1 · x2 · . . . · xn y1

1− x1· 1

1− x2· . . . · 1

1− xnson iguales.

3. El trapecio isosceles ABCD tiene lados paralelos AB y CD. Sabemos que AB = 6,AD = 5 y ∠DAB = 60o. Se lanza un rayo de luz desde A que rebota en CB en el puntoE e intersecta en AD en el punto F . Si AF = 3, calcula el area del triangulo AFE.

4. Sea p ≥ 3 un numero primo y consideramos el triangulo rectangulo de cateto mayorp2 − 1 y cateto menor 2p. Inscribimos en el triangulo un semicırculo cuyo diametro seapoya en el cateto mayor del triangulo y que es tangente a la hipotenusa del triangulo.Encuentra los valores de p para los cuales el radio del semicırculo es un numero entero.

5. ¿Existen m,n numeros naturales de forma que

n2 + 2018mn+ 2019m+ n− 2019m2

es un numero primo?

6. Fijamos un numero natural k ≥ 1. Encuentra todos los polinomios P (x) que cumplan

P (xk)− P (kx) = xkP (x)

para todo valor de x ∈ R.

7. Considera el conjunto de numeros naturales n cumpliendo que 1 ≤ n ≤ 1000000.En ese conjunto, indica si es mayor la cantidad de numeros que pueden expresarse dela forma a3 + mb2, con a, b ∈ N y m ∈ 0, 2, 4, 6, 8 o la cantidad de numeros que nopueden expresarse de esa manera.

8. Prueba que para todo a, b, c > 0 se cumple que

a2

b3c− a

b2≥ c

b− c2

a.

1

Page 839: COMPENDIUM OME - ToomatesEotvos de Hungría, que se iniciaron en 1894, precisamente durante la efervescencia de fin de siglo, consecuencia de la cual fue también el proceso iniciado

¿En que caso se cumple la igualdad?

9. Consideramos un triangulo ABC y un punto D en el lado AC. Si AB = DC = 1,∠DBC = 30o y ∠ABD = 90o, calcula el valor de AD.

2

Page 840: COMPENDIUM OME - ToomatesEotvos de Hungría, que se iniciaron en 1894, precisamente durante la efervescencia de fin de siglo, consecuencia de la cual fue también el proceso iniciado

LV Olimpiada Matematica Espanola (Primera Fase)

Soluciones

1. Para cada numero de cuatro cifras distintas abcd, denotamos por S al numero S =abcd− dcba. Demuestra que S es multiplo de 37 si y solo si las dos cifras centrales delnumero abcd son iguales.

Solucion. Escribimos el numero como abcd como 1000a+ 100b+ 10c+ d y el numerodcba como 1000d+ 100c+ 10b+ a. Por tanto,

S = abcd− dcba = 1000a+ 100b+ 10c+ d− (1000d+ 100c+ 10b+ a) =

999(a− d) + 90(b− c) = 37 · 33(a− d) + 2 · 5 · 32(b− c).

El primer sumando es obviamente multiplo de 37. El segundo sumando no tiene elfactor 37 ya que este es un numero primo y |b − c| ≤ 9. Por tanto, S sera multiplo de37 si y solo si b− c = 0, es decir, si y solo si b = c.

2. Demuestra que para todo n ≥ 2 podemos encontrar n numeros reales x1, x2, · · · , xn 6=1 de manera que los productos

x1 · x2 · . . . · xn y1

1− x1· 1

1− x2· . . . · 1

1− xnson iguales.

Solucion. Dado x 6= 1, notemos que la ecuacion

x =1

1− x⇐⇒ x2 − x+ 1 = 0

no tiene soluciones reales. Sin embargo, dados x, y 6= 1,

x · y =1

1− x· 1

1− y

tiene una solucion sencilla ya que

x · y =1

1− x· 1

1− y=

1

x− 1· 1

y − 1

y las ecuaciones

x =1

x− 1e y =

1

y − 1⇐⇒ x2 − x− 1 = 0 e y2 − y − 1 = 0

sı tienen solucion

x = y =1±√

5

2.

Por tanto, si n es un numero par, podemos agrupar de dos en dos cada termino de cadaproducto y utilizar lo anterior, encontrando las soluciones

x1 = x2 = · · · = xn =1±√

5

2.

3

Page 841: COMPENDIUM OME - ToomatesEotvos de Hungría, que se iniciaron en 1894, precisamente durante la efervescencia de fin de siglo, consecuencia de la cual fue también el proceso iniciado

Si n = 3, consideramos, por ejemplo, x1 = x2 = 2. La igualdad del enunciado nos llevaa la ecuacion

4x3 =

(1

1− 2

)2 1

1− x3=

1

1− x3⇐⇒ 4x23 − 4x3 + 1 = 0

que da la solucion x3 = 12 . Ası, obtenemos (x1, x2, x3) = (2, 2, 12). Si n es cualquier impar

mayor que 3, basta con completar estos tres valores con un numero par de valores delos xk utilizando el caso par anterior.

3. El trapecio isosceles ABCD tiene lados paralelos AB y CD. Sabemos que AB = 6,AD = 5 y ∠DAB = 60o. Se lanza un rayo de luz desde A que rebota en CB en el puntoE e intersecta en AD en el punto F . Si AF = 3, calcula el area del triangulo AFE.

Solucion. Puesto que el trapecio es isosceles y ∠DAB = 60o, podemos alargar loslados AD y BC que intersectan en G, formando ası un triangulo equilatero ABG.

Llamando α = ∠EAB, tendremos que ∠AEB = 120− α. Como el rayo sale simetrica-mente del lado BC, tendremos que ∠GEF = 120− α y, por tanto,

∠FEA = 180− 2(120− α) = 2α− 60.

Como ∠FAE = 60 − α, tendremos que ∠AFE = 180 − α y entonces ∠GFE = α.Esto demuestra que los triangulos GFE y BAE son semejantes. Llamando x = GE,tendremos que EB = 6 − x. Como FG = AG − AF = 6 − 3 = 3, tendremos por lasemejanza de triangulos que

3

x=

6

6− x⇐⇒ 18− 3x = 6x⇐⇒ 9x = 18

de donde obtenemos que x = 2 y, por tanto, GE = 2 y EB = 4. Tendremos

AB/GF = 6/3 = 2 =⇒ Area(BAE)/Area(GFE) = 22 = 4.

El area del triangulo AFE se puede calcular, por ejemplo,

Area(AFE) = Area(AGB)− Area(GFE)− Area(AEB) =

Area(AGB)− 5Area(GFE) =1

2· 6 · 6 · sen 60− 5 · 1

2· 2 · 3 · sen 60 =

(18− 15)

√3

2=

3√

3

2.

4. Sea p ≥ 3 un numero primo y consideramos el triangulo rectangulo de cateto mayorp2 − 1 y cateto menor 2p. Inscribimos en el triangulo un semicırculo cuyo diametro se

4

Page 842: COMPENDIUM OME - ToomatesEotvos de Hungría, que se iniciaron en 1894, precisamente durante la efervescencia de fin de siglo, consecuencia de la cual fue también el proceso iniciado

apoya en el cateto mayor del triangulo y que es tangente a la hipotenusa del triangulo.Encuentra los valores de p para los cuales el radio del semicırculo es un numero entero.

Solucion. En el triangulo rectangulo ABC, consideramos AB = p2 − 1, AC = 2p.

Por el Teorema de Pitagoras, tendremos que BC2

= AC2

+AB2, ası que

BC2

= (2p)2 + (p2 − 1)2 = 4p2 + p4 − 2p2 + 1 = p4 + 2p2 + 1 = (p2 + 1)2

de donde obtenemos que BC = p2 + 1.Llamando r al radio del semicırculo, E el centro del cırculo que esta en el lado AB y Fel punto de tangencia del semicırculo con el lado BC, tendremos que AE = EF = r.Por una parte, el area del triangulo ABC viene dada por

Area(ABC) =1

2AB ·AC = (p2 − 1)p.

Por otra,

Area(ABC) = Area(AEC) + Area(ECB) =1

2AE ·AC +

1

2BC · EF =

1

2r · 2p+

1

2(p2 + 1) · r =

r

2(p2 + 2p+ 1) =

r

2(p+ 1)2.

Igualando las dos expresiones para el area del triangulo ABC, obtenemos que

r

2(p+ 1)2 = (p2 − 1)p

de donde

r =(p2 − 1)2p

(p+ 1)2=

2p(p− 1)

p+ 1.

Es un calculo sencillo comprobar que

2p− 4 <2p(p− 1)

p+ 1< 2p

ası que las unicas posibilidades para r son 2p− 1, 2p− 2 y 2p− 3.

Si r = 2p− 1, entonces

2p− 1 =2p(p− 1)

p+ 1=⇒ 2p2 + 2p− p− 1 = 2p2 − 2p =⇒ p = 1/3

Si r = 2p− 2 = 2(p− 1), entonces

2(p− 1) =2p(p− 1)

p+ 1=⇒ 1 =

2p

p+ 1=⇒ p = 1

5

Page 843: COMPENDIUM OME - ToomatesEotvos de Hungría, que se iniciaron en 1894, precisamente durante la efervescencia de fin de siglo, consecuencia de la cual fue también el proceso iniciado

Si r = 2p− 3, entonces

2p− 3 =2p(p− 1)

p+ 1=⇒ 2p2 + 2p− 3p− 3 = 2p2 − 2p =⇒ p = 3

Por tanto, la unica solucion valida es p = 3, lo que nos da el valor de r = 3.

5. ¿Existen m,n numeros naturales de forma que

n2 + 2018mn+ 2019m+ n− 2019m2

es un numero primo?

Solucion. Tratamos de factorizar la expresion del enunciado. Igualando esta expresiona 0, tendremos

n2 + (2018m+ 1)n+ 2019m− 2019m2 = 0

que podemos tratar como una ecuacion en la variable n, obteniendo que

n =−(2018m+ 1)±

√(2018m+ 1)2 − 4(2019m− 2019m2)

2.

La expresion dentro de la raız viene dada por

(2018m+ 1)2 − 4(2019m− 2019m2) =

20182m2 + 2 · 2018m+ 1− 4 · 2019m+ 4 · 2019m2 =

(20182 + 4 · 2018 + 4)m2 − 2m+ 1 = 20202m2 − 2m+ 1 = (2020m− 1)2

ası que

n =−(2018m+ 1)± (2020m− 1)

2

y, entonces, las soluciones son

n1 =2m− 2

2= m− 1

y

n2 =−4038m

2= −2019m.

Por tanto, podemos factorizar la expresion del enunciado como

n2 + 2018mn+ 2019m+ n− 2019m2 = (n+ 2019m)(n−m+ 1).

En este producto, el primer factor es obviamente mayor que 1. Una condicion necesariapara que esta expresion sea un numero primo es que n −m + 1 = 1, es decir, n = m.En este caso, el primer factor queda de la forma n+ 2019m = 2020n, que es un numerocompuesto ya que 2020 lo es. Por tanto, la expresion del enunciado no sera un numeroprimo para ningun valor de n y de m naturales.

6. Fijamos un numero natural k ≥ 1. Encuentra todos los polinomios P (x) que cumplan

P (xk)− P (kx) = xkP (x)

para todo valor de x ∈ R.

6

Page 844: COMPENDIUM OME - ToomatesEotvos de Hungría, que se iniciaron en 1894, precisamente durante la efervescencia de fin de siglo, consecuencia de la cual fue también el proceso iniciado

Solucion. Fijemonos que una solucion trivial es P (x) = 0 para cualquier valor dek ≥ 1.Para encontrar otras soluciones, fijamos primero k = 1. En ese caso, la ecuacion queda

P (x)− P (x) = xP (x),

por lo que P (x) = 0, que es la solucion anterior.Si k ≥ 2 y P (x) es una constante c, tendremos que c − c = xkc y, por tanto, cxk = 0,imposible a menos que c = 0, que nos da el polinomio trivial P (x) = 0 de nuevo.Supongamos pues que k ≥ 2 y que el grado del polinomio es n ≥ 1. Es obvio queP (xk) tendra grado nk y P (2x) tendra grado n, ası que el termino de la izquierda dela igualdad sera un polinomio de grado nk ya que k ≥ 1. El termino de la derecha seraun polinomio de grado n+k, ası que tendremos que nk = n+k, de donde k(n−1) = ny, por tanto,

k =n

n− 1= 1 +

1

n− 1.

Como k es un numero natural, tendremos que necesariamente n = 2 y, por tanto, k = 2.Escribimos pues P (x) = ax2 + bx+ c y, sustituyendo en la ecuacion, obtenemos:

ax4 + bx2 + c− (4ax2 + 2bx+ c) = ax4 + bx3 + cx2

y simplificando obtenemos (b− 4a)x2 = bx3 + cx2, de donde necesariamente obtenemosque b = 0 y c = −4a. Ası pues, los polinomios cumpliendo la propiedad del enunciadoseran todos los de la forma P (x) = a(x2 − 4) para todo a ∈ R.En resumen, para todo k ≥ 1, una solucion es P (x) = 0. En el caso k = 2, los polinomiosde la forma P (x) = a(x2 − 4) para cualquier a ∈ R tambien son solucion.

7. Considera el conjunto de numeros naturales n cumpliendo que 1 ≤ n ≤ 1000000.En ese conjunto, indica si es mayor la cantidad de numeros que pueden expresarse dela forma a3 + mb2, con a, b ∈ N y m ∈ 0, 2, 4, 6, 8 o la cantidad de numeros que nopueden expresarse de esa manera.

Solucion. Como 0 ≤ a3, b2 ≤ a3 + mb2 ≤ 1000000, tendremos que 0 ≤ a ≤ 100 y0 ≤ b ≤ 1000. Para m = 0, tenemos que a3 +mb2 = a3 y la cantidad de numeros de esaforma sera 100. Para cada m = 2, 4, 6, 8, la cantidad de numeros sera menor o igual que100 · 1000 = 100000. Por tanto, habra, a lo sumo, 100 + 4 · 100000 = 400100 numerosde la forma a3 +mb2 con las condiciones del enunciado. Por tanto, habra mas numerosque no pueden expresarse de esa manera.

8. Prueba que para todo a, b, c > 0 se cumple que

a2

b3c− a

b2≥ c

b− c2

a.

¿En que caso se cumple la igualdad?

Solucion. Multiplicando por ab3c toda la desigualdad para eliminar los denominadores,tendremos que

a3 − a2bc ≥ ac2b2 − c3b3 ⇐⇒ a3 − ac2b2 ≥ a2bc− c3b3 ⇐⇒

a2(a− bc) ≥ c2b2(a− bc)⇐⇒ (a2 − b2c2)(a− bc) ≥ 0.

7

Page 845: COMPENDIUM OME - ToomatesEotvos de Hungría, que se iniciaron en 1894, precisamente durante la efervescencia de fin de siglo, consecuencia de la cual fue también el proceso iniciado

Como la funcion f(x) = x2 es creciente, tendremos que los dos terminos del productode la izquierda deben tener el mismo signo, ası que la desigualdad es cierta. La igualdadse dara si y solo si a− bc = 0, es decir, si a = bc.

9. Consideramos un triangulo ABC y un punto D en el lado AC. Si AB = DC = 1,∠DBC = 30o y ∠ABD = 90o, calcula el valor de AD.

Solucion. Llamando ∠ADB = α, tendremos que ∠BDC = 180 − α. Utilizando elteorema de los senos en el triangulo ADB tenemos que

x

1=

1

sinα=

BD

sin(90− α)

y en el triangulo DBC tendremos que

1

sin 30=

BD

sin(α− 30)=

BC

sin(180− α)

De las primeras igualdades deducimos que BD = cosαsinα y de la segunda que

BD =sin(α− 30)

sin 30= 2 sin(α− 30) = 2(sinα cos 30− cosα sin 30)

y ası, BD =√

3 sinα− cosα. Igualando tenemos

cosα

sinα=√

3 sinα− cosα ⇒ cosα =√

3 sin2 α− sinα cosα

de donde cosα(1+sinα) =√

3 sin2 α y, elevando al cuadrado y denotando por t = sinα,(1−t2)(1+t)2 = 3t4, de donde 4t4+2t3−2t−1 = 0 y, factorizando, 2t3(2t+1)−(2t+1) =0. Por tanto, (2t3 − 1)(2t + 1) = 0 y, por tanto, tenemos dos opciones: Si 2t + 1 = 0,entonces t = −1

2 y x = 1/t = −2, imposible. Por tanto, 2t3 − 1 = 0, ası que t = 13√2

y

deducimos que x = 3√

2.

8

Page 846: COMPENDIUM OME - ToomatesEotvos de Hungría, que se iniciaron en 1894, precisamente durante la efervescencia de fin de siglo, consecuencia de la cual fue también el proceso iniciado

LV Olimpiada Matematica Espanola (Concurso Final)

Enunciados

1. Un conjunto de numeros enteros T es orensano si existen enteros a < b < c tales quea, c pertenecen a T y b no pertenece a T . Hallar el numero de subconjuntos orensanosde 1, 2, . . . , 2019.

2. Determinar si existe un conjunto finito S formado por numeros primos positivos demanera que para cada entero n ≥ 2, el numero 22 +32 + · · ·+n2 sea, al menos, multiplode uno de los elementos de S.

3. Las ternas de numeros reales (a, b, c) son tales que el polinomio p(x) = x4 + ax3 +bx2+ax+c tiene exactamente tres raıces reales distintas, que son iguales a tan y, tan 2yy tan 3y, para algun numero real y. Hallar todos los posibles valores de y, tales que0 ≤ y < π.

4. Calcular todos los pares de enteros (x, y) tales que

3423(x2 + y2

)= x3y3.

5. Calcular el valor mınimo del mayor valor del conjunto A, siendo

A = xy, xy − x− y + 1, x+ y − 2xy, 0 ≤ x ≤ y ≤ 1 .

6. En el triangulo escaleno ABC, la bisectriz de A corta al lado BC en el punto D. Lasrectas que pasan por D tangentes a las circunferencias circunscritas de los triangulosABD y ACD cortan a las rectas AC y AB en los puntos E y F , respectivamente. SiBE y CF se cortan en G, demostrar que ∠EDG = ∠ADF .

9

Page 847: COMPENDIUM OME - ToomatesEotvos de Hungría, que se iniciaron en 1894, precisamente durante la efervescencia de fin de siglo, consecuencia de la cual fue también el proceso iniciado

LV Olimpiada Matematica Espanola (Concurso Final)

Soluciones

1. Un conjunto de numeros enteros T es orensano si existen enteros a < b < c tales quea, c pertenecen a T y b no pertenece a T . Hallar el numero de subconjuntos orensanosde 1, 2, . . . , 2019.

Solucion. El numero de subconjuntos de 1, 2, · · · , 2019 es 22019 como es bien conoci-do. Contemos ahora el numero de estos conjuntos que NO tienen la propiedad pedida.Claramente, el conjunto vacıo, y los subconjuntos de 1, 2, . . . , 2019 con un unico ele-mento no tienen la propiedad pedida, ya que es imposible elegir a, c distintos en ellos.Hay en total (

2019

0

)+

(2019

1

)= 1 + 2019 = 2020

de estos conjuntos. Cualquier otro subconjunto que no tenga la propiedad pedida hade estar formado por elementos consecutivos de 1, 2, . . . , 2019. En efecto, si m,Mson respectivamente el mınimo y el maximo de T , para que T NO tenga la propiedadenunciada, todos los elementos m,m+1,m+2, . . . ,M han de estar en T , pues si algunok no lo estuviera, podemos tomar a = m, b = k y c = M , cumpliendose la propiedad. Elnumero de tales subconjuntos es

(20192

). En efecto, dado un tal subconjunto, su mınimo

m y su maximo M lo determinan biunıvocamente, y hay(20192

)formas distintas de

elegir dos numeros del conjunto 1, 2, . . . , 2019, de forma que el menor sera m, y elmayor M .

Finalmente, se tiene que el numero total de subconjuntos T de 1, 2, · · · , 2019 con lapropiedad pedida es

22019 − 2020−(

2019

2

)= 22019 − 2039191.

2. Determinar si existe un conjunto finito S formado por numeros primos positivos demanera que para cada entero n ≥ 2, el numero 22 +32 + · · ·+n2 sea, al menos, multiplode uno de los elementos de S.

Solucion. Es bien conocido que

12 + 22 + · · ·+ n2 =n(n+ 1)(2n+ 1)

6=

2n3 + 3n2 + n

6,

con lo que restando 1 a ambos lados, resulta

22 + 32 + · · ·+ n2 =2n3 + 3n2 + n− 6

6=

(n− 1)(2n2 + 5n+ 6

)6

.

Esta expresion sugiere tomar p = (n− 1)/6 o n = 6p+ 1 con lo que se obtiene

22 + 32 + · · ·+ n2 = p(72p2 + 54p+ 13

).

10

Page 848: COMPENDIUM OME - ToomatesEotvos de Hungría, que se iniciaron en 1894, precisamente durante la efervescencia de fin de siglo, consecuencia de la cual fue también el proceso iniciado

Ahora elegimos n = 6p + 1, donde p da resto 1 al dividir entre cualquier primo de Sdistinto de 139, y da resto −1 al dividir entre 139. Notese que el resto al dividir entrecualquier primo de S distinto de 139 es 72 + 54 + 13 = 139, que es primo y por lo tantono divisible por ningun primo de S distinto de 139, y el resto al dividir entre 139 es−72 + 54 − 13 = −31, claramente tampoco divisible por 139. Luego esta expresion escoprima con todos los primos de S, y la respuesta es que no existe tal conjunto S.

3. Las ternas de numeros reales (a, b, c) son tales que el polinomio p(x) = x4 + ax3 +bx2+ax+c tiene exactamente tres raıces reales distintas, que son iguales a tan y, tan 2yy tan 3y, para algun numero real y. Hallar todos los posibles valores de y, tales que0 ≤ y < π.

Solucion. Sean r, r, s, t los tres ceros reales y distintos del polynomio p(x). Mediantelas formula de Cardano-Viete, identificando los coeficientes cubico y lineal, se obtiene

2r + s+ t = r2s+ r2t+ 2rst⇔ 2r(1− st) = (r2 − 1)(s+ t)

De la ultima igualdad se deduce que si r2 − 1 = 0 entonces 1 − st = 0 y viceversa. Acontinuacion, bajo la hipoteis r2 = st = 1 y considerando 0 ≤ y < π, analizaremos lossiguientes casos:

Si r = tan y entonces y ∈π

4,3π

4

pero para estos valores s = tan 2y no esta

definido.

Si r = tan 2y entonces y ∈π

8,3π

8,5π

8,7π

8

. Poniendo s = tan y y s = tan 3y se

tiene que para todos los valores anteriores es rs = tan y · tan 3y = 1, lo cual escierto si y + 3y = 4y es un multiplo impar de π/2, y esto ocurre para todos losvalores anteriores.

Si r = tan 3y entonces se debe cumplir rs = tan y · tan 2y = 1 y la tan 3y no estadefinida.

Por tanto solo queda por analizar los casos en que r2 − 1 6= 0 y 1− st 6= 0. Dividiendolos dos lados de la igualdad 2r(1− st) = (r2 − 1)(s+ t) por (r2 − 1)(1− st) resulta

2r

1− r2+

s+ t

1− st= 0.

Estas expresiones por separado serıan las correspondients a las formulas de la tangentedel angulo doble y de la tangente de la suma. Ahora, utilizando la ultima expresion,distinguiremos los siguientes casos:

Si r = tan y, s = tan 2y, t = tan 3y entonces tan 2y+ tan 5y = 0 que admite comosoluciones los multiplos de π

2 inferiores a 7π2 con lo que

y ∈π

7,2π

7,3π

7,4π

7,5π

7,6π

7

.

Si r = tan 2y, s = tan y, t = tan 3y entonces tan 4y+ tan 4y = 0 que admite comosoluciones los multiplos de π

2 inferiores a 4π, con lo que

y ∈π

8,3π

8,5π

8,7π

8

,

11

Page 849: COMPENDIUM OME - ToomatesEotvos de Hungría, que se iniciaron en 1894, precisamente durante la efervescencia de fin de siglo, consecuencia de la cual fue también el proceso iniciado

y hemos rechazado y = π2 ya que entonces tan y no serıa un numero real, y tambien

π4 y 3π

4 ya que entonces tan(2y) tampoco serıa un numero real.

Si r = tan 3y, s = tan y, t = tan 2y entonces tan 6y+ tan 3y = 0 que admite comosoluciones los multiplos de π

2 inferiores a 9π2 , con lo que

y ∈π

9,2π

9,π

3,4π

9,5π

9,2π

3,7π

9,8π

9

.

Estos son todos los valores, y hemos terminado.

4. Calcular todos los pares de enteros (x, y) tales que

3423(x2 + y2

)= x3y3.

Solucion. Notese en primer lugar que si xy = 0, entonces x2+y2 = 0, de donde resultala solucion x = y = 0. Notese tambien que si xy < 0, entonces x2 + y2 < 0, absurdo,luego x, y tienen ambos el mismo signo. Como cambiar simultaneamente de signo a xy a y no altera la ecuacion, podemos asumir a partir de este momento y sin perdidade generalidad que x, y son ambos enteros positivos. Sean x = 3ma e y = 3nb con m,nenteros no negativos y a, b enteros coprimos y no divisibles por 3. Supongamos quem ≥ n. Entonces la ecuacion dad se transforma en

8((3m−na)2 + b2) = 33m+n−4a3b3.

Como todo cuadrado perfecto da resto 0 o 1 al dividir entre 3, como es bien conocido,el miembro de la izquierda no es divisible por 3 y para que se cumpla la ecuacion debeser el exponente de 3 en el termino de la derecha igual a cero. Es decir, 3m+n− 4 = 0y como m ≥ n ≥ 0 esto implica que m = n = 1. La ecuacion ahora se simplifica yqueda

8(a2 + b2) = a3b3.

Por la simetrıa de la expresion podemos suponer que a ≥ b. Entonces

16a2 ≥ a3b3 ⇔ 16 ≥ ab3

y tenemos dos posibilidades (1) b = 2 de donde resulta a = 2, (2) b = 1 pero en este casolos unicos valores posibles de a son 1, 2, 4, 8 y no satisfacen la ecuacion a3−8a2−8 = 0.Se deduce que (a, b) = (2, 2) y (x, y) = (6, 6). Finalmente, se tiene que las unicassoluciones posibles son:

(x, y) = (−6,−6), (x, y) = (0, 0), (x, y) = (6, 6).

5. Calcular el valor mınimo del mayor valor del conjunto A, siendo

A = xy, xy − x− y + 1, x+ y − 2xy, 0 ≤ x ≤ y ≤ 1 .

12

Page 850: COMPENDIUM OME - ToomatesEotvos de Hungría, que se iniciaron en 1894, precisamente durante la efervescencia de fin de siglo, consecuencia de la cual fue también el proceso iniciado

Solucion 1. Haciendo el cambio de variable xy = p, y x+ y = s y escribiendo los treselementos del conjunto A en terminos de s y p, tenemos

a = xy = p, b = xy − x− y + 1 = (1− x)(1− y) = s− 1 + p, c = s− 2p

verificandose que a+ b+ c = 1. Observemos que s2 − 4p = (x− y)2 ≥ 0.

Ahora consideremos los siguientes casos:

Si 0 ≤ x ≤ y ≤ 1

3⇒ p = xy ≤ 1

9⇔ b = (1− x)(1− y) ≥ 4

9.

Si c = s − 2p ≤ 4

9⇒ 0 ≤ s2 − 4p =

(2p+

4

9

)2

− 4p = 4

(p− 1

9

) (p− 4

9

), de

donde se deduce que p ≤ 1

9o p ≥ 4

9. Si a = p ≤ 1

9⇒ b ≥ 4

9, c = s− 2p ≤ 4

9y si

a = p ≥ 4

9⇒ b ≤ 1

9, c ≤ 4

9. Por lo tanto, en cualquier caso

maxA ≥ 4

9

Para hallar el mınimo4

9, las desigualdades deberan ser igualdades y se tiene:

p =1

9, s− 2p =

4

9⇒ s =

2

3⇒ x = y =

1

3,

p =4

9, s− 2p =

4

9⇒ s =

4

3⇒ x = y =

2

3.

Solucion 2. En primer lugar, se observa que xy+ (xy−x− y+ 1) + (x+ y− 2xy) = 1.Ası, si uno de los tres elementos de A vale 1

9 o menos, entonces los otros dos suman 89

o mas, luego si el menor valor de A vale 19 o menos, el mayor valor de A vale 4

9 o mas.

Supongamos que xy ≥ x+y−2xy. Entonces, por la desigualdad entre medias aritmeticay geometrica se tiene que

3xy ≥ x+ y ≥ 2√xy,

√xy ≥ 2

3,

y el mayor valor de A es al menos xy ≥ 49 en este caso.

Supongamos que xy − x − y + 1 ≥ x + y − 2xy. Nuevamente por la desigualdad entremedias aritmetica y geometrica, se tiene que

3xy + 1 ≥ 2(x+ y) ≥ 4√xy ⇔ (3

√xy − 1) (

√xy − 1) ≥ 0.

La ultima desigualdad se verifica cuando√xy ≥ 1, con lo que el mayor valor de A serıa

al menos xy ≥ 1, o bien√xy ≤ 1

3 , para xy ≤ 19 , y por la observacion inicial el mayor

valor de A serıa 49 o mas.

En cualquier otro caso, x+y−2xy es el mayor valor de A, y supongamos que es inferiora 4

9 . Es decir, x+ y − 2xy < 49 . Aplicando la desigualdad entre las medias aritmetica y

geometrica, se obtiene

2xy +4

9> x+ y ≥ 2

√xy ⇔

(√xy − 1

2

)2

>1

36=

(1

6

)2

13

Page 851: COMPENDIUM OME - ToomatesEotvos de Hungría, que se iniciaron en 1894, precisamente durante la efervescencia de fin de siglo, consecuencia de la cual fue también el proceso iniciado

Si√xy − 1

2 > 16 , entonces

√xy > 2

3 y el mayor valor de A es al menos xy > 49 ,

contradiccion. Si 12 −√xy > 1

6 , entonces√xy < 1

3 , con lo que xy < 19 , y por la

observacion inicial el mayor valor de A es mayor que 49 , contradiccion.

Luego el mayor valor de A nunca puede ser menor que 49 , y ese valor es en efecto el

mınimo que toma el maximo de A, pues se puede obtener A =49 ,

19 ,

49

tomando

x = y = 23 , o A =

19 ,

49 ,

49

tomando x = y = 1

3 . Viendo las condiciones de igualdaden los dos primeros casos analizados, se tiene ademas que estos son todos los posiblespares de valores (x, y) para los que el mayor valor de A toma dicho valor mınimo.

6. En el triangulo escaleno ABC, la bisectriz de A corta al lado BC en el punto D. Lasrectas que pasan por D tangentes a las circunferencias circunscritas de los triangulosABD y ACD cortan a las rectas AC y AB en los puntos E y F , respectivamente. SiBE y CF se cortan en G, demostrar que ∠EDG = ∠ADF .

Solucion. Se verifican las igualdades de angulos ∠ADE = ∠B y ∠ADF = ∠C, porser angulos semiinscritos en las circunferencias (ABD) y (ADC), respectivamente. Por

GF E

D CB

A

tanto el cuadrilatero AFDE es inscriptible y de eso se deduce que

∠AFE = ∠ADE = ∠B y ∠AEF = ∠ADF = ∠C,

y por tanto, EF es paralela a BC. Ademas DE = DF por ser cuerdas correspondientesal angulo ∠A/2, ası que el triangulo FDE es isosceles.

Sea M el punto medio de EF , L = AD ∩EF , y H = DG∩EF . Aplicando el teoremade Thales, se obtiene que

LE

LF=DC

BD

Por otro lado, de 4FGH ∼ 4CGD resulta

HF

DC=GH

GD,

de 4EGH ∼ 4BGD, se obtiene

HE

BD=GH

GD

14

Page 852: COMPENDIUM OME - ToomatesEotvos de Hungría, que se iniciaron en 1894, precisamente durante la efervescencia de fin de siglo, consecuencia de la cual fue también el proceso iniciado

y de aquı queHF

HE=DC

DB,

luego H y L son simetricos respecto de M , como vermos mas tarde, y ∠LDM =∠HDM = α siendo

α = 90 − ∠ADC =∠C − ∠B

2si C > B,

y

α = 90 − ∠ADB =∠B − ∠C

2si B > C.

Finalmente, se tiene que

∠GDE = ∠ADE − α = ∠ADF si B > C,

y∠GDE = ∠ADE + α = ∠ADF si B < C.

Ahora falta probar que L y H son simetricos respecto a M . En efecto, si denotamosLF + LE = HF +HE = s, entonces

LE

LF=HF

HE,

de dondes

LF=

s

HE⇒ LF = HE y LE = HF

con lo que HM = ML como se querıa demostrar.

15

Page 853: COMPENDIUM OME - ToomatesEotvos de Hungría, que se iniciaron en 1894, precisamente durante la efervescencia de fin de siglo, consecuencia de la cual fue también el proceso iniciado

BarcelonaTech Mathcontest 2019

Problems

1. Let p ≥ 3 be a prime number. If a < b are positive integers then find the value ofgcd(p2

a+ 1, p2

b+ 1).

2. Let D,E, F denote the feet of the altitudes of an acute triangle ABC, and let(X1, X2), (Y1, Y2), (Z1, Z2) denote the feet of perpendiculars from D,E, F , respectively,upon the other two sides of the triangle. Prove that the six points X1, X2, Y1, Y2, Z1, Z2

lie on a circle.

3. Let OX and OY be two segments in the plane that form an acute angle α. LetP = X1Y1X2Y2 · · ·XnYn (n ≥ 2) be a closed polygonal line with Xi ∈ OX and Yi ∈ OYfor all 1 ≤ i ≤ n. The sides of P are colored red or blue in such a way that XiYi is redand YiXi+1 is blue for all 1 ≤ i ≤ n (subscripts are taken modulo n). If R and B arethe total length of segments colored red and blue respectively, then prove that

R

B≥ sin

α

2.

4. Consider the binomial coefficients(mn

), where 1 ≤ m ≤ 1919 and 0 ≤ n ≤ m

(that is, the numbers corresponding to the rows of Tartaglia’s triangle from 1 to 1919).Determine how many of these numbers are odd.

16

Page 854: COMPENDIUM OME - ToomatesEotvos de Hungría, que se iniciaron en 1894, precisamente durante la efervescencia de fin de siglo, consecuencia de la cual fue también el proceso iniciado

BarcelonaTech Mathcontest 2019

Solutions

1. Let p ≥ 3 be a prime number. If a < b are positive integers then find the value ofgcd(p2

a+ 1, p2

b+ 1).

Solution. Using induction on b we have

p2b − 1 = (p− 1)(p+ 1)(p2 + 1)(p2

2+ 1) . . . (p2

b−1 + 1),

and since a < b then p2a

+ 1 is a divisor of p2b − 1. Thus, the gcd of p2

a+ 1 and p2

b+ 1

divides both p2b − 1 and p2

b+ 1. Therefore, it divides their difference

(p2b

+ 1)− (p2b − 1) = 2,

and it is exactly 2.

2. Let D,E, F denote the feet of the altitudes of an acute triangle ABC, and let(X1, X2), (Y1, Y2), (Z1, Z2) denote the feet of perpendiculars from D,E, F , respectively,upon the other two sides of the triangle. Prove that the six points X1, X2, Y1, Y2, Z1, Z2

lie on a circle.

Solution. Let H be the orthocenter of 4ABC, and let Z2 and Y1 lie on BC, X2 andZ1 on CA, and Y2 and X1 on AB as shown in the figure below.

H

Z2

Z1Y2

Y1

X2

X1

FE

D CB

A

Scheme for solving problem 2

From 4BDX1 ∼ 4BFZ2, we get

BX1

BZ2=BD

BF,

from 4BEY2 ∼ 4BHF , we have

BY2BF

=BE

BH⇔ BY2

BE=BF

BH.

17

Page 855: COMPENDIUM OME - ToomatesEotvos de Hungría, que se iniciaron en 1894, precisamente durante la efervescencia de fin de siglo, consecuencia de la cual fue también el proceso iniciado

Likewise, from 4BEY1 ∼ 4BHD, we obtain

BY1BD

=BE

BH⇔ BY1

BE=BD

BH.

Dividing up the last two equalities, yields

BY1BY2

=BD

BF=BX1

BZ2⇔ BY1 ·BZ2 = BX1 ·BY2.

Hence Y2, X1, Z2, Y1 lie on a circle Γ1; and similarly Z2, Y1, X2, Z1 lie on a circle Γ2;and X2, Z1, Y2, X1 lie on a circle Γ3. If Γ1 and Γ2, for example, coincide, the statementis proved. If they are distinct their radical axis is the line BC. Thus, if our statementis not true, we have three distinct circles Γ1, Γ2 and Γ3, whose radical axes, taking thecircles in pairs, form the sides of triangle ABC, which contradicts the theorem thatsuch radical axes are either concurrent or parallel, as it is well-known.

3. Let OX and OY be two segments in the plane that form an acute angle α. LetP = X1Y1X2Y2 · · ·XnYn (n ≥ 2) be a closed polygonal line with Xi ∈ OX and Yi ∈ OYfor all 1 ≤ i ≤ n. The sides of P are colored red or blue in such a way that XiYi is redand YiXi+1 is blue for all 1 ≤ i ≤ n (subscripts are taken modulo n). If R and B arethe total length of segments colored red and blue respectively, then prove that

R

B≥ sin

α

2.

Solution. Let R =

n∑i=1

XiYi and B =

n∑i=1

YiXi+1. For all 1 ≤ i ≤ n, we have

YiXi+1 ≤ OYi +OXi+1

on account of triangular inequality. Adding up these expressions, yields

B =n∑i=1

YiXi+1 ≤n∑i=1

(OYi +OXi+1)

Using Cosine Law and the AM-GM-QM inequalities, we have

XiY2i = OX2

i +OY 2i − 2OXi ·OYi cosα

≥ OX2i +OY 2

i − (OX2i +OY 2

i ) cosα

= (OX2i +OY 2

i ) (1− cosα)

≥ (OXi +OYi)2

2(1− cosα)

from which

XiYi ≥ (OXi +OYi)

√2

1− cosα=

1

sinα/2

follows. Adding up the above expressions, we get

B ≤n∑i=1

(OYi +OXi+1) ≤1

sinα/2

n∑i=1

XiYi =R

sinα/2

18

Page 856: COMPENDIUM OME - ToomatesEotvos de Hungría, que se iniciaron en 1894, precisamente durante la efervescencia de fin de siglo, consecuencia de la cual fue también el proceso iniciado

from which the statement follows.

4. Consider the binomial coefficients(mn

), where 1 ≤ m ≤ 1919 and 0 ≤ n ≤ m

(that is, the numbers corresponding to the rows of Tartaglia’s triangle from 1 to 1919).Determine how many of these numbers are odd.

Solution. We first prove that the cardinality of the set of odd numbers in a row ofTartaglia’s triangle is given by

2# ones in the binary expansion of n

(for instance, the number of odd number between(5i

)where 0 ≤ i ≤ 5, is 22). To see

this, recall first that (a+b)2 ≡ a2+b2 mod 2. Next, consider the binary decompositionof n, as n = 2a1 + . . .+ 2ak . Therefore,

n∑i=0

(n

i

)xi = (1 + x)n = (1 + x)2

a1+...+2ak ≡ (1 + x2a1

) · · · (1 + x2ak ) (mod 2).

Expanding the latter product we get 2k summands, each of the form x raised to a sumof powers of 2; since each of them is different, we get that in

(ni

)xi there are exactly 2k

non-zero terms, as claimed.Now, we just have to do the corresponding sum between the rows 1 and 1919; forsimplicity, let us first count the number of odd terms in the first 2047 rows (the closestpower of two to 1919). There are

(11i

)numbers between 1 and 2047 with exactly i ones

in its decomposition, for 1 ≤ i ≤ 11. This is because these numbers can be seen aswords of length 11 where each letter is either 0 or 1. Hence, we have that the desiredvalue is given by

11∑i=1

(11

i

)2i.

Observe that

(1 + x)11 =

11∑i=0

(11

i

)xi,

and hence11∑i=1

(11

i

)2i = 311 − 1.

Now, we just have to discount the number of odd numbers in the rows 2047 − i, with0 ≤ i ≤ 127. The number of ones in the binary decomposition of 2047 − i is 11 minusthe number of ones in the binary decomposition of i. But between 0 and 127 we have(7i

)numbers with i ones, 0 ≤ i ≤ 7, and hence we have to subtract

7∑i=0

(7

i

)211−i = 211

( 7∑i=0

(7

i

)(1

2

)i)= 211

( 7∑i=0

(7

i

)(1

2

)i)=

= 211 · (3/2)7 = 24 · 37.

Then, the number of odd numbers in the first 1919 rows is

311 − 24 · 37 − 1.

19

Page 857: COMPENDIUM OME - ToomatesEotvos de Hungría, que se iniciaron en 1894, precisamente durante la efervescencia de fin de siglo, consecuencia de la cual fue también el proceso iniciado

Mediterranean Mathematical Competition 2019

Problems

1. Triangle ABC, in which ∠A = 60 and D ∈ BC is such that AD is the internalbisector of angle ∠A. Let rB, rC and r be the inradius of the triangles ABD, ADC andABC, respectively. Show that

1

rB+

1

rC= 2

(1

a+

1

b+

1

c

),

where a, b and c are the lengths of the sides BC, AC and AB of triangle ABC.

2. Prove that there exist infinitely many positive integers x, y, z for which the sum ofthe digits in the decimal representation of 4x4 + y4 − z2 + 4xyz is 2.

3. Let P be an interior point to an equilateral triangle of altitude one. If x, y, z arethe distances from P to the sides of the triangle, then prove that

x2 + y2 + z2 ≥ x3 + y3 + z3 + 6xyz.

4. Let m1 < m2 < · · · < ms be a sequence of s ≥ 2 positive integers, none of whichcan be written as the sum of (two or more) distinct other numbers in the sequence. Forevery integer r with 1 ≤ r < s, prove that r ·mr +ms ≥ (r + 1)(s+ 1).

20

Page 858: COMPENDIUM OME - ToomatesEotvos de Hungría, que se iniciaron en 1894, precisamente durante la efervescencia de fin de siglo, consecuencia de la cual fue también el proceso iniciado

Mediterranean Mathematical Competition 2019

Solutions

1. Triangle ABC, in which ∠A = 60 and D ∈ BC is such that AD is the internalbisector of angle ∠A. Let rB, rC and r be the inradius of the triangles ABD, ADC andABC, respectively. Show that

1

rB+

1

rC= 2

(1

r+

1

b+

1

c

),

where b and c are the lengths of the sides AC and AB of triangle ABC.

Solution. It is well-known that the bisector AD is given by

AD =2bc

b+ ccos

A

2=bc√

3

b+ c

because ∠A = 60. Let h = AM be the length of the altitude from A in 4ABC. Onaccount of the internal bisector theorem, we have

BD =ac

b+ cand CD =

ab

b+ c.

Let pABC =BD +DA+AB

2be the semiperimeter of 4ABD. If we denote by [ABD]

the area of triangle ABD, then we have

rB =[ABD]

pABD=

h ·BD2 · pABD

=h ·BD

BD +DA+AB=

h·acb+c

acb+c + bc

√3

b+c + c=

ah

2p+√

3 · b,

and likewise, we get

rC =ah

2p+√

3 · b,

where p = (a+ b+ c)/3.

From the above, we have

1

rB+

1

rC=

2p+√

3 · bah

+2p+

√3 · c

ah=

4p+ (b+ c)√

3

2 [ABC]

=2

r+

2 · (b+ c)√

3

2 · bc sin 60= 2

(1

r+

1

b+

1

c

),

and we are done.

2. Prove that there exist infinitely many positive integers x, y, z for which the sum ofthe digits in the decimal representation of 4x4 + y4 − z2 + 4xyz is 2.

21

Page 859: COMPENDIUM OME - ToomatesEotvos de Hungría, que se iniciaron en 1894, precisamente durante la efervescencia de fin de siglo, consecuencia de la cual fue también el proceso iniciado

Solution. We have a lot of solutions. Indeed, we have

4x4 + y4 − z2 + 4xyz = (4x4 + y4 + 4x2y2)− (4x2y2 + z2 − 4xyz)

= (2x2 + y2)2 − (2xy − z)2

= (2x2 + y2 − 2xy + z)(2x2 + y2 + 2xy − z)

The two factors A = 2x2 + y2 − 2xy + z and B = 2x2 + y2 + 2xy − z add up toA+B = 4x2 + 2y2. If we choose the values of x and y so that A = 4x2 = 4 · 52n+2 andB = 2y2 = 2 · 22n, then the product will become AB = 2 · 102n+2 and the sum of thedigits will be equal 2.

Summarizing, we pick up an integer n ≥ 1 and set x = 5n+1 and = 2n. The desiredequation A = 4x2 is equivalent to 2x2 + y2 − 2xy + z = 4x2, and hence

z = 2x2 + 2xy − y2 = 2 · 52n+2 + 10n+1 − 4n.

Note that z indeed is a positive integer. As the described choice of x, y, z then yields

4x4 + y4 − z2 + 4xyz = 2 · 102n+2,

and the proof is complete.

3. Let P be an interior point to an equilateral triangle of altitude one. If x, y, z are thedistances from P to the sides of the triangle, then prove that

x2 + y2 + z2 ≥ x3 + y3 + z3 + 6xyz.

Solution. It is well-known that in an equilateral triangle the sum of the distances froman interior point P to its sides equals the altitude of the triangle, as can be easilyproven. On account of the preceding, we have to prove that if x + y + z = 1 then itholds that

x2 + y2 + z2 ≥ x3 + y3 + z3 + 6xyz

To do it, we begin observing that when x+ y + z = 1, then

xy + yz + zx ≥ 9xyz

Indeed, applying AM-GM inequality, we get

xy + yz + zx = (xy + yz + zx)(x+ y + z) ≥ 3 3√

(xy)(yz)(zx) · 3 3√xyz = 9xyz

orxy + yz + zx− 3xyz ≥ 6xyz

On account of the constrain and the preceding inequality, we obtain

xy + yz + zx− 3xyz = xy(1− z) + yz(1− x) + zx(1− y)

= xy(x+ y) + yz(y + z) + zx(z + x)

≥ 6xyz

Adding 1 to both terms of the last inequality and reordering terms, yields

(x+ y + z)2 + xy(x+ y) + yz(y + z) + zx(z + x)− 6xyz ≥ 1

22

Page 860: COMPENDIUM OME - ToomatesEotvos de Hungría, que se iniciaron en 1894, precisamente durante la efervescencia de fin de siglo, consecuencia de la cual fue también el proceso iniciado

or equivalently,

x2+y2+z2+2xy(1−z)+2yz(1−x)+2zx(1−y)+xy(x+y)+yz(y+z)+zx(z+x) ≥ 1,

and

x2 + y2 + z2 + 3xy(x+ y) + 3yz(y + z) + 3zx(z + x) ≥ 1 = (x+ y + z)3

from whichx2 + y2 + z2 ≥ x3 + y3 + z3 + 6xyz

follows. Equality holds when x = y = z = 1/3. That is, when P is the centroid of thetriangle, and we are done.

4. Let m1 < m2 < · · · < ms be a sequence of s ≥ 2 positive integers, none of whichcan be written as the sum of (two or more) distinct other numbers in the sequence. Forevery integer r with 1 ≤ r < s, prove that r ·mr +ms ≥ (r + 1)(s+ 1).

Solution. For k, ` with 0 ≤ k ≤ r and k + 1 ≤ ` ≤ s, we introduce the auxilairy value

T (k, `) = m` +k∑i=1

mi.

Next, we count the number of elements of the set

A =T (k, `) | 0 ≤ k ≤ r and k + 1 ≤ ` ≤ s

.

For k = 0 we have s possible values for `, for k = 1 we have s− 1 values for `, and soon until for k = r we have s− r values for `. Adding up the preceding, we get

|A| = s+ (s− 1) + (s− 2) + . . .+ (s− r) = (r + 1)s− r(r + 1)

2=

1

2(r + 1)(2s− r).

Now we claim that these 12 (r + 1)(2s − r) auxiliary values are all pairwise distinct.

Indeed, let us assume that T (k, `) = T (u, v). WLOG we may assume that k ≤ u, sothe last equality turns into

m` = mv +

u∑i=k+1

mi.

But then m` can be written as a sum of distinct other numbers in the sequence, unless` = v and k = u holds. Hence the auxiliary values indeed are distinct. As altogetherthere are 1

2 (r + 1)(2s − r) auxiliary values, the largest value T (r, s) must be at least12 (r + 1)(2s− r). This yields

1

2(r + 1)(2s− r) ≤ T (r, s) = ms +

r∑i=1

mi ≤ ms +

r∑i=1

(mr − r + i).

Here we used that mi ≤ mr − r + i, which follows as the sequence is increasing. Theabove inequality can be rewritten into

1

2(r+1)(2s−r) ≤ ms+

r∑i=1

(mr−r+ i)⇔ 1

2(r+1)(2s−r) ≤ ms+rmr−r2 +

r(r + 1)

2

orr ·mr +ms ≥ rs+ s− r ≥ (r + 1)(s− 1),

and we are done.

23

Page 861: COMPENDIUM OME - ToomatesEotvos de Hungría, que se iniciaron en 1894, precisamente durante la efervescencia de fin de siglo, consecuencia de la cual fue también el proceso iniciado

Preparacion Barcelona Contest 2019

Problems

1 On the interior of each side of a triangle ABC draw n distinct points. Join the pointsfrom the sides AB and AC with the points on the side BC. Determine the maximumnumber of intersection points in the interior of 4ABC.

2 Determine all the positive integer solutions of the equation

4a · 5b − 3c · 11d = 1.

3 Determine the smallest M greater than 2019 for which the set

X = 2019, 2020, · · · ,M

can be expressed as a disjoint union of subsets of X, so that in each of them there isan element equal to the sum of the remaining elements of that subset.

4 Cyclic quadrilateral ABC is in scribed in a circle of center O and radius R. PointsE ∈ CD and F ∈ AB verify that EO = FO. Lines AD and BC cut line EF at pointsM and N , respectively. Let P be the symmetric point of M respect to the midpoint ofsegment AE. Prove that triangles FBN and CEP are similar.

24

Page 862: COMPENDIUM OME - ToomatesEotvos de Hungría, que se iniciaron en 1894, precisamente durante la efervescencia de fin de siglo, consecuencia de la cual fue también el proceso iniciado

Preparacion Barcelona Contest 2019

Solutions

1. On the interior of each side of a triangle ABC draw n distinct points. Join the pointsfrom the sides AB and AC with the points on the side BC. Determine the maximumnumber of intersection points in the interior of 4ABC.

Solution. Denote the points in the interior ofAB (starting fromA) with C1, C2, . . . , Cn,the points interior to BG (starting from B) with B1, B2, . . . , Bn and the points inte-rior to CA (starting from C) with A1, A2, · · · , An. Joining B1 with the points Ai andCi (1 ≤ i ≤ n) no intersection point is created. The segment B2Ci, intersects B1Cj if1 ≤ j ≤ i−1 (B2Ci and B1Cj are the diagonals of the convex quadrilateral B1B2CjCi),so there are i − 1 intersections. In the same time B2Ci intersects B1Aj for 1 ≤ j ≤ ncreating n+ i− 1 intersections.The segment B2Ai (1 ≤ i ≤ n) intersects B1Aj if 1 ≤ j ≤ i − 1 (in i − 1 intersectionpoints), so by drawing all the line segments from B2 we obtain

1 + 2 + . . .+ (n− 1) + n+ . . .+ (n+ n− 1) = n(2n− 1)

intersection points on the lines starting from B1. The number of intersection points ofthe segments whose end point is Bk (2 ≤ k ≤ n) with the segments whose end point isB1 (2 ≤ k ≤ n) is the same as the number of the intersection points determined by thesegments starting from B2 and B1. But this is n · (2n− 1). The condition ` < k impliesthe existence of k − 1 points B` , so the segments from Bk determine

(k − 1)n(2n− 1)

intersection points. This implies that by drawing the segments corresponding to all thepoints Bk we obtain

N =n∑k=1

(k − 1)n(2n− 1) =n2(n− 1)(2n− 1)

2

intersection points.

For n = 1 we get N = 0 intersection points. For n = 2 we obtain N = 6 intersectionpoints (see figure ). For n = 3 we illustrate this counting process in the figure:

1. Intersection points of segments starting from B2 with segments starting from B1.

2. Intersection points of segments starting from B3 with segments starting from B2.

3. Intersection points of segments starting from B3 with segments starting from B1.

We obtain N = 45 intersection points.

2. Determine all the positive integer solutions of the equation

4a · 5b − 3c · 11d = 1.

25

Page 863: COMPENDIUM OME - ToomatesEotvos de Hungría, que se iniciaron en 1894, precisamente durante la efervescencia de fin de siglo, consecuencia de la cual fue también el proceso iniciado

B2B1

A2

A1C2

C1

CB

A

B3B2B1

A3

A2

A1C3

C2

C1

CB

A

B3B2B1

A3

A2

A1C3

C2

C1

CB

A

B3B2B1

A3

A2

A1C3

C2

C1

CB

A

Number of intersection points for Problem 1.

Solution. Working modulo 3, we have 5b ≡ 2b ≡ 1, and b is even, say b = 2x, withx ∈ N. Then, we have

4a · 52x − 1 = (2a · 5x − 1) (2a · 5x + 1) = 3c · 11d.

Now, we consider the following cases:

i) 2a · 5x − 1 = 11d, 2a · 5x + 1 = 3c, or

ii) 2a · 5x + 1 = 11d, 2a · 5x − 1 = 3c.

Case (i) it is not possible, because modulo 5, 11d ≡ 1 (and not −1).

Next, we consider the second case. Since 11 = 2 · 5 + 1, then the triple d = 1, x = 1 anda = 1 is a solution. Moreover, 2 · 5− 1 = 9 = 32, and c = 2 from which

(a, b, c, d) = (1, 2, 2, 1)

as a solution follows.

Now, we will see that there are no more solutions. Indeed, if d ≥ 2, then c ≥ 5, and weget

2a · 5x = 11d − 1 = 3c + 1⇒ 11d − 11 = 3c − 9,

or11(

11d−1 − 1)

= 32(3c−2 − 1

).

26

Page 864: COMPENDIUM OME - ToomatesEotvos de Hungría, que se iniciaron en 1894, precisamente durante la efervescencia de fin de siglo, consecuencia de la cual fue también el proceso iniciado

Therefore, 11 divides 3c−2 − 1. That is, 3c−2 ≡ 1 (mod 11). Since the order of 3 in Z11

is 5, then c− 2 is a multiple of 5, and

35 − 1 | 3c−2 − 1.

Therefore, 2 · 112 = 242 = 35 − 1 is a divisor of 11(11d−1 − 1), which is impossible.

3. Determine the smallest M greater than 2019 for which the set

X = 2019, 2020, · · · ,M

can be expressed as a disjoint union of subsets of X, so that in each of them there isan element equal to the sum of the remaining elements of that subset.

Solution. Suppose X = X1 ∪X2 ∪ · · ·Xk. The element ai ∈ Xi is called especial if aiis equal to the sum of the remaining elements of Xi. It is evident that if any of thesesubsets contains some special element, then it must have at least three elements. Thus,the total number of elements of X is M − 2018, and the number k of subsets of thepartition verifies

k ≤ M − 2018

3.

On the other hand, the sum of all elements of X is

S = 2019 + 2020 + · · ·+M =(M + 2019)(M − 2018)

2.

Note that if ai ∈Mi is especial, then ye sum of the elements of Mi is 2ai. So, the sumS of the numbers of all Mi, verifies

S ≤ 2 (M + (M − 1) + (M − 1) + · · · (M − k + 1))

= 2(M +M − k + 1)k

2= (2M − k + 1) k

≤(

2M + 1− M − 2018

3

)· M − 2018

3

=(5M + 2021)(M − 2018)

9⇔ S =

M + 2019

2≤ 5M + 2021

9

on account that k ≤ M − 2018

3). From the preceding, we get

9(M + 2019) ≤ 2(5M + 20121)

andM ≥ 9 · 2019− 2 · 2021 = 7 · 2019− 4 = 14129.

Now, remains to prove that with M = 14129 (the set X would have M − 2018 =14129 − 2018 = 12111 elements) is possible to obtain the partition of the statement,using 4037 subsets, with three elements each one. Indeed, we will take in each subsetthe 4037 largest elements as special, from 7 · 2019− 4 = 14129 to 5 · 2019− 2 = 10093,completing with the following and taking care of maintaining the condition of specialfor the largest element of each subset to obtain:

M1 = 7 · 2019− 4, 5 · 2019− 3, 2 · 2019− 1,M2 = 7 · 2019− 5, 5 · 2019− 5, 2 · 2019,M3 = 7 · 2019− 6, 5 · 2019− 7, 2 · 2109 + 1,

· · · · · ·M4037 = M2·2019−1 = 5 · 2019− 2, 3 · 2019, 2 · 2019− 2.

27

Page 865: COMPENDIUM OME - ToomatesEotvos de Hungría, que se iniciaron en 1894, precisamente durante la efervescencia de fin de siglo, consecuencia de la cual fue también el proceso iniciado

4. Cyclic quadrilateral ABC is in scribed in a circle of center O and radius R. PointsE ∈ CD and F ∈ AB verify that EO = FO. Lines AD and BC cut line EF at pointsM and N , respectively. Let P be the symmetric point of M respect to the midpoint ofsegment AE. Prove that triangles FBN and CEP are similar.

Solution. Let X and Y be the intersection points of the line EF with the circumferencecircumscribed to quadrilateral ABCD. We will prove that ME = FN . Indeed, if Z

Z

Y

X

O

P

N

M

F

ED

C

B

A

is the midpoint of segment XY , the condition OE = OF implies that Z is also themidpoint of EF . Then M and N are symmetric points respect to Z, and ME = FN .

Since EMAP is a parallelogram, we have the equality of vectors−→AP =

−−→ME =

−−→FN ,

and this prove that APNF is also a parallelogram. In particular, quadrilateral PNCEis cyclic because it has sides parallel to ABCE.

Now, we have∠CEP = ∠CDA = 180 − ∠CBA = ∠FBN

and∠EPC = ∠ENC = ∠FBN

because they subtend the same arc.

28

Page 866: COMPENDIUM OME - ToomatesEotvos de Hungría, que se iniciaron en 1894, precisamente durante la efervescencia de fin de siglo, consecuencia de la cual fue también el proceso iniciado

LX Olimpiada Internacional de Matematicas

Enunciados

1. Let Z be the set of integers. Determine all functions f : Z → Z such that, for allintegers a and b,

f(2a) + 2f(b) = f(f(a+ b)).

2. In triangle ABC, point A1 lies on side BC and point B1 lies on side AC. Let Pand Q be points on segments AA1 and BB1, respectively, such that PQ is parallel toAB. Let P1 be a point on line PB1 such that B1 lies strictly between P and P1, and∠PP1C = ∠BAC. Similarly, let Q1 be a point on line QA1 such that A1 lies strictlybetween Q and Q1, and ∠CQ1Q = ∠CBA. Prove that points P , Q, P1, and Q1 areconcyclic.

3. A social network has 2019 users, some pairs of whom are friends. Whenever user Ais friends with user B, user B is also friends with user A. Events of the following kindmay happen repeatedly, one at a time:

Three users A, B, and C such that A is friends with both B and C, but B and C arenot friends, change their friendship statuses such that B and C are now friends, but Ais no longer friends with B, and no longer friends with C. All other friendship statusesare unchanged.

Initially, 1010 users have 1009 friends each, and 1009 users have 1010 friends each.Prove that there exists a sequence of such events after which each user is friends withat most one other user.

4. Find all pairs (k, n) of positive integers such that

k! = (2n − 1)(2n − 2)(2n − 4) · · · (2n − 2n−1).

5. The Bank of Bath issues coins with an H on one side and a T on the other. Harry hasn of these coins arranged in a line from left to right. He repeatedly performs the followingoperation: if there are exactly k > 0 coins showing H, then he turns over the k-th coinfrom the left; otherwise, all coins show T and he stops. For example, if n = 3 the processstarting with the configuration THT would be THT → HHT → HTT → TTT , whichstops after three operations.

(a) Show that, for each initial configuration, Harry stops after a finite number ofoperations.

29

Page 867: COMPENDIUM OME - ToomatesEotvos de Hungría, que se iniciaron en 1894, precisamente durante la efervescencia de fin de siglo, consecuencia de la cual fue también el proceso iniciado

(b) For each initial configuration C, let L(C) be the number of operations beforeHarry stops. For example, L(THT ) = 3 and L(TTT ) = 0. Determine the averagevalue of L(C) over all 2n possible initial configurations C.

6. Let I be the incentre of acute triangle ABC with AB 6= AC. The incircle ω of ABCis tangent to sides BC, CA, and AB at D, E, and F, respectively. The line through Dperpendicular to EF meets ω again at R. Line AR meets ω again at P. The circumcirclesof triangles PCE and PBF meet again at Q.Prove that lines DI and PQ meet on the line through A perpendicular to AI.

30

Page 868: COMPENDIUM OME - ToomatesEotvos de Hungría, que se iniciaron en 1894, precisamente durante la efervescencia de fin de siglo, consecuencia de la cual fue también el proceso iniciado

LX Olimpiada Internacional de Matematicas

Soluciones equipo espanol

The 60th edition of the International Mathematical Olympiad took place in July 2019in Bath (United Kingdom). The number of countries attending the competition withat least one contestant was 114, a new record for the IMO. The number of contestantswas also impressive: there were 643 this year. The competition was developed in twoconsecutive days, and contestants had to solve three problems each day in a maximumtime of four hours and a half. Each problem was graded with an integer mark between0 and 7 points, so the maximum possible score was 42 points, achieved this year bysix students. According to the usual standards, at most half of the students can get amedal, and then these are awarded in the proportion 1:2:3 for gold, silver and bronze,respectively. Under these circumstances, the number of points needed to get a bronzedistinction were 17; for a silver medal, 24 points were necessary, and only those whoreached 31 points got a gold medal. By countries, both the People’s Republic of Chinaand the United States of America got 227 points out of 252, while the Republic ofKorea was just one point below.Spain made an outstanding performance, with a new record of points (110) and relativeposition. They finished in the 42nd position and, for the first time, five of the studentsgot a bronze medal, while the remaining one got an honorable mention. The team wascomposed by Pau Cantos (22 points), Leonardo Costa (21), Albert Lopez (19), OriolBaeza (17), Pablo Soto (17), and Juan Brieva (14). The delegation was completed byMarıa Gaspar, as the chief of the delegation, and by Oscar Rivero, as the deputy leader.We present now the four problems which were solved by at least one Spaniard contestant(problems 1, 2, 4 and 5), and include the solutions given to them by our team (in somecase slightly modified by the deputy leader). In all the cases, the solutions follow theideas presented by the contestants, but we have done some little modifications to easethe exposition.

Problem 1. Let Z be the set of integers. Determine all functions f : Z→ Z such that,for all integers a and b,

f(2a) + 2f(b) = f(f(a+ b)).

Comment. Five of the students gave complete solutions to this question. All of themrequire an algebraic manipulation before getting that any solution to the functionalequation is of the form f(x) = Cx+d, where C and d are integer constants; after that,one has to guess which are the possible values for the constants. There are two mainapproaches for the first (and harder) part: either by different substitutions (as sketchedin the first solution we present) or by passing to a Cauchy equation, whose solutionsare well-known (as presented in the second solution, which assumes certain knowledgeof functional equations by the reader of this note).

Solution by Juan Brieva Ramırez. Setting a = 0 and b = 0, we obtain

3f(0) = f(f(0)). (1)

31

Page 869: COMPENDIUM OME - ToomatesEotvos de Hungría, que se iniciaron en 1894, precisamente durante la efervescencia de fin de siglo, consecuencia de la cual fue también el proceso iniciado

When we evaluate the given expression at (a, 0) and (0, a) and subtract both equations,one gets

f(2a) + f(0) = 2f(a). (2)

Using now (2), from the original equation we have

2f(a) + 2f(b) = f(f(a+ b)) + f(0). (3)

If we do b 7→ −a in (3), and then use (1), we obtain

f(a) + f(−a) = 2f(0). (4)

We finally substitute (−a, a+ 1) in the given equation and, using (4), one gets

f(a+ 1)− f(a) = C,

where C = f(f(1))−4f(0)2 . Since the difference between two consecutive integers is a

constant, we have that all possible functions are of the form f(x) = Cx+ d.From the condition (1),

3d = f(d) = Cd+ d,

and then, either d = 0 or C = 2. In the latter case, an easy check shows that all thesolutions of the form f(x) = 2x+ d are valid and satisfy the given condition. If d = 0,one easyly gets that either C = 2 (and this solution is already included in the previouscase) or C = 0 (and this solution clearly satisfies the given condition).We conclude that the possible solutions are either f(x) ≡ 0, or f(x) = 2x + d, whered ∈ Z.

Solution by Oriol Baeza Guasch. As in the previous solution, a straightforwardsubstitution gives

2f(a) + f(0) = f(f(a)),

and setting a = x+ y,

2f(x+ y) + f(0) = f(f(x+ y)) = 2f(x) + 2f(y)− f(0). (5)

Therefore,f(x+ y)− f(0) = (f(x)− f(0)) + (f(y)− f(0)). (6)

Defining g(x) = f(x)− f(0), equation (6) gives

g(x+ y) = g(x) + g(y). (7)

But (7) is a Cauchy equation, whose unique solutions are given by g(x) = Cx, whereC ∈ Z. Then, all possible solutions have the form f(x) = Cx + f(0). Proceeding asbefore by substituting in the original equation, we get that either f(0) = 0 or C = 2,and by a careful analysis of each case we get that either f(x) ≡ 0, or f(x) = 2x+ f(0),with f(0) ∈ Z.

Problem 2. In triangle ABC, point A1 lies on side BC and point B1 lies on sideAC. Let P and Q be points on segments AA1 and BB1, respectively, such that PQ isparallel to AB. Let P1 be a point on line PB1 such that B1 lies strictly between P andP1, and ∠PP1C = ∠BAC. Similarly, let Q1 be a point on line QA1 such that A1 liesstrictly between Q and Q1, and ∠CQ1Q = ∠CBA. Prove that points P , Q, P1, andQ1 are concyclic.

32

Page 870: COMPENDIUM OME - ToomatesEotvos de Hungría, que se iniciaron en 1894, precisamente durante la efervescencia de fin de siglo, consecuencia de la cual fue también el proceso iniciado

Solution by Pablo Soto Martın. Let X stand for the cut point of the circumcircle ofA1CQ1 with AA1. Similarly, let Y denote the intersection of the circumcircle of B1CP1

with BB1.We begin by claiming that both X and Y belong to the circumcircle of ABC. This istrue because

∠A1XC = ∠A1Q1C = ∠ABC

and∠B1Y C = ∠B1P1C = ∠BAC.

We now prove that the following three quadrilaterals are cyclic: PQXQ1, PQY P1, andPQXY . In particular, these three facts together imply that PQP1Q1XY is cyclic, andwe are done.Begin with PQXQ1. In this case,

∠QPX = ∠BAX = ∠BCX = ∠A1Q1X = ∠QQ1X. (8)

For PQY P1, the proof is symmetric, following the same angle chasing procedure as in(8). Finally, for PQXY we just observe that

∠PQY = ∠ABY = ∠AXY = ∠PXY ,

and hence the conclusion follows.

Problem 4. Find all pairs (k, n) of positive integers such that

k! = (2n − 1)(2n − 2)(2n − 4) · · · (2n − 2n−1).

Notation. As a general piece of notation for the two solutions we present, for a givenprime p, let vp(n) stand for the greatest integer such that pvp(n) divides n but pvp(n)+1

does not. All the approaches to the problem are based on comparing the growth of bothsides: one option is by using two different primes (as shown in the second solution),and the other one is using one prime and the usual absolute value (as shown in the firstsolution, which is maybe the most natural approach to the problem, although in the lastpart it requires some nasty computations). The second solution uses as a well-knownfact the lifting the exponent lemma.

Solution by Pau Cantos Coll. We begin by observing that

v2(k!) =⌊k

2

⌋+⌊k

4

⌋+⌊k

8

⌋+ . . . <

k

2+k

4+k

8+ . . . = k.

Since

v2((2n − 1)(2n − 2)(2n − 4) · · · (2n − 2n−1)) =

n(n− 1)

2,

we obtain thatn(n− 1)

2< k. (9)

On the other hand, we have

2n2> (2n − 1)(2n − 2)(2n − 4) · · · (2n − 2n−1) = k!. (10)

Putting (9) and (10) together, one gets

2n2>(n(n− 1)

2

)!. (11)

33

Page 871: COMPENDIUM OME - ToomatesEotvos de Hungría, que se iniciaron en 1894, precisamente durante la efervescencia de fin de siglo, consecuencia de la cual fue también el proceso iniciado

For n = 7, this is false, since

21! = 218 · 39 · 54 · 73 · 11 · 13 · 17 · 19 > 218 · 29 · 28 · 26 · 23 · 23 · 24 · 24 > 249.

We now show by induction that, indeed, the opposite inequality as that of (11) holdsfor n ≥ 7. For n = 7 it has already been checked. Then, using the induction hypothesis,we obtain ((n+ 1)n

2

)! >

(n(n− 1)

2

)n(n(n− 1)

2

)! > 8n · 2n2

= 2n2+3n > 2n

2+2n+1 = 2(n+1)2 ,

and we have proved that there are no solutions for n ≥ 7.Small cases have to be checked by hand.

When n = 6, the right hand side is 63 · 62 · 60 · 56 · 48 · 32. Since 31 is a primedividing 62, k ≥ 31 and then the left hand side would be clearly greater than theright hand side.

When n = 5, the right hand side is 31 · 30 · 28 · 24 · 16. Since 31 is prime, k ≥ 31and then, as before, the left hand side would be greater than the right hand side.

When n = 4, the right hand side is 15 · 14 · 12 · 8. Then, k ≥ 7; if k = 7, the 2-adicvaluation of both sides does not agree. If k ≥ 14, the left hand side is bigger thanthe right hand side.

When n = 3, the right hand side is 7 · 6 · 4 = 168, which is not a factorial of aninteger number.

When n = 2, the right hand side is 6 = 3!, so (3, 2) is a solution.

When n = 1, we obtain the solution (1, 1).

Therefore, the unique possible solutions are (3, 2) and (1, 1).

Solution by Albert Lopez Bruch. We compare the 5-adic and 7-adic valuationsof both sides to reach a contradiction. We begin with the prime 7. In this case, fordetermining

R7(n) := v7((2n − 1)(2n−1 − 1) · · · (22 − 1)(2− 1)),

we just recall that, by virtue of the lifting the exponent lemma, v7(2` − 1) = 0 when `

is not a multiple of 3, and

v7(2`−1) = v7(`) + 1 for all ` ∈ 3 · N.

Then,

R7(n) =⌊n

3

⌋+⌊ n

3 · 7

⌋+⌊ n

3 · 72⌋

+ . . . ≥ n− 2

3.

Similarly, forR5(n) := v5((2

n − 1)(2n−1 − 1) · · · (22 − 1)(2− 1)),

we have that v5(2` − 1) = 0 if ` is not a multiple of 4, and

v5(2`−1) = v5(`) + 1 for all ` ∈ 4 · N.

34

Page 872: COMPENDIUM OME - ToomatesEotvos de Hungría, que se iniciaron en 1894, precisamente durante la efervescencia de fin de siglo, consecuencia de la cual fue también el proceso iniciado

We have that

R5(n) =⌊n

4

⌋+⌊ n

4 · 5

⌋+⌊ n

4 · 52⌋

+ . . . <5n

16.

Then, note that

5n

16> R5(n) = v5(k!) ≥ v7(k!) = R7(n) ≥ n− 2

3. (12)

Hence, n < 32. This means that, indeed, since only the first two terms can contribute,R5(n) < n

4 + n20 = 3n

10 and, therefore,

3n

10≥ n− 2

3. (13)

Then, n ≤ 20. We distinguish now several cases.

If n is congruent to 0 modulo 3, then equation (12) can be refined to 5n/16 ≥ n/3,which never holds.

If n is congruent to 1 modulo 3, and moreover n < 20, equation (13) becomes

n

4≥ n− 1

3.

When n = 1 we get the solution (1, 1). When n = 4, we proceed as in the previoussolution to see that there are no possible solutions. All the cases correspondingto n > 4 are automatically excluded.

When n = 20, R5(20) = R7(20) = 6. Hence, v5(k!) = 6, so 25 ≤ k ≤ 29. At thesame time, v7(k!) = 6, so 42 ≤ k ≤ 48, and this is a contradicition.

If n < 20 and n is congruent to 2 modulo 3, equation (13) is replaced by

n

4≥ n− 2

3,

so n ≤ 8. If n = 8, R5(8) = R7(8) = 2. In this case, v5(k!) = 2, so 10 ≤ k ≤ 14,but at the same time v7(k!) = 2, and this forces 14 ≤ k ≤ 20. Hence, the uniqueoption is k = 14, but one can easily check that the pair (14, 8) does not work.

The case n = 5 is discarded as before, and when n = 2 we get the solution (3, 2).

Hence, the only possible solutions are (3, 2) and (1, 1).

Problem 5. The Bank of Bath issues coins with an H on one side and a T on theother. Harry has n of these coins arranged in a line from left to right. He repeatedlyperforms the following operation: if there are exactly k > 0 coins showing H, thenhe turns over the k-th coin from the left; otherwise, all coins show T and he stops.For example, if n = 3 the process starting with the configuration THT would beTHT → HHT → HTT → TTT , which stops after three operations.

(a) Show that, for each initial configuration, Harry stops after a finite number ofoperations.

(b) For each initial configuration C, let L(C) be the number of operations beforeHarry stops. For example, L(THT ) = 3 and L(TTT ) = 0. Determine the averagevalue of L(C) over all 2n possible initial configurations C.

35

Page 873: COMPENDIUM OME - ToomatesEotvos de Hungría, que se iniciaron en 1894, precisamente durante la efervescencia de fin de siglo, consecuencia de la cual fue también el proceso iniciado

Comment. Before properly presenting the solutions given by the contestants, we pointout that there are several straightforward ways to solve just part (a) without simulta-neously solving part (b), as it will be the case with the two solutions we discuss here,due to Leonardo Costa and Albert Lopez. For the sake of completeness, we indicate astrategy for part (a), following the script of Pablo Soto.Assume that the process does not finish; since any configuration completely determinesthe subsequent movements, this would mean that we get a cycle. Suppose that we arein a position of the cycle where the number of appearances of the H is minimum, andsay that at that moment we have k times that letter, with k ≥ 1. Therefore, at positionk we have the letter T (otherwise, we would move to a situation with fewer than kH’s). This means that, in the following movement, we continue towards the right of thesequence, until we find a letter H. If there were no more H’s beyond the initial position,that initial letter would be necessarily an H, by the conditions of the problem. Whenwe find the first H, we switch its state, so this becomes a T , and we begin the movementtowards the left, until one finds a new T . Observe in particular that at position k thereis the letter H, and we do pass through it, since all the intermediate positions until themoment were H, by construction. When this happens, it becomes a T . Hence, sincethe state of the intermediate positions has not changed (we have gone through themexactly twice), we get a configuration with strictly fewer H’s than the initial one. Thisis a contradiction, that comes from having supposed that the process never finishes.

Solution by Albert Lopez Bruch. Let cn denote the average number of operationsfor sequences of length n. We proceed by induction to solve both parts of the problem.For n = 1 it is easy, and one gets c1 = 1/2.Consider a sequence of length n + 1. If it finishes with a T , this does not affect thesequence of operations at all, since we never reach that position (it would require n+ 1appearances of the H, including the last one), and hence the number of operations isthe same as those needed for the word of length n obtained by deleting the last coin.Moreover, in this case the process finishes, since the induction hypothesis guaranteesthis.We now take a word of length n + 1 finishing with an H. Say that the word is of theform a1a2 . . . anH. We establish a bijection between words of length n+1 finishing withan H and words of length n by sending

a1a2 . . . anH 7→ an . . . a2a1,

where ai ∈ H,T and ai is H when ai is T and viceversa. This bijection commuteswith the operation described in the statement of the problem; moreover, it is indeed abijection, with the inverse given by

a1a2 . . . an 7→ an . . . a2a1H.

To see that it commutes with the operation, say that after applying it to the initialword we get a1a2 . . . ak . . . anH; if we apply now the bijection we get an . . . ak . . . a2a1.But this is precisely the result of applying the operation to an . . . a2a1, since this wordhas n− (k− 1) = n− k+ 1 times the letter H, and precisely its (n− k+ 1)-th elementis ak, that now gets reversed. Hence, the process arrives to a situation of H . . .HH.Moreover, this shows that, after the same number of operations needed in the caseof length n to get all T ’s, we get a solution of all H’s. From here, and after n + 1operations, we can finish.Therefore, we get

cn+1 =cn2

+cn + n+ 1

2= cn +

n+ 1

2,

36

Page 874: COMPENDIUM OME - ToomatesEotvos de Hungría, que se iniciaron en 1894, precisamente durante la efervescencia de fin de siglo, consecuencia de la cual fue también el proceso iniciado

and by the induction hypothesis one easily concludes. The answer is just

cn =n(n+ 1)

4.

Solution by Leonardo Costa Lesage. We keep the same notations as before, wherecn stands for the expected number of movements. In this case, we proceed again byinduction, but we need to consider two base cases, those being c1 = 1/2 and c2 = 3/2.There are three kinds of sequences of length n + 2: (a) those finishing with a T ; (b)those beginning with an H; and (c) those beginning with a T and finishing with anH. The proportion of the first two kinds is 1/2, but the intersection is non-empty:those sequences beginning with an H and finishing with a T (proportion 1/4) are beingcounted twice!Let us determine the number of operations in all three cases, showing that the processalways finishes. In the first one, since we never reach the last position (it would requiren + 2 appearances of the H), it is the same as the sequence of length n + 1 removingthe last coin. This gives an average number of cn+1. In the second case, the first coinshifts everything by one position, and therefore we can neglect it until the end, wherewe get an extra move. The average is cn+1 + 1. We now consider its intersection, givenby the sequences beginning with a H and finishing with a T . Taking into account theprevious considerations, we take the n central coins, and since until the end the coinsplaced in the extremes do not affect the operations, we reach a configuration of theform HTT . . . TT , and therefore the average number of movements is now cn + 1.In the third and last case, everything goes as in the second one, and after the movementscorresponding to the n central positions, we get TT . . . TH. From here, it takes 2n+ 3moves to finish. Observe that we have established that in all the cases the processterminates by reducing to shorter sequences where the induction hypothesis applies.Therefore,

cn+2 =cn+1

2+cn+1 + 1

2− cn + 1

4+cn + 2n+ 3

4= cn+1 +

n+ 2

2.

We already know that c1 = 1/2 and c2 = 3/2, and hence we can prove by a directinduction argument that

cn =n(n+ 1)

4.

37

Page 875: COMPENDIUM OME - ToomatesEotvos de Hungría, que se iniciaron en 1894, precisamente durante la efervescencia de fin de siglo, consecuencia de la cual fue también el proceso iniciado

XXXIV Olimpiada Iberoamericana de Matematicas

Problemas

1. Para cada entero positivo n, sea s(n) la suma de los cuadrados de los dıgitos den. Por ejemplo, s(15) = 12 + 52 = 26. Determina todos los enteros n ≥ 1 tales ques(n) = n.

2. Determina todos los polinomios P (x) de grado n ≥ 1 con coeficientes enteros talesque para todo numero real x se cumple

P (x) = (x− P (0))(x− P (1))(x− P (2)) · · · (x− P (n− 1)).

3. Sea Γ el circuncırculo del triangulo ABC. La paralela a AC que pasa por B cortaa Γ en D (D 6= B) y la paralela a AB que pasa por C corta a Γ en E (E 6= C). Lasrectas AB y CD se cortan en P , y las rectas AC y BE se cortan en Q. Sea M el puntomedio de DE. La recta AM corta a Γ en Y (Y 6= A) y a la recta PQ en J . La rectaPQ corta al circuncırculo del triangulo BCJ en Z (Z 6= J). Si las rectas BQ y CP secortan en X, demuestra que X pertenece a la recta Y Z.Nota. El circuncırculo de un triangulo es la circunferencia que pasa por los vertices deltriangulo.

4. Sea ABCD un trapecio con AB||CD e inscrito en la circunferencia Γ. Sean P y Qdos puntos en el segmento AB (A,P,Q,B estan en ese orden y son distintos) tales queAP = QB. Sean E y F los segundos puntos de interseccion de las rectas CP y CQ conΓ, respectivamente. Las rectas AB y EF se cortan en G. Demuestra que la recta DGes tangente a Γ.

5. Don Miguel coloca una ficha en alguno de los (n + 1)2 vertices determinados porun tablero de n × n. Una jugada consiste en mover la ficha desde el vertice en que seencuentra a un vertice adyacente en alguna de las ocho posibles direcciones: ↑, ↓, →,←, , , , , siempre y cuando se salga del tablero. Un recorrido es una sucesionde jugadas tal que la ficha estuvo en cada uno de los (n + 1)2 vertices exactamenteuna vez. ¿Cual es la mayor cantidad de jugadas diagonales (,,,) que en totalpuede tener un recorrido?

6. Sean a1, a2, . . . , a2019 enteros positivos y P un polinomio con coeficientes enteros talque, para todo entero positivo n,

P (n) divide a an1 + an2 + · · ·+ an2019.

Demuestra que P es un polinomio constante.

(La duracion de cada sesion fue de 4 horas y 30 minutos, y cada problema fue calificadocon un numero entero entre 0 y 7).

38

Page 876: COMPENDIUM OME - ToomatesEotvos de Hungría, que se iniciaron en 1894, precisamente durante la efervescencia de fin de siglo, consecuencia de la cual fue también el proceso iniciado

XXXIV Olimpiada Iberoamericana de Matematicas

Soluciones

1. Para cada entero positivo n, sea s(n) la suma de los cuadrados de los dıgitos den. Por ejemplo, s(15) = 12 + 52 = 26. Determina todos los enteros n ≥ 1 tales ques(n) = n.

Solucion por Andres Saez Schwedt. Sea Sk la suma maxima para un numero dek cifras, es obvio ver y probar que Sk = 81k. Para k = 4 se tiene que 81k < 10k−1, yaque 81k = 324 es un numero de 3 cifras, por lo que ningun numero n de 4 cifras puedeverificar s(n) = n.Dado que 10k−1 es una funcion exponencial que crece mas rapidamente que 81k (lineal),es previsible que para todo k > 4 se siga cumpliendo que 10k−1 > 81k, por lo quetampoco habra numeros n de k > 4 cifras tales que s(n) = n. Lo probaremos porinduccion:

10k = 10(10k−1) > 10 · 81k > 81(k + 1),

ya que 10k > k + 1 para k ≥ 1 (y estamos considerando k > 4).A continuacion probaremos que no existe ningun numero n de 3 dıgitos verificandos(n) = 3. Si lo hubiera, se tendrıa que s(n) ≤ S3 = 81 · 3 = 243, por lo tanto esimposible que n sea mayor que 243. Y como el maximo valor de s(n) para n en elintervalo [100, 243] es 12 + 92 + 92 = 163, debemos restringirnos al intervalo [100,163].De manera analoga 12 + 52 + 92 = 107 es el mayor valor posible para s(n), por lo quen ≤ 107, y finalmente se comprueba que ningun numero n entre 100 y 107 provoca ques(n) tenga 3 cifras.Para numeros de 2 cifras, sea n = 10a+ b, con a 6= 0. Si fuera s(n) = n, se tendrıa quea2 + b2 = 10a+ b, en particular b2 − b = 10a− a2, o bien b(b− 1) = a(10− a). Pero nohay coincidencias posibles entre los valores de b(b − 1) y a(10 − a), ya que b(b − 1) essiempre par, y los valores pares que puede alcanzar a(10− a) son solamente 2 · 8 = 16y 4 · 6 = 24, pero las ecuaciones b(b− 1) = 16 y b(b− 1) = 24 no tienen solucion.En consecuencia, no hay soluciones para numeros n de 2 cifras, y el unico de una cifraque verifica s(n) = n es n = 1.

2. Determina todos los polinomios P (x) de grado n ≥ 1 con coeficientes enteros talesque para todo numero real x se cumple

P (x) = (x− P (0))(x− P (1))(x− P (2)) · · · (x− P (n− 1)).

Solucion por Andres Saez Schwedt. Sea P (x) = anxn + an−1x

n−1 + · · ·+ a1x+ a0el polinomio en cuestion. Si n = 1 entonces P (x) = x−P (0), con lo cual P (0) = −P (0),es decir P (0) = 0 y P (x) = x. A partir de ahora se supondra n ≥ 2.Notamos primero que a0 = P (0). Ahora, por las relaciones de Cardano-Vieta tenemosque

P (0) = (−1)nP (0)P (1) · · ·P (n− 1).

Consideramos los siguientes casos por separado.

39

Page 877: COMPENDIUM OME - ToomatesEotvos de Hungría, que se iniciaron en 1894, precisamente durante la efervescencia de fin de siglo, consecuencia de la cual fue también el proceso iniciado

a0 6= 0. Esto implica (−1)n = P (1) · · ·P (n − 1), por lo que todas las raıces conexcepcion de a0 deben ser +1 o −1. Pero 1 no puede ser raız pues tendrıamos queP (1) = 0, y 0 no forma parte del conjunto de raıces −1, 1, a0. Ademas, si −1 esraız tendrıamos P (x) = (x−a0)(x+ 1)n−1, por lo que −1 = P (1) = (1−a0)2n−1,de donde 2 divide a −1, lo cual es absurdo. Luego −1 tampoco es raız. Se sigueque el polinomio debe ser de la forma x− a0. Pero entonces P (0) = −a0 tambiendebe ser raız, ası que a0 = −a0, es decir a0 = 0, contradiccion.

a0 = 0: Tenemos P (x) = x(x − P (1))(x − P (2)) · · · (x − P (n − 1)), y tomandox = 1:

P (1) = (1− P (1))(1− P (2)) . . . (1− P (n− 1)),

lo cual es imposible si 1 − P (1) tiene un divisor primo, ya que mcd(P (1), 1 −P (1)) = 1.

Por lo tanto debe ser 1− P (1) = ±1, es decir P (1) = 0 o P (1) = 2.

Si P (1) = 0, signfica que 1 es raız, luego uno de los factores (x− P (k)) debe ser(x−1), es decir, P (k) = 1 para algun k verificando 2 ≤ k ≤ n−1, y sustituyendox = k obtenemos

1 = k2(k − 1)(k − P (2)) . . . (k − P (n− 1)),

lo cual es una contradiccion.

Si P (1) = 2, ya podemos asegurar que n debe ser al menos 3, puesto que si fueran = 2, P (x) serıa (x − P (0))(x − P (1)) = x(x − 2), y P (1) serıa igual a −1 enlugar de 2.

Al sustituir x = 1 se tiene −2 = (1−P (2)) · · · (1−P (n− 1)), lo cual debe ser unproducto de factores 1,−1, 2 o −2. Esto se consigue de dos maneras:

• con un factor 1− P (k) igual a 2 (es decir P (k) = −1), una cantidad impar defactores 1− P (k) = −1 (P (k) = 2, que junto con P (1) formaran un numero parde factores (x− 2)), y el resto de factores 1− P (k) = 1 (P (k) = 0), luego

P (x) = (x+ 1)xa(x− 2)b, con a ≥ 1 y b ≥ 2 par (14)

• o bien con un factor −2 y una cantidad par de factores −1, es decir

P (x) = xa(x− 2)b(x− 3), con a ≥ 1 y b ≥ 1 impar (15)

En el caso (14) el grado de P (x) es n ≥ 4, luego P (3) deberıa ser una de lasraıces, que son −1, 0, 2, pero P (3) tiene valor absoluto mayor o igual que 12,absurdo.

En el caso (15), el grado no puede ser n ≤ 4 porque esta presente el factor (x−3)pero P (0) = 0, P (1) = 2, P (2) = 0 y P (3) = 0, y ninguno de los (x−P (k)) puedeser (x− 3). Y cuando n ≥ 5, P (4) deberıa ser raız, por lo tanto solo puede tomaruno de los valores 0, 2 o 3, sin embargo su valor absoluto es mayor o igual que 8.

En consecuencia, la unica solucion posible es la ya encontrada P (x) = x.

3. Sea Γ el circuncırculo del triangulo ABC. La paralela a AC que pasa por B cortaa Γ en D (D 6= B) y la paralela a AB que pasa por C corta a Γ en E (E 6= C). Las

40

Page 878: COMPENDIUM OME - ToomatesEotvos de Hungría, que se iniciaron en 1894, precisamente durante la efervescencia de fin de siglo, consecuencia de la cual fue también el proceso iniciado

rectas AB y CD se cortan en P , y las rectas AC y BE se cortan en Q. Sea M el puntomedio de DE. La recta AM corta a Γ en Y (Y 6= A) y a la recta PQ en J . La rectaPQ corta al circuncırculo del triangulo BCJ en Z (Z 6= J). Si las rectas BQ y CP secortan en X, demuestra que X pertenece a la recta Y Z.Nota. El circuncırculo de un triangulo es la circunferencia que pasa por los vertices deltriangulo.

Solucion por Andres Saez Schwedt. Los trapecios cıclicos (inscriptibles) son siem-pre isosceles, en este caso se tiene que AB = CD y AC = BE. Y dado que ∠DCA =∠CAB = ∠ABE, los triangulos DCA y ABE son iguales (congruentes) por el criterioLAL, con lo cual AD = AE. En particular, la mediatriz del segmento DE es un eje desimetrıa de la figura por el cual pasan los puntos A,O,M, Y, J , siendo O el centro deΓ. Ademas P esta en la mediatriz comun de AC y BD, recta que pasa por O, y Q estaen la mediatriz comun de AB y CE, la cual tambien pasa por O.

Esquema para resolver el problema 3.

Veamos ahora que los puntos O,B,C, P,Q estan en una misma circunferencia, quellamaremos ω: en efecto, por ser PO bisectriz de ∠BPC y OB = OC, se tiene queB,C, P,O son concıclicos, y dado queQO es bisectriz de ∠BQC, tambien son concıclicosB,C,Q,O.Por otra parte, el teorema de Rheims aplicado a los cuadrilateros cıclicos BCQP yBCED con las alineaciones P,D,C y Q,E,B implica que DE||PQ.Lema: Veremos que Y → S en la homotecia de centro X que lleva DE en PQ.Puesto que los triangulos DEY y PQS son isosceles, basta con ver la igualdad de unangulo correspondiente. Utilizando angulos orientados (dirigidos) en sentido antihorarioy modulo 180, se tiene:

]PSQ = 2]PBQ = 2]ABE = 2]CAB;]DY E = ]DBE = ]AQB = −]BQA = ]QAB + ]ABQ = 2]CAB.

En virtud del lema, los puntos S, Y,X estan alineados en una recta, que corta a PQen Z ′. El problema estara resuelto si probamos que Z ′ = Z, es decir, si los puntosB,C, J, Z ′ son concıclicos. Sea R = BC ∩ PQ, punto diagonal del cuadrilatero cıclicoBCQP , que junto con A y X forma un triangulo autopolar AXR (Brocard) del cualS es el ortocentro. En particular, las rectas AR y XS son perpendiculares y se cortanen un punto T , que pertenece a Γ.

41

Page 879: COMPENDIUM OME - ToomatesEotvos de Hungría, que se iniciaron en 1894, precisamente durante la efervescencia de fin de siglo, consecuencia de la cual fue también el proceso iniciado

La potencia desde R a Γ y al cuadrilatero cıclico ATJZ ′ (inscrito en la circunferenciade diametro AZ ′) permite afirmar que RB · RC = RA · RT = RJ · RZ ′, de donde setiene que BCJZ ′ es cıclico, como se querıa demostrar.Algunos casos especiales: si AB = AC resulta que el circuncırculo de BCJ es tangentea PQ, y no existe otro Z 6= J como pide el enunciado. Si A es recto, coinciden lospuntos D,E,M, Y , mientras que P,Q se van al infinito, pero el enunciado asegura queexisten.

4. Sea ABCD un trapecio con AB||CD e inscrito en la circunferencia Γ. Sean P y Qdos puntos en el segmento AB (A,P,Q,B estan en ese orden y son distintos) tales queAP = QB. Sean E y F los segundos puntos de interseccion de las rectas CP y CQ conΓ, respectivamente. Las rectas AB y EF se cortan en G. Demuestra que la recta DGes tangente a Γ.

Solucion por Andres Saez Schwedt. Notese que el trapecio cıclico ABCD es ne-cesariamente isosceles, siendo C,D puntos simetricos respecto de la mediatriz de AB.Dado que P,Q tambien son puntos simetricos, se tiene que ∠PDC = ∠DCQ.

Esquema para resolver el problema 4.

Ademas, por ser AB||CD se verifica que ∠PDC = ∠DPG, y el hecho de ser CDEFcıclico implica que ∠DCF = 180o − ∠FED = ∠DEG.De todo lo anterior se deduce que

∠DPG = ∠PDC = ∠DCQ = ∠DCF = ∠DEG.

Esto muestra que el cuadrilatero DPEG es cıclico, de donde se obtiene que

∠GDE = ∠GPE = ∠DCE,

y de aquı concluimos (angulos semiinscritos) que DG es tangente a Γ.

42

Page 880: COMPENDIUM OME - ToomatesEotvos de Hungría, que se iniciaron en 1894, precisamente durante la efervescencia de fin de siglo, consecuencia de la cual fue también el proceso iniciado

Mathematics may not teach us to add love or subtracthate, but it gives us hope that every problem has asolution – Anonymous.

Page 881: COMPENDIUM OME - ToomatesEotvos de Hungría, que se iniciaron en 1894, precisamente durante la efervescencia de fin de siglo, consecuencia de la cual fue también el proceso iniciado
Page 882: COMPENDIUM OME - ToomatesEotvos de Hungría, que se iniciaron en 1894, precisamente durante la efervescencia de fin de siglo, consecuencia de la cual fue también el proceso iniciado

/

LVII Olimpiada Matemática Española Concurso Final Nacional

7 y 8 de mayo de 2021

Enunciados

Problema 1

Los vértices, A, B y C, de un triángulo equilátero de lado 1 están en la superficie de una

esfera de radio 1 y centro O. Sea D la proyección ortogonal de A sobre el plano, α,

determinado por B, C y O. Llamamos N a uno de los cortes con la esfera de la recta

perpendicular a α por O. Halla la medida del ángulo DNO .

(Nota: la proyección ortogonal de A sobre el plano α es el punto de corte con α de la recta

que pasa por A y es perpendicular a α.)

Problema 2

Dado un número entero positivo n, definimos λ(n) como el número de soluciones enteras

positivas de la ecuación nyx 22. Diremos que el número n es “olímpico” si

λ(n) = 2021. ¿Cuál es el menor entero positivo que es olímpico? ¿Y cuál es el menor entero

positivo impar que es olímpico?

Problema 3

Tenemos 2021 colores y 2021 fichas de cada color. Colocamos las 20212 fichas en fila. Se

dice que una ficha, F , es “mala” si a cada lado de F quedan un número impar de las 2020 ×

2021 fichas que no comparten color con F .

a) Determina cuál es el mínimo número posible de fichas malas.

b) Si se impone la condición de que cada ficha ha de compartir color con al menos una ficha

adyacente, ¿cuál es el mínimo número posible de fichas malas?

Problema 4

Sean a, b, c, d números reales tales que

a + b + c + d = 0 y a2 + b

2 + c

2 + d

2 = 12.

Halla el valor mínimo y el valor máximo que puede tomar el producto abcd, y determina para

qué valores de a, b, c, d se consiguen ese mínimo y ese máximo.

Problema 5

Disponemos de 2n bombillas colocadas en dos filas (A y B) y numeradas de 1 a n en cada

fila. Algunas (o ninguna) de las bombillas están encendidas y el resto apagadas; decimos que

eso es un “estado”. Dos estados son distintos si hay una bombilla que está encendida en uno

de ellos y apagada en el otro. Diremos que un estado es “bueno” si hay la misma cantidad de

bombillas encendidas en la fila A que en la B.

Demuestra que el número total de estados buenos, EB, dividido por el número total de

estados, ET, es

!2

)12...(753

n

n

ET

EBn

Problema 6

Sea ABC un triángulo con ACAB , sea I su incentro, γ su circunferencia inscrita y D el

punto medio de BC. La tangente a γ por D diferente de BC toca a γ en E. Demuestra que AE

y DI son paralelas.

Page 883: COMPENDIUM OME - ToomatesEotvos de Hungría, que se iniciaron en 1894, precisamente durante la efervescencia de fin de siglo, consecuencia de la cual fue también el proceso iniciado

OLIMPIADA MATEMÁTICA

PROBLEMAS Y SOLUCIONES 2021

REAL SOCIEDAD MATEMÁTICA ESPAÑOLA

Page 884: COMPENDIUM OME - ToomatesEotvos de Hungría, que se iniciaron en 1894, precisamente durante la efervescencia de fin de siglo, consecuencia de la cual fue también el proceso iniciado
Page 885: COMPENDIUM OME - ToomatesEotvos de Hungría, que se iniciaron en 1894, precisamente durante la efervescencia de fin de siglo, consecuencia de la cual fue también el proceso iniciado

12345678910111213141516171819202

1222324

252627

282930

31332333

43536373839404142434445464748495051525354555657

LVII Olimpiada Matemática EspañolaConcurso Final Nacional

7 y 8 de mayo de 2021PROBLEMAS Y SOLUCIONES

Problema 1

Los vértices, A, B y C, de un triángulo equilátero de lado 1 están en la superfi-cie de una esfera de radio 1 y centro O. Sea D la proyección ortogonal de A sobreel plano, α, determinado por B, C y O. Llamamos N a uno de los cortes con laesfera de la recta perpendicular a α por O. Halla la medida del ángulo DNO.

(Nota: la proyección ortogonal de A sobre el plano α es el punto de corte conα de la recta que pasa por A y es perpendicular a α.)

Solución 1:Es obvio que A, B, C y O son vértices de un tetraedro regular de arista igual

a 1, puesto que la distancia entre dos cualesquiera de ellos es 1. Como D es laproyección ortogonal de A sobre la cara opuesta del tetraedro, D es el centro de lacara BCO. Así pues, la distancia de D a O (distancia del centro de un triánguloequilátero de lado 1 a uno de sus vértices) es

d(D,O) =2

3

√3

2=

1√3.

Como el triángulo DNO es rectángulo en O, el cateto OD mide 1/√

3 y elcateto ON mide 1, el ángulo buscado es arc tg 1√

3= 30o.

Solución 2:Elegimos un sistema de coordenadas (x, y, z) de modo que sea O ≡ (0, 0, 0), α

el plano z = 0 y la recta y = z = 0 (es decir, el eje x) que sea la mediatriz deBC por O, estando BC en el semiplano x > 0, z = 0. Tenemos así que, al serBC = 1, son B ≡

(√32, −1

2, 0)y C ≡

(√32, 12, 0). Por simetría respecto del plano

y = 0, tenemos que A ≡ (u, 0, v) con u2 + v2 = 1 para que A esté en la esfera, y(u−

√32

)2+ 1

4+v2 = 1 para que AB = AC = 1. Además, por ser D la proyección

de A sobre α se tiene que D ≡ (u, 0, 0) y OD = |u|. En consecuencia,

u2 = 1− v2 =

(u−√

3

2

)2

+1

4= u2 −

√3u+

3

4+

1

4, OD = u =

1√3.

Luego ODN es un triángulo rectángulo en O, con ON = 1 y OD = 1√3,

concluyéndose que el ángulo buscado es arc tg 1√3

= 30o.

Page 886: COMPENDIUM OME - ToomatesEotvos de Hungría, que se iniciaron en 1894, precisamente durante la efervescencia de fin de siglo, consecuencia de la cual fue también el proceso iniciado

Problema 2

Dado un número entero positivo n, definimos λ(n) como el número de solu-ciones enteras positivas de la ecuación x2 − y2 = n. Diremos que el número nes “olímpico” si λ(n) = 2021. ¿Cuál es el menor entero positivo que es olímpico?¿Y cuál es el menor entero positivo impar que es olímpico?

Solución:Distinguiremos 4 casos, según n sea impar o par y según n sea cuadrado per-

fecto o no.(a) Sea n = pa11 · · · parr un número impar que no es cuadrado perfecto.

Si x2 − y2 = (x + y)(x− y) = n, con x, y > 0, entonces existen enteros positivosa, b, con a > b y teniendo ambos la misma paridad, de manera que x + y = a yx − y = b (con lo que, x = (a + b)/2, y = (a − b)/2). Las formas de escribir ncomo un producto de dos números diferentes de la misma paridad son, en estecaso, la mitad del número de divisores, por lo que tendremos que buscar númeroscon 4042 divisores:

4042 = (a1 + 1) · · · (ar + 1).

La descomposición de 4042 como producto de primos es 4042 = 2·43·47. Por tanto,las opciones de números naturales con 4042 divisores son de la forma: pq42r46;pq2020; p42q93; p46q85; o p4041, en donde p, q, r son primos impares diferentes. Esinmediato comprobar que la opción que da el número más bajo es 346 · 542 · 7.

(b) Consideremos ahora el caso en el que n es impar y cuadrado perfecto. Eneste caso, el número de divisores es impar, y como excluimos el caso de que los dosnúmeros de la factorización sean iguales, tenemos que buscar cuadrados perfectoscon 4043 divisores:

4043 = (a1 + 1) · · · (ar + 1).

La descomposición de 4043 como producto de primos es 4043 = 13 · 311, luegolas únicas opciones son p12q310 y p4042. El número más bajo es 3310 · 512, que esmayor que el que hemos encontrado antes.

(c) Supongamos ahora que n = 2kpa11 · · · parr es par, pero no un cuadrado per-fecto. Nuevamente, tenemos que hacer la descomposición n = ab, siendo a y b de la misma paridad; esto hace que a y b tengan que ser pares, o sea, que k ≥ 2 y no sirven los divisores impares. Por tanto, las opciones para el exponente de 2 en cada divisor son 1, 2, 3, . . . , k − 1, pero nunca 0 ni k. Así pues,

4042 = (k − 1)(a1 + 1) · · · (ar + 1),

y las opciones son 23p42q46; 244pq46; 248pq42; 287p46; 248p85; 295p42; 244q93; 23p2020; 22022p; y 24043 .También es válida la opción 4m, donde m es cualquiera de los números considerados en el apartado (a). El número más bajo es 248 · 342 · 5, que es menor que el encontrado en el apartado (a).

(d) Por último, tenemos el caso en el que n es par y cuadrado perfecto, esto es 4043 = (k − 1)(a1 + 1) · · · (ar + 1) y las opciones son 214p310, 2312p12 y 24044.También es válida la opción 4m, donde m es cualquiera de los números considerados en el apartado (b). Los tres números son mayores que el encontrado en el apartado (c).

Por consiguiente, el menor número olímpico es 248 · 342 · 5 y el menor número impar olímpico es 346 · 542 · 7.

Page 887: COMPENDIUM OME - ToomatesEotvos de Hungría, que se iniciaron en 1894, precisamente durante la efervescencia de fin de siglo, consecuencia de la cual fue también el proceso iniciado

Problema 3

Tenemos 2021 colores y 2021 fichas de cada color. Colocamos las 20212 fichasen fila. Se dice que una ficha, F , es “mala” si a cada lado de F quedan un númeroimpar de las 2020× 2021 fichas que no comparten color con F .

(a) Determina cuál es el mínimo número posible de fichas malas.

(b) Si se impone la condición de que cada ficha ha de compartir color con almenos una ficha adyacente, ¿cuál es el mínimo número posible de fichasmalas?

Solución 1:(a) Como 2020 × 2021 es par, para decidir si una ficha es mala es suficiente

comprobar que el número de fichas a su izquierda con las que no comparte colores impar. Sea A el conjunto de fichas que ocupan una posición impar en la fila, esdecir, las fichas que tienen un número par de fichas a su izquierda. Es claro que|A| = 20212+1

2.

Sea B el conjunto de fichas que, entre las de su color, ocupan una posiciónimpar. Es decir, hay un número par de fichas de su color a su izquierda. Hay 1011fichas de cada color en B, así que |B| = 1011× 2021.

Por construcción, las fichas que están en B pero no en A son malas (tienen unnúmero impar de fichas a su izquierda, de las cuales un número par compartencolor con ella). Por lo tanto, hay al menos |B|−|A| = 2021×1011− 20212+1

2= 1010

fichas malas.Este número se puede conseguir de la siguiente forma: colocamos 2020 fichas

de color 1, luego 2020 fichas de color 2, etc., hasta tener 2020 fichas de cada color;luego colocamos las fichas restantes por orden de color. Las únicas fichas malasen esta configuración son las que son última de cada color par. En la imagen seve esta configuración para 5 colores y 5 fichas de cada color.

malas

(b) Sean X = A ∩ B, Y = A ∩ Bc y Z = Ac ∩ B, donde A y B son losconjuntos descritos en el apartado anterior. Las fichas malas son precisamenteY ∪ Z (contando las fichas a la izquierda que no comparten color con la fichadada, como antes), por lo que tenemos que minimizar el tamaño de este conjunto.

Obsérvese que si la ficha F es adyacente a una ficha F ′ ∈ Z del mismo color,entonces F ∈ Y . Como cada ficha de Z es adyacente a al menos una ficha deY , y cada ficha de Y es adyacente a como mucho dos fichas de Z, tenemos que|Z| ≤ 2|Y |.

Page 888: COMPENDIUM OME - ToomatesEotvos de Hungría, que se iniciaron en 1894, precisamente durante la efervescencia de fin de siglo, consecuencia de la cual fue también el proceso iniciado

Además, tenemos que |Z|− |Y | = (|X|+ |Z|)− (|X|+ |Y |) = |B|− |A| = 1010.Y concluimos que

|Y |+ |Z| = 3(|Z| − |Y |)− 2(|Z| − 2|Y |) ≥ 3× 1010− 2× 0 = 3030.

(La conclusión del último párrafo también se puede obtener haciendo:|Z| = 1010 + |Y | ≥ 1010 + 1

2|Z|, luego |Z| ≥ 2020 y |Z|+ |Y | ≥ 3

2|Z| ≥ 3030.)

Una forma de tener exactamente 3030 fichas malas es colocar primero 2018fichas de color 1, luego 2018 de color 2, etc., y después colocar las tres fichasrestantes de color 1, luego las tres de color 2, etc. Las únicas fichas malas son lastres últimas de cada color par.

Solución 2: Diremos que una ficha que no es mala, es “buena”. Numeramos loscolores de 1 a 2021. Decimos que una ficha es par (impar) si ocupa una posiciónpar (impar) en la fila, y decimos que es color-par (color-impar) si de entre lasfichas de su color, ocupa un lugar par (impar). Si una ficha es par y color-par,eso quiere decir que tiene antes de ella en la fila un número impar de fichas de sucolor, y también un número impar de fichas en total. Luego tiene antes que ellaun número par de fichas de otros colores, y al ser 2020 · 2021 par el número totalde fichas de otros colores, también tiene un número par de fichas de otros coloresdetrás de ella. Luego una ficha par y color-par es buena. De forma análoga, unaficha impar y color-impar es buena, pero son malas todas las fichas pares queson color-impares, y todas las fichas impares que son color-pares. Nótese que elnúmero de fichas color-pares, pares, impares y color-impares son respectivamente

2021 · 1010 <20212 − 1

2<

20212 + 1

2< 2021 · 1011.

Supongamos que hay exactamente u fichas que sean malas por ser impares perocolor-pares. Esto quiere decir que hay exactamente 20212+1

2− u fichas que son

buenas por ser impares y color-impares, luego el número de fichas que son malaspor ser pares y color-impares es exactamente

2021 · 1010−(

20212 + 1

2− u)

= 1010 + u.

Las restantes fichas serán pares y color-pares y por lo tanto buenas, y el númerototal de fichas malas será igual a 1010 + 2u.

(a) Podemos hacer u = 0, y por lo tanto que haya exactamente 1010 fichasmalas, todas ellas pares y color-impares, si colocamos primero 2020 fichas delcolor 1, luego 2020 fichas del color 2, y así sucesivamente hasta el color 2021,y a continuación las 2021 fichas restantes, una de cada color. Nótese que lasprimeras 2020 · 2021 fichas, todas son bien pares y color-pares, bien impares ycolor-impares, luego buenas en cualquier caso. Las 2021 fichas últimas son cadauna de ellas color-impares, pero de ellas 1011 son impares y por lo tanto buenas,y 1010 son pares y por lo tanto malas. El mínimo, alcanzable de esta forma, esentonces igual a 1010.

Page 889: COMPENDIUM OME - ToomatesEotvos de Hungría, que se iniciaron en 1894, precisamente durante la efervescencia de fin de siglo, consecuencia de la cual fue también el proceso iniciado

(b) Nótese que cada ficha mala por ser par y color-impar tiene ahora a su ladouna ficha del mismo color, que por lo tanto será impar y color-par, luego u > 0.Asignemos cada ficha par y color-impar a la ficha contigua de su mismo colorque es impar y color-par. Si una ficha par y color-impar es contigua a dos de sumismo color, la asignamos a una cualquiera de las dos. Nótese que como cadaficha impar y color-par tiene a lo sumo dos fichas vecinas, hay a lo sumo 2u fichasque pueden ser asignadas a u fichas impares y color-pares. Pero sabemos quehay exactamente 1010 + u fichas asignadas, luego 1010 + u ≤ 2u, y por lo tantou ≥ 1010. Concluimos que hay como mínimo 3030 fichas malas, alcanzándose esemínimo si la asignación es completa, es decir si cada ficha mala por ser impar ycolor-par está rodeada de dos fichas malas de su mismo color, y cada ficha malapor ser par y color-impar es contigua sólo a una ficha mala de su mismo color.Esto puede conseguirse por ejemplo colocando primero 2018 fichas del color 1,luego 2018 fichas del color 2, y así hasta el color 2021, colocando después lasrestantes 3 ·2021 fichas, 3 de cada color, en 2021 ternas formada cada una por las3 fichas restantes de cada color. Las primeras 2018 · 2021 fichas son claramentebuenas, y de las 2021 ternas al final de la fila, 1011 ternas tienen su primera fichaimpar y color-impar, y por lo tanto las 3 fichas de la terna son buenas, y 1010ternas tienen su primera ficha par y color-impar, y por lo tanto las 3 fichas de laterna son malas. El mínimo, alcanzable de esta forma, es entonces igual a 3030.

Page 890: COMPENDIUM OME - ToomatesEotvos de Hungría, que se iniciaron en 1894, precisamente durante la efervescencia de fin de siglo, consecuencia de la cual fue también el proceso iniciado

Problema 4

Sean a, b, c, d números reales tales que

a+ b+ c+ d = 0 y a2 + b2 + c2 + d2 = 12.

Halla el valor mínimo y el valor máximo que puede tomar el producto abcd, ydetermina para qué valores de a, b, c, d se consiguen ese mínimo y ese máximo.

Solución 1:De las condiciones del enunciado tenemos que no todos los números tienen el

mismo signo. El producto abcd tomará un valor positivo cuando dos números seanpositivos y dos negativos, así que buscaremos el máximo suponiendo que a, b > 0y c, d < 0. Notemos que

2(ab+ cd) ≤ a2 + b2 + c2 + d2 = 12, (1)

con lo que ab + cd ≤ 6. Utilizando la desigualdad entre las medias aritmética ygeométrica,

(ab) · (cd) ≤(ab+ cd

2

)2

≤ 9. (2)

La igualdad en (2) se alcanza cuando ab = cd = 3, con lo que (1) obliga aque sea (a − b)2 = (c − d)2 = 0; es decir, a = b y c = d. Así pues, a = b =

√3,

c = d = −√

3. El valor máximo de la expresión es por tanto 9.

Para hallar el valor mínimo supondremos que a, b, c > 0 y que d < 0. (Si tres delos números son negativos y el otro positivo, considerando sus opuestos tenemosque el valor de abcd permanece invariante.) Por tanto, d = −(a+ b+ c) y

a2 + b2 + c2 + d2 = 2(a2 + b2 + c2 + ab+ bc+ ca) = 12.

Esto quiere decir que

(a+b+c)2 = a2+b2+c2+ab+bc+ca+ab+bc+ca ≤ 6+a2 + b2 + c2 + ab+ bc+ ca

2≤ 9,

donde se ha usado que, por la desigualdad de Cauchy, ab+ bc+ ca ≤ a2 + b2 + c2.Por tanto, a+ b+ c ≤ 3. El problema es equivalente a encontrar el máximo valorposible de abc(a + b + c). Usando nuevamente la desigualdad entre las mediasaritmética y geométrica,

abc ≤ (a+ b+ c)3

27y por tanto abc(a+ b+ c) ≤ (a+ b+ c)4

27≤ 3.

En consecuencia, el valor mínimo es −3 y se alcanza en los casos (3,−1,−1,−1)y (1, 1, 1,−3) (o permutaciones de estos).

Page 891: COMPENDIUM OME - ToomatesEotvos de Hungría, que se iniciaron en 1894, precisamente durante la efervescencia de fin de siglo, consecuencia de la cual fue también el proceso iniciado

Solución 2:Aplicando la desigualdad de las medias cuadrática y geométrica a |a|, |b|, |c|, |d|,

tenemos que

|a| · |b| · |c| · |d| ≤

(√a2 + b2 + c2 + d2

4

)4

=(√

3)4

= 9,

con igualdad si y sólo |a| = |b| = |c| = |d| =√

3. Con la condición a+ b+ c+ d =0, esto sólo es posible si exactamente dos de entre a, b, c, d son positivos y losotros dos negativos. Como esta cota máxima es independiente de la condicióna + b + c + d = 0, tenemos entonces que el máximo valor posible de abcd es 9, yse obtiene si y sólo si dos de entre a, b, c, d son

√3 y los otros dos son −

√3.

Tomando a = 3, b = c = d = −1, se obtiene abcd = −3, con lo que el valormínimo del producto ha de ser negativo, y se debe conseguir claramente cuandoun número impar de los a, b, c, d es negativo, y los demás son positivos. Comoinvertir simultáneamente los signos de a, b, c, d deja inalteradas las condicionesy el producto, podemos asumir sin pérdida de generalidad que a > 0 > b, c, dpara el mínimo de abcd. Ahora bien, por la desigualdad entre medias aritméticay geométrica aplicadas a |b|, |c|, |d| tenemos que

|bcd| = |b| · |c| · |d| ≤(|b|+ |c|+ |d|

3

)3

=

(−b− c− d

3

)3

=a3

27,

con igualdad si y sólo si b = c = d, mientras que por la desigualdad entre mediasaritmética y cuadrática,

a = |b|+ |c|+ |d| ≤ 3

√b2 + c2 + d2

3= 3

√12− a2

3, a2 ≤ 36− 3a2,

por lo que a ≤ 3, con igualdad si y sólo si b = c = d = −1. Luego cuandoa > 0 > b, c, d, tenemos que

abcd = −a|bcd| ≥ −a4

27≥ −3,

y el mínimo valor posible de abcd es −3 y, restaurando la generalidad, se obtienesi y sólo si (a, b, c, d) es una permutación de (±3,∓1,∓1,∓1).

Page 892: COMPENDIUM OME - ToomatesEotvos de Hungría, que se iniciaron en 1894, precisamente durante la efervescencia de fin de siglo, consecuencia de la cual fue también el proceso iniciado

Problema 5

Disponemos de 2n bombillas colocadas en dos filas (A y B) y numeradas de 1a n en cada fila. Algunas (o ninguna) de las bombillas están encendidas y el restoapagadas; decimos que eso es un “estado”. Dos estados son distintos si hay unabombilla que está encendida en uno de ellos y apagada en el otro. Diremos queun estado es “bueno” si hay la misma cantidad de bombillas encendidas en la filaA que en la B.

Demuestra que el número total de estados buenos, EB, dividido por el númerototal de estados, ET, es

EB

ET=

3 · 5 · 7 · · · (2n− 1)

2n n!.

Solución 1:Es obvio que ET = 22n, puesto que cada una de las 2n bombillas puede estar

apagada o encendida. El número de “estados buenos” con k bombillas encendidas

en cada fila es(nk

)2

, ya que hay(nk

)formas de elegir las k bombillas encendidas

de la fila A y otras tantas de elegir las k bombillas encendidas de la fila B. Enconsecuencia,

EB =n∑k=0

(nk

)2

.

Es conocido que esa suma da como resultado EB =

(2nn

). En cualquier caso,

basta con observar que un estado es “bueno” si hay exactamente k (0 ≤ k ≤ n)bombillas encendidas en la fila A y exactamente n−k bombillas apagadas en la filaB. Cada “estado bueno” se obtiene (de una única manera) eligiendo n bombillasen total y haciendo que estén encendidas las elegidas de la fila A y las no elegidas

de la fila B. Así pues, EB =

(2nn

).

Ahora, tenemos:

EB

ET=

(2nn

)22n

=(2n)!

22nn!n!=

(∏nk=1 2k

)(∏nk=1(2k − 1)

)22nn!n!

=

=2nn!

(∏nk=1(2k − 1)

)22nn!n!

=3 · 5 · 7 · · · (2n− 1)

2n n!.

Page 893: COMPENDIUM OME - ToomatesEotvos de Hungría, que se iniciaron en 1894, precisamente durante la efervescencia de fin de siglo, consecuencia de la cual fue también el proceso iniciado

Solución 2:Si empezamos “desde el final”, notemos que

(2n)!

3 · 5 · 7 · · · (2n− 1)= (2n) · (2n− 2) · · · 4 · 2 = 2n · n · (n− 1) · · · 2 · 1 = 2n · n!,

y como claramente ET = 22n porque cada una de las 2n bombillas puede estar,independientemente de las demás, encendida o apagada, el problema se reduce ademostrar que

EB = 22n · 3 · 5 · 7 · · · (2n− 1)

2n · n!= 22n · (2n)!

(2n · n!)2=

(2n)!

(n!)2=

(2nn

).

La demostración de que EB =

(2nn

)puede hacerse de distintas maneras. Una

de ellas la dada en la Solución 1. Otras maneras son:Considerando el binomio (1 + x)2n = (1 + x)n · (1 + x)n. El coeficiente de xn

es por una parte igual a(

2nn

), y por otra igual a la suma de los productos de

los coeficientes respectivos(nm

)y(

nn−m

)de xm y xn−m en cada (1 + x)n.

También puede verse como el número de caminos en una cuadícula de n× n,desde (0, 0) hasta (n, n), en la que cada desplazamiento lleva desde (x, y) bien a(x + 1, y) bien a (x, y + 1). El número total de caminos, cada uno de los cualesha de tener 2n desplazamientos de los que n son en horizontal y n en vertical,

es(

2nn

). Ahora bien, si consideramos la diagonal “descencente” formada por los

puntos (x, y) tales que x+ y = n, cada camino ha de pasar por exactamente unode ellos. Los que pasan por (m,n−m) tienen, en sus primeros n desplazamientos,m horizontales y n−m verticales, y en sus siguientes n desplazamientos, n−m

horizontales y m verticales, para un total de(nm

)(n

n−m

)=

(2nn

)2

caminos

pasando por ese punto. Sumando las contribuciones de cada elemento de estadiagonal se halla la relación pedida.

Page 894: COMPENDIUM OME - ToomatesEotvos de Hungría, que se iniciaron en 1894, precisamente durante la efervescencia de fin de siglo, consecuencia de la cual fue también el proceso iniciado

Problema 6

Sea ABC un triángulo con AB 6= AC, sea I su incentro, γ su circunferenciainscrita y D el punto medio de BC. La tangente a γ por D diferente de BC tocaa γ en E. Demuestra que AE y DI son paralelas.

Solución 1:(Ver la figura 1) Sea P el punto de tangencia de γ con BC. Sean Q y R los

puntos simétricos de P con respecto a D e I, respectivamente. Tenemos que:

• ER es paralela a DI. Por un lado, DI es perpendicular a PE (es de hechola mediatriz de PE). Por otro lado, por ser R el punto diametralmenteopuesto a P en γ el ángulo ∠PER es recto.

• QR es paralela a DI. Esto es así porque D e I son los puntos medios dePQ y PR.

De ahí se deduce que Q, R y E están alineados, en una recta paralela a DI. Paraconcluir el problema basta demostrar que A, Q y R están alineados.

Trazando la paralela a BC por R obtenemos un triángulo semejante a ABC,que es el resultante de tomar una homotecia de ABC con respecto a A. TantoR como Q son los puntos de tangencia de la circunferencia exinscrita del ladoopuesto de A, por lo que la homotecia envía Q a R. Concluimos que A, Q y Restán alineados.

Nota: (Ver la figura 2) A continuaciónn se incluye la demostración de quelos puntos de tangencia de las circunferencias inscrita y exinscrita con BC sonsimétricos con respecto al punto medio de BC. Usando la notación de la figura,tenemos que demostrar que BP = CX. Por un lado tenemos que BP + CP =BX+CX. Por otro, tenemos CP−BP = CN−BM = CN+AN−BM−AM =AC−AB = (AY −AB)− (AZ−AC) = BY −CZ = BX−CX. De este sistemade ecuaciones se halla BP = CX.

Page 895: COMPENDIUM OME - ToomatesEotvos de Hungría, que se iniciaron en 1894, precisamente durante la efervescencia de fin de siglo, consecuencia de la cual fue también el proceso iniciado

Solución 2:Sea P el punto de tangencia de γ con BC, y sean S y T los puntos medios

de EP y AP , respectivamente. Claramente ST es paralela a AE. Como DI es lamediatriz de EP , tenemos que S está en DI. Por lo tanto, el problema se reducea demostrar que T está en DI.

Sea L el punto en el que la bisectriz AI corta a BC. Aplicando el teoremade Menelao al triángulo APL, tenemos que demostrar que AT

TPPDDL

LIIA

= 1. Pordefinición de T , AT

TP= 1.

Si a, b y c son las longitudes de los lados de ABC, por el teorema de la bisectriztenemos BL

CL= c

b. Como BL + LC = a, tenemos CL = ab

b+c. Aplicando ahora el

teorema de la bisectriz a ALC, tenemos LIIA

= CLCA

= ab+c

.Tenemos también CD = a

2y CP = a+b−c

2, con lo que

PD

DL=CP − CDCL− CD

=a+b−c

2− a

2abb+c− a

2

=b+ c

a.

Sustituyendo, obtenemos ATTP

PDDL

LIIA

= 1, como queríamos demostrar

Page 896: COMPENDIUM OME - ToomatesEotvos de Hungría, que se iniciaron en 1894, precisamente durante la efervescencia de fin de siglo, consecuencia de la cual fue también el proceso iniciado

Solución 3:Sea ω la circunferencia con centro D que pasa por E. Por simetría respecto de

la recta DI, ω también pasa por el punto F de tangencia de γ con BC. El segundopunto, G, donde ω corta a BC es claramente el simétrico de F con respecto alpunto medio D de BC, que como es conocido es el punto donde la circunferenciaA−exinscrita es tangente al lado BC. Es también conocido que existe una ho-motecia con centro A que convierte γ en esta circunferencia A−exinscrita, y poresta homotecia, los puntos A y G están alineados con el punto F ′ diametralmenteopuesto a F en γ. Ahora bien, como FG es diámetro de ω y FF ′ es diámetrode γ estando E en ambas circunferencias, se tiene que ∠FEF ′ = ∠GEF = 90o,luego F ′, E,G están alineados, y por lo tanto la recta AE es perpendicular a FE.Como E,F son los puntos de corte de γ y ω, son simétricos respecto a la rectaDI que une sus centros. Luego DI es perpendicular a FE, y por lo tanto paralelaa AE, como queríamos demostrar.

Solución 4:Sea un sistema de coordenadas cartesianas tal que el eje horizontal sea la recta

BC, y el eje vertical pase por I. Se tiene entonces que D ≡ (δ, 0) para un ciertoδ 6= 0 y que como D es el punto medio de BC, B ≡ (−∆ + δ, 0) y C ≡ (∆ + δ, 0),donde ∆ = a

2. Además, I ≡ (0, r), donde r es el inradio.

El origen O que es el punto de tangencia de BC con γ y E son claramentesimétricos respecto a la recta DI ≡ y = r − rx

δ. Luego E está en la recta y = δx

r,

y sobre la circunferencia γ, que es x2 + (y − r)2 = r2. Se tiene entonces porsustitución, y descartando la solución x = 0 que produce O, que

E ≡(

2r2δ

r2 + δ2,

2r2δ

r2 + δ2

).

De forma análoga podemos hallar el punto de tangencia de γ con el lado AB,sin más que sustituir la coordenada horizontal δ deD por la coordenada horizontal−∆ + δ de B, obteniéndose que este punto es

E ≡(

2r2(δ −∆)

r2 + (δ −∆),

2r(δ −∆)2

r2 + (δ −∆)2

).

y por lo tanto la recta AB, que pasa por este punto y por B, tiene ecuación

y =2r(δ −∆)

r2(δ −∆)2(x− δ + ∆).

Como la coordenada horizontal de C es igual a la de B pero con signo opuestopara ∆, invirtiendo el signo de ∆ en la anterior ecuación, obtenemos la de la rectaAC:

y =2r(δ −∆)

r2 − (δ + ∆)2(x− δ −∆).

Resolviendo el sistema formado por ambas ecuaciones, obtenemos

Page 897: COMPENDIUM OME - ToomatesEotvos de Hungría, que se iniciaron en 1894, precisamente durante la efervescencia de fin de siglo, consecuencia de la cual fue también el proceso iniciado

A ≡(

2r2δ

r2 + δ2 −∆2,

2r(δ2 −∆2)

r2 + δ2 −∆2

).

La pendiente de la recta AE viene entonces dada por la diferencia entre suscoordenadas y,

2r3∆2

(r2 + δ2)(r2 + δ2 −∆2)

dividida entre la diferencia de sus coordenadas x,

2r2δ∆2

(r2 + δ2)(r2 + δ2 −∆2),

que es claramente igual a − rδ, y por lo tanto igual a la pendiente de DI, y hemos

terminado.Nota: En esta solución puede haber en algún momento denominadores que

se anulen, casos que por lo tanto han de ser tratados aparte. En concreto, losdenominadores que podrían a priori anularse son r2 − (δ −∆)2, r2 − (δ + ∆)2 yr2 + δ2 −∆2. El que r2 = (δ −∆)2 no es sin embargo ningún problema. En esecaso, la recta AB sería vertical, y la coordenada x de A sería

2(δ −∆)2δ

(δ −∆)2) + δ2 −∆2,

idéntica a la de B, quedando determinada la coordenada y de A por la recta AC.De forma análoga, si r2 = (δ − ∆)2, la recta AC sería vertical, siendo la

coordenada x de A igual a δ + ∆, y quedando determinada su coordenada y porla recta AB. Finalmente, como ∆ = a

2y δ = |a

2− c+a−b

2| = |b−c|

2, tenemos que

r2 = ∆2 − δ2 es equivalente a

r2 =a2 − (b− c)2

4=a2 − b2 − c2 + 2bc

4= bc sin2 A

2= r2 cot

B

2cot

C

2,

donde hemos usado el conocido resultado r = 4R sin A2

sin b2

sin C2. Ahora bien, a

su vez esto sería equivalente a cos B+C2

= 0, absurdo pues B+C < 90o. Obsérveseque la condición para que ese tercer denominador se anule es B+C = 180. Estoes intuitivamente consistente con que el denominador al hallar las coordenadasde A se anule: el que este denominador sea nulo es equivalente a que el punto Aesté “en el infinito”, que a su vez es equivalente a que las rectas AB y AC seanparalelas, es decir B + C = 180.

En definitiva, los denominadores que pueden realmente anularse no suponenun problema en la solución propuesta, sino que simplemente indican que dos delas rectas definidas en la solución son verticales.

Page 898: COMPENDIUM OME - ToomatesEotvos de Hungría, que se iniciaron en 1894, precisamente durante la efervescencia de fin de siglo, consecuencia de la cual fue también el proceso iniciado
Page 899: COMPENDIUM OME - ToomatesEotvos de Hungría, que se iniciaron en 1894, precisamente durante la efervescencia de fin de siglo, consecuencia de la cual fue también el proceso iniciado

OLIMPIADA I

Page 900: COMPENDIUM OME - ToomatesEotvos de Hungría, que se iniciaron en 1894, precisamente durante la efervescencia de fin de siglo, consecuencia de la cual fue también el proceso iniciado

II OLIMPIADAPARTICIPANTES DE LA SEGUNDA

FASE

Page 901: COMPENDIUM OME - ToomatesEotvos de Hungría, que se iniciaron en 1894, precisamente durante la efervescencia de fin de siglo, consecuencia de la cual fue también el proceso iniciado

CARTEL DE LA PRIMERA OLIMPIADA

Page 902: COMPENDIUM OME - ToomatesEotvos de Hungría, que se iniciaron en 1894, precisamente durante la efervescencia de fin de siglo, consecuencia de la cual fue también el proceso iniciado

ADAPTACIÓN DE LOS CARTELES ALOS PLANES DE ESTUDIO

Page 903: COMPENDIUM OME - ToomatesEotvos de Hungría, que se iniciaron en 1894, precisamente durante la efervescencia de fin de siglo, consecuencia de la cual fue también el proceso iniciado

Compilación realizada por Gerard Romo durante el confinamiento del Covid-19 del año 2020.

Maials (Lleida), 24 de Abril del 2020

Actualizado el 27/7/2020.

Segunda edición el 8/2/2021 (toda la fase local se separa en un documento aparte).

Actualizado el 9/5/2021.